Fundamentals Final

Ace your homework & exams now with Quizwiz!

A nurse is caring for a group of 6 and 7 year olds. The nurse remembers that, according to Kohlberg, moral development is a component of psychosocial development. Moral development depends on the childs ability to do which of the following? (Select all that apply.) a.Accept social responsibility. b.Respect the integrity and rights of others. c.Integrate principles of justice and fairness. d.Use symbols and objects on the way to abstract thinking. e.Perform repetitive motion responses.

A, B, C Moral development depends on the childs ability to accept social responsibility and integrate personal principles of justice and fairness. In addition, the childs knowledge of right and wrong and behavioral expression of this knowledge must be founded on respect and regard for the integrity and rights of others. Piagets theory, not Kohlbergs, states that as the child grows from infancy into adolescence, the intellectual development progresses, starting with reflex and repetitive motion responses, to the use of symbols and objects from the childs point of view, to logical thinking, and finally to abstract thinking.

A nursing student is concerned with sensory deprivation among the patients in the nursing home during the clinical rotation. Which of the following could be caused by sensory deprivation? (Select all that apply.) a.Confusion b.Anxiety c.Disorientation d.Panic e.Aggressiveness

A, B, C Sensory deprivation occurs when inadequate quality or quantity of stimuli impairs perception. These effects sometimes produce cognitive changes, such as the inability to solve problems, poor task performance, and disorientation. Affective changes, which include boredom, restlessness, increased anxiety, or emotional ability, can occur. Symptoms of sensory overload include panic, confusion, and aggressiveness.

The nurse is caring for a patient who has just been found in respiratory arrest. Which laboratory values will the RN expect to find in the patient's chart? (Select all that apply.) a. pH 7.28 b. PaO2 65 mm Hg c. PaCO2 58 mm Hg d. HCO3 - 15 mEq/L

A, B, C Normal arterial blood pH value is 7.35 to 7.45 (acidic is less than 7.35, and alkalotic is greater than 7.45). Respiratory acidosis is an increased PaCO2 and an increased hydrogen ion concentration (pH below 7.35) that reflect the excess carbonic acid (H2CO3) in the blood. Hypoventilation produces respiratory acidosis. The elevated PaCO2 level is a result of the patient not breathing (normal range is 35 to 45 mm Hg). The low PaO2 is also a result of not breathing (normal range is 80 to 100 mm Hg). The low HCO3 - is not expected as a result of respiratory arrest (normal range is 22 to 26 mEq/L)

Blood pressure depends on the interrelationships of which of the following? (Select all that apply.) a.Cardiac output b.Peripheral vascular resistance c.Blood volume d.Blood viscosity e.Pulse pressure

A, B, C, D

A home care nurse is conducting a home assessment. The nurse is looking for the presence of sensory alterations. Factors to assess include if any changes have occurred in which of the following? (Select all that apply.) a.Activities of ADLs b.Health promotion c.Has person had visitors d.Is person wearing hearing aids and glasses e.Ability to follow a conversation

A, B, C, D, E When assessing for sensory alterations, home care nurses need to assess sensory status, self-care management, and health promotion activities, as well as lifestyle and socialization.

A middle-age patient was admitted to the trauma intensive care unit after a motor vehicle accident. The nurse notes that the patient becomes increasingly agitated when visitors stay for an extended period or after nursing interventions. The nurse identifies this as sensory overload. Which of the following would most likely help the patient? (Select all that apply.) a.Reducing the number of visitors to her room b.Performing dressing changes with the bath c.Providing a dedicated period of rest time each afternoon d.Requesting that health care providers do rounds when the family is available e.Coordination with other departments for tests and examinations

A, B, C, E Reduce sensory overload by organizing the patients care to control for excessive stimuli. Combining activities such as dressing changes, bathing, and vital sign assessment in one visit prevents the patient from becoming overly fatigued. Coordination with other departments will reduce the time needed for tests and examinations. The patient needs time for rest and quiet. Although it is important for health care professionals to communicate to family members, it will not likely reduce sensory overload for the patient.

Which life events should the nurse recognize as being spiritually life changing? (Select all that apply.) a. Births b. Weddings c. Medical diagnoses d. Career day to day job duties e. Loss of independence

A, B, C, E The meaning and significance of the event might only be experienced by one individual; others who might be participants in the event might be left virtually untouched and unchanged. These life changing spiritual events include just about any occurrence that has intense and personal relevance to those involved in the event. Examples of spiritually life changing events include births, deaths, weddings, divorces, illnesses, diagnoses, and loss of abilities, loss of independence, death and so many more. These events, having the power to change individuals and families, also have the power to draw people toward the transcendent—for many people that transcendent is known as God but this is not universal. Day-to-day activities are not the best examples of spiritually life changing events.

The nurse is caring for an older adult patient who has a constant urge to void due to a bladder infection. Which are the appropriate nursing diagnoses for this patient? (Select all that apply.) a. Sleep deprivation related to frequent need to use the toilet during the night b. Risk for falls related to getting up frequently to use the bathroom at night c. Impaired urinary elimination: frequency related to urinary tract inflammation d. Risk for infection related to bacterial invasion of urinary tract e. Risk for urge urinary incontinence related to urinary tract inflammation

A, B, C, E Sleep deprivation is appropriate for the patient due to frequent need to use the toilet at night. The elderly patient is at risk for falls when ambulating to the toilet. Urinary frequency is appropriate as the patient has a constant urge to void. Risk for urge urinary incontinence is also appropriate as urinary tract inflammation can lead to loss of urine. Risk for infection is not appropriate as the patient already has a bladder infection.

A nurse is caring for a patient who suffers from a sleep pattern disturbance. To promote adequate sleep, what are the most appropriate nursing interventions? (Select all that apply.) a. Provide personal hygiene before bedtime. b. Straighten and change any soiled bed linens. c. Assist the patient to use the toilet before bed. d. Administer sleep aids every night at the same time. e. Synchronize the schedule for medications and vital signs.

A, B, C, E You will make the patient more comfortable in an acute care setting by providing personal hygiene before bedtime. Have patients void before going to bed so they are not kept awake by a full bladder. Clean, dry linens make the patient comfortable for falling asleep. Medications and vital signs should be scheduled to wake the patient as infrequently as possible. Sleep aids should be avoided as they carry risk of side effects and long-term dependency.

The nurse is caring for a patient who becomes agitated when visitors stay for extended periods or the hospital unit becomes noisy. The nurse identifies this as sensory overload. Which interventions will be of benefit to the patient? (Select all that apply.) a. Reduce the number of visitors to the patient's room. b. Provide a dedicated period of rest time each afternoon. c. Institute a unit-wide quiet time at 10:00 p.m. each night. d. Turn on the television to drown out noise from other patients. e. Coordinate therapies and tests with other departments and providers.

A, B, C, E Reduce sensory overload by organizing the patient's care to control for excessive stimuli. Reducing the number of visitors to the patient's room and instituting a unit-wide quiet time can help the patient to rest comfortably. Coordination with other departments will reduce the time needed for therapies and tests.

Which therapeutic communication techniques should the nurse use while communicating with a small child? (Select all that apply.) a. Sit at the child's eye level. b. Use simple, direct language. c. Use drawings and toys as needed. d. Tell the child exactly what they can do. e. Avoid sudden movements or gestures.

A, B, C, E Sudden movements or gestures can be frightening so they need to be avoided. When giving explanations or directions, use simple, direct language and be honest. Meet a child at eye level. Drawing and playing with young children allows them to communicate nonverbally (making the drawing) and verbally (explaining the picture). Telling the child exactly what they can do is inappropriate. Remain calm and gentle and, if possible, let a child make the first move.

When completing the FICA tool for spiritual assessment, which questions should the nurse ask the patient? (Select all that apply.) a. What things do you believe in that give meaning to life? b. Are you connected with a faith center in your community? c. How has your illness affected your personal beliefs? d. When was the last time you have been to church? e. What can I do for you?

A, B, C, E The FICA tool for spiritual assessment stands for Faith or beliefs, Importance and influence, Community, and Address. "When was the last time you have been to church?" is not a question included in the FICA assessment. The patient may attend community activities, besides church, that foster his/her spiritual well-being.

Any injury to the hypothalamus impairs heat loss mechanisms. Educate patients at risk for hyperthermia to do which of the following? (Select all that apply.) a.Avoid strenuous exercise in hot, humid weather. b.Avoid exercising in areas with poor ventilation. c.Drink clear fluids before and after exercising, not during. d.Wear light, loose-fitting clothing. e.Do not visit hot climates.

A, B, D

While being tested for orthostatic hypotension, the patient stands up and begins to feel light-headed and feels faint. The nurse should do which of the following? (Select all that apply.) a.Have the patient lie down. b.Report findings to the health care provider. c.Have the nursing assistive personnel (NAP) check orthostatic blood pressure. d.Instruct the patient not to get out of bed without assistance. e.Take the BP in each arm and use the arm with the lowest systolic reading.

A, B, D

An adult collapsed at the grocery store. Before being transported to the emergency department, an automated external defibrillator (AED) was used. The nurse remembers learning that the AED is effective in saving a life because of which factors? (Select all that apply.) a.Use of an AED strengthens the chain of survival. b.The AED can be used by nonmedical personnel. c.The AED sends heart rhythm to the closest emergency room (ER) for the physician to analyze and give orders. d.Every minute without defibrillation decreases the survival rate by 7% to 10%. e.The AED will automatically deliver a shock to the victim after announcing, Everyone stand back.

A, B, D The following is information on the use of an automated external defibrillator (AED): The automated external defibrillator (AED) is a device used to administer an electrical shock through the chest wall to the heart. The AED has a built-in computer that assesses the victims heart rhythm and determines if defibrillation is needed. The rescuer delivers a shock to the victim after announcing, Everyone stand back. The AED can be used by nonmedical personnel. Use of an AED strengthens the chain of survival. Every minute of a sudden cardiac death without defibrillation decreases the survival rate by 7% to 10% (American Heart Association, 2005).

Which characteristics helped promote the development of the patient's tortuous varicose veins (Select all that apply.) a. Smoking 2 packs of cigarettes daily for the last 15 years b. Frequently sitting with the legs crossed c. Taking naproxen sodium daily for arthritis pain d. Standing for long periods of time at work e. Eating a gluten-free, low-sodium diet for the last 10 years

A, B, D Smoking, sitting with the legs crossed, and standing for long periods all contribute to the development of varicose veins. Taking NSAIDs daily and eating a gluten-free, low-sodium diet will not increase the patient's risk of developing varicose veins.

Which interventions are appropriate for the patient with the nursing diagnosis decreased cardiac output related to reduced stroke volume and contractility? (Select all that apply.) a. Frequent lung sound assessment and continuous pulse oximetry. b. Strict intake and output monitoring with daily weights before breakfast. c. Administer oxygen to maintain pulse oximetry levels between 90% and 92%. d. Provide stool softeners and encourage dietary fiber to prevent constipation. e. Encourage the patient to consume additional salt to maintain blood pressure.

A, B, D Lung sounds and pulse oximetry will monitor for impaired oxygenation and pulmonary edema. Strict intake and output will monitor for weight gain seen with congestive heart failure. Constipation should be avoided as straining at stool can lead to dysrhythmias. The patient's pulse oximetry should be kept at least 95% to promote oxygenation of tissues. Additional salt will increase cardiac workload and lead to worsening of heart failure

On entering the room, a nurse finds the patient sitting upright in bed with the upper torso resting on the over-bed table. The nurse assesses that this patient is experiencing acute hypoxemia. Which of the following are symptoms of acute hypoxemia? (Select all that apply.) a.Cyanosis b.Arrhythmias c.Eupnea d.Restlessness e.Diaphoresis

A, B, D, E Symptoms of acute hypoxemia include changes in respiration (tachypnea, dyspnea); blood pressure (hypertension, hypotension); color (pallor, cyanosis); mental status (headache, anxiety, impaired judgment, confusion, euphoria, lethargy); motor function (loss of coordination, weakness, tremors, hyperactive reflexes, restlessness, stupor, coma [around 30 mm Hg], death); arrhythmias (tachycardia, bradycardia), diaphoresis, blurred or tunnel vision, and nausea/vomiting. Eupnea is a normal breathing rate.

When a nurse is conducting a health history assessment, which information would be viewed as most important as related to the patient's elimination status? (Select all that apply.) a. Recent changes in elimination patterns b. Changes in color, consistency, or odor of stool or urine c. Time of day patient defecates d. Discomfort or pain with elimination e. List of medications taken by patient f. Patient's preferences for toileting

A, B, D, E Recent changes in elimination patterns, color, consistency, or odor are important for the nurse to know concerning elimination. Discomfort or pain during elimination is important for the nurse to know. A nurse should also know which medications the patient is on as this may affect elimination. Personal preferences are not the most important data the nurse needs to collect.

On admission a patient is noted to have an alteration in skin integrity on the right heel. The nurse uses the Braden Scale. Which areas will the nurse assess when using this scale? (Select all that apply.) a. Mobility b. Nutrition c. Infection d. Activity e. Friction

A, B, D, E The Braden Scale is a highly reliable scale that uses six subscales to identify patients at greatest risk for pressure injuries: sensory perception, moisture, activity, mobility, nutrition, and friction and shear. Infection is not an area that is assessed on the Braden Scale.

A nurse is about to administer a medication and notices that the physicians or primary health care providers order looks incorrect regarding the amount of the medication. What should the nurse do? (Select all that apply.) a.Notify the physician or health care provider. b.Do not carry out the order. c.Document the suspicion that the dosage is incorrect. d.Administer the medication. e.Notify the supervisor or nurse manager

A, B, E Nurses are responsible for carrying out medical treatment unless the physicians or health care providers order is in error, violates hospital policy, or is harmful to the patient. Therefore it is imperative to assess all orders and, if they appear to be erroneous or harmful to the patient, to obtain further clarification from the physician or health care provider. Do not carry out the order if there is a risk that harm will come to your patient; therefore do not administer the medication. Inform the nurse manager or the nursing supervisor. The nurse does not document suspicions or opinions, just objective, factual information.

A postoperative abdominal surgery patient has been admitted to the surgical floor. The nurse is aware that wound healing is delayed due to complications. Which conditions would prevent normal wound healing at the surgical site? (Select all that apply.) a. Dehiscence b. Evisceration c. Debridement d. Hemostasis e. Hemorrhage

A, B, E Complications of wound healing include any of the following: hemorrhage, hematoma, infection, dehiscence, and evisceration. Hemostasis is a normal response to healing, not a complication. Debridement is the removal of dead or infected tissue from the wound which will promote healing

Which situations demonstrate oppression in health care? (Select all that apply.) a. A pediatrician who refuses to treat children of parents in same-sex marriages b. A medical practice refuses to interpret for patients who do not speak English c. An emergency room physician refuses to administer opiates for minor pain d. A pediatrician who refuses to treat children whose parents refuse vaccinations e. A social services agency refuses to allow any Asian couples to adopt children

A, B, E Oppression in health care occurs when providers treat patients unfairly on the basis of ethnic, cultural, financial, or other characteristics. Refusing to treat children of same-sex parents, refusing to interpret for patients, and refusing Asian couples to adopt children are examples of oppression. It is appropriate for an emergency room physician to refuse to administer opiates for minor pain. Pediatricians may refuse to treat children whose parents refuse vaccinations as they can endanger the health of the other patients in the practice.

Which of the following would be included in the assessment of a patient with diabetes mellitus who is experiencing a hypoglycemic reaction? (Select all that apply.) a. Tremors b. Nervousness c. Extreme thirst d. Flushed skin e. Profuse perspiration f. Constricted pupils

A, B, E When hypoglycemia occurs, blood glucose levels fall, resulting in sympathetic nervous system responses such as tremors, nervousness, and profuse perspiration. Dilated pupils would also occur, not constricted pupils. Extreme thirst, flushed skin, and constricted pupils are consistent with hyperglycemia.

Which assessment findings put the patient at high risk for development of vision problems? (Select all that apply.) a. Takes insulin glulisine for type 1 diabetes. b. Takes metoprolol to treat hypertension. c. Takes docusate sodium for constipation. d. Takes acetaminophen for osteoarthritis pain. e. Takes prednisone for multiple sclerosis

A, B, E History of diabetes and hypertension are both significant risk factors for eye diseases such as glaucoma and retinopathy. Prednisone is associated with early development of cataracts. Constipation, osteoarthritis, acetaminophen, and docusate sodium do not put the patient at high risk for vision problems.

Which are expected physical assessments finding for a middle-aged adult? (Select all that apply.) a. Difficulty hearing female voices b. Diminished breath sounds bilaterally c. Gradual loss of senses of taste and smell d. Need for reading glasses to see small print e. Decreased ability to solve practical problems

A, B, E Middle adulthood usually refers to those years between 40 and 65. Expected physical changes include a slow, progressive decrease in skin turgor, decreased abdominal strength, decreased visual acuity, and loss of accommodation of lens to focus light on near objects. The ability to solve practical problems based on experience peaks at midlife because of the ability for integrative thinking. Cranial nerves are unaffected so senses of taste and smell remain intact. Middle-aged patients often report difficulty hearing female voices as they are higher in pitch than male voices.

Which medications can lead to development of tinnitus? (Select all that apply.) a. Furosemide b. Vancomycin c. Insulin glulisine d. Docusate sodium e. Naproxen sodium

A, B, E Ototoxic medications, such as analgesics, antibiotics (such as vancomycin and aminoglycosides), or diuretics, affect hearing acuity, balance, or both, with the most common symptom being tinnitus (ringing in the ears). Surgical anesthesia, morphine, and docusate sodium do not have the side effect of ototoxicity or tinnitus.

Which assessment findings place the patient at high risk of pulmonary embolism due to immobility? (Select all that apply.) a. The patient has had polycythemia vera for the last 5 years. b. The patient refuses daily injections of enoxaparin. c. The patient has worn compression hose since the day of surgery. d. The patient has an indwelling urinary catheter to bedside drainage. e. The patient cannot tolerate intermittent sequential compression devices.

A, B, E Polycythemia vera increases blood viscosity, increasing the risk of DVT and PE. Enoxaparin is an anticoagulant used to prevent formation of DVT. The patient's refusal to take enoxaparin increases risk of DVT and PE. Inability to tolerate SCDs also increases risk of DVT and PE as venous return is impaired by immobility. Compression (TED) hose will reduce risk of DVT and PE. Indwelling urinary catheter does not increase risk of DVT and PE.

The patient has recent bilateral, above-the-knee amputations and has developed C. difficile diarrhea. What assessments should the nurse use to detect ECV deficit in this patient? (Select all that apply.) a. Test for skin tenting. b. Measure rate and character of pulse. c. Measure postural blood pressure and heart rate. d. Check Trousseau sign. e. Observe for flatness of neck veins when upright. f. Observe for flatness of neck veins when supine.

A, B, F ECV deficit is characterized by skin tenting; rapid, thready pulse; and flat neck veins when supine, which can be reviewed in this patient. Although ECV deficit also causes postural blood pressure drop with tachycardia, this assessment is not appropriate for a patient with recent bilateral, above-the-knee amputations. Trousseau sign is a test for increased neuromuscular excitability, which is not characteristic of ECV deficit. Flat neck veins when upright is a normal finding.

The nurse is beginning an assessment of a newly admitted patient. What are some recommended comprehensive assessment approaches the nurses can use? (Select all that apply.) a.Functional Health Patterns b.Nursing Diagnosis c.Problem-Focused Approach d.Nursing Intervention Classification e.Nursing Outcome Classification

A, C

Which outcomes are appropriate for the patient with the nursing diagnosis risk for impaired skin integrity related to immobility and muscle weakness? (Select all that apply.) a. The patient's skin will remain intact without redness or ulceration. b. The nurse will assess the patient's skin daily for any sign of breakdown. c. The patient will verbalize at least two methods to prevent skin breakdown. d. The patient's wounds will be kept clean and will not develop signs of infection. e. The nurse will reposition the patient every 2 hours and pad bony prominences.

A, C Appropriate goals for risk for impaired skin integrity include maintenance of intact skin and patient verbalization of methods to prevent skin breakdown. Daily wound assessment and repositioning are interventions. The presence of wounds makes the risk for impaired skin integrity nursing diagnosis inappropriate as wounds have already developed.

The nurse is reviewing a patient's medical history. Which medication findings are likely to cause insomnia? (Select all that apply.) a. Takes a beta-adrenergic blocker b. Takes a muscle relaxant c. Has antihistamine abuse d. Has a diuretic ordered in the a.m. e. Takes a benzodiazepine

A, C Beta-adrenergic blockers and antihistamines when used in excess can cause insomnia. Muscle relaxants and benzodiazepine can cause drowsiness. Diuretics do not cause insomnia but if administered late in the day can lead to nocturia, causing nighttime awakenings

Which possible reasons does the nurse identify that contribute to the patient's black stools? a. Takes ferrous sulfate 325 mg PO BID. b. Hemorrhoids are irritated and bleeding. c. Bleeding from a perforated gastric ulcer. d. Incomplete small bowel obstruction. e. Development of a Clostridium difficile infection

A, C Iron supplements and upper gastrointestinal bleeding will cause black stools. Hemorrhoids will cause streaks of fresh red blood on the stool. Clostridium difficile infection will cause watery diarrhea. Incomplete small bowel obstruction will not cause black stools.

Which positions may be used to perform a rectal examination on the patient? (Select all that apply.) a. Sim's b. Supine c. Lithotomy d. Lateral recumbent e. Dorsal recumbent

A, C The patient may be placed in the Sim's or lithotomy position for a rectal examination. The supine and recumbent positions do not allow visualization of the rectal area.

Which assessment findings show that the patient has a history of chronic arterial insufficiency in the legs? (Select all that apply.) a. The nurse must use a Doppler to find the patient's pedal pulses. b. The patient has calf pain when the knee is flexed and foot pointed downward. c. The patient's legs are cool to the touch and show no hair growth. d. The patient has 3+ pitting pedal edema extending up to the knees. e. The patient has tortuous varicose veins from the hip to the ankle.

A, C The patient with a history of chronic lower extremity arterial insufficiency will have weak pedal pulses. The pulses may be so weak that the nurse may have to use a Doppler to find them. The legs will be cool to the touch and hairless due to the poor blood flow to the area. Calf pain when the knee is flexed and foot pointed downward is a positive Homans' sign that may indicate deep vein thrombosis. 3+ pitting pedal edema may be due to lymphatic blockage or congestive heart failure. Tortuous varicose veins indicate chronic venous insufficiency.

Which assessment findings indicate to the nurse that the patient is hypoxic? (Select all that apply.) a. Heart rate is 55 beats/minute and irregular. b. Urine output is 300 mL over the last 8 hours. c. The patient is drowsy and difficult to arouse. d. Hands and feet are pale and cool to the touch. e. Abdomen is soft with bowel sounds 4 quadrants.

A, C, D Bradycardia, impaired mental status, and poor circulation all indicate that the patient is hypoxic. Soft abdomen with bowel sounds 4 quadrants and urine output at least 30 mL/hour are normal findings.

A nurse wants to follow the American Nurses Associations Social Media Policy (2011). Which actions should the nurse take? (Select all that apply.) a.Never name or describe a patient. b.Never have a blog. c.Never post an image of the patient. d.Never disparage a fellow employee. e.Never report breaches of privacy

A, C, D The American Nurses Association has developed a Social Media Policy (2011), which recommends that when using social media sites, a nurse should never name or describe a patient, never post an image of a patient, and never disparage a fellow employee or employer. In addition, the professional nurse has an obligation to report breaches of privacy and confidentiality. Never having a blog is not a recommendation.

Which assessment findings lead the nurse to conclude that the patient has had chronic obstructive pulmonary disease (COPD) for many years? (Select all that apply.) a. The patient's nails have a clubbed shape. b. The patient has splinter hemorrhages under the nails. c. The patient's chest appears rounded and bulging. d. The patient is short of breath with minimal activity. e. The patient has soft, spongy gums that bleed easily.

A, C, D Clubbed nails are an indicator of chronic lack of oxygen and COPD. The patient with COPD has difficulty exhaling air so the chest takes on a rounded, barrel shape over time. As the airways and alveoli become damaged, the patient becomes short of breath with minimal activity. Soft, spongy gums that bleed easily indicate lack of vitamin C. Splinter hemorrhages under the nails are caused by minor trauma or endocarditis.

A nurse has been temporarily assigned to the night shift. A change in this circadian rhythm may cause which of the following? (Select all that apply.) a. Anxiety b. Weight gain c. Decreased appetite d. Impaired judgment e. Increased periods of sleep

A, C, D When the sleep-wake cycle becomes disrupted (e.g., by working rotating shifts), other physiological functions change as well. For example, a new nurse who starts working the night shift experiences a decreased appetite and loses weight, not weight gain. Anxiety, restlessness, irritability, and impaired judgment are other common symptoms of sleep cycle disturbances. Decreased, not increased, periods of sleep can occur. Failure to maintain an individual's usual sleep-wake cycle negatively influences the person's overall health.

A patient has just undergone an abdominal aortic aneurysm repair. The patient is pulling at the Foley catheter, nasogastric tube, central line, and abdominal dressing, and a wrist restraint is applied after an order is received. Later, the patient reports tingling and numbness in the fingers and hand. Which actions should the nurse take? (Select all that apply.) a. Stay with the patient. b. Medicate the patient for pain. c. Notify the health care provider. d. Remove the restraint immediately. e. Tell the patient to relax and it will feel better shortly.

A, C, D If a patient has altered neurovascular status (tingling and numbness) remove the restraint immediately, stay with the patient, and notify the health care provider. Tingling and numbness will not decrease with time; it will continue to cause damage. The patient does not need pain medication; the restraint is too tight and needs to be removed.

Which abilities are required for moral development of the child? (Select all that apply.) a. Accept social responsibility. b. Perform repetitive motion responses. c. Respect the integrity and rights of others. d. Integrate principles of justice and fairness. e. Use symbols and objects for abstract thinking

A, C, D Moral development depends on the child's ability to accept social responsibility and integrate personal principles of justice and fairness. In addition, the child's knowledge of right and wrong and behavioral expression of this knowledge must be founded on respect and regard for the integrity and rights of others. Piaget's theory, not Kohlberg's, states that as the child grows from infancy into adolescence, the intellectual development progresses, starting with reflex and repetitive motion responses, to the use of symbols and objects from the child's point of view, to logical thinking, and finally to abstract thinking.

A registered nurse is caring for a patient who was admitted to the hospital after being involved in a motor vehicle accident. The patient has undergone two surgeries and now has a health careacquired infection. Multiple medications were ordered. Which of the following would be appropriate for the nurse to administer to reduce the fever without masking signs of infection? (Select all that apply.) a.Acetaminophen b.Corticosteroid c.Ibuprofen d.Indomethacin e.Salicylates

A, C, D, E

Which assessment findings are social determinants of health? (Select all that apply.) a. Living in a house containing lead paint b. Genetic predisposition to spinal osteoarthritis c. Growing up in a neighborhood without grocery stores d. Annual income 125% above the federal poverty level e. Completed high school and 1 year of community college

A, C, D, E Genetic makeup is not a social determinant of health. Living conditions, neighborhood, annual income, and educational level are all social determinants of health.

The nurse is caring for a patient with diarrhea caused by Clostridium difficile infection. Which are the priority interventions of the nurse? (Select all that apply.) a. Perform hand hygiene with soap and water. b. Increase the patient's dietary intake of fiber. c. Maintain strict contact isolation precautions. d. Accurate calculation of patient's intake and output. e. Liberally apply skin barrier cream to the perineal area. f. Give loperamide 4 mg after each loose stool.

A, C, D, E Hand hygiene with soap and water and maintenance of contact isolation precautions are essential to prevent spread of Clostridium difficile infection. Accurate calculation of intake and output is needed to assess for fluid volume deficit from diarrhea. Skin barrier cream to the perineal area will help prevent skin breakdown. Increasing dietary fiber is not recommended as it may further irritate the colon. Antidiarrheal medication such as loperamide is not administered as it will prevent removal of the Clostridium difficile infection from the colon through the diarrhea

Which medications are appropriate for a patient with chronic pain and cannot swallow pills? (Select all that apply.) a. Morphine sulfate liquid b. Crushed extended-release morphine sulfate c. Fentanyl nasal spray d. Acetaminophen suppository e. Fentanyl transdermal patch

A, C, D, E Morphine sulfate liquid, fentanyl nasal spray, acetaminophen suppository, and fentanyl transdermal patch are all options for patients who cannot swallow pills. Crushed morphine sulfate is an extended-release drug that should never be crushed.

The nurse has assessed the need to check the patient for orthostatic hypotension. Conditions that would cause the nurse to be concerned about this would be which of the following? (Select all that apply.) a.Dehydration b.Obesity c.Recent blood loss d.Cigarette smoking e.Prolonged bed rest

A, C, E

A patient has just undergone an abdominal aortic aneurysm repair. The patient is pulling at the Foley catheter, nasogastric tube, central line, and abdominal dressing and a wrist restraint is applied after an order is received. Later, the patient reports tingling and numbness in the fingers and hand. Which actions should the nurse take? (Select all that apply.) a.Remove the restraint immediately. b.Remind the patient this will decrease with time. c.Notify the health care provider. d.Medicate the patient for pain. e.Stay with the patient.

A, C, E If a patient has altered neurovascular status (tingling and numbness) remove the restraint immediately, stay with the patient, and notify the health care provider. Tingling and numbness will not decrease with time; it will continue to cause damage. The patient does not need pain medication; the restraint is too tight and needs to be removed.

A group of teenagers are attending a preparation class for babysitters. Which statements by the teenagers indicate a correct understanding of the teaching about safety issues? (Select all that apply.) a.Home fires are a major cause of death and injury. b.Bacterial food contamination cannot be controlled. c.There should be working batteries in the smoke detector. d.Temperature changes do not affect the childs safety. e.Toddlers are very curious and like to put objects in their mouths.

A, C, E The best intervention is to prevent fires. Home fires are a major cause of death and injury. Another problem related to fatal fires is a failure to keep fresh batteries in home smoke detectors. The improper use of cooking equipment and appliances, particularly stoves, is another source for in-home fires. Smoke detectors and carbon monoxide detectors need to be placed strategically throughout a home. Multipurpose fire extinguishers need to be near the kitchen and any workshop areas. Children at these early stages are curious; they explore their environment, and because of an increase in oral activity, put objects in their mouths. If food is prepared and stored properly, food poisoning risk can be decreased. Temperature changes can lead to hypothermia and/or heatstroke or heat exhaustion.

The nurse would identify which body systems as directly involved in the process of normal gas exchange? (Select all that apply.) A Neurologic system B Endocrine system C Pulmonary system D Immune system E Cardiovascular system F Hepatic system

A, C, E The neurologic system controls respiratory drive; the respiratory system controls delivery of oxygen to the lung capillaries; and the cardiac system is responsible for the perfusion of vital organs. These systems are primarily responsible for the adequacy of gas exchange in the body. The endocrine and hepatic systems are not directly involved with gas exchange. The immune system primarily protects the body against infection.

The nurse would identify which body systems as directly involved in the process of normal gas exchange? (Select all that apply.) a. Neurologic system b. Endocrine system c. Pulmonary system d. Immune system e. Cardiovascular system f. Hepatic system

A, C, E The neurologic system controls respiratory drive; the respiratory system controls delivery of oxygen to the lung capillaries; and the cardiac system is responsible for the perfusion of vital organs. These systems are primarily responsible for the adequacy of gas exchange in the body. The endocrine and hepatic systems are not directly involved with gas exchange. The immune system primarily protects the body against infection.

The patient's electricity has been shut off after failure to pay the utility bills. Which actions by the patient pose health hazards? (Select all that apply.) a. The gas oven is used to warm the kitchen. b. Extra layers of warm clothing are worn. c. Food is smelled to determine if it is edible. d. Dry ice is used to keep milk and eggs cold. e. Lit taper candles are placed around the house

A, C, E The oven should never be used to warm the house, especially since a gas oven is the only option without electricity. Food should be thrown out if there is any doubt about its safety, rather than relying on its smell. Flashlights should be used for light as candles can cause fires. Dry ice is an option for keeping milk and eggs cold when there is no electricity. Extra layers of warm clothing should definitely be worn to prevent hypothermia.

Which interventions are appropriate for the patient at risk for peripheral neurovascular dysfunction related to casted right ankle? (Select all that apply.) a. Regularly assess the patient's toes for warmth and capillary refill. b. Apply graduated compression stockings to both legs to prevent DVT. c. Encourage the patient to frequently wiggle the toes to increase circulation. d. Educate the patient that numbness and tingling of the area is to be expected. e. Elevate the ankle above the level of the heart and apply ice to the ankle area.

A, C, E The toes should be regularly assessed for warmth and capillary refill. The patient should be encouraged to frequently wiggle the toes to increase circulation. The ankle should be elevated and iced to reduce swelling that can impede circulation. Numbness and tingling is not to be expected and should be reported to the physician immediately. TED hose cannot be applied to the casted leg.

Which assessment findings lead the nurse to clarify the order for ibuprofen 600 mg PO every 8 hours? (Select all that apply.) a. The patient has a gastrointestinal bleed. b. The patient has allergies to shellfish, strawberries, and iodine. c. The patient takes 30 mg morphine sulfate daily. d. The patient has a history of diabetes and early renal failure. e. The patient has severe joint pain due to aggressive arthritis.

A, D Ibuprofen and other NSAID medications should not be taken by patients who have bleeding tendencies including anticoagulation with warfarin. Ibuprofen can be toxic to the kidneys and should be avoided by patients with renal failure. Taking ibuprofen with morphine increases the pain-relieving effects of the morphine. Ibuprofen and other NSAID medications are very effective for arthritic joint pain. Allergies to shellfish, strawberries, and iodine do not contraindicate administration of ibuprofen.

Which assessment findings indicate to the nurse that the older adult patient has a urinary tract infection? (Select all that apply.) a. Confusion and irritability b. Urinalysis is positive for hyaline casts and ketones. c. Urinalysis is negative for nitrites and white blood cells. d. Reports frequency and burning with urination e. Has had two uncharacteristic episodes of incontinence

A, D, E Mental status changes, incontinence, and urinary frequency are all signs of urinary tract infection in the elderly. In the presence of a urinary tract infection, a urinalysis will be positive for nitrites and white blood cells. Hyaline casts are found when protein is lost through the urine due to renal disease. Ketones are present in the urine when the blood sugar is elevated.

The nurse is caring for a patient whose pituitary secretes excessive amounts of antidiuretic hormone (ADH). Which findings will the nurse expect to see during assessment of the patient? (Select all that apply.) a. The patient's serum sodium level is 122 mEq/L. b. The patient's serum chloride level is 116 mEq/L. c. The patient is voiding large amounts of watery urine. d. The patient's mucus membranes are dry and cracked. e. The patient has gained 5 pounds and has swollen feet.

A, E Excessive amounts of ADH (SIADH) will lead to hyponatremia with a serum sodium level of 122 mEq/L. Fluid retention from excess ADH will cause weight gain and swollen feet. Diabetes insipidus (DI) causes hyperchloremia, dry mucus membranes, and excessive urination due to a shortage of ADH

Which patients can receive a unit of A+ blood? (Select all that apply.) a. Type A+ b. Type A c. Type B+ d. Type B e. Type AB+ f. Type AB

A, E Patients with type A+ and AB+ blood may receive a unit of A+ blood. Type A-, B+, B-, ABpatients would not be compatible with A+ blood.

1.A nurse is presenting at an interdisciplinary meeting about the multiple external forces that are influencing nursing today. Which examples should the nurse include? (Select all that apply.) a.Health care reform b.Threat of bioterrorism c.Population demographics d.Role of nurse manager e.Nursing shortage

A,B,C,E Multiple external forces affect nursing today, including health care reform, demographic changes of the population, increasing numbers of medically underserved, need for emergency preparedness, threat of bioterrorism, workplace issues, and the nursing shortage. Role of nurse manager is not an external force affecting nursing, but is one role of the registered nurse.

Which assessment findings indicate that the patient is at risk for developing ventricular fibrillation? (Select all that apply.) a. Serum potassium level 7.6 mEq/L b. Long history of coronary artery disease c. Impaired conduction through the SA node d. Recent incidents of ventricular tachycardia e. Vagal stimulation from removal of fecal impaction

A,B,D Hyperkalemia, coronary artery disease, and recent incidents of ventricular tachycardia are all risk factors for development of ventricular fibrillation. Impaired SA node conduction and vagal stimulation would lead to bradycardia rather than fibrillation.

Risk factors for breast cancer in women include which of the following? (Select all that apply.) a.Age greater than 40 b.BRCA 1 and 2 gene mutations c.Late onset menarche d.Early menopause e.Use of contraceptives

A,B,E

A nurse wants to become an advanced practice registered nurse (APRN) and have a higher degree of independence. Which advanced roles could the nurse pursue? (Select all that apply.) a.Clinical nurse specialist b.Nurse manager c.Nurse practitioner d.Nurse midwife e.Nurse anesthetist

A,C,D,E The advanced practice registered nurse (APRN) is the most independently functioning nurse. An APRN has a masters degree in nursing; advanced education in pathophysiology, pharmacology, and physical assessment; and certification and expertise in a specialized area of practice. There are four core roles for the APRN: clinical nurse specialist (CNS), nurse practitioner (NP), certified nurse midwife (CNM), and certified RN anesthetist (CRNA). Nurse managers do not require an advanced degree.

A 45-year-old mother of three children is at the doctors office and her blood pressure 152/92. This is the first time that she has ever shown an elevated reading. She is concerned that she has hypertension. The nurses best response would be: a.A single reading may not mean anything. We will take it again at your next visit. b.It looks like you have high blood pressure now. Well check it again in 3 months. c.Fortunately, hypertension isnt related to other diseases and is easily treated. d.You may have hypertension, but there is little else that can be done except medicines.

A- A single reading may not mean anything. We will take it again at your next visit. The diagnosis of hypertension in adults is made on the average of two or more readings taken at each of two or more visits after an initial screening. One blood pressure recording revealing a high SBP or DBP does not qualify as a diagnosis of hypertension

The infection control nurse is presenting an in-service presentation on infection prevention and control. A participating nurse identifies what patient as most susceptible to acquiring an infection? a.An 81-year-old patient with a fractured hip b.A 10-month-old patient with a first-degree burned hand c.A 40-year-old patient with a recent uncomplicated laparoscopic cholecystectomy d.A 16-year-old athlete with a repair of the medial collateral ligament

A- An 81-year-old patient with a fractured hip When a person ages, there are normal physiological changes that influence susceptibility to infection. First-degree burns leave the skin intact and do not breach the barrier to infection. Any surgical procedure involves a break in the bodys defenses against infection, and procedures such as cholecystectomy and ligament repair pose a risk for infection; however, the age of the patients place them at a lower level of risk.

A man has been admitted to the hospital with lethargy. He was placed on the telemetry unit and is being continuously monitored. He is due to receive his dose of digoxin. The nurse knows that the medication is to be held if the pulse rate is less than 60 beats per minute. The nurse will use which site to assess the patients pulse? a.Apical b.Radial c.Brachial d.Carotid

A- Apical When a patient takes a medication that affects the heart rate, the apical pulse provides a more accurate assessment of heart rate

A nurse is admitting a 79-year-old woman with a fractured hip to the orthopedic unit. Her husband states that she broke her hip when she tripped in her garden. Upon examination, the nurse notes purple, green, and yellow bruises on the back and arms. The patient states that those were received when she fell. The nurse should do which of the following? a.Ask the husband to wait in the waiting room. b.Ignore the bruises because the patient has provided an explanation. c.Realize that the patient may be abused, but that is a family issue. d.Prepare to discharge the patient home once treatment is complete.

A- Ask the husband to wait in the waiting room Patients are more likely to reveal problems when the suspected abuser is not present in the room. Psychological abuse as well as obvious physical injury or neglect (e.g., evidence of malnutrition or presence of bruising on the extremities or trunk) should be assessed. If you have suspicion of abuse, find a way to interview the patient privately

.A nurse is collecting data on a patient who is being admitted into hospice care. The nurse collects data from both the patient and the family so that a clear picture of the patient status is obtained. The nurse is currently involved in which step of the nursing process? a.Assessment b.Implementation c.Evaluation d.Diagnosing

A- Assessment Assessment is the deliberate and systematic collection of data about a patient. The data will reveal a patients current and past health status, functional status, and present and past coping patterns. A nursing diagnosis is a clinical judgment about individual, family, or community responses to actual and potential health problems or life processes that the nurse is licensed and competent to treat. Implementation is the performance of nursing interventions necessary for achieving the goals and expected outcomes of nursing care. Evaluation is crucial to deciding whether, after interventions have been delivered, a patients condition or well-being improves.

A 73-year-old patient with hypertension is awaiting a triple cardiac bypass surgery. The patient is hard of hearing and did not understand what the surgeon said regarding the surgery. The daughter is concerned that the patient does not understand the risks of the surgery. If not clarified, this would be a violation of what principle? a.Autonomy b.Justice c.Fidelity d.Nonmaleficence

A- Autonomy Autonomy refers to a persons independence. As a principle in bioethics, autonomy represents an agreement to respect a patients right to determine a course of action. Justice refers to the principle of fairness: fair treatment and fair distribution of health care resources. Fidelity refers to the agreement to keep promises and is based on the virtue of caring. Nonmaleficence is actively seeking to do no harm.

A nurse is evaluating care for a patient. Which action should the nurse take? a.Compares patient findings with the goals and outcomes b.Determines if interventions were completed c.Develops a nursing diagnosis d.Writes a care plan

A- Compares patient findings with the goals and outcomes During evaluation you compare your findings with the goals and expected outcomes set for your patient. You conduct an evaluation to determine if expected outcomes are met, not if nursing interventions were completed. Develops a nursing diagnosis is the second step of the nursing process (diagnosis), not the last (evaluation). Writes a care plan occurs in the planning phase.

Although a registered nurse has been working for several years as a staff nurse on an adult oncology unit, the nurse recently transferred to a pediatric unit in the hospital. The nurse will be in orientation for several days to learn about the different systems and will need to demonstrate proficiency in various pediatric areas such as medication administration. Which behavior is the nurse demonstrating? a.Competency b.Judgment c.Advocacy d.Utilitarianism

A- Competency In the practice of nursing, competence ensures the provision of safe nursing care (proficiency in pediatric medication administration). The agreement to practice with competence is a common denominator for all state regulations and is in the nursing code of ethics. Judgment refers to the ability to form an opinion or draw sound conclusions. Advocacy involves speaking up for patient care issues from your unique perspective and advocating for humane and dignified care. You use a utilitarian ethic when determining the value of something based primarily on its usefulness and effects or consequences. The greatest good for the greatest number of people is the guiding principle for action in this system (utilitarian).

Only one nurse was scheduled to care for 12 postsurgical patients with a nursing assistant. The nurse is concerned for the safety of the patients and the nursing license. What is the most appropriate first step in this situation? a.Contacting the nursing supervisor and documenting the action b.Refusing to care for the patients without appropriate help and leaving c.Contacting the State Board of Nursing and documenting the action d.Contacting the hospital administrator on call to complain and documenting the action

A- Contacting the nursing supervisor and documenting the action If a nurse is assigned to care for more patients than is reasonable for safe care, he or she should notify the nursing supervisor. If the nurse is required to accept the assignment, he or she must document this information in writing and provide the document to nursing administrators. Although documentation does not relieve a nurse of responsibility if patients suffer harm because of inattention, it shows that the nurse attempted to act appropriately

A postoperative patient is continuing to have incisional pain. As part of the nurses assessment, the nurse notes that the patient is grimacing when he or she changes position. The patients grimace can be useful in the assessment and can be described as which of the following? a.Cue b.Inference c.Diagnosis d.Health pattern

A- Cue Grimacing is a cue. A cue is information that a nurse obtains through use of the senses. An inference is your judgment or interpretation of these cues. Gordons functional health patterns are a type of database format to obtain a comprehensive assessment. A nursing diagnosis is a clinical judgment about individual, family, or community responses to actual and potential health problems or life processes that the nurse is licensed and competent to treat

Which behavior best indicates that the nurse is fulfilling ethical responsibilities? a.Delivers competent care b.Applies the scientific process c.Forms interpersonal relationships d.Evaluates new computerized technologies

A- Delivers competent care Delivers competent care is the best example because the American Nurses Association (ANA) and the International Council of Nurses (ICN) publish codes of ethics for nurses that set principles of behavior for nurses to embrace. They reflect common underlying principles that shape professional nursing practice, including responsibility, accountability, respect for confidentiality, competency, judgment, and advocacy. The scientific process, interpersonal relationships, and new technologies do not indicate ethical behavior like competent care.

A registered nurse is preparing to perform a physical examination on a 5-year-old child. To make the child feel safer during the examination the nurse should do which of the following? a.Examine the childs fingernails before listening to his breath sounds. b.Question only the child so as to avoid unwanted parental influence. c.Perform palpation before visual inspection. d.Calls the parents by their first names to establish a more trusting bond

A- Examine the childs fingernails before listening to his breath sounds. Children will feel safer during an examination if it is initiated from the periphery and then moves to the central. For example, examine the extremities before moving to the chest. It also helps to perform parts of the examination that you can do visually before actually touching the child

There was an outbreak of Salmonella poisoning at a nursing home. Several residents were hospitalized as a result of their infections. What is the best term to describe this infection? a.Exogenous infection b.Endogenous infection c.Community-acquired infection d.Asepsis

A- Exogenous infection An exogenous infection comes from microorganisms found outside the individual, such as Salmonella , Clostridium tetani , and Aspergillus . They do not exist as normal flora

A 47-year-old patient is being treated for hyperthyroidism. When she goes to the health care provider for a check-up, the nurse in the clinic expects to see which of the following physical symptoms? a.Exophthalmos b.Strabismus c.Photophobia d.Diplopia

A- Exophthalmos Bulging eyes (exophthalmos) usually indicate hyperthyroidism. The crossing of eyes (strabismus) results from neuromuscular injury or inherited abnormalities

A nurse is employed by a health care agency that provides an informal training session on how to properly use a new vital sign monitor. Which type of education did the nurse receive? a.In-service education b.Advanced education c.Continuing education d.Registered nurse education

A- In-service education In-service education programs are instruction or training provided by a health care agency or institution designed to increase the knowledge, skills, and competencies of nurses and other health care professionals employed by the institution. Some roles for RNs in nursing require advanced graduate degrees, such as a clinical nurse specialist or nurse practitioner. There are various educational routes for becoming a registered nurse (RN), such as associate, diploma, and baccalaureate. Continuing education involves formal, organized educational programs offered by universities, hospitals, state nurses associations, professional nursing organizations, and educational and health care institutions.

The registered nurse is precepting a first-year nursing student. She is demonstrating how to appropriately auscultate. Auscultation is defined as which of the following? a.Listening with a stethoscope to sounds produced by the body b.Tapping the body with the fingertips to produce a vibration c.Becoming familiar with the nature and source of body odors d.Using the hands to touch body parts to make a sensitive assessment

A- Listening with a stethoscope to sounds produced by the body Auscultation is listening for sounds produced by the body. Percussion involves tapping the body with the fingertips to produce a vibration that travels through body tissues

The nurse is caring for a 22-year-old woman who has come in for a routine examination. The patient asks about how often she should perform breast self-examinations and get a mammogram. The nurse should inform the patient of which of the following? a.Monthly breast self-examinations are optional for women in their twenties and thirties. b.Women less than 40 need a clinical breast exam by a health care provider yearly. c.Asymptomatic women need mammograms yearly until age 40. d.Women greater than age 40 need mammograms every 6 months

A- Monthly breast self-examinations are optional for women in their twenties and thirties. Monthly BSE is an option for women in their twenties and thirties. Women 20 years of age and older need to report any breast changes to a health care provider immediately. Women need a clinical breast examination by a health care provider every 3 years from age 20 to 40, and yearly for women greater than age 40

A patient died from suspicious circumstances. What should the nurse do next? a.Notify the coroner. b.Notify the newspaper. c.Chart what the nurse thinks happened. d.Chart opinions from the health care staff.

A- Notify the coroner. State statutes specify that, when there are reasonable grounds to believe that a patient died as a result of violence, homicide, suicide, accident, or death occurring in any unusual or suspicious manner, you need to notify the coroner. Notifying the newspaper would break confidentiality. Charting must be objective and factual, not what the nurse thinks happened or opinions

A 17 year old was taken to the emergency department when his mother found him unresponsive. Upon examination, his pupils were found to be pinpoints. This is a common sign of which of the following? a.Opioid intoxication b.Arcus senilis c.Cataracts d.Opioid withdrawal

A- Opioid intoxication Pinpoint pupils are a common sign of opioid intoxication. A thin white ring along the margin of the iris, called an arcus senilis, is common with aging but is abnormal in anyone less than age 40

A nurse is interviewing a patient being admitted to the hospital for surgery. During the interview, the nurse introduces self and explains that will be gathering some information. The nurse is in which phase of the interview? a.Orientation b.Working c.Assessment d.Termination

A- Orientation The orientation phase begins with introducing oneself and ones position and explaining the purpose of the interview. The nurse explains to patients why the data are being collected and assures the patient that the information will remain confidential and will be used only by health care professionals who provide his or her care. During the working phase you gather information about a patients health status. When the interview comes to an end, this is called termination. Assessment is the first step in the nursing process, not the first step in an interview.

A nurse is teaching the staff about the phases of the interview process. Which information should the nurse include in the teaching session? a.Orientation, working, termination b.Orientation, assessment, evaluation c.Planning, assessment, termination d.Planning, assessment, evaluation

A- Orientation, working, termination The three phases of the interview process are orientation, working, and termination. Assessment, evaluation, and planning are phases in the nursing process.

The nursing student is performing a physical examination on a 6-year-old patient who is being admitted to the pediatric unit with abdominal pain. When would be the most appropriate time in the examination to palpate the patients abdomen? a.Palpate tender areas last. b.Palpate tender areas first to get it over. c.Palpate tender areas before inspection. d.Palpate before auscultation

A- Palpate tender areas last Because palpation involves the use of the hands to touch body parts and make sensitive assessments, palpate tender areas last . Palpation typically occurs right after inspection. When examining the abdomen, however, palpation occurs after auscultation. Palpate the abdomen for tenderness, distention, or masses

A patient is complaining of nonspecific abdominal pain. What is the technique the nurse uses to assess tenderness? a.Palpation b.Percussion c.Auscultation d.Olfaction

A- Palpation Palpation primarily detects areas of abdominal tenderness, distention, or masses. Percussion involves tapping the body with the fingertips to produce a vibration that travels through body tissues

A nurse completes an occurrence report. Which is the best way for the nurse to document this occurrence? a.Patient found lying on right side on floor. No noted injuries, patient stated, I slipped on a wet spot on the floor. I dont think I am injured. b.Patient slipped on a wet spot on the floor. No noted injuries, physician notified. c.Patient in too much of a hurry and was walking too fast and fell. Was not injured. Patient instructed to slow down and not be in such a hurry. Health care provider notified. d.Patient fell while going outside to smoke. Patient denied any injuries. Health care provider notified. Patient counseled

A- Patient found lying on right side on floor. No noted injuries, patient stated, I slipped on a wet spot on the floor. I dont think I am injured. Objectively record the details of the event and any statements the patient makes. An example is as follows: Patient found lying on floor on right side. Abrasion on right forehead. Patient stated, I fell and hit my head. Patient slipped on wet spot and patient fell while going outside should not be charted unless the nurse actually observed the event; otherwise, chart what found: Patient lying on floor. Patient in too much of a hurry includes subjective assumptions and statements; assigning blame or fault is inappropriate when completing the report.

A nurse is teaching the staff about the International Council of Nurses Code of Ethics. Which major element of the Code should the nurse include in the teaching session? a.People b.Pride c.Power d.Problems

A- People The major elements of the Code include: Nurses and People; Nurses and Practice; Nurses and the Profession; and Nurses and Co-workers. It does not include pride, power, and problems.

When a nurse suspects child abuse or neglect, which action must the nurse take? a.Report it to the proper legal authority. b.Inform the parents that their actions are illegal. c.Call the security department to handle the problem. d.Prevent the parents from seeing the child during hospitalization

A- Report it to the proper legal authority. Health care providers are required to report incidents such as child, spousal, or elder abuse; rape; gunshot wounds; attempted suicide; and certain communicable diseases. Health care providers are provided legal immunity if the report is made in good faith. Not reporting suspected child abuse or neglect can cause a nurse to be liable in civil or criminal legal actions. It is not the nurses responsibility to inform the parents of illegal activity or to prevent the parents from seeing the child. The nurse is responsible for reporting the suspected abuse, not call security to handle the problem

The patient has just returned to the postsurgical unit after undergoing surgery to remove a lung tumor. During one of the postoperative vital sign checks, the nurse notes that the patients systolic blood pressure had dropped by 30 points. In addition to the drop in systolic blood pressure, the patients skin is pale and clammy. The nurse should do which of the following? a.Report the findings to the health care provider immediately. b.Understand that the patients arteries are constricting, causing pallor. c.Wait to see if the blood pressure increases in 30 minutes. d.Nothing; this is a normal occurrence following a thoracic surgery.

A- Report the findings to the health care provider immediately Signs and symptoms associated with hypotension include pallor, skin mottling, clamminess, confusion, dizziness, chest pain, increased heart rate, and decreased urine output.

.A nurse decides to withhold a medication because it will further lower a patients respiratory rate. In this case, the nurse is practicing what principle? a.Responsibility b.Privacy c.Ethics d.Moral behavior

A- Responsibility Responsibility refers to the execution of duties associated with a nurses role. For example, when administering a medication, you are responsible for assessing the patients need for the medication, giving it safely and correctly, and evaluating the patients response to it. Moral behavior refers to judgment about right and wrong behavior. Ethics refers to the consideration of standards of conduct, particularly the study of right and wrong behavior. A fundamental right of patients is the right to privacy. Privacy becomes a focus of increasing interest as health care becomes digitized, but it is not a focus of this scenario.

The patients temperature has reached 103.4 F. The nurse prepares to draw a blood culture before giving the patient an antipyretic medication. What is the best reason to draw a blood culture before giving an antipyretic medication? a.The causative organism is most prevalent during a spike in temperature. b.Elevated temperatures slow metabolic rate and improve blood oxygenation. c.Increased blood flow leads to moist mucous membranes making blood draws easier. d.Venous distention is greater because of fluid retention secondary to hyperthermia.

A- The causative organism is most prevalent during a spike in temperature. Obtain blood specimens at the same time as a temperature spike, when the causative organism is most prevalent. Satisfy requirements for increased metabolic rate

A 5-year-old child was admitted for a diagnosis of meningitis with a fever of 104.5 F and nuchal rigidity. She responded to antipyretics that were ordered. In addition, the patients mother was asked to help reduce the fever by limiting the number of blankets covering the patient. After interventions, the childs temperature is 100.5 F. The nurse recognized that the mother has an understanding of the patients condition when she states which of the following? a.The high temperature is useful in fighting bacteria and viruses as long as its not too high. b.You need to get her temperature down quickly. Shes so uncomfortable. c.Her fever is dropping because she is shivering. She must be cold. d.She probably picked up a bacteria. Thats what kids do. Thats why they get infected.

A- The high temperature is useful in fighting bacteria and viruses as long as its not too high. A fever is usually not harmful if it stays below 39 C (102.2 F) in adults or 40 C (104 F) in children. Increased temperature reduces the concentration of iron in the blood plasma, causing bacterial growth to slow.

To establish the elements of malpractice against a nurse, which must be proved by the patient? a.The patient must have been harmed as a result of the injury. b.The patient must have paid for the health care services. c.The patient must show evidence of malicious intent. d.The patient must demonstrate personal accountability

A- The patient must have been harmed as a result of the injury. To establish the elements of malpractice, the patient or plaintiff must prove the following: (1) the nurse defendant owed a duty to the patient, (2) the nurse breached that duty, (3) the patient was injured because of the nurses breach of duty, and (4) the patient has accrued damages as a result of the injury. The patient paying, showing evidence of malicious intent, and demonstrating personal accountability are not elements of malpractice.

The nurse has just completed an assessment on a patient with a fractured right femur. Which data will the nurse categorize as objective? a.The patients toes of right foot are warm and pink. b.The patient reports a dull ache in the right hip. c.The patient says feels tired all the time. d.The patient is concerned about insurance coverage.

A- The patients toes of right foot are warm and pink Toes pink and warm are objective data. Objective data are observations or measurements of a patients health status. Subjective data are patients verbal descriptions of their health problems. Only patients provide subjective data.

A nurse is caring for a patient and performs several interventions. Which action by the nurse is an independent nursing intervention? a.Turning every 2 hours b.Administering a medication c.Inserting an indwelling catheter d.Starting an intravenous (IV) for intravenous fluids

A- Turning every 2 hours According to state Nurse Practice Acts, independent nursing interventions pertain to ADLs (turning), health education and promotion, and counseling. Nurse-initiated interventions are the independent nursing interventions or actions that nurses initiate. Physician-initiated interventions are dependent nursing interventions or actions that require an order from a physician or another health care professional. Administering a medication, implementing an invasive procedure (catheter and intravenous fluids), and preparing a patient for diagnostic tests are examples of such interventions.

A nurse wants to provide patient-centered care to a patient of another culture. Which question is the most culturally sensitive when talking about a patients illness? a.What do you call your problem? b.How long has your child had the runs? c.When did you last void today? d.Has anyone else in your family had diarrhea?

A- What do you call your problem? To start an assessment, Seidel and others (2011) offer useful questions to begin to explore a patients illness or health care problem in context of the patients culture: What do you call your problem? A different culture may not know what the runs means. Most people do not know what void (urinate) means. Has anyone else in your family had diarrhea is not as culturally sensitive as finding out what the problem is according to the patients culture.

The nurse is admitting a patient with a methicillin-resistant Staphylococcus aureus (MRSA) infection isolated in his stage III pressure ulcer. The nurse places the patient on: a.contact precautions. b.airborne precautions. c.droplet precautions. d.protective environment

A- contact precautions.

When the ventricles relax, the blood remaining in the arteries exerts a minimum or: a.diastolic pressure. b.pulse pressure. c.hypertension. d.systolic pressure.

A- diastolic pressure. When the ventricles relax, the blood remaining in the arteries exerts a minimum or diastolic pressure. Under high pressure, the left ventricle ejects blood into the aorta; the peak pressure is known as systolic pressure. The difference between systolic and diastolic pressure is the pulse pressure

The movement of oxygen and carbon dioxide between the alveoli and the red blood cells is known as: a.diffusion. b.perfusion. c.respiration. d.eupnea.

A- diffusion Respiration is the mechanism the body uses to exchange gases between the atmosphere, blood, and cells. Respiration involves three processes: ventilation (the mechanical movement of gases into and out of the lungs), diffusion (the movement of oxygen [O 2 ] and carbon dioxide [CO 2 ] between the alveoli and the red blood cells), and perfusion (the distribution of red blood cells to and from the pulmonary capillaries). The normal rate and depth of ventilation is known as eupnea.

A patient is admitted for treatment of a home-acquired pressure ulcer. The patient is incontinent of urine and has Alzheimer disease. A Foley catheter is inserted. The nurse recognizes that the best way to break the infection chain is to: a.discontinue the Foley as soon as possible. b.wear a mask when working with the patient if she or he has a cold. c.wear sterile gloves if there is a chance of contact with blood. d.use surgical asepsis when handling body fluids.

A- discontinue the Foley as soon as possible. To control or eliminate infection in reservoir sites, eliminate sources of body fluids, drainage, or solutions that possibly harbor microorganisms such as a Foley collection bag

A regular interval interrupted by an early beat, late beat, or missed beat indicates an abnormal rhythm or: a.dysrhythmia. b.tachycardia. c.bradycardia. d.a pulse deficit

A- dysrhythmia A regular interval interrupted by an early beat, late beat, or a missed beat indicates an abnormal rhythm (not rate) or dysrhythmia. Pulse rate assessment often reveals variations in heart rate

A nurse can best auscultate the point of maximum impulse (PMI) in a teenager at the intercostal space, midclavicular line. a.fifth; left b.fourth; left c.fifth; right d.fourth; right

A- fifth; left The point of maximal impulse (PMI) is palpable at the fifth intercostal space at the left midclavicular line in adults and children older than 7 years of age

An elevated body temperature related to the bodys inability to promote heat loss or reduce heat production is known as: a.hyperthermia. b.heat stroke. c.hypothermia. d.fever of unknown origin.

A- hyperthermia. An elevated body temperature related to the bodys inability to promote heat loss or reduce heat production is hyperthermia (hyper- means excessive ; therm means heat ). Prolonged exposure to the sun or high environmental temperatures overwhelms the bodys heat loss mechanisms

A nursing student is assisting with ambulation of a blind patient. The patient has hemiplegia of the right side. The best position for the student nurse to assume when ambulating is by standing on the patients side and walking a half step the patient. a.left; ahead b.right; ahead c.left; behind d.right; behind

A- left; ahead You will need to assist patients with acute visual impairments with walking. Stand on the patients dominant, stronger, or uninjured side. The patient grasps your elbow or upper arm. You then walk one half step ahead and slightly to the patients side. The patients shoulder is directly behind your shoulder. Relax and walk at a comfortable pace.

.To determine the patients actual temperature, the nurse is aware that: a.the patients core temperature is not affected by environmental temperature b.surface temperatures are the most stable c.circadian rhythms keep the body temperature stable throughout the day. d.gender and age have no effect on body temperature

A- the patients core temperature is not affected by environmental temperature Despite environmental temperature extremes and physical activity, temperature-control mechanisms of human beings keep the bodys core temperature , or temperature of deep tissues, relatively constant during sleep, exposure to cold, and strenuous exercise.

A patient states, Im burning up, and I have a fever. The nurse takes the patients temperature, observes the skin for flushing, and feels the skin temperature. This is an example of subjective data. a.validating b.clustering c.reviewing d.documenting

A- validating Validation of assessment data is the comparison of data with another source to confirm accuracy. The nurse reviews data to validate that measurable, objective physical findings support subjective data. A data cluster is a set of signs or symptoms that are grouped in a logical order. When a nurse reviews a patients subjective data, the nurse is examining the patients own interpretation of his or her condition. Documenting information includes the written details of the assessment.

A nurse delegates the task of obtaining vital signs to a nursing assistant. The nurse reminds the nursing assistant that blood pressure: a.will be lower if the cuff is too wide. b.is not affected by cuff length. c.should be taken slowly for more accuracy. d.should be taken with the arm above heart level

A- will be lower if the cuff is too wide A bladder or cuff that is too wide will yield false-low readings. A cuff that is wrapped too loosely or unevenly will yield false-high readings. Deflating a cuff too slowly will produce false-high diastolic readings. The arm should be at the level of the heart. Having the arm above heart level will produce a false-low reading

A 67-year-old male patient of French heritage is admitted to the hospital. The patient is interviewed by a nurse from a Korean family. The nurse did not make eye contact with the patient while conducting the interview. This disturbed the patient because the patient thought that the nurse might be trying to hide something. Which factor most likely influenced the behavior of the nurse and patient? a.Culture b.Validation c.Collaborative problem d.Defining characteristics

A-.Culture Communication and culture are interrelated in the way individuals express feelings verbally and nonverbally. When a nurse learns the variations in how people of different cultures communicate, he or she will likely gather more accurate information from patients. Validation of assessment data is the comparison of data with another source to confirm accuracy. A collaborative problem is an actual or potential physiological complication that nurses monitor to detect the onset of changes in a patients status. Defining characteristics are the clinical criteria or assessment findings that support an actual nursing diagnosis

The nurse is assessing a patients differential white blood cell count. What implications would this test have on evaluating the adequacy of a patients gas exchange? A An elevation of the total white cell count indicates generalized inflammation. B Eosinophil count will assist to identify the presence of a respiratory infection. C White cell count will differentiate types of respiratory bacteria. D Level of neutrophils provides guidelines to monitor a chronic infection.

A. An elevation of the total white cell count indicates generalized inflammation. Elevation of total white cell count is indicative of inflammation that is often due to an infection. Upper respiratory infections are common problems in altering a patients gas exchange. Eosinophil cells are increased in an allergic response. Neutrophils are more indicative of an acute inflammatory response. White cells do not assist to differentiate types of respiratory bacteria. Monocytes are an indicator of progress of a chronic infection.

A staff nurse reports a medication error, failure to administer a medication at the scheduled time. An appropriate response of the charge nurse would be A Well do a root cause analysis. B That means youll have to do continuing education. C Why did you let that happen? D Youll need to tell the patient and family.

A. Well do a root cause analysis. In a just culture the nurse is accountable for their actions and practice, but people are not punished for flawed systems. Through a strategy such as root cause analysis the reasons for errors in medication administration can be identified and strategies developed to minimize future occurrences. Requiring continued education may be an appropriate recommendation but not until data is collected about the event. Telling the patient is part of transparency and the sharing and disclosure among stakeholders, but it is generally the role of risk management staff, not the staff nurse.

The acid-base status of a patient is dependent on normal gas exchange. Which patient would the nurse identify as having an increased risk for the development of respiratory acidosis? A patient with A chronic lung disease with increased carbon dioxide retention B acute anxiety, hyperventilation, and decreased carbon dioxide retention C decreased cardiac output with increased serum lactic acid production D gastric drainage with increased removal of gastric acid

A. chronic lung disease with increased carbon dioxide retention Respiratory acidosis is caused by an increase in retention of carbon dioxide, regardless of the underlying disease. A decrease in carbon dioxide retention may lead to respiratory alkalosis. An increase in production of lactic acid leads to metabolic acidosis. Removal of an acid (gastric secretions) will lead to a metabolic alkalosis.

The nurse and a student nurse are discussing the effects of bed immobility on patients. The nurse knows that the student nurse understands the concept of mobility when she states, Patients with impaired bed mobility A have an increased risk for pressure ulcers. B like to have extra visitors. C need to have a mechanical soft diet. D are prone to constipation.

A. have an increased risk for pressure ulcers. Patients who cannot move themselves in bed are more susceptible to pressure ulcers because they cannot relieve the pressure they feel. Extra visitors or diet consistency do not have any bearing on mobility. Constipation should not be a by-product of immobility if a bowel regimen is instituted.

Which patient would the nurse identify as being at an increased risk for altered transport of oxygen? A patient with A hemoglobin level of 8.0 B bronchoconstriction and mucus C peripheral arterial disease D decreased thoracic expansion

A. hemoglobin level of 8.0 Altered transportation of oxygen refers to patients with insufficient red blood cells to transport the oxygen present. Bronchoconstriction and decreased thoracic expansion (spinal cord injury) would result in impairment of ventilation. Peripheral vascular disease would result in inadequate perfusion.

The nurse would identify which patient as having a problem of impaired gas exchange secondary to a perfusion problem? A patient with A peripheral arterial disease of the lower extremities B chronic obstructive pulmonary disease (COPD) C chronic asthma D severe anemia secondary to chemotherapy

A. peripheral arterial disease of the lower extremities Perfusion relates to the ability of the blood to deliver oxygen to the cellular level and return the carbon dioxide to the lung for removal. COPD and asthma are examples of a ventilation problem. Severe anemia is an example of a transport problem of gas exchange.

A patient who has been in the hospital for several weeks is about to be discharged. The patient is weak from the hospitalization and asks the nurse to explain why this is happening. The nurses best response is AYou are weak because your iron level is low. This is known as anemia. B of your immobility in the hospital. This is known as deconditioning. C of your poor appetite. This is known as malnutrition. D of your medications. This is known as drug induced weakness.

B of your immobility in the hospital. This is known as deconditioning. When a person is ill and immobile the body becomes weak. This is known as deconditioning. Anemia, malnutrition, and medications may have an adverse effect on the body, but this is not known as deconditioning.

The home health nurse is caring for a patient with a diagnosis of acute immunodeficiency syndrome (AIDS) who has chronic diarrhea. Which assessments should the nurse use to detect the fluid and electrolyte imbalances for which the patient has highest risk? (Select all that apply.) a. Bilateral ankle edema b. Weaker leg muscles than usual c. Postural blood pressure and heart rate d. Positive Trousseau sign e. Flat neck veins when upright f. Decreased patellar reflexes

B, C, D Chronic diarrhea has high risk of causing ECV deficit, hypokalemia, hypocalcemia, and hypomagnesemia because it increases fecal excretion of sodium-containing fluid, potassium, calcium, and magnesium. Appropriate assessments include postural blood pressure and heart rate for ECV deficit; weaker leg muscles than usual for hypokalemia; and positive Trousseau sign for hypocalcemia and hypomagnesemia. Bilateral ankle edema is a sign of ECV excess, which is not likely with chronic diarrhea. Flat neck veins when upright is a normal finding. Decreased patellar reflexes is associated with hypermagnesemia, which is not likely with chronic diarrhea.

A patient reports chills, dizziness, and feeling hot during a blood transfusion. What are the appropriate actions of the nurse? (Select all that apply.) a. Slow the rate of infusion. b. Stop the blood transfusion. c. Check the patient's vital signs. d. Notify the physician and blood bank. e. Restart the patient's IV at a different site.

B, C, D STOP (do not slow down) the transfusion immediately even when you just suspect a reaction. Remain with the patient, observing signs and symptoms and monitoring vital signs as often as every 5 minutes. Immediately notify the health care provider or emergency response team and the blood bank. The patient's IV should not be restarted at a different site.

Which statements said by patients indicate that the nurse's teaching regarding prevention of acid-base imbalances is successful? (Select all that apply.) a. "Baking soda is an effective and inexpensive antacid." b. "I should take my insulin on time every day." c. "My aspirin is on a high shelf away from children." d. "I have reliable transportation to dialysis sessions." e. "Fasting is a great way to lose weight rapidly."

B, C, D Taking insulin as prescribed helps prevent diabetic ketoacidosis. Safeguarding aspirin from children prevents metabolic acidosis from increased acid intake. Regular dialysis reduces the risk of metabolic acidosis from decreased renal excretion of metabolic acid. Baking soda is sodium bicarbonate and should not be used as an antacid due to the risk of metabolic alkalosis. Fasting without carbohydrate intake is a risk factor for starvation ketoacidosis.

The nurse is working with a patient diagnosed with posttraumatic stress disorder related to childhood sexual abuse. The patient is crying and states, "I should be over this by now; this happened years ago." Which response(s) by the nurse will facilitate communication? (Select all that apply.) a. "Why do you think you are so upset?" b. "I can see that this situation really bothers you." c. "The abuse you endured is very painful for you." d. "Crying is a way of expressing the hurt you're experiencing." e. "Let's talk about something else, since this subject is upsetting you."

B, C, D Reflecting and giving information are therapeutic techniques. "Why" questions often imply criticism or seem intrusive or judgmental. They are difficult to answer. Changing the subject is a barrier to communication

The bodys tissues and cells function efficiently within a relatively narrow temperature range, from 36 to 38 C (96.8 to 100.4 F). Factors that cause fluctuations in body temperature include which of the following? (Select all that apply.) a.The bodys core temperature b.Age c.Time of day d.Circadian rhythm e.physical activity

B, C, D, E

The patient has been diagnosed with progressive Alzheimers disease. Characteristics of this disease include which of the following? (Select all that apply.) a.Delirium b.Agnosia c.Apraxia d.Aphasia e.Amnesia

B, C, D, E Alzheimers disease is the most common form of dementia. Alzheimers disease is a progressive loss of memory (amnesia), loss of ability to recognize objects (agnosia), loss of the ability to perform familiar tasks (apraxia), and loss of language skills (aphasia). As the disease progresses, some patients also experience changes in personality and behavior, such as anxiety, suspiciousness, or agitation, as well as delusions or hallucinations. Delirium is an acute confusional state and requires prompt assessment. It is a potentially reversible cognitive impairment that is often a result of physiological causes. Some of these causes include electrolyte imbalance, hypoglycemia, infection, and medications.

A nurse is working with the parents of a seriously ill newborn. Surgery has been proposed for the infant, but the chances of success are unclear. To help the parents resolve this conflict, which steps should the nurse take to process the ethical dilemma? (Select all that apply.) a.Identifying people who can solve this dilemma b.Gathering all relevant information surrounding this dilemma c.Clarifying own values and opinions about the issues d.Consulting a professional ethicist regarding how to proceed with this dilemma e.Considering possible courses of action

B, C, E The nurse should gather all relevant information, clarify own values and opinions about the issue, and consider possible courses of action. Seven steps are used when solving an ethical dilemma: (1) Asking is it an ethical dilemma?, (2) gathering all information, (3) examining and determining ones own values and opinions about the issue, (4) stating the problem clearly, (5) considering possible courses of action, (6) negotiating an outcome, and (7) evaluating the action. Identifying people who can solve this dilemma and consulting a professional ethicist are not steps of the process.

Which nursing interventions are appropriate for a terminally ill patient who is a devout Jew? (Select all that apply.) a. Arrange for a minister to provide the sacrament of anointing of the sick. b. Facilitate bedside Sabbath services for the patient and the family members. c. Allow the patient's family to maintain a vigil throughout the day and night. d. Provide snacks and meals for the patient and family only from sunset until dawn. e. Respect the decision of the patient and family not to put the patient on life support

B, C, E Sabbath services are very important for people of Jewish faith so the nurse should attempt to facilitate bedside Sabbath services for the patient and the family members. Constant vigil at the bedside is also very important so the nurse should allow the patient's family to maintain a vigil throughout the day and night. Life support is often discouraged within the Jewish faith and the nurse must respect the wishes of the patient and family. It would only be appropriate for the nurse to arrange for a minister to provide the sacrament of anointing of the sick when the patient is Catholic and requests to receive the sacrament. Muslims do not eat anything from dawn until sunset during the month of Ramadan. Providing snacks and meals for the patient and family only from sunset until dawn would be inappropriate for a Jewish patient and family.

The nurse is caring for an adolescent with a BMI of 22 and the nursing diagnosis imbalanced nutrition, more than body requirements related to caloric intake greater than metabolic needs. Which nursing interventions will the nurse use to help the adolescent achieve a balanced diet? (Select all that apply.) a. Use sugary treats as a reward for getting good grades in school. b. Encourage the adolescent to gradually increase physical activity. c. Recommend intake of at least 2 to 3 L of water each day. d. Remind the adolescent of how body image affects relationships. e. Encourage the adolescent to eat breakfast every day before school.

B, C, E Gradually increasing physical activity, drinking 2 to 3 L of water daily, and eating breakfast daily will help the adolescent balance the diet and manage weight. Using sugary treats as a reward will worsen the adolescent's imbalanced diet. The adolescent does not need to be reminded about how body image affects relationships.

The nurse is obtaining an oxygen saturation reading from a female patient who was brought in after collapsing at a party. Factors that may lead to inaccurate SpO 2 readings include which of the following? (Select all that apply.) a.Using a pulsatile area to attach the probe b.The patient wearing fingernail polish c.Measuring the level intermittently d.The patient being anemic e.Measuring the level continuously

B, D

The patient is hyperventilating from anxiety and abdominal pain. Which assessment findings should the nurse attribute to respiratory alkalosis? (Select all that apply.) a. Skin pale and cold b. Tingling of fingertips c. Heart rate of 102 d. Numbness around mouth e. Cramping in feet

B, D, E Hyperventilation is a risk factor for respiratory alkalosis. Respiratory alkalosis can cause perioral and digital paresthesias and pedal spasms. Pallor, cold skin, and tachycardia are characteristic of activation of the sympathetic nervous system, not respiratory alkalosis.

A 65-year-old female patient has been admitted to the medical/surgical unit. The nurse is assessing the patients risk for falls so that falls prevention can be implemented if necessary. Select all the risk factors that apply from this patients history and physical. (Select all that apply.) A Being a woman B Taking more than six medications C Having hypertension D Having cataracts E Muscle strength 3/5 bilaterally F Incontinence

B, D, E, F Adverse effects of medications can contribute to falls. Cataracts impair vision, which is a risk factor for falls. Poor muscle strength is a risk factor for falls. Incontinence of urine or stool increases risk for falls. Men have a higher risk for falls. Hypertension itself does not contribute to falls. Dizziness does contribute to falls.

A 65-year-old female patient has been admitted to the medical/surgical unit. The nurse is reviewing the patient's risk for falls so that falls prevention can be implemented if necessary. Select all the risk factors that apply from this patient's history and physical. (Select all that apply.) a. Being a woman b. Taking more than six medications c. Having hypertension d. Having cataracts e. Muscle strength 3/5 bilaterally f. Incontinence

B, D, E, F Adverse effects of medications can contribute to falls. Cataracts impair vision, which is a risk factor for falls. Poor muscle strength is a risk factor for falls. Incontinence of urine or stool increases risk for falls. Men have a higher risk for falls. Hypertension itself does not contribute to falls. Taking medications to treat hypertension that may lead to hypotension and dizziness is a fall risk. Dizziness does contribute to falls.

The nurse is assessing a patient for the adequacy of ventilation. What assessment findings would indicate the patient has good ventilation? (Select all that apply.) A Respiratory rate is 24 breaths/min. B Oxygen saturation level is 98%. C The right side of the thorax expands slightly more than the left. D Trachea is just to the left of the sternal notch. E Nail beds are pink with good capillary refill. F There is presence of quiet, effortless breath sounds at lung base bilaterally.

B, E, F Oxygen saturation level should be between 95 and 100%; nail beds should be pink with capillary refill of about 3 seconds; and breath sounds should be present at base of both lungs. Normal respiratory rate is between 12 and 20 breaths/min. The trachea should be in midline with the sternal notch. The thorax should expand equally on both sides.

The nurse is reviewing a patient for the adequacy of ventilation. What review findings would indicate the patient has good ventilation? (Select all that apply.) a. Respiratory rate is 24 breaths/min. b. Oxygen saturation level is 98%. c. The right side of the thorax expands slightly more than the left. d. Trachea is just to the left of the sternal notch. e. Nail beds are pink with good capillary refill. f. There is presence of quiet, effortless breath sounds at lung base bilaterally.

B, E, F Oxygen saturation level should be between 95 and 100%; nail beds should be pink with capillary refill of about 3 seconds; and breath sounds should be present at base of both lungs. Normal respiratory rate is between 12 and 20 breaths/min. The trachea should be in midline with the sternal notch. The thorax should expand equally on both sides.

A nurse sets up a sterile field. A break in the sterile field occurs when the nurse does which of the following? (Select all that apply.) a.Drops a sterile capped needle onto the sterile field b.Spills solution onto the sterile field c.Keeps the top of the table above waist level d.Keeps sterile objects within a 1-inch border of the field

B,D

.As part of the admission process the nurse asks if the patient has an advance directive. The patient doesnt know for sure. What is the nurses best response? a.It is autopsy permission. b.It is a living will. c.It is informed consent. d.It is an organ donation card.

B- .It is a living will. Many times the decision regarding lifesaving treatment is in writing in the patients living will or advance directive. Living wills are documents instructing the health care provider to withhold or withdraw life-sustaining procedures in a patient who is terminally ill. Advanced directives are not an organ donation card, nor informed consent, nor autopsy permission

Which patient would the nurse consider to be competent to give informed consent? a.A 27-year-old unconscious patient b.A 16-year-old emancipated minor c.A 43-year-old patient who is drunk d.A 33-year-old patient who has been declared legally incompetent

B- A 16-year-old emancipated minor Even though an emancipated minor has not achieved the legal age of consent, he or she may give consent for procedures and treatment. If a patient is unconscious, you need to obtain consent from a person legally authorized to give consent on his or her behalf. A patient who is legally incompetent needs to have the consent of a legal guardian, which is determined through a legal proceeding. A person who is drunk cannot fully understand the procedure and cannot sign the consent form

A nurse listens to a patients lungs and determines that the patient needs to cough and deep breath. The nurse has the patient cough and deep breath. Which concept did the nurse demonstrate? a.Accountability b.Autonomy c.Licensure d.Certification

B- Autonomy Autonomy is essential to professional nursing and involves the initiation of independent nursing interventions without medical orders. Accountability means that you are professionally and legally responsible for the type and quality of nursing care provided. To obtain licensure in the United States, RN candidates must pass the NCLEX RN examination administered by the individual State Boards of Nursing to obtain a nursing license. Beyond the NCLEX-RN, some nurses choose to work toward certification in a specific area of nursing practice.

A registered nurse is working on a pediatric oncology unit and caring for four children undergoing chemotherapy. Today a new nursing assistive personnel (NAP) who has passed a competency examination is assigned to the team. The nurse will delegate a portion of the fundamental nursing tasks to the NAP during the shift, but realizes that he or she is still responsible for his or her own actions and is accountable for the care. The nurse is following which principle of behavior? a.Ethical dilemma b.Code of ethics c.Bioethics d.Feminist ethics

B- Code of ethics The code of ethics reflects underlying principles that include responsibility, accountability, respect for confidentiality, competency, judgment, and advocacy. An ethical dilemma exists when the right thing to do is not clear or when members of the health care team cannot agree on the right thing to do. The study of bioethics represents a particular branch of ethics (i.e., the study of ethics within the field of health care). Feminist ethics proposes that we routinely ask how ethical decisions will affect women as a way to repair a history of inequality (Lindeman, 2005)

A nurse is maintaining precise records regarding the dispensing, wasting, and storage of a drug that is securely locked. Which drug is the nurse administering? a.Routine medication b.Controlled substance c.Over-the-counter medication d.Substance not requiring an order

B- Controlled substance Controlled substances are securely locked away, and only authorized personnel have access to them. Maintain precise records regarding the dispensing, wasting, and storage of controlled substances. There are criminal penalties for the misuse of controlled substances. Routine and over-the-counter drugs are not controlled substances. Controlled substances required an order by a licensed physician or in some states advanced practice nurses.

A patient has met the goals and outcomes mutually agreed upon for improvement of ventilatory status. What should the nurse do next? a.Modify the care plan. b.Discontinue the care plan. c.Create a nursing diagnosis that states goals have been met. d.Reassess the patients response to care and evaluate interventions.

B- Discontinue the care plan After a nurse determines that expected outcomes and goals have been met and evaluation confirms it, the nurse discontinues that portion of the care plan. The nurse modifies a care plan when goals are not met. Create a nursing diagnosis occurs after assessment, not during evaluation. Reassessing the patient occurs if the goals are not met

.Which action by the nurse is the final step in a complete assessment? a.Forming diagnostic conclusions b.Documentation of findings c.Auscultation d.Palpation

B- Documentation of findings

A 6-year-old was taken to the hospital after having a seizure at home. The patients mother tells the nurse that she has been ill for the past week and has had a fever with nausea and vomiting. The patients mother believes that the seizure was caused by a fever of 99.5 F, which the patient had during the course of her illness. What is the nurses best response? a.With a temperature that high, we can only hope that there is no permanent damage. b.Fevers in this range are part of the bodys natural defense system c.Febrile seizures are common in children Nancys age. d.The child will need antibiotics. Does she have any allergies?

B- Fevers in this range are part of the bodys natural defense system Fever serves as an important defense mechanism. Therefore most health care providers will not treat an adults fever until it is greater than 39 C (102.2 F).

A clinic nurse stopped at an automobile accident to assist. There was one victim who was not breathing. The nurse provided CPR at the scene, but the victim died. The victims family sued the nurse. Which will provide the best protection to the nurse in this case? a.Clinics malpractice insurance policy b.Good Samaritan Law c.State Board of Nursing d.Institute of Medicine

B- Good Samaritan Law The Good Samaritan Law protects the nurse because CPR is within a nurses scope of practice. Although Good Samaritan Laws provide immunity to the nurse who does what is reasonable to save a persons life, if the nurse performs a procedure for which he or she has no training, the nurse will be liable for any injury resulting from that act. Therefore, provide only care that is consistent with your level of expertise. The insurance policy, state boards of nursing, and Institute of Medicine do not provide protection to the nurse under the Good Samaritan Law.

A patient with a history of seizures is being admitted to the hospital after a grand mal seizure took place at a shopping mall. The patients spouse accompanied the patient to the hospital and is being interviewed by the nurse. Which question should the nurse ask to quickly focus on the patients symptoms? a.What made you choose this hospital? b.How long did the seizure last? c.Tell me how the seizure disorder has affected the family. d.Tell me why you brought your spouse to the hospital today.

B- How long did the seizure last? How long did the seizure last? is the question that will quickly focus on the patients symptoms. Once patients tell their story, use a problem-seeking interview technique. This approach takes the information provided in the patients story and then more fully describes and identifies specific problem areas. For example, focus on the symptoms the patient identifies and ask closed-ended questions that limit the patients answers to one or two words such as yes or no or a number or frequency of a symptom. What made you choose this hospital does not focus on the seizure. Tell me will not get information quickly as these are open-ended.

A postoperative patient has denied the need for pain medication. The nurse has noted that the patient describes the pain as a 1 on a 0 to 10 scale. The nurse also notes that the patient grimaces when he or she changes position and guards the incision. The nurse believes that the patient is experiencing pain based on the information gathered in the assessment. What is this phenomenon known as? a.Cue b.Inference c.Diagnosis d.Health pattern

B- Inference The nurse made a judgment, which is an inference, that the patient is experiencing pain. An inference is a nurses judgment or interpretation of a cue. A cue is information that you obtain through use of the senses. Gordons functional health patterns are a type of database format to obtain a comprehensive assessment. A nursing diagnosis is a clinical judgment about individual, family, or community responses to actual and potential health problems or life processes that the nurse is licensed and competent to treat such as impaired tissue perfusion

A nurse uses four basic skills during a physical assessment. While assessing the abdomen the nurse must begin with which of the following skills? a.Light palpation b.Inspection c.Deep palpation d.Auscultation

B- Inspection The order of abdominal assessment differs from previous assessments. First the nurse begins by inspection, followed by auscultation. By using auscultation before palpation (light or deep), there is less chance of altering the frequency and character of bowel sounds

A nurse develops a nursing diagnosis for a patient. What is the rationale for the nurses actions? a.It allows a nurse to compete with physicians or health care providers. b.It allows a nurse to develop an individualized plan of care. c.It allows a nurse to treat nursing problems and medical problems. d.It allows a nurse to manage patient care for the entire health team

B- It allows a nurse to develop an individualized plan of care The diagnostic process results in the formation of a total diagnostic statement that allows a nurse to develop an appropriate, patient-centered plan of care. A nursing diagnosis provides direction for nursing, not for medical problems or for the entire health team. It is not used to compete with physicians or health care providers

A nurse is preparing to perform a physical exam on a patient. She has found that it is best to perform the physical with a head-to-toe approach. Why is this important? a.The head-to-toe format excludes unnecessary body systems. b.It is a methodical way to include all body systems. c.It reduces time by allowing examination of only one side. d.It requires that painful procedures be done first.

B- It is a methodical way to include all body systems. A head-to-toe approach includes all body systems and helps to anticipate each step. In an adult, a nurse begins by assessing the head and neck, progressing methodically down the body to include all body systems. Both sides of the body must be compared for symmetry. Any painful procedures should be performed near the end of the examination.

.A 45-year-old mother of two children has cirrhosis of the liver and is on a waiting list for a liver transplant. She had to meet certain criteria to be eligible to receive a liver. She understands that she is next on the list for a donor liver that matches. This is an example of which ethical principle? a.Autonomy b.Justice c.Fidelity d.Nonmaleficence

B- Justice Justice refers to the principle of fairness. In health care, the term is used to reflect a commitment to fair treatment and fair distribution of health care resources. You may find reference to this principle during discussion about issues of access to care. It is not always clear just how to achieve a fair distribution of resources. Autonomy refers to independence and self-determination. Fidelity refers to the agreement to keep promises and is based on the virtue of caring. Nonmaleficence refers to the fundamental agreement to do no harm.

A 64-year-old house painter who is seeing his health care provider for his annual checkup. When the nurse asks the patient if they have any health concerns, the patient states, I dont think my vision is as good as it used to be, things look more yellow than they used to. The nurse knows that this is a visual change in older adults caused by which of the following? a.Iris yellows b.Lens yellows c.Retina is hypersensitive d.Need for less light to see than when they were in young adulthood

B- Lens yellows Visual changes often include reduced visual fields, increased glare sensitivity, impaired night vision, reduced accommodation, reduced depth perception, and reduced color discrimination. Many of these symptoms occur because the pupils in the older adult take longer to dilate and constrict secondary to weaker iris muscles. Color vision decreases because the retina is duller and the lens yellows. Eventually, older adults may require three times as much light to see things as they did when they were in young adulthood.

15.A single mother with three school-age children has recently noticed that the second child has been scratching his head and complains that it itches. She asks the school nurse to examine him. The school nurse notes that the child has head lice. Which of the following is the best description of head lice? a.Lice are easy to see and look like little white crabs. b.Lice are difficult to see, but their eggs are small oval particles. c.Lice and their eggs are on the hair shafts and look like dandruff d.Treatment for lice can wait until the entire family can be checked

B- Lice are difficult to see, but their eggs are small oval particles. Head and crab lice attach their eggs to hair. Lice eggs look like oval particles of dandruff. The lice themselves are difficult to see. Observe for bites or pustular eruptions in the follicles and in areas where skin surfaces meet, such as behind the ears and in the groin. The discovery of lice requires immediate treatment and family education

A nurse is assisting with a female genitalia examination. The nurse will assist the patient to which position? a.Supine b.Lithotomy c.Knee-chest d.Dorsal recumbent

B- Lithotomy The lithotomy position provides maximal exposure of the rectal area. The supine position is used to assess the head, neck, anterior thorax, lungs, breasts, axillae, heart, abdomen, extremities, and pulses. It is the most normally relaxed position

The student nurse caring for a postsurgical patient who has developed a health careacquired wound infection that has become systemic. Which of the following should be the student nurses top priority? a.Providing emotional support b.Managing vital signs c.Providing patient education d.Providing personal hygiene

B- Managing vital signs Give special attention to any urgent needs the infection creates

A nurse must follow legal laws that protect public health, safety, and welfare. Which law is the nurse following? a.Code of Ethics b.Nurse Practice Act c.Standards of practice d.Quality and safety education for nurses

B- Nurse Practice Act In the United States each State Board of Nursing oversees its Nurse Practice Act (NPA), which regulates the scope of nursing practice for the state and protects public health, safety, and welfare. The ANAs Code of Ethics for Nurses: Interpretation and Application (2010) provides a guide (not a law) for carrying out nursing responsibilities to ensure high-quality nursing care and provide for the ethical obligations of the profession. The purpose of a standard of care is to describe the common level of professional nursing care to judge the quality of nursing practice. The Robert Wood Johnson Foundation sponsored the Quality and Safety Education for Nurses (QSEN) initiative to respond to reports about safety and quality patient care by the Institute of Medicine.

A patient is admitted to the hospital after a motorcycle accident. The nurse in the emergency room is assessing vital signs, general appearance and behavior, and performing a head-to-toe examination of all body systems. What is the nurse doing? a.Making a medical diagnosis b.Performing a physical examination c.Making an evaluation d.Performing data validation

B- Performing a physical examination A physical examination involves use of the techniques of inspection, palpation, percussion, auscultation, and smell. A complete examination includes a patients height, weight, vital signs, general appearance and behavior, and a head-to-toe examination of all body systems. Nurses make nursing diagnoses, not medical diagnoses, after assessment of data. Evaluation is the last step of the nursing process. Evaluation is crucial to deciding whether, after interventions have been delivered, a patients condition or well-being improves. Validation of assessment data is the comparison of data with another source to confirm accuracy.

What term refers to the distribution of red blood cells to and from the pulmonary capillaries? a.Diffusion b.Perfusion c.Respiration d.Eupnea

B- Perfusion Respiration is the mechanism the body uses to exchange gases between the atmosphere, blood, and cells. Respiration involves three processes: ventilation (the mechanical movement of gases into and out of the lungs), diffusion (the movement of oxygen [O 2 ] and carbon dioxide [CO 2 ] between the alveoli and the red blood cells), and perfusion (the distribution of red blood cells to and from the pulmonary capillaries). The normal rate and depth of ventilation is known as eupnea

The nurse is having difficulty hearing his patients apical pulse with his stethoscope. Which of the following would best maximize the sound quality of what is heard through the stethoscope? a.Positioning the diaphragm very lightly on the area to which he is listening b.Placing the stethoscope chest piece directly on the patients skin c.Make sure that the earpieces fit loosely in the ear canals d.Use a stethoscope with the longest tubing available

B- Placing the stethoscope chest piece directly on the patients skin Always place the stethoscope directly on the skin because clothing obscures the sound. Position the diaphragm to make a tight seal against the patients skin

Which behavior is the best way for a nurse to avoid being liable for malpractice? a.Purchasing quality malpractice insurance coverage on a yearly basis b.Practicing nursing that meets the generally accepted standard of care c.Not sharing his or her last name with patients and families d.Not delegating any tasks to unlicensed assistive personnel

B- Practicing nursing that meets the generally accepted standard of care The best way to avoid being liable for malpractice is to give nursing care that meets the generally accepted standard of care. In a malpractice lawsuit the law uses nursing standards of care to measure nursing conduct and determine whether the nurse acted as any reasonably prudent nurse would act under the same or similar circumstances. Purchasing insurance, not sharing last name, and not delegating tasks are not appropriate behaviors to avoid malpractice.

What is the difference between systolic and diastolic pressure? a.Diastolic pressure b.Pulse pressure c.Hypertension d.Systolic pressure

B- Pulse pressure The difference between systolic and diastolic pressure is the pulse pressure. For a blood pressure (BP) of 120/80 mm Hg, the pulse pressure is 40. When the ventricles relax, the blood remaining in the arteries exerts a minimum or diastolic pressure

A patient has an outcome of ambulating three times a day. The patient does not ambulate the entire day. What should the nurse do next? a.Walk the patient. b.Reassess the patient. c.Change the goal for the patient. d.Continue with the plan for the patient.

B- Reassess the patient When there is failure to achieve a goal, no matter what the reason, repeat the entire nursing process sequence for that nursing diagnosis to discover changes the plan needs. A complete reassessment of all patient factors relating to the nursing diagnosis and etiology is necessary when modifying a plan. The plan cannot continue because the goal was not met. The goal cannot be changed and walking the patient cannot occur until reassessment has been completed

While auscultating a patient with pneumonia, a nurse hears low-pitched, rumbling coarse sounds during inspiration and expiration. These sounds can best be described as which of the following? a.Crackles b.Rhonchi c.Wheezes d.A friction rub

B- Rhonch Rhonchi are loud, low-pitched, rumbling coarse sounds heard either during inspiration or expiration.

A family member is accompanying the elderly patient to their follow-up appointment after a recent hospitalization for gastrointestinal problems. The nurse interrupts a discussion between the family member and the patient regarding rancid food in the patients refrigerator. The family member looks at the nurse and states, She was trying to eat spoiled food for lunch, it spelled terrible, and she still wanted to eat it. What is the most likely physiological reason that the patient not realizes that the food is spoiled? a.She has xerostomia. b.She has a diminished sense of smell. c.She has a diminished sense of taste. d.She has a limited vision.

B- She has a diminished sense of smell. Olfactory changes begin around age 50 and include a loss of cells in the olfactory bulb of the brain and a decrease in the number of sensory cells in the nasal lining. Reduced sensitivity to odors is common. A small decrease in the number of taste cells occurs with aging, beginning around age 60. Reduced sour, salty, and bitter taste discrimination is common. The ability to detect sweet tastes seems to remain intact. Xerostomia is the decrease in salivary production that leads to thicker mucus and a dry mouth. This interferes with the ability to eat and leads to appetite and nutritional problems.

A 26-year-old man was helping a friend replace a roof on his backyard shed after work on a hot July afternoon. His friend brought him to the hospital after the patient complained of severe muscle cramps and became confused. Which of the following should the admitting nurse do first when assessing the patient? a.Place the patient in a tub of iced water. b.Take the patients temperature. c.Remove fans to prevent premature chilling. d.Apply a hyperthermia blanket to lower temperature slowly.

B- Take the patients temperature Assessment includes taking the patients temperature. The nurse then uses that measurement to guide care of that patient. Placing the patient in a tub of iced water, removing fans to prevent premature chilling, and applying a hyperthermia blanket to lower temperature slowly are not assessments but interventions.

A patient is suffering from shortness of breath. How should the nurse write the expected outcome for this patient? a.The patient will be comfortable by the morning. b.The patient will breathe unlabored at 14 to 18 breaths per minute by the end of the shift. c.The patient will not complain of breathing problems. d.The patient will appear less short of breath

B- The patient will breathe unlabored at 14 to 18 breaths per minute by the end of the shift. Each patient outcome contains the following aspects in order to be correctly written: (1) patient-centered, (2) singular, (3) observable, (4) measurable, (5) time limited, (6) mutual factors, and (7) realistic. Comfortable is not measurable. Outcome that deals with no complaints of breathing is lacking the time limited guideline. Patient will appear less short of breath is not a correct statement because there is no specific observable behavior for appears less short of breath.

.A student nurse is helping admit a 77-year-old man to the surgical floor of an acute care hospital. The patient has an abdominal tumor that is scheduled to be removed. He has had nothing by mouth (NPO) since midnight in preparation for his surgery. The student nurse notes that the patients skin is very dry and scaly. Which of the following should the student nurse investigate further as the most likely cause for the patients dry skin? a.Excess humidity in the patients living environment b.The use of excessive soap when bathing c.Lack of sun exposure leading to decreased stimulation of sweat glands d.Decreased levels of stress

B- The use of excessive soap when bathing Excessively dry skin is common in older adults and persons who use excessive amounts of soap during bathing. Other factors causing dry skin include lack of humidity, exposure to sun, smoking, stress, excessive perspiration, and dehydration. Excessive dryness worsens existing skin conditions.

Which situation will enable a nurse to use restraints? a.To punish a patient b.To ensure the patients safety c.To retaliate against poor behavior d.To ensure staff convenience

B- To ensure the patients safety Regulations set the standard that all patients have the right to be free from seclusion and physical or chemical restraints except to ensure the patients safety in emergency situations. The standards specifically prohibit restraining patients for staff convenience, punishment, or retaliation.

A 45-year-old gay man who lives with a partner of 19 years recently has developed a persistent cough and night sweats from which he wakes up soaked. The nurse who is caring for the man should be most concerned about which of the following conditions? a.Lung cancer b.Tuberculosis c.Orthopnea d.Cardiopulmonary disease

B- Tuberculosis Persistent cough and night sweats are symptoms of tuberculosis (TB). Review risk factors for TB and/or HIV infection and assess for symptoms, including persistent cough, hemoptysis, unexplained weight loss, fatigue, anorexia, night sweats, and fever

A registered nurse has recently been reassigned to the gynecology unit at the hospital. The nurse is strongly against abortion because of religious beliefs and contacts the nursing supervisor regarding the assignment because the unit cares for women who are undergoing abortions. The nurse is having a conflict in which area? a.Confidentiality b.Values c.Social networking d.Culture

B- Values The nurse is having a conflict in values because of religious beliefs and abortion. A value is a personal belief about the worth of an idea, a custom, or an object. Confidentiality is not the issue because no confidences have been broken. Social networking is online communication, which is not the issue in this scenario, values are the issue. The nurse is not having a conflict in culture, but in beliefs and values.

A 15-year-old girl was taken to a small rural hospital by her mother. The family had been camping, and it had become very cold during the night. The mother had difficulty waking her daughter in the morning, and she was shivering uncontrollably. The patient is still unconscious. Which of the following interventions should the admitting nurse do first? a.Have the patient drink hot liquids. b.Wrap the girl in warm blankets. c.Uncover the head to allow the head to warm. d.Place heating pads on the bottom of the feet.

B- Wrap the girl in warm blankets. The priority treatment for hypothermia is to prevent a further decrease in body temperature. Removing wet clothes, replacing them with dry ones, and wrapping the patient in blankets are strategic nursing interventions

When the fever breaks, the temperature returns to an acceptable range and the patient becomes: a.febrile. b.afebrile. c.fever of unknown origin (FUO). d.pyrexic.

B- afebrile When the fever breaks, the temperature returns to an acceptable range and the patient becomes afebrile (a- means not or without, so afebrile means without fever )

A nurse inspects the thorax of an older adult patient who has chronic lung disease. An expected finding would be a(n): a.round shape of the chest. b.barrel-shaped chest. c.AP diameter greater than the transverse diameter. d.AP diameter less than the transverse diameter.

B- barrel-shaped chest. In adults, a barrel-shaped chest (AP diameter = transverse) characterizes chronic lung disease. Normally the chest is symmetrical, with the anteroposterior diameter one third to one half the size of the transverse diameter. Infants have an almost round shape with a 1:1 ratio between the AP and transverse diameters. In addition, a more rounded chest is associated with older age

The nurse is taking the pulse of an adult patient and finds that the patients heart rate is 48. He knows that this is considered: a.tachycardia. b.bradycardia. c.a normal heart rate for an infant. d.a normal heart rate for an adult.

B- bradycardia

A nurse is teaching the staff about how to process an ethical dilemma. Which order should the nurse use to present the steps? a. Evaluate the action. b. Negotiate the outcome. c. State the problem clearly. d. Gather all relevant information. e. Examine own values and opinions. f. Consider possible courses of action.

B- d, e, c, f, b, a

A patient with poor vision is ready to be discharged. The nurse is educating the patient and family regarding ways to improve vision. The nurse teaches the patient and family to avoid reading materials with shiny surfaces. The rationale for this intervention is which of the following? a.Glare causes headaches. b.Glare will reduce visual acuity. c.Shiny surfaces reflect damaging rays. d.Too much light is damaging to the eyes.

B.Glare will reduce visual acuity. When a patient ages, the pupil loses the ability to adjust to light. Therefore reducing the amount of bright light in the patients environment will assist vision. Reduce glare by eliminating waxed floors and shiny surfaces exposed to bright sunlight, tint glass, install sheer curtains over windows, and use soft and diffused lighting.

The home care nurse is trying to determine the necessary services for a 65-year-old patient who was admitted to the home care service status after left knee replacement. Which tool(s) will assist with this determination? A Minimum Data Set (MDS) B Functional Status Scale (FSS) C 24-Hour Functional Ability Questionnaire (24hFAQ) D The Edmonton Functional Assessment Tool

C 24-Hour Functional Ability Questionnaire (24hFAQ) The 24hFAQ assesses the postoperative patient in the home setting. The MDS is for nursing home patients. The FSS is for children. The Edmonton is for cancer patients.

Which term will the nurse use to document the patient's age-related hearing loss? a. Tinnitus b. Meniere's disease c. Presbycusis d. Presbyopia

C Presbycusis Hearing changes often associated with aging include decreased hearing acuity, speech intelligibility, and pitch discrimination, which is referred to as presbycusis. Low-pitched sounds are easiest to hear, but it is difficult to hear conversation over background noise. A decrease in active sebaceous glands causes the cerumen to become dry and completely obstruct the external auditory canal. Tinnitus is commonly caused by ototoxicity and patients experience the sensation of ringing in the ears. Presbyopia refers to the gradual decline in ability of the lens to accommodate or focus on close objects and reduces ability to see near objects clearly. Although the cause of Meniere's disease is unknown, the symptoms include progressive low-frequency hearing loss, vertigo, tinnitus, and a full feeling or pressure in the affected ear.

Which is the priority nursing diagnosis for a patient who has been diagnosed with Meniere's disease? a. Nausea related to constant sensation of noxious taste b. Acute confusion related to delirium and disorientation c. Risk for falls related to unsteadiness and loss of balance d. Autonomic dysreflexia related to distention of bowel or bladder

C Risk for falls related to unsteadiness and loss of balance Meniere's disease causes vertigo, a sensation of the environment spinning. This causes the patient to be unsteady and at high risk for falls. Nausea is less of a priority than falls. Meniere's disease does not cause confusion or autonomic dysreflexia.

A patient is talking with the nurse about hip fractures. The patient would like to know the best approach to strengthen the bones. The nurses best response is which of the following? A Walk at least 5 miles every day for exercise. B Wear proper fitting shoes to prevent tripping. C Talk with your physician about a calcium supplement. D Stand up slowly so you dont feel faint.

C Talk with your physician about a calcium supplement. Calcium strengthens the bones. A calcium supplement will help strengthen bones as they may be affected by aging, illness, or trauma. Walking several miles will help strengthen the bones but a calcium supplement is a good addition. Wearing proper shoes and standing slowly to prevent dizziness is important but they will not prevent fractures.

To plan early intervention and care for a child with a developmental delay, the nurse would consider knowledge of the concepts most significantly impacted by development, including A culture. B environment. C functional status. D nutrition.

C functional status. Function is one of the concepts most significantly impacted by development. Others include sensory-perceptual, cognition, mobility, reproduction, and sexuality. Knowledge of these concepts can help the nurse anticipate areas that need to be addressed. Culture is a concept that is considered to significantly affect development; the difference is the concepts that affect development are those that represent major influencing factors (causes), hence determination of development and would be the focus of preventive interventions. Environment is considered to significantly affect development. Nutrition is considered to significantly affect development.

The most appropriate response of the nurse when a mother asks what the Denver II does is that it A can diagnose developmental disabilities. B identifies a need for physical therapy. C is a developmental screening tool. D provides a framework for health teaching.

C is a developmental screening tool. The Denver II is the most commonly used measure of developmental status used by health care professionals; it is a screening tool. Screening tools do not provide a diagnosis. Diagnosis requires a thorough neurodevelopment history and physical examination. Developmental delay, which is suggested by screening, is a symptom, not a diagnosis. The need for any therapy would be identified with a comprehensive evaluation, not a screening tool. Some providers use the Denver II as a framework for teaching about expected development, but this is not the primary purpose of the tool.

Essential elements of a standard order set to verify a medication order include A volume only. B number of tablets. C metric dose/strength. D hour of administration

C metric dose/strength. The ISMP recommendations for standardized medication order sets include such elements as the drug name (generic followed by brand when appropriate), metric dose/strength, frequency and duration, route, and indication. Although a prescription may include volume or number of tablets, the essential component is dose or strength, because the volume or number of tablets may vary by manufacturer. The exact hour of administration can be based on factors such as the frequency, agency protocols, and patient preferences.

The nurse preparing a teaching plan for a preschooler knows that, according to Piaget, the expected stage of development for a preschooler is A concrete operational. B formal operational. C preoperational. D sensorimotor.

C preoperational. The expected stage of development for a preschooler (3 to 4 years old) is preoperational. Concrete operational describes the thinking of a school-age child (7 to 11 years old). Formal operational describes the thinking of an individual after about 11 years of age. Sensorimotor describes the earliest pattern of thinking from birth to 2 years old.

A 17-year-old girl is hospitalized for appendicitis, and her mother asks the nurse why she is so needy and acting like a child. The best response of the nurse is that in the hospital, adolescents A have separation anxiety. B rebel against rules. C regress because of stress. D want to know everything

C regress because of stress. Regression to an earlier stage of development is a common response to stress. Separation anxiety is most common in infants and toddlers. Rebellion against hospital rules is usually not an issue if the adolescent understands the rules and would not create childlike behaviors. An adolescent may want to know everything with their logical thinking and deductive reasoning, but that would not explain why they would act like a child.

The nurse is assigned a group of patients. Which patient would the nurse identify as being at increased risk for impaired gas exchange? A patient A with a blood glucose of 350 mg/dL B who has been on anticoagulants for 10 days C with a hemoglobin of 8.5 g/dL D with a heart rate of 100 beats/min and blood pressure of 100/60

C with a hemoglobin of 8.5 g/dL The hemoglobin is low (anemia), therefore the ability of the blood to carry oxygen is decreased. High blood glucose and/or anticoagulants do not alter the oxygen carrying capacity of the blood. A heart rate of 100 beats/min and blood pressure of 100/60 are not indicative of oxygen carrying capacity of the blood.

Which medications are classified as nonsteroidal antiinflammatory medications? (Select all that apply.) a. Tramadol b. Acetaminophen c. Aspirin d. Ibuprofen e. Codeine

C, D Aspirin and ibuprofen are examples of nonsteroidal antiinflammatory (NSAID) medications. Codeine, acetaminophen, and tramadol are not NSAIDs.

Which assessment findings prevent the nurse from starting an IV in the patient's right arm? (Select all that apply.) a. The patient is right-handed. b. The IV pole will be on the left side of the bed. c. The patient has a dialysis fistula in the right arm. d. The patient has right-sided paralysis after a stroke. e. The patient had a right mastectomy 6 years ago.

C, D, E Dialysis fistula, history of mastectomy, and paralysis all prevent the nurse from starting the patient's IV in the right arm. The patient's preferences for the right hand and IV pole position have no bearing on where to start the IV.

Which are the appropriate interventions for the patient with the diagnosis impaired bed mobility related to hemiplegia after stroke? (Select all that apply.) a. Assist the patient to reposition in the bed at least every 3 hours. b. Maintain the patient's bed in high Fowler's position whenever possible. c. Instruct the patient how to use the over-bed trapeze to reposition self in bed. d. Utilize a sliding sheet to minimize skin trauma during repositioning of the patient. e. Position the patient's affected limbs in neutral alignment between ranges of motion.

C, D, E The patient should be taught how to use the over-bed trapeze to reposition self in bed to encourage independence and frequent repositioning. A sliding sheet will minimize skin trauma during repositioning of the patient. The patient's limbs should be positioned in neutral alignment between ranges of motion to prevent contractures. The patient should be repositioned at least every 2 hours. Maintaining the bed in high Fowler's position will facilitate the development of pressure injuries on the patient's coccyx, sacrum, and back

A patient is admitted from the emergency department. The nurse notices that the patient and family are anxious. During the admission process the nurse should do which of the following? (Select all that apply.) a.Continue with the examination even if the patient is anxious to get it done. b.Disregard cultural differences to gather therapeutic data. c.Provide a thorough explanation of the purpose of each assessment. d.Provide a thorough explanation of the steps of each assessment. e.Maintain a formal professional approach.

C,D

Two common abnormalities in heart rate are: a.dysrhythmia. b.apical pulse. c.tachycardia. d.bradycardia. e.pulse deficit.

C,D

Which information indicates the nurse has an accurate understanding of the State Nurse Practice Act? a.It is a federal senate bill. b.It is a law enacted by the federal government. c.It is a statute enacted by state legislature. d.It is a judicial decision.

C- .It is a statute enacted by state legislature. Nurse Practice Acts are examples of statutes enacted by state legislatures to regulate the practice of nursing. Common laws are based on judicial decisions or case law precedent. An example of a judicial decision that guides health care practice is Roe v. Wade , but not the nurse practice act . An example of a federal statute that affects health care practice is the Americans with Disabilities Act, but not the nurse practice act. The nurse practice act is a state law, not a federal senate bill.

An 89-year-old patient who lives in a nursing home has been admitted to the hospital for observation after falling, and is exhibiting confusion and malaise in the nursing home. He had a urinary catheter inserted 2 weeks ago when he complained of difficulty urinating. Lab work was ordered and the nurse notes that his neutrophil count is elevated. She knows that this, combined with the other clinical signs and symptoms, most likely indicates what condition? a.Tuberculosis b.Parasitic infection c.Acute bacterial infection d.Viral infection

C- Acute bacterial infection Neutrophil counts are elevated in the presence of acute bacterial infection. Lymphocytes are increased in viral infections. Monocytes are elevated in tuberculosis infections. Eosinophils are usually elevated in parasitic infections

A patient does not want the treatment that was prescribed. The nurse helps the patient talk to the primary health care provider and even talks to the primary health care provider when needed. The nurse is acting in which professional role? a.Educator b.Manager c.Advocate d.Provider of care

C- Advocate As an advocate you act on behalf of your patient, securing and standing up for your patients health care rights. As an educator you explain concepts and facts about health, describe the reason for routine care activities, demonstrate procedures such as self-care activities, reinforce learning or patient behavior, and evaluate the patients progress in learning. Most nurses provide direct patient care in an acute care setting, and this describes the role of provider of care. A manager coordinates the activities of members of the nursing staff in delivering nursing care and has personnel, policy, and budgetary responsibility for a specific nursing unit or agency

The nurse is assessing an older adult who has smoked for the past 47 years and has been diagnosed with chronic obstructive pulmonary disease. Which of the following would the nurse expect to see upon physical examination of the patient? a.Pallor and hardening of the nail bed b.Jaundice in the nail bed c.An angle greater than 160 degrees between the nail and nail bed d.An angle less than 160 degrees between the nail and nail bed

C- An angle greater than 160 degrees between the nail and nail bed Inspection of the angle between the nail and nail bed normally reveals an angle of 160 degrees. A larger angle and softening of the nail bed indicate chronic oxygenation problems. An ongoing bluish or purplish cast to the nail bed occurs with cyanosis. A white cast or pallor results from anemia. Observe for jaundice (yellow-orange), which indicates liver disease.

A 36-year-old African-American patient has been admitted to the hospital with diabetic ketoacidosis. The nurse who is admitting him notes that his blood pressure is 164/98. Which of the following should the nurse do next? a.Call the health care provider because the patients values differ from the standard range. b.Immediately call the health care provider and request antihypertensive medication. c.Ask the patient what his blood pressure normally measures for comparison. d.Do nothing; this is within a normal range for a patient with diabetic ketoacidosis

C- Ask the patient what his blood pressure normally measures for comparison. Know the patients usual range of vital signs. A patients usual values sometimes differ from the standard range for that age or physical state

A senior nursing student is doing her community clinical rotation. When visiting a young family to whom she has been assigned, the mother of a 3-year-old child states that her daughter does not feel well. The nursing student feels her skin, which is warm. She asks the mother if she has taken her temperature to which the mother replies, Yes, I used the same thermometer that was my great-grandmothers; it has been used by my family for years. Her oral temperature was 102.3 F. The most important action for the nursing student to perform is to do which of the following? a.Teach that temporal artery thermometers are more accurate than others. b.Tell the mother that hospitals still use mercury thermometers. c.Ask to see the thermometer. d.Recommend a chemical thermometer for greater accuracy

C- Ask to see the thermometer. Inspect the thermometer to make sure that it is mercury. The mercury-in-glass thermometers are obsolete in the health care setting because of the environmental hazards of mercury

The nurse is admitting a patient to the unit and asks the patient about the health history. The nurse is engaged in which component of the nursing process? a.Evaluation b.Diagnosis c.Assessment d.Planning

C- Assessment The nurse is in the assessment phase. An assessment database includes a patients comprehensive health history, which includes information about a patients physical and developmental status, emotional health, social practices and resources, goals, values, lifestyle, and expectations about the health care system. The database also includes physical examination findings and a summary of results from laboratory and diagnostic testing. A nursing diagnosis is a clinical judgment about individual, family, or community responses to actual and potential health problems or life processes that the nurse is licensed and competent to treat. Evaluation is crucial to deciding whether, after interventions have been delivered, a patients condition or well-being improves. Planning involves setting priorities, identifying patient-centered goals and expected outcomes, and prescribing nursing interventions.

As a nurse is obtaining a health history from a patient, the nurse uses comments such as go on. Which technique is the nurse using? a.Cues b.Inferences c.Back-channeling d.Termination

C- Back-channeling This is known as back-channeling, which is the practice of giving positive comments such as all right, go on, or uhhuh to the speaker. These indicate that a nurse has heard what the patient says and is attentive to hear the full story. A cue is information that you obtain through use of the senses. An inference is your judgment or interpretation of these cues. Termination is the last phase of the interview in which the interview comes to an end.

A 38-year-old postoperative patient is suddenly unresponsive but is still breathing. The nurse will use which site to assess the patients pulse? a.Apical artery b.Radial artery c.Carotid artery d.Brachial artery

C- Carotid artery When a patients condition suddenly deteriorates, use the carotid site to quickly locate a pulse

.An RN has been caring for a patient. The nurse received an erroneous order for a medication. The primary health care provider has a reputation for impatience and irritability. Knowing this health care providers nature, which action by the nurse would be most appropriate? a.Clarify the order with the pharmacy. b.Ask the patient to remember. c.Clarify the order with the primary health care provider. d.Ask another nurse to look at the order to try to clarify it.

C- Clarify the order with the primary health care provider. A nurse will assess all physician or health care provider orders, and if the nurse determines they are erroneous or harmful, obtain clarification from that physician or health care provider. Calling pharmacy, asking the patient, and asking another nurse are not the best ways to handle erroneous orders.

Which task can a nurse safely delegate to a student nurse who is working as a nursing assistant? a.Distributing medications to patients b.Administering insulin injections c.Collecting intake and output data d.Assessing patients

C- Collecting intake and output data During the time when a student nurse works as an employee of a health care facility, perform only tasks that appear in a job description for a nurses aide or nursing assistant. For example, even if a student nurse has learned how to administer intramuscular medications, do not perform this task as a nurses aide

A nursing assistant asks the nurse why she needs to bathe a febrile patient. The best response is that this intervention increases heat loss through which of the following? a.Convection b.Radiation c.Conduction d.Evaporation

C- Conduction Heat loss occurs through conduction, which is the transfer of heat from one object to another with direct contact.

A 56-year-old grandmother has been admitted to the hospital with a fever of 103.2 F after caring for her 5-yearold granddaughter who also developed a fever. The health care provider has ordered blood cultures, antibiotics, antipyretics, a clear liquid diet, and a chest radiograph. Which of the orders should the nurse do first? a.Administer antibiotic. b.Administer antipyretic. c.Draw blood cultures. d.Apply water cooled blankets.

C- Draw blood cultures Before antibiotic therapy, obtain blood cultures when ordered. Obtain blood specimens at the same time as a temperature spike, when the causative organism is most prevalent.

A nursing student is working on a surgical unit in the hospital. Included in her job description is to assist in the cleaning and disinfection of equipment stored on the unit. Which of the following is the best explanation of disinfection? a.Removing organic material b.Removing inorganic material c.Eliminating almost all pathogenic organisms d.Destroying all forms of microbial life

C- Eliminating almost all pathogenic organisms Disinfection eliminates almost all pathogenic organisms, with the exception of bacterial spores

The patient recently had a cast applied to his left lower leg after a football injury in which he broke his tibia. After the application of the cast, the nurse felt the toes of his left and right feet. To assess circulation in the left foot, the nurse should do which of the following? a.Use the tips of her fingers to assess temperature. b.Expect that the temperature in the left leg will be lower than that in the right. c.Expect that the temperature of the left foot to be the same as the right foot. d.Expect that the left foot will be warmer than the left.

C- Expect that the temperature of the left foot to be the same as the right foot. Compare symmetrical body parts, which should be the same in assessment. Always assess skin temperature for patients at risk for impaired circulation, such as after a cast application or vascular surgery. Normally the skin temperature is warm. Skin temperature is the same throughout the body. Accurately assess temperature by palpating the skin with the dorsum , or back, of the hand

A registered nurse who works on an oncology unit discussed pain control options that the primary health care provider had ordered with a patient undergoing treatment for pancreatic cancer. The patient requested that the intravenous (IV) pain medication be given on a regular basis. The nurse agreed to provide the IV pain medication as requested and continued to reevaluate the pain levels. The nurse is following which ethical principle? a.Autonomy b.Justice c.Fidelity d.Nonmaleficence

C- Fidelity Fidelity refers to the agreement to keep promises. The principle of fidelity also promotes the obligation of a nurse to follow through with the care offered to patients. Autonomy refers to independence and self-determination, which is what the patient followed, but the question asked for which principle the nurse followed. Justice refers to fairness or equity of health care resources. Nonmaleficence refers to the fundamental agreement to do no harm.

Which of the following situations is most likely to contribute to a health careacquired infection? a.A closed urinary drainage system b.Use of aseptic technique during dressing changes c.Foley catheter drainage bag touching the floor d.Changing IV access site when site is red and warm

C- Foley catheter drainage bag touching the floor Causes of health careacquired infection include contact between drainage bag port and contaminated surface, an open urinary drainage system, failure to use aseptic technique during dressing changes, and failure to change IV access site when inflammation first appears.

A nurse wants to follow nursing standards of care. Which document should the nurse follow? a.World Health Organization guidelines b.National League for Nursing brochure c.Health care facilitys written procedure manual d.Department of Health and Human Services guidelines

C- Health care facilitys written procedure manual The health care facilitys written procedure manual is defined as a standard of care. Standards of care are defined by the following: (1) State Nurse Practice Acts, (2) state and federal hospital licensing laws and accreditation rules, (3) professional and specialty organizations, and (4) written policies and procedures of the nurses health care facility. Brochures are not standards of care. World Health Organization and Department of Health and Human Services are not state or federal hospitals or professional and specialty organizations for nurses

An adolescent is examined and told that she has a sexually transmitted disease (STI). The patient is shocked and states, How can that be. I would know if I had a problem down there wouldnt I? What is the most appropriate response by the nurse? a.You should have known. Its pretty hard to miss. b.Usually STIs show up within 2 to 3 days of exposure. c.If youd like, we can teach you some self-examination techniques. d.Its just bad luck. Well clear this up and that will be the end of it.

C- If youd like, we can teach you some self-examination techniques Patients who are at risk for contracting STIs need to learn to perform a genital self-examination. The purpose of the examination is to detect any signs or symptoms of STIs

A patient has lost 10 pounds in the last 2 months from breast cancer and chemotherapy. The chemotherapy has caused the patient to not eat. Which nursing diagnosis should the nurse use to develop the plan of care? a.Imbalanced Nutrition: Less Than Body Requirements Related to Decreased Food Intake b.Imbalanced Nutrition: Less Than Body Requirements Related to Cancer c.Imbalanced Nutrition: Less Than Body Requirements Related to Loss of Weight d.Imbalanced Nutrition: Less Than Body Requirement Related to Insufficient Prescription of Chemotherapy

C- Imbalanced Nutrition: Less Than Body Requirements Related to Decreased Food Intake

A nurse is delegating care of patients to the nursing assistant personnel (NAP) and a licensed practical nurse (LPN). Which situation indicates the nurse needs more instruction on delegation? a.LPN to change a sterile dressing b.NAP to provide skin care c.NAP to insert an indwelling catheter d.LPN to administer an enema

C- NAP to insert an indwelling catheter The question indicates the nurse made an incorrect delegation assignment. An NAP cannot insert indwelling catheter, an LPN or RN can do that skill

An older adult African-American woman has gone to the clinic where a RN volunteers twice a week. She is a diabetic and has some skin breakdown on the calf of her right leg. Her skin is very darkly pigmented. To best examine the patients skin, the nurse should use which of the following? a.Halogen lighting b.Artificial warming to increase room temperature c.Natural sunlight d.Air conditioning to lower room temperature

C- Natural sunlight The recommended light is natural sunlight, with halogen lighting being another option. Sunlight is the best choice for detecting skin changes in patients with darker skin. A room that is too warm causes superficial vasodilation, resulting in an increased redness of the skin. Patients who become too cold by air conditioning can develop cyanosis (bluish color) around the lips and nail beds.

A 2-year-old patient is being admitted to the outpatient surgery for a tonsillectomy. Which will provide the best primary source of information for what comforts the patient when stressed? a.Patient chart b.Patient c.Parents d.Surgeon

C- Parents

On examination of an 18-month-old child, the nurse found the right eardrum pink and bulging. How should a normal eardrum appear? a.Pink b.Red c.Pearly gray d.White

C- Pearly gray The normal eardrum is translucent, shiny, and pearly gray. It is free from tears or breaks. A pink or red bulging membrane indicates inflammation. A white color reveals pus behind it

A nurse is preparing to perform a physical examination on a patient who has mobility issues. In preparing for the examination, the nurse should do which of the following? a.Be sure that a well-equipped examination room is available. b.Tune the radio to the nurses favorite station to relax the patient. c.Perform thorough hand hygiene before preparing equipment. d.Instruct the patient on the safest way to transfer onto the examination table

C- Perform thorough hand hygiene before preparing equipment. Perform hand hygiene before equipment preparation and the examination. A well-equipped examination room is preferable, but often the examination occurs in the patients room

.A home health nurse is providing care to a patient. Which action by the nurse is a physical care technique? a.Dressing a patient b.Assisting a patient to learn how to shop c.Performing range-of-motion exercises d.Administering cardiopulmonary resuscitation

C- Performing range-of-motion exercises Physical care techniques involve the safe and competent administration of nursing procedures (e.g., inserting a urinary catheter, performing range-of-motion exercises). Dressing a patient is an activity of daily living. Shopping is an instrumental activity of daily living. Cardiopulmonary resuscitation is a lifesaving measure

A 63-year-old welder who has gone to the clinic for an annual checkup. The patient shares a concern regarding difficulty hearing conversations at the coffee shop in the mornings. After looking in his ears to determine if there is a build-up of cerumen, the nurse tells the patient that the hearing loss may be associated with his occupation or it may be associated with aging. The nurse is aware that hearing loss associated with the aging process is known as which of the following? a.Tinnitus b.Mnires disease c.Presbycusis d.Presbyopia

C- Presbycusis Hearing changes often associated with aging include decreased hearing acuity, speech intelligibility, and pitch discrimination, which is referred to as presbycusis. Low-pitched sounds are easiest to hear, but it is difficult to hear conversation over background noise. A decrease in active sebaceous glands causes the cerumen to become dry and completely obstruct the external auditory canal. Tinnitus is commonly caused by ototoxicity and patients experience the sensation of ringing in the ears. Presbyopia refers to the gradual decline in ability of the lens to accommodate or focus on close objects and reduces ability to see near objects clearly. Although the cause of Mnires disease is unknown the symptoms include progressive low-frequency hearing loss, vertigo, tinnitus, and a full feeling or pressure in the affected ear.

The young mother of an 8-month-old patient brought her daughter to the clinic after taking the little girls temperature rectally and obtaining a reading of 100.4 F. The mother was concerned that her daughter might be ill. Which of the following is the best response? a.Children usually run lower rather than higher temperatures when ill. b.Because of her age, it is probably a bacterial infection. c.Rectal temperatures are higher than temperatures obtained orally. d.When taking multiple temperatures, the sites should be rotated

C- Rectal temperatures are higher than temperatures obtained orally Depending on the site, temperatures will normally vary between 36 C (96.8 F) and 38 C (100.4 F). It is generally accepted that rectal temperatures are usually 0.5 C (0.9 F) higher than oral temperatures. Children have immature temperature control mechanisms, so temperatures sometimes rise rapidly. Most fevers in children are of viral origin, lasting only briefly, and have limited effects. Use the same site when repeated measurements are needed

A senior nursing student is working on a community health project for the local homeless shelter. There are several indigent men who come to the shelter in cold weather to sleep for the night. The student nurse knows that these men do not bathe on a regular basis. One of the men has been sick several times recently with skin infections. Which of the following is the best way for the student nurse to explain the importance of personal hygiene to this individual? a.You dont have to shower every day. You only need to take a shower when you feel like youre going to be sick. b.Take a shower. If you dont take a shower, you will continue to get sick. c.Showering regularly will remove germs that cause skin infections. What do you think we should do about these skin infections of yours? d.Showering with warm water is enough to wash away bacteria. Soap is not needed if you dont like it.

C- Showering regularly will remove germs that cause skin infections. What do you think we should do about these skin infections of yours? Identify patients expectations about their care, and involve them in planning their care. Encourage patients to verbalize their expectations so that you are able to establish interventions to meet patients priorities

A student nurse is responsible for assessing a patient, who is abrupt and requests that the assessment be done later by a nurse. As the student nurse charts the interaction, which statement is the best way to document what happened? a.Appears to be in pain as evidenced by grouchy behavior b.Behavior is inappropriate, requests registered nurse do the assessment c.States, I want a registered nurse to do my assessment d.Is grumpy, registered nurse notified

C- States, I want a registered nurse to do my assessment When a nurse collects objective data, he or she should apply critical thinking intellectual standards (e.g., clear, precise, and consistent). Nurses do not include personal interpretive statements. The timely, thorough, and accurate communication of facts is necessary to ensure continuity and appropriateness of patient care. If you do not report or record an assessment finding or problem interpretation, it is lost and unavailable to anyone else caring for the patient. Grouchy, inappropriate, and grumpy are personal interpretive statements and should be avoided

A registered nurse is caring for a patient 2 days after a colon resection. The patient called for assistance to go to the bathroom. Instead of waiting for help, the patient decided to get up without help. The patient fell but was not injured. After contacting the patients primary health care provider, which action should the nurse take next? a.Nothing; the patient was not injured. b.Call the ethics committee. c.Submit an incident report. d.Insist that the patient have a radiograph done

C- Submit an incident report. When there is a deviation from the standard of care, such as a patient or visitor falls or an error is made, a nurse makes specific documentation of the event or incident in the form of an occurrence/incident report. The nurse should complete an occurrence report when anything unusual happens that could potentially cause harm to a patient, visitor, or employee. Just because the patient was not injured does not mean the report can be neglected. The health care provider orders follow-up care or treatment when necessary, not the nurse. The ethics committee is involved in ethical dilemmas, not occurrence/incident reports.

A nurse uses effective strategies to communicate and handle conflict with nurses and other health care professionals. Which Quality and Safety Education for Nurses (QSEN) competency is the nurse demonstrating? a.Informatics b.Quality improvement c.Teamwork and collaboration d.Evidence-based practice

C- Teamwork and collaboration uses effective strategies to communicate and handle conflict. Informatics includes navigating electronic health records. Quality improvement uses tools such as flow charts and diagrams to improve care. Evidence-based practice integrates best current evidence with clinical expertise and patient/family preferences and values for delivery of optimal health care.

Which question or comment should the nurse initially use that would best gather the most information during a health history assessment? a.Let us help you. b.Did you seek help when it first started? c.Tell me about the problems you are having. d.Do you have a family history of this problem?

C- Tell me about the problems you are having. Initially use open-ended questions/comments. The use of open-ended questions/comments prompts patients to describe a situation in more than one or two words. This technique leads to a discussion in which patients actively describe their health status. Once patients tell their story, focus on the symptoms that the patient identifies and ask closed-ended questions that limit his or her answers to one or two words such as yes or no or a number or frequency of a symptom. The questions that start with Do and Did are closed-ended. Let us help you will not get the patients perspective

A registered nurse was accused of patient abandonment when the nurse became angry, quit the job, and left the hospital before the end of the shift. This is an example of violating legal standards/guidelines set by which organization? a.The State Department of Health b.The Joint Commission c.The State Board of Nursing d.The National League for Nursing

C- The State Board of Nursing Nurse Practice Acts permit the State Board of Nursing to set rules, regulations, and guidelines that specifically define the standard of care in nursing practice. An example is the guidelines that define patient abandonment. The State Department of Health, the Joint Commission, and the National League for Nursing do not set the legal rules and regulations for patient abandonment.

Upon a patients admission to the nursing unit, the registered nurse delegated to the nursing assistive personnel to take her vital signs. What is the registered nurses responsibility regarding delegating this task? a.This is inappropriate delegation; the nurse should always take the vital signs. b.Have the NAP repeat the measurement if vital signs appear abnormal. c.The nurse should review and interpret the vital sign measurements. d.This task has been delegated so the nurse is not responsible

C- The nurse should review and interpret the vital sign measurements.

A mother of five children is admitted to the hospital for abdominal pain. The nurse asks a series of questions before performing a physical assessment. The patient answers the questions. When asking the patient some other questions, the patients spouse starts to answer. As the admission process progresses and the nurse gathers subjective data, the nurse requests that the patient answer the next questions. What is the rationale for the nurses behavior? a.The patient is exhibiting confusion. b.The spouse is being obnoxious. c.The patient is the best source of information. d.The spouse is too controlling

C- The patient is the best source of information. A patient is usually the best source of information. A patient who is alert and answers questions appropriately provides the most accurate information about health care needs, lifestyle patterns, present and past illnesses, perception of symptoms, and changes in activities of daily living. There is no evidence in the scenario to indicate confusion on the patients part or that the spouse was obnoxious or too controlling. The nurse needs more data before saying the spouse is obnoxious or controlling.

The student nurse who is developing a plan of care for a postoperative patient who underwent abdominal surgery to remove a tumor. The student has chosen Risk for Infection as a nursing diagnosis. Which of the following is the most appropriate goal for this diagnosis? a.The patients wound drainage will decrease in 2 days. b.The patient will report decrease in incisional pain by discharge. c.The progression of infection will be controlled or decreased. d.The patient will describe signs/symptoms of wound infection.

C- The progression of infection will be controlled or decreased In an acute care setting the goal for the diagnosis Risk for Infection is to control or decrease the progression of infection

A community health nurse states, I wish we had just a portion of the dollars spent repairing atherosclerotic hearts to teach the community about cardiovascular risk factors. The nurses statement stems from what philosophy? a.Deontology b.Feminist ethic c.Utilitarianism d.Ethics of care

C- Utilitarianism Utilitarianism determines the value of something based primarily on its usefulness. Deontology defines actions as right or wrong according to principles. The feminist ethic asks how ethical decisions will affect women. The ethics of care suggests that health care workers solve ethical dilemmas by the promotion of the fundamental act of caring.

A nurse bases ethical decisions on the effect, or consequences, an act will have and uses the following guidelines: the greatest good for the greatest number of people. Which ethical system is the nurse using? a.Legal b.Deontology c.Utilitarianism d.Ethics of care

C- Utilitarianism Utilitarianism guides us to measure the effect, or consequences, that an act will have. The greatest good for the greatest number of people is the guiding principle for action in this system. By comparison, deontology focuses less on consequences and looks to the presence of pure principles of autonomy, justice, fidelity, beneficence, and nonmaleficence. Ethical issues differ from legal issues. Legal issues are resolved by reference to laws that tend to be concrete and publicly determined. Ethics of care suggest that health care workers resolve ethical dilemmas by paying attention to relationships and stories of the participants and by promoting a fundamental act of caring

The patient has a methicillin-resistant Staphylococcus aureus (MRSA) infection in an abdominal surgical wound. The patient is in a private room, is receiving vancomycin (Vancocin) for the MRSA, and pain is well controlled with a morphine sulfate patient-controlled analgesia (PCA) pump, and is receiving docusate sodium (Colace) to prevent constipation. During the nurses rounds, the patient begins complaining of ringing in the ears. Which is the most likely cause for the patients tinnitus? a.Surgical anesthesia b.Morphine sulfate c.Vancomycin d.Docusate sodium

C- Vancomycin Ototoxic medications, such as analgesics, antibiotics (such as vancomycin and aminoglycosides), or diuretics, affect hearing acuity, balance, or both, with the most common symptom being tinnitus (ringing in the ears). Surgical anesthesia, morphine, and docusate sodium do not have the side effect of ototoxicity or tinnitus.

An emergency department nurse is caring for a patient with a laceration on the lower leg that has become infected. On assessment the nurse realizes that a sign of an inflammatory response consists of: a.wound blanching b.coolness at the site of injury. c.a vascular reaction that delivers fluid, blood, and nutrients to the area. d.decreased pain sensation

C- a vascular reaction that delivers fluid, blood, and nutrients to the area. Inflammation is a protective vascular reaction that delivers fluid, blood products, and nutrients to interstitial tissues in an area of injury. This process neutralizes and eliminates pathogens or necrotic tissues and establishes a means of repairing body cells and tissues. Signs of inflammation include swelling, redness (not blanching), heat, pain or tenderness, and loss of function in the affected body part.

A nurse must ask a family member to consider an organ donation. In which order should the nurse contact the individuals? a. Spouse b. Parent c. Guardian d. Grandparent e. Adult son or daughter f. Adult brother or sister a.a, c, e, f, b, d b.a, e, f, b, d, c c.a, e, b, f, d, c d.a, b, e, f, d, c

C- a, e, b, f, d, c You approach individuals in the following order to consider organ or tissue donations: (1) spouse, (2) adult son or daughter, (3) parent, (4) adult brother or sister, (5) grandparent, and (6) guardian

The nurse is assessing a patients functional ability. Which activities most closely match the definition of functional ability? A Healthy individual, works outside the home, uses a cane, well groomed B Healthy individual, college educated, travels frequently, can balance a checkbook C Healthy individual, works out, reads well, cooks and cleans house D Healthy individual, volunteers at church, works part time, takes care of family and house

D Healthy individual, volunteers at church, works part time, takes care of family and house Functional ability refers to the individuals ability to perform the normal daily activities required to meet basic needs; fulfill usual roles in the family, workplace, and community; and maintain health and well-being. The other options are good; however, each option has advanced or independent activities in the context of the option.

The nurse is developing an interdisciplinary plan of care using the Roper-Logan-Tierney Model of Nursing for a patient who is currently unconscious. Which interventions would be most critical to developing a plan of care for this patient? A Eating and drinking, personal cleansing and dressing, working and playing B Toileting, transferring, dressing, and bathing activities C Sleeping, expressing sexuality, socializing with peers D Maintaining a safe environment, breathing, maintaining temperature

D Maintaining a safe environment, breathing, maintaining temperature The most critical aspects of care for an unconscious patient are safe environment, breathing, and temperature. Eating and drinking are contraindicated in unconscious patients. Toileting, transferring, dressing, and bathing activities are BADLs. Sleeping, expressing sexuality, and socializing with peers are a part of the Roper-Logan-Tierney Model of Nursing; however, these are not the most critical for developing the plan of care in an unconscious patient.

Which clinical management prevention concept would the nurse identify as representative of secondary prevention? A Decreasing venous stasis and risk for pulmonary emboli B Implementation of strict hand washing routines C Maintaining current vaccination schedules D Prevention of pneumonia in patients with chronic lung disease

D Prevention of pneumonia in patients with chronic lung disease Prevention of and treatment of existing health problems to avoid further complications is an example of secondary prevention. Primary prevention includes infection control (hand washing), smoking cessation, immunizations, and prevention of postoperative complications.

The nurse is caring for a patient experiencing an allergic reaction to a bee sting who has an order for BenaDRYL. The only medication in the patients medication bin is labeled BenaZEPRIL. The nurse contacts the pharmacy for the correct medication to avoid what type of error? A Communication B Diagnostic C Preventive D Treatment

D Treatment The nurse avoided a treatment error, giving the wrong medication. Benazepril is an ace inhibitor used to treat blood pressure. According to Leape, treatment errors occur in the performance of an operation, procedure, or test; in administering a treatment; in the dose or method of administering a drug; or in avoidable delay in treatment or in responding to an abnormal test. Communication errors refer to those that occur from a failure to communicate. Diagnostic errors are the result of a delay in diagnosis, failure to employ indicated tests, use of outmoded tests, or failure to act on results of monitoring or testing. Preventive errors occur when there is inadequate monitoring or failure to provide prophylactic treatment or follow-up of treatment.

The nurse is talking to the unlicensed assistive personnel about moving a patient in bed. The nurse knows the unlicensed assistive personnel understands the concept of mobility and proper moving techniques when he or she states, Patients must A have a trapeze over the bed to move properly. B move themselves in bed to prevent immobility. C always have a two-person assist to move in bed. D be moved correctly in bed to prevent shearing.

D be moved correctly in bed to prevent shearing Patients must be moved properly in bed to prevent shearing of the skin. Having a trapeze over the bed is only functional is the patient can assist in the moving process. A two-person assist is good, but the patient still needs to be moved properly. A patient may move himself or herself if he or she is able, but shearing may still occur.

To promote safety, the nurse manager sensitive to point of care (sharp end) and systems level (blunt end) exemplars works closely with staff to address the point of care exemplars such as A care coordination. B documentation. C electronic records. D fall prevention.

D fall prevention. The most common safety issues at the sharp end include prevention of decubitus ulcers, medication administration, fall prevention, invasive procedures, diagnostic workup, recognition of/action on adverse events, and communication. These are the most common issues the staff nurse providing direct patient care encounters. Each of the other options are classified as systems level exemplars.

The school nurse talking with a high school class about the difference between growth and development would best describe growth as A processes by which early cells specialize. B psychosocial and cognitive changes. C qualitative changes associated with aging. D quantitative changes in size or weight.

D quantitative changes in size or weight. Growth is a quantitative change in which an increase in cell number and size results in an increase in overall size or weight of the body or any of its parts. The processes by which early cells specialize are referred to asdifferentiation. Psychosocial and cognitive changes are referred to as development. Qualitative changes associated with aging are referred to as maturation.

A sentinel event refers to an event that A could have harmed a patient, but serious harm didnt occur because of chance. B harms a patient as a result of underlying disease or condition. C harms a patient by omission or commission, not an underlying disease or condition. D signals the need for immediate investigation and response.

D signals the need for immediate investigation and response. A sentinel event is an unexpected occurrence involving death or serious physical or psychologic injury or the risk thereof called sentinel, because they signal the need for immediate investigation and response. A near-miss refers to an error or commission or omission that could have harmed the patient, but serious harm did not occur as a result of chance. Harm that relates to an underlying disease or condition provides the rationale for the close monitoring and supervision provided in a health care setting. An adverse event is one that results in unintended harm because of the commission or omission of an act.

The strategy to avoid medication errors endorsed by the Institute for Safe Medication Practices (ISMP) to differentiate products with look-alike names is referred to as A automatic alerts. B bar coding. C computer order entry. D tallman lettering.

D tallman lettering. Tallman lettering is a term coined by ISMP to describe the practice of using unique letter characteristics of similar drug names known to have been confused with one another. Tallman lettering is used to differentiate products with look-alike names such as BenaDRYL (antihistamine) and BenaZEPRIL (ace inhibitor). The other options are examples of safety- enhancing technologies strategies designed to minimize drug errors, but they are not directed at look-alike medications. Automatic alerts are computer-generated alarms that can be programmed to occur with such things as allergies and incompatible medications. Bar coding is used with medication administration systems that can be programmed to match patient identification bracelets with documentation. Computer order entry systems are designed to include components of a standard medication order.

Core temperatures are obtained via which of the following? (Select all that apply.) a.Axillae b.Rectal c.Oral d.Pulmonary artery e.Urinary bladder

D, E

Which actions of the nurse demonstrate correct administration of a soapsuds enema? (Select all that apply.) a. The enema is administered while the patient is in the right Sims' position. b. Liquid antibacterial soap is added to the enema bag before administration. c. The tip of the enema tube is lubricated with petroleum jelly before insertion. d. The enema bag is lowered when the patient reports abdominal cramping. e. The nurse removes the patient's fecal impaction before administering the enema.

D, E The enema bag is lowered when the patient complains of abdominal cramping to slow the rate and reduce discomfort. Fecal impactions should be removed before administration of soapsuds enemas. The tip of the enema tube should be lubricated with water-soluble jelly rather than petroleum jelly. Only castile soap may be used for soapsuds enemas. Liquid antibacterial soap can cause damage to the colon. The enema should be administered while the patient is in the left Sims' position.

Which statements demonstrate that the patient is at the acceptance stage of learning? (Select all that apply.) a. "I do not have to learn how to do the dressing. My wife will do it for me." b. "I feel like such a failure for not consulting a podiatrist earlier about my foot." c. "I'll try to do the exercises you described if you will give me a cookie afterward." d. "I want to learn how to do this myself so I do not have to go to a rehab center." e. "I know that I have to give myself the injections because I could get a blood clot."

D, E The patient indicates acceptance by wanting to learn and understanding the importance of the teaching. Referring the care to the spouse, feeling like a failure, and wanting rewards do not demonstrate acceptance.

Which chart entry by a nurse would require follow up? a.0815 Patient found on floor. b.0816 Patient assessed and helped back to bed. c.0818 Physician notified of incident. d.0820 Occurrence report completed.

D- 0820 Occurrence report completed Do not document in the nurses notes that an occurrence report was completed . All the other entries are accurate. Objectively record the details of the event and any statements the patient makes. At the time of the event, always assess the patient thoroughly, and then contact the health care provider to examine him or her.

At a point during the admission process of a young female patient, the nurse asks the patient about whether or not she performs self-breast examinations. The patient asks about the best time of the month to perform this examination. What is the best response? a.Usually the first day of your menstrual period is best. b.At the same time every month. c.If youre pregnant there is no need to do it. d.A few days after your menstrual period ends.

D- A few days after your menstrual period ends. The best time for BSE is when the breasts are not tender or swollen, usually a few days after a menstrual period ends. If the woman is postmenopausal, advise her to check her breasts on the same day each month. A pregnant woman should also check her breasts on a monthly basis.

The nurse is working for a postsurgical unit. He is caring for four postsurgical patients, all of whom have been in the hospital for 3 days or more. Which of the following patients should he be most concerned about regarding a health careassociated infection? a.An asymptomatic elderly patient with bacteria in his urine b.A middle-aged woman with a white blood cell count of 10,000/mm 3 c.A young adult woman who is 1 day postoperative with redness at incision site d.A middle-aged man with temperature of 101.3 F and complaints of malaise

D- A middle-aged man with temperature of 101.3 F and complaints of malaise By assessing existing signs and symptoms (e.g., the condition of a wound, the presence of fever), you will determine whether a patients clinical condition indicates the onset or extension of a systemic infection

An elderly patient with diabetes is seeing the health care provider for complaints of visual changes. The patient explains to the nurse that visual changes include distortion that makes the edges of objects appear wavy. The nurse knows that this is an early sign of which of the following? a.Cataracts b.Glaucoma c.Diabetic retinopathy d.Age-related macular degeneration

D- Age-related macular degeneration Age-related macular degeneration occurs when the macula (specialized portion of the retina responsible for central vision) degenerates as a result of aging and loses its ability to function efficiently. An early sign includes distortion that causes edges or lines to appear wavy. In later stages, patients may see dark or empty spaces that block the center of vision. Cataract is clouding of the lens in the eye that affects vision. Interferes with passage of light through the lens and reduces the light that reaches the retina. Cataracts usually develop gradually and often result in cloudy or blurry vision, glare, double vision, and poor night vision. Glaucoma is a slowly progressive increase in intraocular pressure that causes progressive pressure against the optic nerve. At first, vision stays normal, and there is no pain. If left untreated, there may be a loss of peripheral (side vision). Diabetic retinopathy are pathological changes of the blood vessels of the retina secondary to increased pressure resulting in hemorrhage, macular edema, and reduced vision or vision loss.

.A patient is confused and is attempting to get out of the hospital bed. The nurse is tired after working for more than 10 hours and is concerned for the patients safety. What is the best action that the nurse should take to prevent the patient from harm? a.Restrain the patient with wrist restraints. b.Place the patient with a belt restraint in a chair. c.Sedate the patient with medication. d.Ask a family member to sit with the patient.

D- Ask a family member to sit with the patient. Asking a family member to sit with the patient is the best answer because it does not restrain the patient physically or chemically. The Joint Commission has set guidelines for the use of restraints in hospitals. These regulations set the standard that all patients have the right to be free from seclusion and physical or chemical restraints except to ensure the patients safety in emergency situations. The standards specifically prohibit restraining patients for staff convenience, punishment, or retaliation.

.Which action is the nurse required by law to perform when a patient is admitted? a.Notify the family. b.Notify the attorney. c.Ask how payment will be made. d.Ask about advance directives

D- Ask about advance directives The Patient Self-Determination Act (1991) requires health care institutions to inquire whether a patient has created an advance directive, give patients information on advance directives, and document whether a patient states that he or she has an advance directive. Notifying the family and attorney is breaking confidentiality. Asking how payment will be made is not required by law and is not the responsibility of the nurse.

A 9-year-old patient was severely burned and has been undergoing whirlpool treatments to debride the wounds. The patient is crying and does not want to go to the physical therapy department for treatment. The registered nurse caring for the patient knows that, even though it is uncomfortable, the patient needs to have the therapy for the wounds to heal properly. The nurse is demonstrating which ethical principle? a.Autonomy b.Bioethics c.Justice d.Beneficence

D- Beneficence The principle of beneficence promotes taking positive, active steps to help others. It encourages a nurse to do good for the patient. Beneficence guides decisions in which the benefits of a treatment pose a risk to the patients wellbeing or dignity. Autonomy refers to independence and self-determination. The study of bioethics represents a particular branch of ethics (i.e., the study of ethics within the field of health care). Justice refers to the principle of fairness. In health care the term is used to reflect a commitment to fair treatment and fair distribution of health care resources

A patient is being seen in the health clinic for abdominal pain. The nurse will be doing a physical assessment. After using light palpation to examine the patient, the nurse uses deep palpation. With deep palpation the nurse does which of the following? a.Performs a completely safe method of examination b.Should use two hands only c.Uses the upper hand to exert an upward pressure d.Can examine the condition of organs

D- Can examine the condition of organs After light palpation, use deeper palpation to examine the condition of organs. Depress the area you are examining deeply and evenly

.A registered nurse is required to participate in a simulation to learn how to triage patients who are arriving to the hospital after exposure to an unknown gas. This is an example of a response to what type of influence on nursing? a.Workplace hazards b.Nursing shortage c.Professionalism d.Emergency preparedness

D- Emergency preparedness Many health care agencies, schools, and communities have educational programs to prepare for nuclear, chemical, or biological attack and other types of disasters. Nurses play an active role in emergency preparedness. Workplace hazards include violence, harassment, and ergonomics. A person who acts professionally is conscientious in actions, knowledgeable in the subject, and responsible to self and others. There is an ongoing global nursing shortage, which results from insufficient qualified registered nurses (RNs) to fill vacant positions and the loss of qualified RNs to other professions.

A woman went to the public health clinic with a fever that had persisted for several days. Upon palpation of the neck, a registered nurse found several large lymph nodes. What is the nurses best response to this patient upon palpating her lymph nodes? a.There is no need to worry; enlarged lymph nodes are normal findings. b.Lymph nodes are not really significant because they really have no function. c.Most people have a few enlarged lymph nodes. d.Enlarged lymph nodes sometimes indicate an infection.

D- Enlarged lymph nodes sometimes indicate an infection. Lymph nodes that are large, fixed, inflamed, or tender indicate a problem such as local infection, systemic disease, or neoplasm. Enlarged lymph nodes are not normal. Normally lymph nodes are not easily palpable. An extensive system of lymph nodes collects lymph from the head, ears, nose, cheeks, and lips

A nurse is processing an ethical dilemma by focusing on relationships and stories of the participants. Which ethical system is the nurse using? a.Deontology b.Utilitarianism c.Feminist ethics d.Ethics of care

D- Ethics of care Ethics of care suggest that health care workers resolve ethical dilemmas by paying attention to relationships and stories of the participants and by promoting a fundamental act of caring. Attention to relationships distinguishes the ethics of care from other ethical viewpoints because it does not necessarily apply universal principles that are intellectual or analytical. Deontology defines actions as right or wrong based on right-making characteristics such as truth and justice. You use utilitarian ethics when determining the value of something based primarily on its usefulness and effects or consequences. Feminist ethics proposes that we routinely ask how ethical decisions will affect women as a way to repair a history of inequality

An RN suffers from chronic back pain that was the result of an injury suffered when pulling a patient up in bed. The nurse is addicted to pain medication and has recently been accused of stealing narcotics. This is an example of which violation of the law? a.Misdemeanor b.Tort c.Malpractice d.Felony

D- Felony A felony is a serious offense that results in significant harm to another person or society in general. Felony crimes may carry penalties of monetary restitution, imprisonment for greater than 1 year, or death. Examples of Nurse Practice Act violations that may carry criminal penalties include practicing nursing without a license and misuse of controlled substances. A misdemeanor is a crime that, although injurious, does not inflict serious harm. Torts are civil wrongful acts or omissions against a person or a persons property that are compensated by awarding monetary damages to the individual whose rights were violated. Malpractice is an example of negligence, sometimes referred to as professional negligence . The law defines nursing malpractice as the failure to use the degree of care that a reasonable nurse would use under the same or similar circumstances.

The mother of a 45-year-old patient is a retired physician and requests to discuss the patients plan of care with the nurse caring for the patient. What is the nurses best response to this request? a.I will need to ask permission from my supervisor before I can share that information. b.I will show you the chart, just follow me and we can discuss your questions and concerns. c.I would suggest that you leave me out of your family problems. I am here to care for the patient. d.I will have to get the patients permission before I can share that information.

D- I will have to get the patients permission before I can share that information. Even family members or friends of the patient are not permitted access to the patients personal health information without the patients consent. Federal legislation known as HIPAA (Health Insurance Portability and Accountability Act of 1996) requires that those with access to personal health information not disclose the information to a third party without patient consent. The nurse does not need to ask permission from the supervisor because HIPAA laws state what the nurse can do. I would suggest that you leave me out of your family problems is inappropriate because it ignores the request of the family member. Showing the chart and discussing the care is a violation of HIPAA.

A patient is obese. At the bedside is a standard-size blood pressure cuff. The nurse realizes that the use of this cuff will provide which of the following? a.Accurate readings as long as it is 20% of the circumference of the midpoint of the limb. b.Indistinct readings if the bladder encircles 80% of the adults arm. c.A falsely low reading if the cuff is wrapped too loosely around the arm. d.Inaccurate readings and needs to be replaced with a larger cuff

D- Inaccurate readings and needs to be replaced with a larger cuff When the incorrect size cuff is used, it is possible to obtain a false reading. The size selected is proportional to the circumference of the limb being assessed. Ideally select a cuff that is 40% of the circumference (or 20% wider than the diameter) of the midpoint of the limb being used to be obtain measurements

A patient with bilateral pneumonia is admitted to the intensive care unit. The nurse who initially prepared the plan of care identified that the patient had the collaborative problem of Potential complications: hypoxemia . What made the nurse classify this as a collaborative problem? a.It requires ensuring adequate hydration. b.It requires monitoring for signs of acid-base imbalance. c.It requires evaluating the effects of positioning on oxygenation. d.It requires both nursing and physician-prescribed intervention

D- It requires both nursing and physician-prescribed intervention A collaborative problem is an actual or potential physiological complication that nurses monitor to detect the onset of changes in a patients status. When collaborative problems develop, nurses collaborate with personnel from other health care disciplines, such as social workers and dietitians and physicians. Adequate hydration, acid-base imbalance, and oxygenation do not make a collaborative problem

A nurse is working on a medical unit in an acute care hospital. One of the patients she is caring for has a fever of 100.6 F. Which of the following is the best reason why the patient should not receive an antipyretic at this time? a.A temperature of 100.3 F is within the normal range. b.Shivering is a more effective way to dissipate heat energy. c.Corticosteroids are safer to use than antipyretics. d.Mild fevers are an important defense mechanism of the body

D- Mild fevers are an important defense mechanism of the body Fever, or pyrexia, is an important defense mechanism. Therefore most health care providers will not treat an adults fever until it is higher than 39 C (102.2 F)

A student nurse must pass the NCLEX before practicing as a registered nurse. NCLEX stands for Examination. a.Nursing Council of Licensing b.Nightingale Code of Licensure c.Nursing Code of Licensure d.National Council Licensure

D- National Council Licensure To be licensed in a state, a nurse must have a passing score on the National Council Licensure Examination (NCLEX) to obtain the initial license and meet the educational requirements set by the state. Nursing Council of Licensing, Nightingale Code, and Nursing Code examinations do not exist to practice as a nurse.

A patient falls out of bed because the nurse did not raise the side rails. Which action did the nurse commit? a.Felony b.Assault c.Battery d.Negligence

D- Negligence Negligence is conduct that falls below the generally accepted standard of care of a reasonably prudent person. A felony is a serious offense that has a penalty of imprisonment for greater than a year or possibly even death such as practicing nursing without a license. Assault is any intentional threat to bring about harmful or offensive contact with another individual. Battery is any intentional touching without consent.

Which assessment finding indicates that the patient has developed diabetes-induced peripheral neuropathy? a. The nurse must speak louder than usual to be understood by the patient. b. Fine tremors of the hands that worsen when the patient tries to eat or write c. Painful muscle spasms with hyperreactive Achilles and quadriceps reflexes. d. No pain is felt when the patient's feet are burned after walking on hot pavement.

D- No pain is felt when the patient's feet are burned after walking on hot pavement. Diabetic peripheral neuropathy may present with pain or loss of sensation in the extremities, particularly the feet. The patient who does not feel pain when the feet are burned is experiencing peripheral neuropathy. Diabetic peripheral neuropathy does not affect hearing or cause tremors or muscle spasms.

A patient is about to undergo a new, controversial bone marrow transplant procedure. The procedure may cause periods of pain and suffering. Although nurses agree to do no harm, this procedure may be necessary to promote health. This is an example of which ethnical principle? a.Autonomy b.Justice c.Fidelity d.Nonmaleficence

D- Nonmaleficence Nonmaleficence refers to the fundamental agreement to do no harm. The principle of nonmaleficence promotes a continuing effort to consider the potential for harm even when it is necessary to promote health. Autonomy refers to a persons independence. Justice refers to the principles of fairness. Fidelity refers to the agreement to keep promises.

.A registered nurse knows that an oncology patient undergoing a bone marrow transplant will spend weeks in isolation in the hospital. During that time the patient will be at an increased risk for infection and other complications and may not recover. The nurse ensures that the patient has been given information regarding the risks and potential benefits of the procedure. The nurse is following which ethical principle? a.Autonomy b.Justice c.Fidelity d.Nonmaleficence

D- Nonmaleficence The principle of nonmaleficence (do no harm) promotes a continuing effort to consider the potential for harm even when it is necessary to promote health. It is helpful in guiding your discussions about new or controversial technologies. Autonomy deals with independence and self-determination. Justice refers to fairness or equity of health care resources. Fidelity refers to maintaining promises and faithfulness.

When admitting a patient to the hospital, the nurse asks if has problems eating since the patient had a stroke. The patient denies any problems and states that does not require assistance. After lunch, the nurse notes that the patient has not eaten most of the food and has spilled much of the food. These cues lead the nurse to believe that the patient is not functioning at the level indicated upon admission. The nurse is using which type of information to make this deduction? a.Verbal behavior b.Physical assessment c.Nursing diagnosis d.Nonverbal behavior

D- Nonverbal behavior

A nurse is directing the care and staffing of three cardiac units. The nurse is practicing in which nursing role? a.Advanced practice registered nurse b.Nurse researcher c.Nurse educator d.Nurse administrator

D- Nurse Administrator A nurse administrator manages patient care and the delivery of specific nursing services within a health care agency. An advanced practice registered nurse has a masters degree in nursing; advanced education in pathophysiology, pharmacology, and physical assessment; and certification and expertise in a specialized area of practice. A nurse educator works primarily in schools of nursing, staff development departments of health care agencies, and patient education departments. The nurse researcher investigates problems to improve nursing care and further define and expand the scope of nursing practice.

Upon assessment, the nurse finds that a patient has a heart rate of 66 beats per minute, a respiratory rate of 12 breaths per minute, and a blood pressure of 120/80 mm Hg. The nurse obtained which type of data? a.Personal b.Demographic c.Subjective d.Objective

D- Objective Objective data are observations or measurements of a patients health status. Personal and demographic data refer to patients name, age, sex, and so on. Subjective data are patients verbal descriptions about their health problems. Demographic data includes birth, gender, address, family members names and addresses

An older adult patient complains of thirst, headache, and weight loss. The patient appears emaciated. On physical assessment the nurse finds that the patients skin does not return to normal shape after being assessed. This finding is consistent with which of the following? a.Pallor b.Cyanosis c.Erythema d.Poor skin turgor

D- Poor skin turgor Turgor is the skins elasticity. To assess skin turgor, grasp a fold of skin on the back of the forearm or sternal area with the fingertips and release

The nurse is assessing an elderly patient and notices a significant drooping of the eyelid. The nurse recognizes this as which of the following? a.Ectropion of the eyelid b.Entropion of the eyelid c.Impairment of the fourth cranial nerve d.Ptosis of the eyelid

D- Ptosis of the eyelid An abnormal drooping of the lid over the pupil is called ptosis , caused by edema or impairment of the third cranial nerve. In the older adult, ptosis results from a loss of elasticity that accompanies aging. An older adult frequently has lid margins that turn out (ectropion) or in (entropion).

What is the nurses best proof against malpractice? a.The nurse supervisors memory of the event b.Recorded documentation written carelessly c.The nurses memory of the event d.Recorded documentation of nursing care

D- Recorded documentation of nursing care Documentation of nursing care is the only record of what actually was done for a patient and will serve as proof that a nurse acted reasonably and safely. Nursing notes written at the time of the event, are seen as better evidence of the facts of the event than any one persons memory. Nurses notes written carelessly and without regard to detail or hospital standards of documentation do not reflect well on the health care providers credibility or appearance of accountability to a judge or jury.

Which example demonstrates a breach of confidentiality and a violation of the Health Insurance Portability and Accountability Act (HIPAA) of 1996? a.Giving a report to the oncoming nurse in a conference room b.Discussing a patients diagnosis with the patients health care provider c.Providing patient information to the nursing assistant caring for the patient d.Sharing with other nurses in the cafeteria that a patient is HIV positive

D- Sharing with other nurses in the cafeteria that a patient is HIV positive Although HIPAA does not require such things as soundproof rooms in hospitals, it does mandate that nurses and health care providers avoid discussing patients in public hallways and provide reasonable levels of privacy in communicating with and about patients in any matter. Issues of disclosure, privacy, and confidentiality are important concerns when working with patients or peers infected with blood-borne illnesses such as human immunodeficiency virus (HIV) or acquired immunodeficiency virus (AIDS), hepatitis, and sexually transmitted illnesses. Providing continuity of care, giving reports, talking to the health care provider, and providing information to the nursing assistant do not violate HIPAA.

.A nurse is collecting data during the assessment of a patient. During the assessment, the nurse collects both subjective and objective data. Which information should the nurse consider as subjective data? a.Heart rate of 96 b.Incisional erythema c.Emesis of 150 mL d.Sharp, burning pain

D- Sharp, burning pain Sharp, burning pain is subjective. Subjective data are patients verbal descriptions of their health problems. Only patients provide subjective data. Heart rate, incisions, and emesis are all objective data. Objective data are observations or measurements of a patients health status.

The nursing student is obtaining the patients vital signs. The patient has gone to the clinic seeking help because she is having chest pain. Which of the following vital signs are most important to obtain? a.Temperature, pulse, respirations b.Temperature, pulse, respirations, oxygen saturation c.Temperature, pulse, respirations, blood pressure, oxygen saturation d.Temperature, pulse, respirations, blood pressure, oxygen saturation, pain

D- Temperature, pulse, respirations, blood pressure, oxygen saturation, pain

A nurse is ready to take the temperature of an adult patient rectally. The nurses realizes that rectal temperatures are which of the following? a.Preferable to oral temperatures b.Safer than oral temperatures if the patient has neutropenia c.The best way to obtain temperatures in newborns d.That readings can be influenced by impacted stool

D- That readings can be influenced by impacted stool Rectal temperature readings are sometimes influenced by impacted stool. A rectal temperature is argued to be more reliable than alternative sites when oral temperature is difficult or impossible to obtain, but are not used for patients with diarrhea or those who have had rectal surgery, rectal disorders, bleeding tendencies, or neutropenia, and are not used for routine vital signs in newborns.

The code of ethics for nursing sets forth ideals of nursing conduct and was developed by what organization? a.The Board of Nursing b.The American Medical Association c.The National League for Nursing d.The American Nurses Association

D- The American Nurses Association The American Nurses Association (ANA) and the International Council of Nurses (ICN) publish codes of ethics for nurses that set principles of behavior for them to embrace. The Board of Nursing regulates nursing programs and nursing practice. The American Medical Association deals with physicians. The National League for Nursing is an agency concerned with nursing education.

A nurse is writing a care plan for a newly admitted patient. Which outcome statement did the nurse correctly write? a.The patient will eat 80% of all meals. b.The nursing assistant will set up the patient for a bath every day. c.The nursing assistant will ambulate the patient three times a day by May 30. d.The patient will identify the need to increase dietary intake of fiber by July 4.

D- The patient will identify the need to increase dietary intake of fiber by July 4

The student nurse has been assigned to the pediatric unit for her clinical training this semester. She is assisting with the admission of a 5-month-old infant admitted with pneumonia. The student nurse is responsible for taking the childs vital signs and weighing and measuring the child. The infants mother is very concerned when the student nurse tells her that the baby weighs 14 pounds. The mother states that the baby has lost a significant amount of weight because the previous week she weighed 16 pounds at home. What is the student nurses best response to the mothers concern? a.To get an accurate weight, babies are weighed at different times of the day. b.Variations occur because we place our hand firmly on the child. c.Even if we use the same scale, the variation can be 1 to 2 pounds. d.Weight measurements can vary with different scales

D- Weight measurements can vary with different scales Different scales can give different weights for patients. To ensure accurate clinical decisions, weigh patients at the same time of day, on the same scale, and in the same clothes. Hold a hand lightly above the infant to prevent accidental falls. The scale can measure in weight increments to the nearest 0.1 pound or 0.1 kg.

Which information indicates the nurse has an accurate understanding of when the institutions malpractice insurance covers the nurse? a.While driving to work b.While driving home from work c.While tending to people in the neighborhood d.While working within the scope of employment

D- While working within the scope of employment If a nurse works for a health care institution, generally the institutions insurance will cover the nurse during employment. Malpractice insurance usually provides nurses with an attorney, payment of those fees, and payment of any judgment or settlement if a patient sues a nurse for medical malpractice. If a nurse provides care on a voluntary basis outside the health care facility, hospital-provided malpractice insurance would not cover the nurse. The nurse will need to carry additional insurance. Driving to and from work is not malpractice.

An elderly patient was admitted to the hospital after falling in the nursing home. The patient has a fractured right femur and is awaiting surgery. The surgeon orders bed rest. The patient asks the nurse what this means. What is th nurses best explanation? a.You are to be immobile. b.You cannot move. c.You need restraints. d.You have to remain in bed.

D- You have to remain in bed. A patients mobility can be restricted for therapeutic reasons, such as when bed rest is ordered. Therapeutic reasons for bed rest include decreasing the bodys oxygen needs, reducing cardiac workload, reducing pain, and allowing the debilitated or ill patient to rest. The duration of bed rest depends on the type and nature of the illness or injury and the patients prior state of health. Bed rest does not mean immobile, cannot move, or that restraints are needed.

An inefficient contraction of the heart that fails to transmit a pulse wave to the peripheral pulse site creates: a.dysrhythmia. b.tachycardia. c.bradycardia. d.a pulse deficit

D- a pulse deficit An inefficient contraction of the heart that fails to transmit a pulse wave to the peripheral pulse site creates a pulse deficit. A regular interval interrupted by an early beat, late beat, or a missed beat indicates an abnormal rhythm (not rate) or dysrhythmia

A nurse is coaching a student on the proper method of applying surgical gloves. One step in the proper donning of sterile gloves requires the nurse to: a.with thumb and first two fingers of nondominant hand, touch only the gloves outer surface. b.with gloved dominant hand, slip fingers inside the second glove and pull onto the nondominant hand. c.carefully pull the glove over the dominant hand, leaving a cuff and being sure the cuff rolls up over the wrist. d.carefully pull the glove over the dominant hand, leaving a cuff and being sure the cuff does not roll up over the wrist

D- carefully pull the glove over the dominant hand, leaving a cuff and being sure the cuff does not roll up over the wrist

A patient with a history of poor nutrition and chronic illness is admitted to the medical unit. The nurse caring for this patient is preparing to provide the patient with a bed bath and recalls that normal body flora: a.are only found on the skin surface. b.are beneficially aided by the use of antibiotics. c.are primary sources of infection when balanced. d.help to maintain health.

D- help to maintain health. Normal flora usually does not cause disease, but instead help to maintain health. The number and variety of flora maintain a sensitive balance with other microorganisms to prevent infection

The nurse is caring for a patient who has ongoing headaches, nausea, dizziness, and fatigue since the weather turned cold and snowy. Which assessment question is most important to ask the patient? a. "Has your furnace been inspected lately?" b. "Have you checked your roof for any leaks?" c. "When was the last time your house was painted?" d. "When did you change your smoke detector batteries?"

a. "Has your furnace been inspected lately?" A furnace, stove, or fireplace that is not properly vented introduces carbon monoxide into the environment. This gas binds strongly with hemoglobin, preventing the formation of oxyhemoglobin and thus reducing the supply of oxygen delivered to the tissues. Low concentrations cause nausea, dizziness, headache, and fatigue. The importance of having a proper working smoke detector will decrease the chance of smoke inhalation and potential death owing to a fire but does not produce the symptoms listed. A leaking roof and lead paint would not cause the patient's symptoms.

A nurse is conducting a therapeutic session with a patient in the inpatient psychiatric facility. Which remark by the nurse would be an appropriate way to begin an interview session? a. "How shall we start today?" b. "Shall we talk about losing your privileges yesterday?" c. "Let's get started discussing your marital relationship." d. "What happened when your family visited yesterday?"

a. "How shall we start today?" The interview is patient centered; thus, the patient chooses issues. The nurse assists the patient by using communication skills and actively listening to provide opportunities for the patient to reach goals. In the distracters, the nurse selects the topic.

The patient asks the nurse to explain the function of the sinoatrial node in the heart. What is the nurse's best response? a. "It stimulates the heart to beat in a normal rhythm." b. "It protects the heart from atherosclerotic changes." c. "It provides the heart with oxygenated blood." d. "It protects the heart from infection."

a. "It stimulates the heart to beat in a normal rhythm." The sinoatrial node is the natural pacemaker of the heart, and it assists the heart to beat in a normal rhythm. The sinoatrial node does not protect from atherosclerotic changes or infection, and it does not directly provide the heart with oxygenated blood.

A nurse is conducting community education classes on skin cancer. One participant says to the nurse: "I read that most melanomas occur on the face and arms in fair-skinned women. Is this true?" How should the nurse respond? a. "That is not correct. Melanoma is more commonly found on the torso or the lower legs of women." b. "That is correct, because the face and arms are exposed more often to the sun." c. "That is not correct. Melanoma occurs on the top of the head in men but is rare in women." d. "That is incorrect. Melanoma is most commonly seen in dark-skinned individuals."

a. "That is not correct. Melanoma is more commonly found on the torso or the lower legs of women." Melanoma is more commonly found on the torso or the lower legs in women. Melanoma can occur anywhere and is not associated with direct exposure. For example, an individual can have melanoma under the skin and on the soles of the feet. Dark-skinned individuals are less likely to get melanoma.

A new mother is exhausted because her 4-week-old infant is unable to sleep through the night. Which is the best recommendation of the nurse? a. "Try to sleep when your baby is sleeping." b. "Try keeping the room warm during the night." c. "Try not to let your baby nap during the day." d. "Try laying the baby on his stomach at night."

a. "Try to sleep when your baby is sleeping." Infants usually develop a nighttime pattern of sleep by 3 to 4 months of age. The neonate and infant up to the age of 3 months average about 16 to 18 hours of sleep a day. The mother should be encouraged to sleep when the baby is sleeping. Infants sleep best when the room temperature is 18° C to 21° C (64° F to 70° F). The infant is likely to become irritable and overtired if napping during the day is prevented. Infants should be laid on their backs to sleep to prevent Sudden Infant Death Syndrome.

A staff nurse reports a medication error due to failure to administer a medication at the scheduled time. What is the charge nurse's best response? a. "We'll conduct a root cause analysis." b. "That means you'll have to do continuing education." c. "Why did you let that happen?" d. "You'll need to tell the patient and family."

a. "We'll conduct a root cause analysis." In a just culture the nurse is accountable for their actions and practice, but people are not punished for flawed systems. Through a strategy such as root cause analysis the reasons for errors in medication administration can be identified and strategies developed to minimize future occurrences. Requiring continued education may be an appropriate recommendation but not until data is collected about the event. Telling the patient is part of transparency and the sharing and disclosure among stakeholders, but it is generally the role of risk management staff, not the staff nurse.

Which question is best suited for determining the patient's chief complaint? a. "What brings you to the hospital today?" b. "How long have you been having chest pain?" c. "Did your doctor tell you to come to the hospital?" d. "Have you ever experienced this problem before?"

a. "What brings you to the hospital today?" Asking what brought the patient to the hospital today is a good way for the nurse to determine the patient's chief complaint. Asking about the duration of chest pain, doctor referral, or recurrence of the problem should be asked after the chief complaint is identified.

What is the nurse's best response about developing diabetes to the patient whose father has type 1 diabetes mellitus? a. "You have a greater susceptibility for development of the disease because of your family history." b. "Your risk is the same as the general population, because there is no genetic risk for development of type 1 diabetes." c. "Type 1 diabetes is inherited in an autosomal dominant pattern. Therefore the risk for becoming diabetic is 50%." d. "Because you are a woman and your father is the parent with diabetes, your risk is not increased for eventual development of the disease. However, your brothers will become diabetic."

a. "You have a greater susceptibility for development of the disease because of your family history." Even though type 1 diabetes does not follow a specific genetic pattern of inheritance, those with one parent with type 1 diabetes are at an increased risk for development of the disease.

Which patient does the nurse identify that would benefit from administration of an oil-retention enema? a. A constipated patient with a fecal impaction b. A patient with Clostridium difficile diarrhea c. A patient with a positive fecal occult blood test d. A patient with a serum potassium level of 7.1 mEq/L

a. A constipated patient with a fecal impaction A patient with a fecal impaction would benefit from an oil-retention enema as the hard stool would soften and make it easier to pass. Oil-retention enemas do not benefit patients with diarrhea, positive fecal occult blood test, or a serum potassium level of 7.1 mEq/L. A sodium polystyrene sulfonate enema is administered to treat severe hyperkalemia

Which patient does the nurse identify that would benefit from a nasogastric tube to low intermittent suction? a. A patient who is vomiting due to a complete large bowel obstruction b. A patient with constipation who has not had a bowel movement in 6 days c. A patient with constant diarrhea due to side effects of antibiotic therapy d. A patient with extensive skin irritation due to a leaking colostomy appliance

a. A patient who is vomiting due to a complete large bowel obstruction A patient who is vomiting due to a complete small bowel obstruction would benefit from a nasogastric tube as it will decompress the stomach and eliminate the need for vomiting. A nasogastric tube will not be of benefit for patients with constipation, diarrhea, or leaking ostomy appliance.

Which patient would benefit from education about pursed-lip breathing? a. A patient with emphysema after many years of smoking b. A patient with a pneumothorax and a chest tube to suction c. A patient with a tracheostomy following throat cancer surgery d. A patient with respiratory muscle paralysis after spinal cord injury

a. A patient with emphysema after many years of smoking A patient with emphysema would benefit from pursed-lip breathing as it facilitates exhalation of air from the lungs. Pursed-lip breathing is not recommended for patients with chest tubes. Patients with tracheostomy or respiratory muscle paralysis would not be able to do pursed-lip breathing

Which patient would benefit from the use of a hand roll? a. A patient with paralysis of the right hand after a left-sided stroke b. A paraplegic patient who requires assistance moving up in the bed c. A confused patient who is attempting to pull out the urinary catheter d. A patient with lymphedema of the right arm and hand after mastectomy

a. A patient with paralysis of the right hand after a left-sided stroke A hand roll is a rolled washcloth that is placed in the palm of the hand to keep the fingers in a slightly flexed position. The hand roll is useful for preventing development of hand muscle contractures. The hand roll is not used for restraining patients, treating lymphedema, or to assist with repositioning.

Which is the first intervention of the nurse for changing the dressing to a painful burn? a. Administer pain medication 30 minutes beforehand. b. Gently irrigate the wound using sterile normal saline. c. Loosen the tape gently by pressing the skin away from it. d. Observe the wound bed for presence of granulation tissue

a. Administer pain medication 30 minutes beforehand When you plan a dressing change, consider giving the patient an analgesic at least 30 minutes beforehand. The tape should be loosened gently by pressing the skin away from it. The wound bed should be observed and then irrigation should be performed.

The nurse instructs a patient with type 1 diabetes mellitus to avoid which of the following drugs while taking insulin? a. Aldactone (Spironolactone) b. Dicumarol (Bishydroxycoumarin) c. Reserpine (Serpasil) d. Cimetidine (Tagamet)

a. Aldactone (Spironolactone) Aldactone is a loop potassium-sparing diuretic and can affect serum glucose levels and also lead to hypokalemia; its use is contraindicated with insulin. Dicumarol, an anticoagulant; reserpine, an anti-hypertensive; and cimetidine, an H2 receptor antagonist do not affect blood glucose levels.

Which technique by the nurse will facilitate communication with an older adult? a. Allow reminiscing. b. Use long sentences. c. Ask several questions in a row. d. Play soft music in the background.

a. Allow reminiscing Allow older adults the opportunity to reminisce. Reminiscing has therapeutic properties that increase the sense of well-being. During conversation maintain a quiet environment that is free from background noise (turn off the TV). Allow time for conversation; do not ask several questions in a row. Avoid long sentences to explain the subject. Try to keep it short, simple, and to the point.

The nurse is caring for a patient who took 60 acetaminophen tablets. Which resource will the nurse contact for treatment guidelines when the patient arrives in the emergency room? a. American Association of Poison Control Centers b. Centers for Disease Control and Prevention c. Agency for Healthcare Research and Quality d. Institute for Safe Medication Practices

a. American Association of Poison Control Centers The American Association of Poison Control Centers supports the Poison Help Line and should be contacted for information about acetaminophen overdose. The CDC manages infectious diseases. The AHRQ conducts research about health care practices. The ISMP promotes safe administration of medications by practitioners.

The nurse is reviewing a patient's differential white blood cell count. What implications would this test have on evaluating the adequacy of a patient's gas exchange? a. An elevation of the total white cell count indicates generalized inflammation. b. Eosinophil count will assist to identify the presence of a respiratory infection. c. White cell count will differentiate types of respiratory bacteria. d. Level of neutrophils provides guidelines to monitor a chronic infection.

a. An elevation of the total white cell count indicates generalized inflammation Elevation of total white cell count is indicative of inflammation that is often due to an infection. Upper respiratory infections are common problems in altering a patient's gas exchange. Eosinophil cells are increased in an allergic response. Neutrophils are more indicative of an acute inflammatory response. White cells do not assist to differentiate types of respiratory bacteria. Monocytes are an indicator of progress of a chronic infection

A nurse tells a patient with a recent back injury that damage to the nerves is comparable to a water hose that has been pinched off and that time is needed to allow normal nerve transmission. Which technique did the nurse use? a. Analogy b. Discovery c. Role playing d. Demonstration

a. Analogy Analogies add to verbal instruction by providing familiar images that make complex information more real and understandable. Discovery is a useful tool for teaching problem solving and is a technique for cognitive learning. During role play your patients play themselves or someone else in the situation. Demonstrations are useful when teaching psychomotor skills.

The patient has red, itchy eyes with thick yellow drainage. What will the nurse plan to do as a result of this assessment finding? a. Apply antibiotic eyedrops and use alcohol-based hand sanitizer before and after caring for the patient. b. Assess the patient's visual fields and compare results from each of the patient's eyes. c. Ask the patient about any history of hyperthyroid disease, hypertension, or Addison's disease. d. Instruct the patient not to pluck or wax the eyebrows until the eye infection symptoms have resolved completely

a. Apply antibiotic eyedrops and use alcohol-based hand sanitizer before and after caring for the patient The patient's symptoms are consistent with conjunctivitis, a highly contagious bacterial infection. The nurse should apply antibiotic eyedrops and use alcohol-based hand sanitizer before and after caring for the patient to prevent spread of the infection. None of the other interventions are indicated for treatment of conjunctivitis

Which intervention will be most effective for prevention of foot drop in an immobile patient? a. Apply high-top tennis shoes to the patient's feet. b. Obtain a podiatry consult for all toenail and foot care. c. Encourage the patient to wear sturdy shoes when ambulating. d. Apply moisturizing lotion to the heels of the feet and between the toes

a. Apply high-top tennis shoes to the patient's feet High-top tennis shoes can help prevent foot drop by keeping the feet in a neutral position without plantar flexion. Toenail care and lotion will not prevent foot drop. Using sturdy shoes for ambulation is not appropriate for the immobile patient

The patient is scheduled for hip replacement surgery and significant blood loss is expected. What is the best possible action of the patient to reduce the risk of transfusion complications? a. Arrange for an autologous blood donation. b. Take an iron supplement daily prior to the surgery. c. Expect transfusions will come from a directed donor. d. Request that donated blood be screened twice by the blood bank

a. Arrange for an autologous blood donation. The patient should be advised to arrange for an autologous blood donation. This way the patient will receive his/her own blood rather than from a donor. Iron supplementation will not be sufficient to compensate for the blood loss expected with hip replacement surgery. Patients cannot request extra screening precautions for transfusions. The patient should not be instructed to expect that transfusions will come from a directed donor.

A nurse is caring for a patient who cannot speak clearly. Which technique should the nurse use to enhance conversation with this patient? a. Ask questions that require "yes" or "no" answers. b. Avoid communication aids to prevent embarrassment. c. Speak loudly and slowly to facilitate patient understanding. d. Finish the patient's sentences when the patient is unable to.

a. Ask questions that require "yes" or "no" answers. For patients who are mute, unable to speak, or cannot speak clearly, ask simple questions that require "yes" or "no" answers. Use normal volume and do not shout or speak too loudly. Do not finish the patient's sentences. Use communication aids as needed; do not avoid them.

A patient has a morphine sulfate patient-controlled analgesia (PCA) to control postoperative pain. When the nurse enters the room, the patient complains of pain. The nurse's first response is which of the following? a. Ask the patient to rate the pain on a 0-to-10 scale. b. Check the patency of the patient's intravenous line. c. Call the physician or health care provider immediately. d. Speak to the patient in a calming tone to reduce anxiety.

a. Ask the patient to rate the pain on a 0-to-10 scale Nurses need to assess the patient first. Next, assess the lines, catheter, and infusion pump. Notify the physician or health care provider or follow protocols should a problem exist. Speaking in a calm voice demonstrates caring behavior.

The nurse is assessing a patient with suspicious bruises. Which action is most appropriate to facilitate an accurate account of the patient's injuries? a. Ask the patient's family member to wait outside in the waiting room. b. Ignore the bruises because the patient has provided an explanation. c. Realize that the patient may be abused, but that is a family issue. d. Provide referrals for health counseling once the assessment is complete.

a. Ask the patient's family member to wait outside in the waiting room. Patients are more likely to reveal problems when the suspected abuser is not present in the room. Psychological abuse as well as obvious physical injury or neglect (e.g., evidence of malnutrition or presence of bruising on the extremities or trunk) should be assessed. If you have suspicion of abuse, find a way to interview the patient privately. If you assess a pattern of findings indicating abuse, most states mandate a report to a social service center. (Refer to state guidelines.) Obtain immediate consultation with a health care provider, social worker, and other support staff to facilitate placement in a safer environment.

The nurse is caring for an elderly patient with the nursing diagnosis loneliness related to recent loss of spouse and limited contact with significant others. Which stage of Maslow's hierarchy of needs is addressed with this diagnosis? a. Belonging b. Self-esteem c. Self-actualization d. Safety and security

a. Belonging The nursing diagnosis risk for loneliness relates to Maslow's need for belongingness and love. Self-esteem relates to feelings of accomplishment. Self-actualization is about achieving one's full potential. Safety and security include protection from cold, illness, or injury

Which bedtime snack will best help the patient to fall asleep easily? a. Bowl of cereal with milk b. Ham sandwich, fruit, and juice c. Glass of red wine with wheat crackers d. Chocolate chip cookies and a cup of hot tea

a. Bowl of cereal with milk A bedtime snack containing protein and carbohydrates such as cereal and milk or cheese and crackers, which contain L-tryptophan, may help to promote sleep. A full meal before bedtime often causes gastrointestinal upset and interferes with the ability to fall asleep. Coffee, tea, cola, and chocolate cause a person to stay awake or wake up throughout the night. Alcohol disrupts sleep patterns and can make it difficult to fall asleep.

The nurse is caring for a patient with the nursing diagnosis impaired religiosity related to inability to participate in religious services while hospitalized. Which nursing intervention is most appropriate for this patient? a. Call the patient's religious leader to coordinate bedside services for the patient. b. Help the patient to make a list of important values and ideals in the patient's life. c. Use therapeutic touch and authentic presence to support the patient spiritually. d. Encourage the patient to focus on physical healing before meeting spiritual needs.

a. Call the patient's religious leader to coordinate bedside services for the patient. The patient is distressed because of the inability to participate in religious services while hospitalized. The best way for the nurse to address the religious need of the patient is to call the patient's religious leader to coordinate bedside services for the patient. This will facilitate meeting the patient's religious needs. Helping the patient to list values and using therapeutic touch will not meet the patient's religious needs. Encouraging the patient to focus on physical healing before meeting spiritual needs minimizes the patient's priorities and may be seen as condescending.

An 80-year-old male patient who is in the intensive care unit has suffered a fractured femur. You are making rounds and notice he is somnolent, with no response to verbal or physical stimulation. He has been on round the clock opioid doses q 4 hours. What is the nurse's first action? a. Call the rapid response team to care for the patient immediately. b. Discontinue the opioids on the medication administration record. c. Review the patient's blood pressure and pain level. d. Start a second intravenous line with a large bore catheter.

a. Call the rapid response team to care for the patient immediately. After establishing unresponsiveness, the next action is to call a Rapid Response. The patient is not able to subjectively describe pain if unresponsive. Another IV line may be needed, but first the nurse should call for help. The opioids should be discontinued on the MAR; however the priority action is to call for help

The nurse works on the cardiac unit of a hospital. The health care provider has ordered 20 mEq of KCl per L to be added to the fluids that the patient is receiving. The patient currently has a bag of D5W IV fluid infusing. Which is the priority action of the nurse? a. Check the patient's potassium level before hanging the new IV solution. b. Administer 20 mEq KCl diluted in 5 mL of fluid by IV push in 5 minutes. c. Give the KCl undiluted by IV push in 5 minutes for the most rapid action. d. Estimate the amount of fluid in the IV bag and add KCl to equal 20 mEq/L.

a. Check the patient's potassium level before hanging the new IV solution. Remember that failure to verify that a patient has adequate renal function and urine output before administering an IV solution containing potassium could cause hyperkalemia. Under no circumstances should KCl be given in an IV push. A direct IV infusion of KCl may be fatal. Intravenous administration of KCl requires dilution in solution and infusion slowly over a period of time. In most hospitals, nurses do not add KCl to IV bags. Usually a pharmacist prepares the solution

The nurse is caring for a patient who has been vomiting. The patient's sodium level is 124 mEq/L. Which laboratory result will the nurse also expect to see in the patient's chart? a. Chloride 81 mEq/L b. Calcium 11.1 mg/dL c. Phosphate 5.1 mg/dL d. Magnesium 2.3 mEq/L

a. Chloride 81 mEq/L Chloride levels are closely associated with sodium levels. When the serum sodium level is low, the chloride level will drop as well. The normal sodium level is 135 to 145 mEq/L so the patient's 124 mEq/L level indicates hyponatremia. The nurse will expect to find the patient's chloride level to be 81, below the 95 to 105 normal range. Hypercalcemia, normal magnesium level, and hyperphosphatemia are not expected with hyponatremia due to vomiting.

The acid-base status of a patient is dependent on normal gas exchange. Which patient would the nurse identify as having an increased risk for the development of respiratory acidosis? a. Chronic lung disease with increased carbon dioxide retention b. Acute anxiety, hyperventilation, and decreased carbon dioxide retention c. Decreased cardiac output with increased serum lactic acid production d. Gastric drainage with increased removal of gastric acid

a. Chronic lung disease with increased carbon dioxide retention Respiratory acidosis is caused by an increase in retention of carbon dioxide, regardless of the underlying disease. A decrease in carbon dioxide retention may lead to respiratory alkalosis. An increase in production of lactic acid leads to metabolic acidosis. Removal of an acid (gastric secretions) will lead to a metabolic alkalosis.

Aspects of safety culture that contribute to a culture of safety in a healthcare organization include which component? a. Communication b. Fear of punishment c. Malpractice implications d. Team nursing

a. Communication Aspects that contribute to a culture of safety include leadership, teamwork, an evidence base, communication, learning, a just culture, and patient-centered care. Fear of professional or personal punishment and concern about malpractice implications are considered barriers to a culture of safety. No model of nursing care has been related to a culture of safety

The nurse is reviewing a patient's functional performance. What assessment parameters will be most important in this assessment? a. Continence assessment, gait assessment, feeding assessment, dressing assessment, transfer assessment b. Height, weight, body mass index (BMI), vital signs assessment c. Sleep assessment, energy assessment, memory assessment, concentration assessment d. Health and well-being, amount of community volunteer time, working outside the home, and ability to care for family and house

a. Continence assessment, gait assessment, feeding assessment, dressing assessment, transfer assessment Functional impairment, disability, or handicap refers to varying degrees of an individual's inability to perform the tasks required to complete normal life activities without assistance. Height, weight, BMI, and vital signs are part of a physical assessment. Sleep, energy, memory, and concentration are part of a depression screening. Healthy, volunteering, working, and caring for family and house are functional abilities, not performance

Which assessment findings lead the nurse to suspect that the patient has obstructive sleep apnea? a. Daytime sleepiness, snoring, and obesity b. Insomnia, inability to concentrate, and anemia c. Early morning awakening, shift work, and attention deficit disorder (ADD) d. Latex allergy, stressful career, and recent divorce

a. Daytime sleepiness, snoring, and obesity Daytime sleepiness and snoring in an overweight patient indicates the strong possibility of sleep apnea. The patient should be referred for further testing. Insomnia, early morning awakening, and stressful career do not lead to sleep apnea

Which nursing diagnosis is most appropriate for a toddler who is not walking by the age of 24 months? a. Delayed growth and development related to failure to achieve age-appropriate motor skills b. Impaired walking related to inability to maintain a steady gait along uneven surfaces c. Sedentary lifestyle related to preferred lifestyle that requires minimal physical activity d. Impaired parenting related to unrealistic expectations for age-appropriate motor skills

a. Delayed growth and development related to failure to achieve age-appropriate motor skills If a child does not walk by the age of 20 months, there is delayed gross-motor ability. Therefore delayed growth and development related to failure to achieve age-appropriate motor skills is the appropriate nursing diagnosis. Delayed development has nothing to do with sedentary lifestyle or unrealistic parenting expectations. Impaired walking is not the appropriate nursing diagnosis for a developmentally delayed toddler.

The nurse is caring for an elderly patient who has just been diagnosed with a bladder infection. The normally alert and appropriate patient thinks that she is going to bake cookies with her mother when she gets home after school this afternoon. Which term best describes the patient's mental status? a. Delirious b. Dejected c. Depressed d. Demented

a. Delirious The patient is delirious due to the urinary tract infection. The patient is normally alert and appropriate so the patient is not demented. Alzheimer's disease and other forms of dementia cause long-term mental status changes. The patient is not depressed or dejected as there is no sadness evident in her plan to bake cookies with her mother.

The nurse informs the patient that a code pink is paged overhead when an infant is abducted from the hospital. What is the best description of the use of the term code pink in this situation? a. Denotative meaning b. Perceptual stereotype c. Emotional inflection d. Sender territoriality

a. Denotative meaning Code pink in this instance is a denotative meaning in that the term refers to infant abduction to providers within the agency. Individuals who use a common language share the denotative meaning of a word or phrase. Perceptual stereotyping prevents accurate interpretation of messages from others. Inflection refers to the tone and pitch of the voice during verbal communication. Territoriality refers to the physical space between the sender and the receiver

The patient's urinalysis indicates small amounts of protein in the urine. Which diagnosis does the nurse anticipate to see in the patient's electronic health record? a. Diabetes mellitus b. Diabetes insipidus c. Hypothyroid disease d. Hyperparathyroid disease

a. Diabetes mellitus Diabetes mellitus can cause damage to the glomerular membrane allowing protein to seep into the urine. Diabetes insipidus, hypothyroid disease, and hyperparathyroid disease do not cause proteinuria.

The nurse is reviewing the patient's arterial blood gas results. The PaO2 is 96 mm Hg, pH is 7.20, PaCO2 is 55 mm Hg, and HCO3 is 25 mEq/L. What might the nurse expect to observe on assessment of this patient? a. Disorientation and tremors b. Tachycardia and decreased blood pressure c. Increased anxiety and irritability d. Hyperventilation and lethargy

a. Disorientation and tremors The patient is experiencing respiratory acidosis (pH and PaCO2) which may be manifested by disorientation, tremors, possible seizures, and decreased level of consciousness. Tachycardia and decreased blood pressure are not characteristic of a problem of respiratory acidosis. Increased anxiety and hyperventilation will cause respiratory alkalosis, which is manifested by an increase in pH and a decrease in PaCO2

Which nighttime activity will keep the patient from being able to fall asleep easily? a. Doing crossword puzzles on a tablet computer b. Turning down the temperature in the bedroom c. Listening to recordings of soothing classical music d. Always going to bed at approximately the same time

a. Doing crossword puzzles on a tablet computer Doing crossword puzzles on a tablet computer engages the brain and makes it hard to fall asleep. In addition, the bright background light of the computer screen interferes with the brain's ability to fall asleep. Turning down the temperature in the bedroom and listening to soothing music will facilitate falling asleep. Keeping a consistent bedtime will also help the body to relax and fall asleep.

Which activity should be avoided by older adults due to age-related vision changes? a. Driving after dark b. Digital photography c. Typing on the computer d. Doing crossword puzzles

a. Driving after dark Visual changes often include reduced visual fields, increased glare sensitivity, impaired night vision, reduced accommodation, reduced depth perception, and reduced color discrimination. Many of these symptoms occur because the pupils in the older adult take longer to dilate and constrict secondary to weaker iris muscles. For this reason the older adult should be encouraged to avoid driving after dark. Reading glasses are often required for typing and writing. Digital photography is not affected by age-related vision changes.

Which is the appropriate nursing action after the patient's fecal occult blood test is positive? a. Educate the patient about colonoscopy preparation. b. Obtain an order for a STAT complete blood count (CBC). c. Check the patient's rectum for the presence of impacted stool. d. Draw blood for type and cross-match testing by the blood bank.

a. Educate the patient about colonoscopy preparation. Any patient with positive fecal occult blood tests should have a colonoscopy performed to check for the presence of colon cancer. Fecal occult blood is the presence of minute amounts of blood in the stool so no CBC or transfusion is needed. There is no need for the nurse to check the patient's rectum for impacted stool.

The nurse is conducting a patient assessment. The patient tells the nurse that he has smoked two packs of cigarettes per day for 27 years. The nurse may find which data upon assessment? a. Elevated blood pressure b. Bounding pedal pulses c. Night blindness d. Reflux disease

a. Elevated blood pressure Smokers have a constriction of the blood vessels due to the tar and nicotine in cigarettes. This constriction may lead to hypertension. Bounding pulses, night blindness, and reflux disease do not have a direct link to smoking.

Which is the highest priority of the nurse immediately after withdrawing the needle from the IV catheter after venipuncture? a. Engage the safety device to cover the needle. b. Check for blood return in the flashback chamber. c. Stabilize the catheter with the nondominant hand. d. Align the catheter on the top of vein at a 15-degree angle.

a. Engage the safety device to cover the needle. The nurse's highest priority after withdrawing the needle from the IV catheter after venipuncture is to engage the safety device and cover the needle. It is absolutely essential for this to be done in order to prevent needlestick injury. The nurse will check for blood return in the flashback chamber prior to withdrawing the needle. Stabilizing the catheter with the nondominant hand is less important than engaging the safety device. The catheter is aligned on top of the vein prior to venipuncture

Which technique will the nurse use to facilitate the history and physical examination of a small child? a. Examine the child's hands and feet before listening to breath sounds. b. Direct assessment questions to the child to avoid unwanted parental influence. c. Gently palpate the child's abdomen before auscultating for bowel sounds. d. Call the parents by their first names to establish a more trusting bond.

a. Examine the child's hands and feet before listening to breath sounds Children will feel safer during an examination if it is initiated from the extremities and then moves to the torso. For example, examine the extremities before moving to the chest. It also helps to perform parts of the examination that you can do visually before actually touching the child. When obtaining histories of infants and children, the nurse should gather all or part of the information from parents or guardians. Call children by their preferred name, and address parents formally (e.g., as "Mr. and Mrs. Brown") rather than by first names.

Which assessment finding will the nurse expect to note during physical examination of the patient with a history of uncontrolled hyperthyroid disease? a. Exophthalmos b. Strabismus c. Entropion d. Diplopia

a. Exophthalmos Bulging eyes (exophthalmos) usually indicate hyperthyroidism. The crossing of eyes (strabismus) results from neuromuscular injury or inherited abnormalities. Entropion is the turning inward of eyelashes so that they irritate the conjunctiva and cornea. Strabismus refers to crossed eyes and diplopia refers to double vision.

The patient can follow directions but cannot pronounce words to express needs or thoughts. Which term will the nurse use to document this assessment finding? a. Expressive aphasia b. Receptive aphasia c. Primary dysphagia d. Vocal dysphagia

a. Expressive aphasia Expressive aphasia occurs when the patient can understand written and/or verbal speech but cannot write or speak to express needs or thoughts. Receptive aphasia occurs when the patient cannot understand written or verbal speech. Dysphagia refers to difficulty swallowing rather than communication

Which medication order will provide the most consistent control of the patient's chronic pain? a. Fentanyl transdermal patch 25 mcg b. Hydromorphone 0.5 mg IV c. Fentanyl oral lozenge 200 mcg d. Morphine sulfate liquid 10 mg

a. Fentanyl transdermal patch 25 mcg The fentanyl transdermal patch provides consistent release of the medication for 3 days. IV administration of pain medication works quickly but also wears off quickly. Morphine sulfate liquid and the fentanyl oral lozenge are used to treat breakthrough pain as they are short-acting medications

Which medication does the nurse identify as most likely to cause a patient's constipation? a. Ferrous sulfate 325 mg PO BID b. Cefaclor 500 mg PO TID c. Warfarin 5 mg PO daily d. Prednisone 10 mg PO daily

a. Ferrous sulfate 325 mg PO BID Ferrous sulfate (iron supplement) is known to cause constipation. Cefaclor and other antibiotics are known to cause diarrhea. Warfarin and prednisone do not cause constipation.

The nurse is providing discharge instructions to the patient. Which grade level should the instructions be written at as the nurse does not know the patient's educational background? a. Fifth-grade b. Seventh-grade c. Ninth-grade d. Eleventh-grade

a. Fifth-grade Individualize teaching materials to meet the patient's needs and match the patient's reading level; if a nurse does not know the patient's reading level, information should be provided at a fifth-grade or lower level. Sixth-, eighth-, and ninth-grade levels are too high.

The nurse is reviewing a patient's functional abilities and asks the patient, "How would you rate your ability to prepare a balanced meal?" "How would you rate your ability to balance a checkbook?" "How would you rate your ability to keep track of your appointments?" Which tool would be indicated for the best results of this patient's perception of their abilities? a. Functional Activities Questionnaire (FAQ) b. Mini Mental Status Exam (MMSE) c. 24hFAQ d. Performance-based functional measurement

a. Functional Activities Questionnaire (FAQ) The FAQ is an example of a self-report tool which provides information about the patient's perception of functional ability. The MMSE reviews cognitive impairment. The 24hFAQ is used to review functional ability in postoperative patients. Performance-based tools involve actual observation of a standardized task, completion of which is judged by objective criteria.

The patients with health insurance receive higher dosages of pain medications than patients who are uninsured. Which term best describes this problem? a. Health disparity b. Intersectionality c. Cultural dissonance d. Linguistic incompetence

a. Health disparity Health disparity occurs when a population is treated differently as a result of a cultural, financial, or ethnic characteristic. Administration of lower pain medication dosages to uninsured patients is an example of a health disparity. Intersectionality is the coming together of a person's cultural, political, financial, educational, and ethnic contexts. Culturally congruent care is tailored to meet the ethnic, cultural, and linguistic needs of the individual. Dissonance is an incompatibility between a person's beliefs and actions. Linguistic competence is the ability to effectively communicate with individuals or groups from other cultural or ethnic backgrounds.

Which laboratory finding indicates that the patient is likely to experience hypoxemia? a. Hematocrit 31% b. Serum creatinine 0.8 mg/dL c. Glycosylated hemoglobin 7% d. White blood cell count 4600/mm3

a. Hematocrit 31% Hemoglobin transports most oxygen and serves as a carrier for both oxygen and carbon dioxide. A hemoglobin level of 8.4 is dangerously low and puts the patient at risk of hypoxemia due to impaired oxygen carrying capacity of the blood. Glycosylated hemoglobin reflects the average blood sugar level for the last 6 weeks rather than oxygen carrying capacity. White blood cell count is a laboratory test to measure infection and is within normal range. Creatinine level of 0.8 mg/dL reflects normal kidney function

Which laboratory finding indicates that the body is attempting to compensate for the patient's end-stage chronic obstructive pulmonary disease (COPD)? a. Hemoglobin 22.1 g/dL b. Serum sodium 130 mEq/L c. Serum cholesterol 236 mg/dL d. Serum albumin level 4.8 g/dL

a. Hemoglobin 22.1 g/dL Elevated hemoglobin level indicates that the body has increased oxygen carrying capacity to compensate for end-stage COPD. Serum sodium 130 mEq/L and albumin level 4.8 g/dL are within normal limits. Elevated cholesterol levels do not compensate for the patient's end-stage chronic obstructive pulmonary disease

Which patient finding would the nurse identify as being a risk factor for altered transport of oxygen? a. Hemoglobin level of 8.0 b. Bronchoconstriction and mucus c. Peripheral arterial disease d. Decreased thoracic expansion

a. Hemoglobin level of 8.0 Altered transportation of oxygen refers to patients with insufficient red blood cells to transport the oxygen present. Bronchoconstriction and decreased thoracic expansion (spinal cord injury) would result in impairment of ventilation. Peripheral vascular disease would result in inadequate perfusion

The patient just sustained a deep laceration that is bleeding profusely. Which stage of healing describes the current state of the patient's wound? a. Hemostasis phase b. Proliferative phase c. Inflammation phase d. Remodeling phase

a. Hemostasis phase Hemostasis phase controls bleeding. Inflammation phase establishes a clean wound bed and obtain bacterial balance. Proliferative phase produces new tissue, epithelialization, and contraction. Remodeling phase reorganizes the collagen to produce a more elastic, stronger collagen for the scar tissue.

A 30-year-old male is admitted to the hospital with acute pancreatitis. He is in acute pain described as a 10/10, which is localized to the abdomen, periumbilical area, and some radiation to his back. The abdomen is grossly distended. He is restless and agitated, with elevated pulse and blood pressure. An appropriate pain management plan of care may include which medication(s)? a. IV Dilaudid q 4 hours prn, hydrocodone 5/500 PO q 6 hours prn, and acetaminophen b. Norco 5/500 q 4 hours PO and Benadryl 25 mg PO q 6 hours c. Phenergan 25 mg IM q 6 hours d. Tylenol 325 mg q 6 hours

a. IV Dilaudid q 4 hours prn, hydrocodone 5/500 PO q 6 hours prn, and acetaminophen A variety of routes of administration are used to deliver analgesics. A principle of pain management is to use the oral route of administration whenever feasible. All of the first-line analgesics used to manage pain are available in short-acting and long-acting formulations. For patients who have continuous pain, a long-acting analgesic, such as modified-release oral morphine, oxycodone, or hydromorphone, or transdermal fentanyl, is used to treat the persistent baseline pain. A fast-onset, short-acting analgesic (usually the same drug as the long-acting) is used to treat breakthrough pain if it occurs. When the oral route is not possible, such as in patients who cannot swallow or are NPO or nauseated, other routes of administration are used, including intravenous (IV), subcutaneous, transdermal, and rectal. Norco, Benadryl, Phenergan, and Tylenol are not appropriate solo choices for acute pancreatitis with pain reported as 10/10

The patient is distressed because hospitalization has prevented participation in church services and adherence to important prayer rituals. Which nursing diagnosis is most appropriate for this patient? a. Impaired religiosity related to inability to participate in religious services b. Noncompliance related to failure to adhere to prescribed treatment regimens c. Disturbed thought processes related to delirium and altered level of consciousness d. Ineffective role performance related to discrimination and inadequate coping

a. Impaired religiosity related to inability to participate in religious services Impaired religiosity is an appropriate diagnosis for this patient because hospitalization has prevented participation in the patient's religious services. Nothing in the scenario indicates that the patient is noncompliant, has experienced discrimination, or is delirious.

Which is the most appropriate nursing diagnosis to use for a patient with expressive aphasia following a stroke? a. Impaired verbal communication related to inability to speak and reply b. Readiness for enhanced comfort related to drooling and facial droop c. Deficient diversional activity related to lack of stimuli in hospital room d. Noncompliance related to inability to verbally answer questions

a. Impaired verbal communication related to inability to speak and reply Impaired verbal communication is the nursing diagnostic label used for a patient who has limited or no ability to communicate verbally. This diagnosis is useful for a wide variety of patients with special problems and needs related to communication. It is defined as difficulty or inability to use or understand language in interpersonal reactions. Readiness for enhanced comfort, deficient diversional activity, and noncompliance do not directly relate to the patient's inability to speak.

When planning to evaluate a patient's satisfaction with a teaching activity, what is the most appropriate strategy? a. Include a survey instrument. b. Observe for level of skill mastery. c. Present information more than one time. d. Provide for a return demonstration.

a. Include a survey instrument. A survey or questionnaires can be used to measure affective behavior change as well as patient satisfaction with the teaching experience. Observing for level of skill mastery would evaluate achievement of a psychomotor goal rather than satisfaction with the experience. Repeating information more than one time or in more than one way may be appropriate strategies to include in the teaching plan but would provide no evaluation data. Providing for a return demonstration would help in evaluating achievement of a psychomotor goal, not satisfaction with the activity

A young adult is balancing the need to live and work independently with the desire to have a girlfriend for companionship and love. Which developmental stage is the young adult experiencing? a. Intimacy versus isolation b. Obedience and orientation c. Self-control and independence d. Generosity versus self-absorption

a. Intimacy versus isolation The young adult is at the intimacy versus isolation stage of Erikson's theory of psychosocial development. The young adult is balancing the need for independence and personal growth with the desire to form intimate relationships with others. Generativity versus self-absorption is experienced during the middle adult years. Obedience and orientation is experienced during the early childhood/toddler years. Self-control and independence is also experienced during the early childhood/toddler years.

The nurse is admitting an older adult with decompensated congestive heart failure. The nursing assessment reveals adventitious lung sounds, dyspnea, and orthopnea. Which physician order should the nurse question? a. Intravenous (IV) 500 mL of 0.9% NaCl at 125 mL/hr b. Furosemide (Lasix) 20 mg PO now c. Oxygen via face mask at 8 L/min d. KCl 20 mEq PO two times per day

a. Intravenous (IV) 500 mL of 0.9% NaCl at 125 mL/hr A patient with decompensated heart failure has extracellular fluid volume (ECV) excess. The IV of 0.9% NaCl is normal saline, which should be questioned because it would expand ECV and place an additional load on the failing heart. Diuretics such as furosemide are appropriate to decrease the ECV during heart failure. Increasing the potassium intake with KCl is appropriate, because furosemide increases potassium excretion. Oxygen administration is appropriate in this situation of near pulmonary edema from ECV excess.

The nurse and the patient are conversing face to face. What communication technique is being demonstrated? a. Linguistic b. Paralinguistic c. Explicit d. Metacommunication

a. Linguistic Conversing face to face, reading newspapers and books, and even texting are all common forms of linguistic communication. Paralinguistics include less recognizable but important means of transmitting messages such as the use of gestures, eye contact, and facial expressions. Explicit communication is not a therapeutic communication technique. Metacommunication factors that affect how messages are received and interpreted would include internal personal states (such as disturbances in mood), environmental stimuli related to the setting of the communication, and contextual variables (such as the relationship between the people in the communication episode).

The nurse is caring for a child with tonsillar enlargement. What is the nurse's priority concern? a. Low oxygen saturation b. Daytime fatigue c. Increased temperature d. Antibiotic administration

a. Low oxygen saturation Tonsillar enlargement in children often leads to obstructive sleep apnea which can cause decreased oxygen saturation levels. Low oxygen is a priority concern which carries the highest safety risk to the child. Obstructive sleep apnea occurs in an estimated 1% to 3% of children, though the causative factors may differ with tonsillar enlargement being a significant component in children. Infection that leads to an increased temperature and requires antibiotic therapy is a concern, but the priority health concern is low oxygen levels. Obstructive sleep apnea can interfere with sleep patterns and lead to daytime fatigue, but the highest priority of care is low oxygen

The nurse is caring for a patient who has perineal skin breakdown after sitting in wet underclothes for many hours. Which term will be used to document the patient's condition in the medical record? a. Maceration b. Dehiscence c. Evisceration d. Debridement

a. Maceration Maceration is skin breakdown due to extended exposure to moisture. Dehiscence is the partial or total separation of layers of skin and tissue above the fascia in a wound that is not healing properly. Evisceration occurs when wound layers separate below the fascial layer, and visceral organs protrude through the wound opening. Debridement is removal of dead or infected tissue from a wound.

Which is the appropriate nursing action after administering a bisacodyl suppository? a. Make sure that the bedside commode is next to the patient's bed. b. Inform the patient to expect a bowel movement in the morning. c. Check the patient's colon for the presence of a fecal impaction. d. Educate the patient about methods to relieve excess gas formation.

a. Make sure that the bedside commode is next to the patient's bed A bisacodyl suppository will cause the patient to have a bowel movement in approximately 30 minutes so the bedside commode should be ready next to the patient's bed. The colon should be checked for fecal impaction prior to administration of a laxative suppository. There is no need to educate the patient about methods to relieve excess gas formation.

A patient complains of not being able to fall asleep at night and asks the nurse if there is a safe, non-prescription medication he can try. After consulting the healthcare provider, the nurse should recommend which naturally occurring hormone? a. Melatonin b. Cortisol c. Luteinizing hormone d. Estrogen

a. Melatonin A rise in the hormone melatonin at the onset of sleep helps to promote and maintain sleep, and a drop in levels leads to eventual awakening. The immune system is enhanced as well during sleep as proteins associated with fighting illness are produced. The circadian rhythm, or the typical 24 hour (more or less) cycle through which the body passes, including both awake and sleep cycles, is responsible for regulating all of the physiologic processes in the body. An adequate amount and quality of sleep, therefore, is essential to all of the regulatory mechanisms that take place in the body.

Which is the safest sleep aid for the elderly patient with insomnia? a. Melatonin b. Trazodone c. Temazepam d. Triazolam

a. Melatonin Melatonin is the safest option for an elderly patient with insomnia although it is meant for short-term use. Trazodone can cause orthostatic hypotension. Temazepam and triazolam can cause cognitive impairment in older adults.

The nurse is caring for a diabetic patient with a blood sugar of 879 mg/dL, and notes the patient has fruity breath, and Kussmaul respirations. Which acid-base imbalance should the nurse expect? a. Metabolic acidosis b. Metabolic alkalosis c. Respiratory acidosis d. Respiratory alkalosis

a. Metabolic acidosis Diabetic ketoacidosis is a common cause of metabolic acidosis that occurs in patients with dangerously high blood sugars. Kussmaul respirations are a compensatory mechanism of the body during metabolic acidosis to raise the serum pH by blowing off additional carbon dioxide

The nurse is caring for a patient who was pulled unconscious from the pool after a near drowning. Which lung sounds will the nurse expect to hear upon auscultation? a. Moist crackles b. Expiratory wheezes c. Friction rub d. Coarse rhonch

a. Moist crackles Fluid in the alveoli causes high-pitched crackles. Wheezes would be expected for a patient with asthma or an allergic reaction. Friction rub is heard with irritation of the pleura. Rhonchi is caused by the presence of thick secretions in the airways.

A 19-year-old male has sustained a transaction of C-7 in a motor vehicle crash rendering him a quadriplegic. He describes his pain as burning, sharp, and shooting. What type of pain is this patient describing? a. Neuropathic pain b. Visceral pain c. Eudynic pain d. Nociceptive pain

a. Neuropathic pain Neuropathic pain results from the abnormal processing of sensory input by the nervous system as a result of damage to the brain, spinal cord, or peripheral nerves. Simply put, neuropathic pain is pathologic. Examples of neuropathic pain include postherpetic neuralgia, diabetic neuropathy, phantom pain, and post stroke pain syndrome. Patients with neuropathic pain use very distinctive words to describe their pain, such as "burning," "sharp," and "shooting." Visceral pain arises from the body cavities and responds to stretching, swelling, and decreased oxygen levels. Eudynic pain refers to the normal transmission of pain via nociceptive receptors. Nociceptive pain refers to the normal functioning of physiological systems that leads to the perception of noxious stimuli (tissue injury) as being painful. Patients describe this type of pain as aching, cramping, or throbbing

Which technique is used to assess fine-motor skills in a middle-school child? a. Observe the child drawing a picture on a piece of paper. b. Watch the child sit on the floor and rise to a standing position. c. Ask the child to walk a straight line in a heel-toe (tandem) gait. d. Ask the child to stand on one leg and hop in place several times

a. Observe the child drawing a picture on a piece of paper. Fine-motor skills can be assessed by watching the child draw a picture on a piece of paper. This allows the nurse to see how the pencil is held, check for tremors, and observe hand muscle strength. Rising to a standing position assesses muscle strength. Walking heel-toe and standing on one leg assesses balance

Which intake and output measurement can the nurse delegate to the nursing assistant? a. Oral fluid intake b. Nasogastric tube intake c. Wound drainage output d. Intravenous fluid intake

a. Oral fluid intake You can delegate portions of intake and output measurement to nursing assistants. In many agencies nursing assistants can record oral intake but not intake through tubes or IVs. Nursing assistants can record urine, diarrhea, and vomitus output, but not drainage through wound drainage tubes.

The patient becomes acutely short of breath when lying flat in bed. Which term will the nurse use to document this assessment finding? a. Orthopnea b. Atelectasis c. Emphysema d. Stridor

a. Orthopnea Difficulty breathing while lying flat is termed orthopnea. Atalectasis is collapse of alveolar lung tissue. Emphysema is destruction of the alveoli. Stridor is a crowing sound heard as the patient struggles to breathe through swollen upper airways.

The nurse is caring for a patient with a pulmonary embolism that prevents blood flow to the lower lobe of the right lung. Which breathing process is impaired for this patient? a. Perfusion b. Diffusion c. Respiration d. Ventilation

a. Perfusion Perfusion refers to the movement of blood into and out of the lungs to the organs and tissues of the body. The patient's blood flow through the lungs is blocked by the clot so the patient's pulmonary perfusion is impaired. Diffusion is the movement of gases between air spaces and the bloodstream. Ventilation refers to the movement of air in and out of the lungs. Respiration is the exchange of oxygen and carbon dioxide during cellular metabolism

The nurse would identify which patient condition as a problem of impaired gas exchange secondary to a perfusion problem? a. Peripheral arterial disease of the lower extremities b. Chronic obstructive pulmonary disease (COPD) c. Chronic asthma d. Severe anemia secondary to chemotherapy

a. Peripheral arterial disease of the lower extremities Perfusion relates to the ability of the blood to deliver oxygen to the cellular level and return the carbon dioxide to the lung for removal. COPD and asthma are examples of a ventilation problem. Severe anemia is an example of a transport problem of gas exchange.

The patient lives in an apartment and has difficulty getting on and off of the toilet. Which is the best intervention to protect the patient's safety? a. Place a bedside commode over the toilet. b. Screw grab bars into the wall by the toilet. c. Attach suction cup grab bars to the wall by the toilet. d. Instruct the patient to lean forward when rising to stand

a. Place a bedside commode over the toilet. b. Screw grab bars into the wall by the toilet A bedside commode should be placed over the patient's toilet to make it safer for the patient. Grab bars should not be screwed into the wall as the patient is renting the apartment. Suction cup grab bars may not attach securely to the wall. The patient should not be instructed to lean forward when rising to stand

Which intervention will help prevent the development of footdrop for a comatose patient? a. Place high-top tennis shoes on the patient's feet. b. Place pillows under the legs to keep the heels off the mattress. c. Apply sequential compression devices to the patient's feet. d. Assist the patient to a lateral position whenever possible.

a. Place high-top tennis shoes on the patient's feet High-top tennis shoes will help keep the patient's feet from becoming locked in plantar flexion known as footdrop. Elevating the heels off the mattress will prevent the development of decubitus ulcers on the heels. Sequential compression devices will help prevent formation of blood clots. Lateral position will not help prevent footdrop.

Which is the highest priority nursing diagnosis for a patient with a spinal cord injury and no pain sensation below the waist? a. Potential for injury related to lack of sensation and protective reflexes b. Disturbed body image related to loss of body function and sensation c. Readiness for enhanced self-care related to desire for increased independence d. Risk for loneliness related to discomfort in social situations due to disability

a. Potential for injury related to lack of sensation and protective reflexes The patient is at high risk of injury to the lower body due to the inability to feel pain or other sensations. The patient will not be able to tell when a body part is being pinched or burned before significant damage occurs. Risk for injury is higher priority than the patient's psychosocial needs for body image, self-care, or loneliness

Controlling pain is important to promoting wellness. Unrelieved pain has been associated with complication? a. Prolonged stress response and a cascade of harmful effects system-wide b. Large tidal volumes and decreased lung capacity c. Decreased tumor growth and longevity d. Decreased carbohydrate, protein, and fat destruction

a. Prolonged stress response and a cascade of harmful effects system-wide Pain triggers a number of physiologic stress responses in the human body. Unrelieved pain can prolong the stress response and produce a cascade of harmful effects in all body systems. The stress response causes the endocrine system to release excessive amounts of hormones, such as cortisol, catecholamines, and glucagon. Insulin and testosterone levels decrease. Increased endocrine activity in turn initiates a number of metabolic processes, in particular, accelerated carbohydrate, protein, and fat destruction, which can result in weight loss, tachycardia, increased respiratory rate, shock, and even death. The immune system is also affected by pain as demonstrated by research showing a link between unrelieved pain and a higher incidence of nosocomial infections and increased tumor growth. Large tidal volumes are not associated with pain while decreased lung capacity is associated with unrelieved pain. Decreased tumor growth and longevity are not associated with unrelieved pain. Decreased carbohydrate, protein, and fat are not associated with pain or stress response.

What is the primary advantage of a hydrogel dressing for wound healing? a. Provide moisture needed for wound healing. b. Act as an absorbent to collect wound drainage. c. Provide negative pressure to promote healing. d. Provide protection from the external environment.

a. Provide moisture needed for wound healing. Hydrogels maintain moisture in some wounds for 1 to 3 days. Hydrogel dressings are available in sheets or in a gel in a tube (amorphous). They contain a high percentage of water and are indicated for wounds that require moisture, either a wound with granulation (maintaining the moist wound environment needed for healing) or a wound that has a high percentage of necrotic tissue (the hydrogel facilitates debridement by softening the dead tissue). Negative-pressure wound therapy supports wound healing by evacuating wound fluids, stimulating granulation tissue formation, reducing the bacterial burden of a wound, and maintaining a moist wound environment. Gauze dressings are best for wounds with moderate drainage, deep wounds, undermining, and tunnels. You apply gauze either moist or dry. The moistened gauze increases the absorptive ability of the dressing to collect exudate. A transparent or hydrocolloid dressing protects against the external environment

The nurse is discontinuing the patient's indwelling urinary catheter. The catheter is not easily withdrawn after the balloon is deflated. Which is the appropriate nursing action? a. Reattach the syringe and attempt to withdraw more water from the balloon. b. Ask the patient to bear down as the catheter is withdrawn with gentle pressure. c. Review the patient's chart to see how much water was inserted into the balloon. d. Explain to the patient that removal of the catheter may cause significant discomfort.

a. Reattach the syringe and attempt to withdraw more water from the balloon. The syringe should be reattached to withdraw any remaining water from the balloon. The nurse should not ask the patient to bear down; catheter removal should be painless. The nurse should check the balloon for more water rather than checking the chart to see how much water was instilled.

The nurse is caring for a patient who is having a heart attack. The patient tells the nurse that the pain is down his left arm rather than in his chest. What type of pain is the patient experiencing? a. Referred b. Psychogenic c. Peripheral d. Chronic

a. Referred Referred pain is when the patient experiences discomfort at an area other than the source of the pain. In this case, the patient has left arm pain even though the source of the pain is the heart. Psychogenic pain is discomfort that is associated with psychological disorders. Peripheral pain is a type of neuropathic pain due to damaged peripheral nerves of the hands and feet. A heart attack is an example of acute pain rather than chronic pain

A patient uses rosary beads and attends mass once a week. This expression of spirituality is best described with which term? a. Religiosity b. Faith c. Belief d. Authenticity

a. Religiosity There are a few similar and related terms to spirituality worth mentioning to provide distinction and clarification. Faith, as defined by Dyess, refers to an "evolving pattern of believing, that grounds and guides authentic living and gives meaning in the present moment of inter-relating." Religiosity, another similar term, is an external expression (public or private), in the form of practicing a belief or faith, whereas spirituality is an internalized spiritual identity (or experiential). Specifically, religiosity is defined as "the adherence to religious dogma or creed, the expression of moral beliefs, and/or the participation in organized or individual worship, or sacred practices."

A patient smokes in the hospital bathroom and starts a fire. Which is the nurse's first response? a. Remove the patient to a safe area. b. Close the door to contain the fire. c. Call the operator to activate the fire alarm. d. Utilize a fire extinguisher to put out the fire.

a. Remove the patient to a safe area. b. Close the door to contain the fire Use the mnemonic RACE to set priorities in case of fire: R—Rescue and remove all patients in immediate danger. A—Activate the alarm. Always do this before trying to extinguish even a minor fire. C—Confine a fire by closing doors and windows and turning off oxygen and electrical equipment. E—Extinguish a fire using an appropriate extinguisher. Reporting, attempting to extinguish, and closing the door all occur after assisting patients to a safe area.

The emergency room staff insists that a female Muslim patient disrobe completely and put on a hospital gown before being treated by the physician. Which nursing diagnosis is most applicable in this situation? a. Risk for compromised human dignity related to cultural attire requirements b. Ineffective role performance related to inadequate direction and preparation c. Noncompliance related to expected behavior of patients in the emergency room d. Impaired environmental interpretation syndrome related to busy emergency room

a. Risk for compromised human dignity related to cultural attire requirements Many female Muslim patients believe in covering the entire body at all times except for the face, hands, and feet. If the emergency room staff insists that the patient disrobe completely for the physician, the patient's human dignity may be put at risk. The patient has not demonstrated any noncompliance behaviors and has not shown any difficulty interpreting the emergency room environment. The ineffective role performance diagnosis does not apply to the patient

Which is the highest priority nursing diagnosis for a patient who developed sleep-related eating disorder when taking zolpidem? a. Risk for falls related to ambulating to kitchen while asleep b. Wandering related to cognitive impairment from sleeping aid c. Powerlessness related to inability to keep from eating during sleep d. Risk for imbalanced nutrition: more than body requirements related to sleep eating

a. Risk for falls related to ambulating to kitchen while asleep The highest priority nursing diagnosis is the risk for falls as serious injury may result. Wandering and powerlessness are less important than falls. Weight gain is less dangerous than the patient's risk of falling. Significant weight gain should not occur once the patient stops taking the medication.

Which is the priority nursing diagnosis for a patient with orthostatic hypotension after several days of bed rest? a. Risk for falls related to light-headedness upon standing and getting out of bed b. Dressing/grooming self-care deficit related to cognitive impairment and fatigue c. Impaired bed mobility related to inability to move from supine to sitting position d. Excess fluid volume related to insensible fluid loss due to prolonged immobility

a. Risk for falls related to light-headedness upon standing and getting out of bed The patient with orthostatic hypotension is at risk for falls due to light-headedness when standing. Dressing/grooming self-care deficit is not a high priority diagnosis. Impaired bed mobility and excess fluid volume are not relevant for this patient.

Which is the priority nursing diagnosis for a patient with a continuous epidural infusion of fentanyl and bupivacaine? a. Risk for impaired gas exchange related to respiratory suppression b. Activity intolerance related to generalized weakness and bed rest c. Impaired physical mobility related to presence of epidural catheter d. Delayed surgical recovery related to need for continuous pain management

a. Risk for impaired gas exchange related to respiratory suppression Epidural infusion of fentanyl and bupivacaine can lead to respiratory suppression and risk for impaired gas exchange. Activity intolerance, impaired physical mobility, and delayed surgical recovery are less important than gas exchange.

Which is the highest priority nursing diagnosis for a patient who is starting CPAP therapy for sleep apnea? a. Risk for impaired skin integrity related to tight-fitting mask on face b. Impaired bed mobility related to presence of CPAP mask on face c. Health-seeking behaviors related to expressed desire for better sleep d. Risk for powerlessness related to inability to breathe regularly during sleep

a. Risk for impaired skin integrity related to tight-fitting mask on face The CPAP mask can cause impaired skin integrity to the face unless it is applied and fitted correctly. The CPAP mask on the face will not cause impaired bed mobility. The patient is at low risk for powerlessness. Health-seeking behaviors are less important than the risk of skin breakdown.

Which nursing diagnosis is the highest priority for a sexually active adolescent? a. Risk for infection related to participation in unprotected sexual activity b. Disturbed body image related to depersonalization and fear of rejection c. Spiritual distress related to inability to integrate sexuality with church teaching d. Risk for compromised human dignity related to loss of respect from peer group

a. Risk for infection related to participation in unprotected sexual activity Risk for infection related to participation in unprotected sexual activity is the highest priority nursing diagnosis as the patient's physical health is in danger. Disturbed body image, spiritual distress, and risk for compromised human dignity may be addressed once the patient has demonstrated understanding of safe sex practices.

A patient's spouse smokes cigarettes in the kitchen while the patient uses supplemental oxygen in the bedroom. Which is the priority nursing diagnosis for this family? a. Risk for injury related to smoking near supplemental oxygen b. Risk-prone health behavior related to inability to quit smoking c. Ineffective health maintenance related to continued use of cigarettes d. Ineffective family therapeutic regimen management related to noncompliance

a. Risk for injury related to smoking near supplemental oxygen The highest priority nursing diagnosis is risk for injury (burns, fire) related to smoking near supplemental oxygen. A life-threatening fire could easily develop as the supplemental oxygen is near the spouse's cigarettes. Risk-prone health behavior, ineffective health maintenance, and ineffective family therapeutic regimen management are not the priority nursing diagnoses; these can be addressed after the safety risk has been mitigated.

The nurse is caring for a patient with a poorly controlled hypoparathyroid condition. Which is the highest priority nursing diagnosis for this patient? a. Risk for injury/fracture related to bone fragility b. Disturbed energy field related to chronic illness c. Deficient knowledge related to importance of exercise d. Disturbed body image related to hormonal changes

a. Risk for injury/fracture related to bone fragility Hypoparathyroid disease can lead to low serum calcium levels and osteoporosis, putting the patient at risk for developing a pathological fracture. Risk for fracture is the highest priority diagnosis for this patient. Disturbed energy field, deficient knowledge, and disturbed body image are not as important as the risk for fracture

The nurse observes a postoperative patient trying to take a friend's oxycodone pain pills in addition to the pain medication administered by the nurse. Which is the priority nursing diagnosis for this patient? a. Risk for poisoning b. Situational low self-esteem c. Ineffective impulse control d. Readiness for enhanced comfort

a. Risk for poisoning The patient is at risk for overdose of pain medication by taking the friend's medication as well as what is administered by the nurse. The nurse needs to institute safety precautions to prevent the patient from taking excessive amounts of pain medications. Low self-esteem, impulse control, and enhanced comfort may be addressed after the risk for poisoning has been resolved

Which nursing diagnosis is the highest priority for a family with small children? a. Risk for suffocation related to unattended swimming pool in back yard b. Risk for caregiver role strain related to four children under 6 years of age c. Readiness for enhanced immunization status related to request for medical records d. Readiness for enhanced parenting related to attachment between family members

a. Risk for suffocation related to unattended swimming pool in back yard Risk for suffocation related to unattended swimming pool in back yard is the highest priority nursing diagnosis because drowning could lead to permanent injury or death. Risk for caregiver role strain, readiness for enhanced immunization status, and readiness for enhanced parenting are lower priority diagnoses as they address psychosocial needs. They can be attended to after the immediate risk for danger is addressed

The nurse is caring for a patient who feels the urge to urinate but is unable to void. Which is the appropriate action of the nurse? a. Scan the patient's bladder to see how much urine is present. b. Obtain a urine sample for urinalysis, culture, and sensitivity. c. Perform a focused physical assessment of the patient's perineum. d. Help the patient to utilize absorbent undergarments for protection

a. Scan the patient's bladder to see how much urine is present. The appropriate action of the nurse is to scan the patient's bladder to determine how much urine is present. The nurse may need to catheterize the patient if a very large amount of urine is in the bladder or encourage fluids if there is minimal urine present. Assessment of the perineum will not assist the patient to void. The nurse will not be able to send a urine sample until the patient voids or is catheterized. Absorbent undergarments are not needed if the patient is unable to void

Which is the appropriate action of the nurse when selecting a vein for IV placement? a. Select a vein that appears to be well dilated. b. Elevate the extremity to visualize the vein. c. Tap and rub the vein vigorously with friction. d. Stroke from proximal to distal above the site

a. Select a vein that appears to be well dilated Use the most distal site in the nondominant arm, if possible. Select a well-dilated vein. Methods to foster venous distention include place the extremity in a dependent position if possible and stroke from distal to proximal below the proposed venipuncture site. Apply warmth to the extremity for several minutes, for example, with a warm washcloth. Vigorous friction and multiple tapping of a vein, especially in older adults, can cause hematoma and/or venous constriction. Avoid vein selection in areas with tenderness, pain, infection, or wounds, or extremities affected by previous stroke (CVA), paralysis, mastectomy, or dialysis graft. Choose a site that will not interfere with the patient's activities of daily living (ADLs)

Which assessment finding indicates to the nurse that the patient is at high risk for developing a pressure injury? a. Serum total protein level of 4.6 g/dL b. Braden Scale score of 22 c. Cetirizine 5 mg PO daily d. Fasting serum glucose level 84 mg/dL

a. Serum total protein level of 4.6 g/dL Poor nutrition, specifically severe protein deficiency, causes soft tissue to become susceptible to breakdown. Serum protein level of 4.6 g/dL leads to edema or swelling, which contributes to problems with the transportation of oxygen and nutrients. A hospitalized adult with a score of 16 or below and an older adult at 18 or below are at risk for pressure injury development; a score of 22 does not place the patient at risk. Cetirizine is a nonsedating antihistamine medication that would not place the patient at risk for developing pressure injuries. Fasting serum glucose level of 84 mg/dL is within normal limits and does not indicate risk of pressure injury development.

A 60-year-old Italian immigrant presents for an annual physical. He is counseled about diagnostic testing including laboratory testing, colonoscopy, influenza vaccination, and pneumococcal vaccination. His reply is "If it isn't broke, don't try to fix it." When developing a plan of care, the nurse should consider which cultural orientation for this patient? a. Short term b. Long term c. Leisurely term d. Noncommittal

a. Short term Short-term cultural orientation focuses on the present or past and emphasizes quick results. Long-term cultural orientation focuses on the future and long-term rewards. Long-term-oriented cultures favor thrift, perseverance, and adopting to changing circumstances. Leisurely term and noncommittal are undefined in cultural orientation

Why is it important for a postoperative patient to get enough sleep after being discharged from the hospital? a. Sleep restores biological processes. b. Sleep stimulates appetite on waking. c. Sleep causes a mental and physiological calm. d. Sleep produces dreams that decrease epinephrine.

a. Sleep restores biological processes. Because the patient is postoperative, the primary reason for sleep is to help the body to heal by restoring biological processes. Sleep allows the body to restore biological processes. During deep slow-wave (NREM N3) sleep, the body releases human growth hormone for the repair and renewal of epithelial and specialized cells such as brain cells. Protein synthesis and cell division for the renewal of tissues also occur during rest and sleep. The basal metabolic rate is lowered during sleep, which conserves the body's energy supply. REM sleep is important for cognitive restoration. During REM sleep, patients experience rapid eye movement, fluctuation in heart and respiratory rate, increased/fluctuating blood pressure, loss of skeletal muscle tone, and increase in gastric secretions. (This is not a mental and physical calm.) During REM sleep, there is increased oxygen consumption and epinephrine is released, not decreased. Sleep does not cause an increase in appetite as the metabolic rate slows down.

Which health care professional will be of most assistance to help the patient with aphasia following a stroke? a. Speech therapist b. Medical interpreter c. Physical therapist d. Mental health nurse specialist

a. Speech therapist Speech therapists help patients with aphasia. The nurse should collaborate with other health care providers who have expertise in communication strategies. Medical interpreters are invaluable when a patient speaks a foreign language. Mental health advanced practice nurses help in communicating with angry or highly anxious patients. Physical therapists help with mobility issues

The nurse is caring for a patient who expresses anger about being abandoned and unloved by God since becoming ill. Which nursing diagnosis is appropriate for this patient? a. Spiritual distress related to perceived alienation from God b. Risk for loneliness related to social and physical isolation c. Acute confusion related to hallucinations and misperceptions d. Impaired memory related to inability to remember familiar prayers

a. Spiritual distress related to perceived alienation from God The nursing diagnosis spiritual distress related to perceived alienation from God is appropriate for this patient due to feelings of alienation from God since becoming ill. The patient scenario does not indicate social or physical isolation, impaired memory, or hallucinations. Feeling abandoned by God is not a hallucination.

The patient has a nonblanchable area of redness on the right malleolus. Which pressure injury stage will be recorded in the patient's chart? a. Stage 1 b. Stage 2 c. Stage 3 d. Stage 4

a. Stage Stage 1: Intact skin with nonblanchable redness of a localized area, usually over a bony prominence Stage 2: Partial-thickness loss of dermis presenting as a shallow open ulcer with a red-pink wound bed, without slough; may also present as an intact or open/ruptured serum-filled blister Stage 3: Full-thickness tissue loss; subcutaneous fat may be visible, but bone, tendon, or muscle is not exposed; slough may be present but does not obscure the depth of tissue loss; may include undermining and tunneling Stage 4: Full-thickness tissue loss with exposed bone, tendon, or muscle; slough or eschar may be present; often includes undermining and tunneling

The nurse enters a patient's room and the patient startles easily and appears to jerk his arms and legs before awakening. Which stage of non-rapid eye movement sleep did the patient most likely awaken from? a. Stage 1 b. Stage 2 c. Stage 3 d. Stage 4

a. Stage 1 In Stage 1 (N1 or Non-REM Stage 1), referred to as light sleep, the individual can be awakened easily. In this stage, brain waves slow and, on EEG, the slower wave pattern known as alpha waves appears. The individual at this point is likely to drift in and out of sleep and can be awakened easily. Sensations known as hypnagogic hallucinations can occur during this stage. A common sensation of this type is the feeling of falling. Uncontrolled muscle jerks sometimes occur at this stage along with sudden movements that can startle the individual and restore wakefulness.

The patient knocked over the specimen container and spilled some of the urine that had been collected for a 24-hour urine analysis. Which is the appropriate action of the nurse? a. Start the collection over again with a new container. b. Inform the patient that the test will have to be canceled c. Replace the lid on the container and continue the collection. d. Extend the collection period by 2 hours for a replacement void.

a. Start the collection over again with a new container The nurse should start the collection over again with a new container as the specimen collection will not be accurate. The test does not need to be canceled. The nurse cannot extend the collection period as that will lead the test to be inaccurate.

The nurse is providing discharge instructions to a patient with memory loss after a head injury. What is the most appropriate action of the nurse? a. Teach the patient and a responsible family member at the same time. b. Teach the patient using simple terminology and a louder tone of voice. c. Teach the patient the most important information first followed by lesser facts. d. Teach the patient immediately before discharge so the patient will remember it.

a. Teach the patient and a responsible family member at the same time. The discharge information should be provided to the patient's responsible family member as well as the patient since the patient may not remember it. Speaking loudly will not help the patient to remember the information. Giving the most important information first immediately before discharge will not help the patient to remember the information.

Which assessment charting indicates that the wound is healing by primary intention? a. The 4-inch incision edges are well approximated with intact sutures. b. Ulcerated 3-inch 1-inch area has thick yellow slough present in the center. c. Incision is 5 inch long 1 inch deep 1 inch wide with granulation tissue present. d. Superficial 3-inch 3-inch abrasion has no active bleeding, drainage or debris.

a. The 4-inch incision edges are well approximated with intact sutures. A clean surgical incision with sutures is healing by primary intention. Open wounds such as open incisions, ulcers, and abrasions heal by secondary intention.

Which respiratory problem is experienced by premature infants due to lack of surfactant in their lungs? a. The alveoli shrivel and are unable to exchange oxygen for carbon dioxide. b. Weakness of the respiratory muscles limits airflow in and out of the lungs. c. Swelling of abdominal organs limits expansion and contraction of the diaphragm. d. Insufficient hemoglobin impairs delivery of oxygen to tissues throughout the body.

a. The alveoli shrivel and are unable to exchange oxygen for carbon dioxide. Surfactant works to keep alveoli expanded so that oxygen can be exchanged for carbon dioxide. Lack of surfactant leads the alveoli to shrivel up preventing diffusion of gases across the alveolar membrane. Lack of surfactant does not affect diaphragm function, hemoglobin levels, or respiratory muscle strength.

The nurse is caring for a pregnant mother of two small children who is on bed rest for several months until the baby is born. Which outcome is most appropriate to address the diagnosis of compromised family coping related to mother's prolonged bed rest? a. The family will verbalize need for support and identify available resources. b. The family will discuss alternatives to bed rest with the health care provider. c. The mother will verbalize need for bed rest to minimize risk of premature birth. d. The mother will report increased psychological comfort with each passing week.

a. The family will verbalize need for support and identify available resources. The entire family is affected when the patient must be on bed rest for several months. The most appropriate outcome is for the family to verbalize need for support and identify available resources. Discussing alternatives to bed rest and verbalization of need for bed rest do not meet the family's need for assistance with coping. The mother's report of increased psychological comfort does not reflect the needs of the entire family.

The nurse hears a distinctive swooshing sound with each heartbeat when auscultating the patient's heart sounds. What is the cause of this assessment finding? a. The mitral valve is leaking and some blood leaks backward with each heartbeat. b. The nurse's stethoscope is not placed firmly enough against the patient's skin. c. The patient's tricuspid valve was replaced with a mechanical prosthetic valve. d. The patient is having short runs of premature supraventricular contractions.

a. The mitral valve is leaking and some blood leaks backward with each heartbeat. A distinctive swooshing sound with each heartbeat indicates there may be damage to the heart valves, e.g., leakage from the mitral valve. Incorrect placement of the stethoscope would limit the nurse's ability to hear any heart sounds at all. Mechanical prosthetic valves cause clicking sounds rather than swooshing sounds. Runs of premature supraventricular contractions would cause an irregular heart rhythm not swooshing sounds.

Which is the most appropriate learning goal for new parents who are learning infant CPR? a. The parents will demonstrate infant CPR skills. b. The parents will be able to understand CPR skills. c. The infant will not require further hospitalization. d. The parents will call the hospital if the infant stops breathing.

a. The parents will demonstrate infant CPR skills. A learning objective describes what the patient or guardian(s) will be able to do after successful instruction. The objective contains an active verb describing what the learner will do after the objective is met (demonstrate). Understand does not specify the behavior or content to be learned and is not an active verb. The parent's "understanding" is not measureable, and learning goals need to be measureable. The best learning goal in the case of a skill is to demonstrate that skill. The learning objectives should focus on the parents as they are the learners; it should not focus on the infant. The parents should call the hospital for help but this does not relate to the skill being taught, CPR.

Which assessment finding leads the nurse to clarify the patient's order for morphine sulfate controlled release 60 mg PO every 12 hours? a. The patient cannot swallow pills. b. The patient is allergic to latex and NSAIDs. c. The patient's platelet count is 200,000/mm3 . d. The patient does not have an intravenous line

a. The patient cannot swallow pills. Morphine sulfate controlled release is an extended-release tablet that may not be crushed. This assessment finding leads the nurse to clarify the order. Allergies to latex and NSAIDs, normal platelet count and lack of an IV line are not contraindications for administration of morphine sulfate controlled release

Which assessment finding explains the patient's tachycardia? a. The patient drinks at least eight cans of diet cola every day. b. The patient takes digoxin 0.125 mg PO daily. c. The patient has a history of untreated hypothyroid disease. d. The patient takes metoprolol 50 mg PO daily

a. The patient drinks at least eight cans of diet cola every day Caffeine in diet cola causes elevation of the heart rate and tachycardia. Untreated hypothyroid disease, digoxin, and metoprolol cause bradycardia.

Which assessment finding indicates that the patient developed osteoporosis after a long period of immobility? a. The patient fractured three ribs after coughing on spicy food. b. The patient is unable to dorsiflex the foot, leading to an unsteady gait. c. The patient has a steppage gait with exaggerated flexion of the hip and knee. d. The patient's knees curve inward and bump against each other with ambulation.

a. The patient fractured three ribs after coughing on spicy food. Fracturing three ribs due to coughing indicates weakened bone structure consistent with osteoporosis. Inability to dorsiflex the foot, walking with a steppage gait, and knock knees do not demonstrate osteoporosis

The nurse is caring for a patient who is very malnourished and laboratory results show that the patient's albumin level is extremely low. Which finding would the nurse expect to observe during the patient's physical assessment? a. The patient has generalized edema. b. The patient is confused and irritable. c. The patient has an irregular heartbeat. d. The patient's eyes are red, irritated, and itchy

a. The patient has generalized edema. Albumin is responsible for keeping water within the bloodstream using oncotic pressure. Low albumin levels allow water to seep from the bloodstream into the tissues causing generalized edema. Confusion, irregular pulse, and irritated eyes are not expected with low albumin levels.

The nurse notes that the patient has hyperactive bowel sounds in all four quadrants of the abdomen. Which factor from the patient's history will account for this assessment finding? a. The patient has had gastroenteritis for the last 4 days with watery diarrhea. b. The patient takes iron supplements daily to treat chronic anemia.. c. The patient has a paralytic ileus after undergoing abdominal surgery. d. The patient takes ibuprofen 3 times daily for arthritis pain

a. The patient has had gastroenteritis for the last 4 days with watery diarrhea. Gastroenteritis with watery diarrhea will cause hyperactive bowel sounds due to increased motility. Iron supplements and NSAIDs like ibuprofen will not cause hyperactive bowel sounds. No bowel sounds are expected when the patient has a paralytic ileus.

Which intrinsic assessment finding could lead a patient to fall? a. The patient has orthostatic hypotension and faints when standing too quickly. b. The patient's room is located at the end of the hall far from the nursing station. c. The patient's roommate sometimes spills the contents of a urinal on the floor. d. The patient's room is crowed with walkers, wheelchairs, and bedside commodes.

a. The patient has orthostatic hypotension and faints when standing too quickly Intrinsic factors that can lead to falls are patient-related such as orthostatic hypotension. Extrinsic factors are environmentally related, such as spills, clutter, and patient room location.

Which assessment findings lead the nurse to suspect that the patient has a history of alcohol abuse? a. The patient has prominent tiny blood vessels running across the face. b. The patient's skin is abnormally dry and flaky, especially on the legs. c. The patient's nails have splinter hemorrhages along the nail bed. d. The patient's breath smells faintly of fruity chewing gum.

a. The patient has prominent tiny blood vessels running across the face Spider angiomas are prominent tiny blood vessels commonly running across the face and upper part of the body. They are commonly seen on patients with a long history of alcohol abuse. Dry flaky skin is commonly seen with aging. Splinter hemorrhages are seen with endocarditis. Breath that smells faintly of fruity chewing gum indicates diabetic ketoacidosis.

Which assessment finding indicates that the patient is at higher risk for a motor vehicle accident? a. The patient is 18 years of age. b. The patient drives a bright blue sedan. c. The patient is the youngest of six children. d. The patient has a history of juvenile arthritis.

a. The patient is 18 years of age According to the CDC, the risk of motor vehicle accidents is higher among 16- to 19-year-old drivers than any other age-group. Therefore the patient's age of 18 years indicates a higher risk of a motor vehicle accident. Driving a bright blue sedan, being the youngest child and a history of arthritis do not put the patient at higher risk of motor vehicle accident

Which assessment finding indicates that the patient is at risk for developing hypothermia? a. The patient is disoriented due to senile dementia. b. The patient is 5-foot 6-inch tall with a BMI of 35. c. The patient has a history of congestive heart failure. d. The patient takes three different antihypertensive medications.

a. The patient is disoriented due to senile dementia. Elderly confused patients are at risk for hypothermia due to wandering and unawareness of surroundings. The patient's BMI of 35 indicates obesity which would not put the patient at risk for hypothermia. Antihypertensive medications and a history of CHF do not increase risk of hypothermia.

Which assessment findings will the nurse communicate to the physician using the SBAR tool? a. The patient is having difficulty breathing and the pulse oximetry is 75%. b. The patient has not had a bowel movement since surgery eight hours ago. c. The patient's family member initially refused to learn how to perform the dressing changes. d. The patient sent the breakfast tray back to the kitchen because the food was cold.

a. The patient is having difficulty breathing and the pulse oximetry is 75%. SBAR stands for situation, background, assessment, and recommendation. Use of common language when communicating critical information helps prevent misunderstandings. SBAR has become a best practice for standardizing communication between health care providers. SBAR is not used to communicate low-priority assessment findings such as constipation, initial refusal to learn, or cold breakfast trays.

Which assessment finding indicates to the nurse that the patient is experiencing difficulty with proprioception? a. The patient must hold on to the railing when ambulating in the hallway. b. The patient must add extra seasoning to food in order for it to have any flavor. c. The patient did not smell smoke even though the smoke detector was alarming. d. The patient suffered a first-degree burn when a heating pad was left on too long

a. The patient must hold on to the railing when ambulating in the hallway. Proprioception is the patient's ability to balance and maintain position. The patient's proprioception is affected when the patient is unable to ambulate without holding on to the handrail. Proprioception does not affect taste, smell, or sensation.

Which patient is appropriate for the nursing diagnosis readiness for enhanced knowledge related to the prescribed treatment regimen? a. The patient who asks the nurse how a pill organizer can help to ensure that all medications are taken on time. b. The patient who insists that the blood sugar levels will never stabilize no matter how many medications are taken. c. The patient who believes that influenza was contracted as a result of the flu immunization last year. d. The patient who was just diagnosed with diabetes and has no idea about how to inject insulin.

a. The patient who asks the nurse how a pill organizer can help to ensure that all medications are taken on time Readiness for enhanced knowledge related to the prescribed treatment regimen indicates that the patient is already knowledgeable and wishes to learn more. Readiness for enhanced knowledge is appropriate for the patient who asks the nurse how a pill organizer can help to ensure that all medications are taken on time. Readiness for enhanced knowledge is not appropriate for patients who do not want to learn or who have not obtained a basic understanding of the concepts already

The nurse is caring for a patient with a new, unexpected sigmoid colostomy. The nursing diagnosis knowledge deficit related to colostomy care is included in the patient's care plan. Which is the appropriate outcome for the patient? a. The patient will empty and change the colostomy appliance. b. The patient will resume a sexual relationship with the spouse. c. The patient will verbalize feelings about presence of colostomy. d. The patient will use clothing to effectively conceal the colostomy.

a. The patient will empty and change the colostomy appliance The appropriate outcome for the nursing diagnosis knowledge deficit related to colostomy is that the patient will empty and change the colostomy appliance. Resuming a sexual relationship, verbalizing feelings, and using clothing to conceal the ostomy do not relate to the patient's need to learn how to care for the colostomy.

Which is an appropriate outcome for an elderly patient with the nursing diagnosis adult failure to thrive related to placement in extended care agency after right hip fracture? a. The patient will participate in social activities and maintain usual weight. b. The patient will exercise both legs to minimize effects of unilateral neglect. c. The patient will verbalize need for extended nursing care during rehabilitation. d. The patient will not demonstrate symptoms of deep vein thrombus development.

a. The patient will participate in social activities and maintain usual weight Adult failure to thrive is a continual deterioration of mental and cognitive function often seen after traumatic injury such as hip fracture. The appropriate outcome is that the patient will participate in social activities and maintain usual weight. Unilateral neglect is the unawareness of a body part following a stroke. Verbalizing need for extended nursing care will not minimize mental and cognitive decline. DVT prevention is not an outcome for the adult failure to thrive diagnosis

The nurse is caring for a patient with metabolic acidosis due to severe hyperglycemia. Which assessment finding indicates to the nurse that the patient's body is attempting to compensate for the acidosis? a. The patient's breathing is very deep and rapid. b. The patient's temperature has been rising steadily. c. The patient's skin is flushed and warm to the touch. d. The patient is urinating large amounts of light colored urine

a. The patient's breathing is very deep and rapid The patient's deep breathing is an attempt by the body to blow off additional carbon dioxide which converts to carbonic acid. Reduction of carbonic acid in the body will raise the pH and compensate for the acidosis. Elevated temperature, flushed skin, and increased urine output will not compensate for the acidosis.

Which factor in the female patient's history places her at higher risk for the development of breast cancer? a. The patient's first period started at age 10. b. The patient has three children under the age of 12. c. The patient used condoms exclusively for contraception. d. The patient's breasts are tender before each period

a. The patient's first period started at age 10 Early menarche (first period before age 12) is a risk factor for developing breast cancer. Never having children and recent use of oral contraceptives are also risk factors. It is typical for female patients to experience breast tenderness and swelling before each menstrual cycle. That is why it is recommended to perform self-breast examinations a few days after the period ends.

Which is the priority outcome for the patient with pulmonary embolism and the nursing diagnosis impaired gas exchange related to impaired pulmonary perfusion? a. The patient's pulse oximetry will stay at least 93%. b. The patient's lung sounds will remain clear bilaterally. c. The patient will verbalize understanding of oxygen therapy. d. The patient will walk 50 feet in the hallway without dyspnea.

a. The patient's pulse oximetry will stay at least 93% Maintenance of pulse oximetry greater than 93% is the highest priority outcome as it demonstrates adequate oxygenation in the patient with pulmonary embolism. Lung sounds may remain clear with pulmonary embolism as the problem is caused by blockage of blood vessels within the lung. Verbalizing understanding of oxygen therapy and walking in the hall without dyspnea are less important outcomes.

Which outcome is appropriate for the patient with risk for urinary tract infection related to recent urinary catheterization? a. The patient's urine will remain free from white blood cells and bacteria. b. The patient will take prescribed antibiotics until the urinary symptoms are gone. c. The patient will have serial urine cultures to ensure that the infection is resolved. d. The patient will carefully wipe the perineal area from front to back after voiding.

a. The patient's urine will remain free from white blood cells and bacteria. An appropriate outcome for the risk for urinary tract infection diagnosis is that the patient's urine will remain free from white blood cells and bacteria. Antibiotics should not be prescribed unless an infection is present, which negates the risk for infection diagnosis. Serial urine cultures will take place if an infection is present. Careful perineal care is an intervention, not an outcome.

Which assessment finding is expected for a patient presenting with a middle ear infection? a. The right tympanic membrane is pink and bulging. b. The patient becomes dizzy when sitting upright. c. The pinna is red, swollen, and tender to palpation. d. The eardrum is a translucent pearly gray color.

a. The right tympanic membrane is pink and bulging. The normal eardrum is translucent, shiny, and pearly gray. It is free from tears or breaks. A pink or red bulging membrane indicates inflammation consistent with middle ear infection. Dizziness indicates an inner ear infection. Inflammation of the pinna indicates an external ear infection.

The nurse is making a home visit to a child who has a chronic disease. Which finding has the most implication for acid-base aspects of this patient's care? a. Urine output is very small today. b. Whites of the eyes appear more yellow. c. Skin around the mouth is very chapped. d. Skin is sweaty under three blankets.

a. Urine output is very small today. Oliguria decreases the excretion of metabolic acids and is a risk factor for metabolic acidosis. Jaundice requires follow-up but is not an acid-base problem. Perioral chapped skin needs intervention but is not an acid-base issue. With three blankets, diaphoresis is not unusual.

Which nursing intervention is appropriate for a patient with a history of parasomnia? a. Use of a bed alarm during the night b. Padded side rails for the patient's bed c. Use of a CPAP machine during the night d. Continuous pulse oximetry during the night

a. Use of a bed alarm during the night The parasomnias are sleep disorders that can occur during arousal from REM or partial arousal from NREM sleep. They include sleep walking, night terrors, nightmares, teeth grinding, and bed-wetting. A bed alarm is useful so that the nursing staff will be notified if the patient attempts to sleepwalk. Padded side rails, CPAP machine, and continuous pulse oximetry are not needed for parasomnia.

The nurse is to transfer an obese, hemiplegic patient from the bed to the chair. Which is the safest way for the nurse to do this? a. Use of a mechanical patient lift device b. Use of a gait belt around the patient's waist c. Use of three staff members to assist the patient d. Use of a roller board to slide from the bed to the chair

a. Use of a mechanical patient lift device The safest option is for the nurse to utilize a mechanical patient lift device for moving the obese hemiplegic patient. Use of a gait belt, multiple staff members, or a roller board could result in injury to the patient and/or staff members

The nurse manager of a pediatric clinic could confirm that the new nurse recognized the purpose of the HEADSS Adolescent Risk Profile when the new nurse responds that it is used to review for needs related to a. anticipatory guidance. b. low-risk adolescents. c. physical development. d. sexual development.

a. anticipatory guidance. The HEADSS Adolescent Risk Profile is a psychosocial assessment screening tool which reviews home, education, activities, drugs, sex, and suicide for the purpose of identifying high-risk adolescents and the need for anticipatory guidance. It is used to identify high-risk, not low-risk, adolescents. Physical development is reviewed with anthropometric data. Sexual development is reviewed using physical examination.

Understanding cultural differences in health care is important because it will help the nurse to understand the manner in which people decide on obtaining treatments and medical care. In independent cultures an individual will a. put himself first. b. consult family members for advice. c. ask for a second opinion. d. travel great distances to receive the best care.

a. put himself first. In independent cultures, an individual will put himself first in the case of a life-threatening illness, whereas even in dire circumstances, members of collectivist cultures may still consult other family members for the best course of action. In independent cultures, an individual will not consult with other family members, ask for a second opinion, or travel great distances to receive the best care

A pediatric nurse is assessing a patient for a routine physical. The nurse identifies that the parents need additional safety teaching when the parents mentions which of the following? a.A 2-year-old child can safely sit in the front seat of a car. b.Teenagers need to practice safe sex. c.Children need to wear a helmet and safety pads when in-line skating. d.Children need to learn to swim even if parents do not have a swimming pool.

a.A 2-year-old child can safely sit in the front seat of a car. A 2-year old cannot sit in the front seat safely, so the nurse needs to correct this misinformation. All children 2 years or older, or those younger than 2 years who have outgrown the rear-facing weight or height limit for their car safety seat (CSS), should use a forward-facing CSS with a harness for as long as possible, up to the highest weight or height allowed by the manufacturer of their CSS. Teenagers practicing safe sex, children wearing a helmet and safety pads when skating, and children learning to swim even if a home pool is not present are all correct and do not need additional teaching.

A nurse is caring for a 5-year-old child who is hospitalized for stabilization of asthma. To provide age-specific care, which of the following is the most appropriate action by the nurse? a.Allow the child to handle medical equipment. b.Have a parental presence at all times. c.Have the child lie flat and still during procedures. d.Tell the child as little as possible about expectations and care.

a.Allow the child to handle medical equipment. These strategies can be used to reduce preschoolers fears when they are hospitalized: allowing children to sit up when performing assessments and procedures; allowing the child to see and handle equipment; allowing the child to assist with the procedure if appropriate; giving simple and factual information to these children because they have a great sense of imagination. Although preschoolers have developed object permanence and recognize their parents still exist when out of sight, most tolerate only short absences without becoming distressed. Encourage parents to tell the child when they are leaving and when they will return in terms the child can understand (e.g., I am leaving and will be back after lunch.).

The mother of an 8-year-old girl has brought her daughter to the health clinic for her annual check-up. She is concerned about the high blood pressure in her family and asks the nurse if there is some way to know if the child is at risk for hypertension. What is the nurses best response? a.Blood pressure elevation in childhood is the single best predictor of adult hypertension. b.Well lets take her blood pressure and see if its up. If it is, she has hypertension. c.She looks pretty plump to me, and that indicates good health. As long as shes eating, she should be OK. d.If you think that shes gaining weight, put her on an exercise program, but wait until shes in her teens.

a.Blood pressure elevation in childhood is the single best predictor of adult hypertension. Blood pressure elevation in childhood is the single best predictor of adult hypertension. This recognition has reinforced the significance of making blood pressure measurement a part of every annual assessment of the child. Measure on at least three separate occasions with the appropriate-size cuff and in a relaxed situation before concluding that the childs blood pressure is elevated and needs further medical attention. Childhood obesity is a prominent health problem, which increases the childs risk for hypertension, diabetes, coronary artery disease, and other chronic health problems. Daily exercise and maintaining normal body weight are important as both interventions and prevention even while in the preteen years.

An elderly nursing home resident fell 2 weeks ago and has been on bed rest. The patient has become increasingly fatigued during activities of daily living (ADLs). The family is concerned about the patients declining condition. The best explanation that the nurse can give the family is that the patients fatigue is caused by which of the following? a.Decreased muscle endurance caused by immobility b.Advanced age c.Increased metabolism d.Urinary stasis

a.Decreased muscle endurance caused by immobility The body loses muscle strength when muscles are inactive. The rate of muscle decline varies with the degree of immobility, but it is rapid while mobility and weight bearing are restricted. These effects are devastating to patients who are marginally functional with their ADLs. This fatigue is a result of the bed rest, not from advanced age or urinary stasis. Reduced metabolism, not increased, leads to a loss of muscle and body mass, causing fatigue with prolonged activity. Urinary stasis does not affect activity level, but can lead to urinary tract infections.

A patient is recovering from an abdominal aortic bypass graft. To reduce the effects of orthostatic hypotension, what is the most appropriate action for the nurse to take? a.Encourage moving positions slowly. b.Perform isometric exercises. c.Decrease the number of ankle pumps. d.Participate in chest physiotherapy.

a.Encourage moving positions slowly. Moving positions slowly will help with orthostatic hypotension by allowing the body to adapt. Orthostatic hypotension is an increase in heart rate of more than 15% and a drop of 15 mm Hg or more in systolic blood pressure or a decrease of 10 mm Hg in diastolic blood pressure when the patient rises from a lying or sitting position to a standing position. Isometric exercises, which are activities that involve muscle tension without muscle shortening, do not have any effect on preventing orthostatic hypotension, but improve activity tolerance. Ankle pumps help to prevent deep vein thrombosis. Participating in chest physiotherapy assists patients with decreasing effects of pulmonary complications.

The nurse working with a new nursing assistive personnel (NAP), is explaining about the importance of repositioning immobile patients to prevent pressure ulcers. At a minimum, the nurse tells the NAP to reposition patients how often? a.Every 2 hours b.Every 3 hours c.Every 4 hours d.Once every shift

a.Every 2 hours Immobilized patients require vigilant nursing care, such as repositioning at least every 2 hours, to avoid physical complications. Every 3 or 4 hours or once every shift is too long.

is the measurable aspect of a persons increase in physical dimensions. a.Growth b.Development c.Maturation d.The latency stage of development

a.Growth Growth is the measurable aspect of a persons increase in physical dimensions. Measurable growth indicators include changes in height, weight, teeth and bone, and sexual characteristics. Development is an interaction of biological, sociological and psychological forces. It occurs gradually and refers to changes in skill and capacity to function. These changes are qualitative in nature and difficult to measure in exact units. Maturation is the biological plan for the predictable milestones for growth and development. Physical growth and motor development are a function of maturation. The latency stage of development is part of Freuds psychoanalytic model of personality development. In the latency stage, Freud believed that the sexual urges from the earlier phallic stage are repressed and channeled into productive activities that are socially acceptable.

The patient is experiencing shortness of breath. Which of the following lab tests indirectly indicates the oxygen level in the blood system? a.Hemoglobin b.White blood cell count c.Electrolytes d.Creatinine

a.Hemoglobin Hemoglobin transports most oxygen and serves as a carrier for both oxygen and carbon dioxide. White blood cell count is a lab test to measure infection. Electrolytes do not indicate oxygen levels but do indicate electrolytes like sodium and potassium. Creatinine levels measure kidney function.

A patient was admitted to the surgical unit after surgical removal of an abdominal tumor. As the nurse performs a postsurgical assessment, the patients blood pressure is 90/54, heart rate is 94, and respiratory rate is 22. Based on these values, the nurse should be most concerned with which of the following conditions? a.Hypovolemia b.Left-sided heart failure c.Right-sided heart failure d.Hypervolemia

a.Hypovolemia Hypovolemia is a reduced circulating blood volume resulting from extracellular fluid losses that occurs in conditions such as shock and severe dehydration. If the fluid loss is significant, the body tries to adapt by increasing the heart rate and constricting peripheral vessels to increase the volume of blood returned to the heart and increase the cardiac output. The patient is experiencing shock (low blood pressure, increased pulse and respirations). Left-sided heart failure is characterized by impaired functioning of the left ventricle. This is usually caused by increased preload (fluid volume overload) or afterload (increased systemic vascular resistance such as hypertension). Right-sided heart failure results from impaired functioning of the right ventricle, which is typically caused by pulmonary disease or pulmonary hypertension. Fluid volume overload or hypervolemia may lead to vascular congestion in patients with heart, kidney, or lung diseases. The patient has lost fluid, not gained fluid

A registered nurse works in a small rural health clinic. During a routine well baby visit, a new mother questions the need to have her infant immunized. Which of the following is the best explanation for why it is recommended that her child receive immunizations? A registered nurse works in a small rural health clinic. During a routine well baby visit, a new mother questions the need to have her infant immunized. Which of the following is the best explanation for why it is recommended that her child receive immunizations? a.Immunization increases resistance to an infectious disease. b.It provides a small amount of a live, strong organism to protect against disease. c.Immunization will definitely keep your child well. d.It will provide active immunity by providing antibodies to your child. b.It provides a small amount of a live, strong organism to protect against disease. c.Immunization will definitely keep your child well. d.It will provide active immunity by providing antibodies to your child.

a.Immunization increases resistance to an infectious disease. Immunization is the process by which resistance to an infectious disease is produced or increased. A weakened or dead organism and modified toxins from the organism is injected into the body, not a live, strong organism. Immunizations do not definitely keep a child well. Passive immunity occurs when antibodies produced by other people or animals are introduced into a persons bloodstream, whereas active immunity is from a weakened or dead organisms or modified toxins from the organism.

Which assessment finding should the nurse expect to observe on an immobilized patient? a.Increased serum glucose levels b.Decreased urine excretion c.Positive nitrogen balance d.Increased serum potassium levels

a.Increased serum glucose levels A patients basal metabolic rate (BMR) decreases in response to reduced cellular energy because of the bodys decreased ability to produce insulin and metabolize glucose. In the immobilized patient, a major shift in blood volume occurs, which causes diuresis (increased urine excretion). Diuresis causes the body to lose electrolytes, such as potassium and sodium. When the body is unable to metabolize glucose, it begins to break down protein stores for energy, resulting in negative nitrogen balance, not positive.

In an outpatient surgery center, the preoperative nurse has the responsibility of starting IVs prior to the patients surgeries. One of the surgeons who works at the center orders a different type of IV fluid than the rest of the surgeons. Which of the following should the nurse be most concerned about in this situation? a.Procedure-related accident b.Patient-inherent accident c.Patient confusion from medications d.Potential electrolyte imbalance

a.Procedure-related accident Procedure-related accidents are caused by health care providers and include medication and fluid administration errors, improper application of external devices, and improper performance of procedures such as dressing changes. Following an organizations policies and procedures and standards of nursing practice helps prevent procedure- related accidents. Patient-inherent accidents are those in which a patient is the primary reason for the accident. The primary issue is a procedure-related accident because of the surgeons use of a different type of IV, which could cause the nurse to improperly administer the medication; it is not an electrolyte imbalance or confusion. Electrolyte imbalance and confusion from medications are all potential issues that are out of the nurses control.

A nurse is caring for a patient in Bucks traction on bed rest for a fracture of the femur. Which action should the nurse take to help preserve skin integrity? a.Provide meticulous skin care. b.Use pain medication to prevent excessive movement. c.Limit range of joint motion so the patient is not disturbed. d.Reduce the amount of protein intake so renal function can be preserved.

a.Provide meticulous skin care. Interventions aimed at prevention of pressure ulcers are positioning, skin care, and the use of pressure-relief devices. Change the immobilized patients position according to the patients activity level, perceptual ability, status of peripheral circulation, treatment protocols, and daily routines. A patient needs adequate, not reduced, protein intake to ensure wound healing and tissue growth and to prevent a negative nitrogen balance. Patients whose mobility is restricted require ROM to reduce the hazards of immobility; ROM should not be limited. Pain medication is to relieve pain, not to prevent excessive movement.

Nurses implement therapeutic immobilization for patients to achieve which result? a.Reducing pain b.Restraining an unstable patient in bed c.Increasing active movement of the body d.Strengthening joints and muscles

a.Reducing pain Therapeutic reasons for bed rest include decreasing the bodys oxygen needs, reducing cardiac workload, reducing pain, and allowing the debilitated or ill patient to rest. Restraining an unstable patient in bed is not a reason for therapeutic immobilization. Restraining is a last resort. The body loses muscle strength when muscles are inactive; therapeutic immobilization does not strengthen joints and muscles. Bed rest

The school nurse is performing periodic screening on preschool children. She is aware that the most common visual problem in childhood is which of the following? a.Refractive errors b.Strabismus c.Congenital blindness d.Color blindness

a.Refractive errors Periodic screening of all children, especially newborns through preschoolers, should be performed for congenital blindness and visual impairment caused by refractive errors and strabismus. The most common visual problem during childhood is a refractive error such as nearsightedness. The school nurse is usually responsible for vision testing of school-age and adolescent children. Your role as a nurse is one of detection, education, and referral. Parents need to know the signs of visual impairment such as failure to react to light and reduced eye contact from the infant. Instruct parents to report signs of visual impairment to their health care provider

This model of personality development is grounded in the belief that two internal biological forces drive the psychological change in a child: sexual (libido) and aggressive energies. Who is responsible for developing this theory? a.Sigmund Freud b.Jean Piaget c.Erik Erikson d.Lawrence Kohlberg

a.Sigmund Freud Sigmund Freud (1856-1939) provided the first formal structured theory of personality development. Freuds psychoanalytic model of personality development is grounded in the belief that two internal biological forces drive the psychological change in a child: sexual (libido) and aggressive energies. Jean Piaget (1896-1980) developed the theory of cognitive development, which describes childrens intellectual organization and how they think, reason, and perceive the world. The theory includes four periods: sensorimotor, preoperational, concrete operations, and formal operations. Erik Erikson divided life into eight stages, known as Eriksons eight stages of development. According to this theory, individuals need to accomplish a particular task before successfully completing each stage. Each task is framed with opposing conflicts, such as trust versus mistrust. Each stage builds upon the successful attainment of the previous developmental conflict. Lawrence Kohlberg (1927-1987) expanded on Piagets work. According to Kohlberg (1964), moral development is one component of psychosocial development. It involves the reasons an individual makes a decision about right and wrong behaviors within a culture.

The nurse identifies that one of the IV pumps has been malfunctioning and was placed outside a patient room until it could be repaired. To prevent an equipment-related accident from occurring, which action should the nurse take first? a.Tag the pump and remove it from the area. b.Initiate a work order on the pump. c.Clean the pump and put it in the equipment closet. d.Call the pump manufacturer.

a.Tag the pump and remove it from the area. Initiating the work order on the pump is important, but the first priority is to tag and remove the pump from service. Leaving the pump in the equipment closet could allow the pump to mistakenly be put back into service without be fixed. It is not the nurses job to call the pump manufacturer to report the issues. Accidents that are equipment related result from the malfunction, disrepair, or misuse of equipment or from an electrical hazard. To avoid accidents, do not operate medical equipment without adequate instruction. If you discover a faulty piece of equipment, replace it with the proper working equipment, place a tag on the faulty one, take it out of service and

Which patient is most at risk for developing a urinary tract infection? a.Teenage comatose patient on a ventilator lying supine b.Middle-age adult after abdominal surgery sitting in a chair c.Elderly adult with Alzheimer disease who is wandering at night d.Middle-age adult postcardiac catheterization being discharged home

a.Teenage comatose patient on a ventilator lying supine The patient most at risk is the teenage comatose patient on a ventilator lying supine. When the patient is in bed, the kidneys and ureters move toward a more level plane, and urine tries to move from the kidney to the bladder against gravity. Because the peristaltic contractions of the ureters are not strong enough to overcome gravity when the patient is reclining, the renal pelvis fills before urine enters the ureters, which increases the patients risk for urinary tract infection (UTI) and renal calculi. The elderly adult with Alzheimer and the middle-age adult post cardiac catheterization are both upright, which decreases chance for UTI. The middle-age adult after abdominal surgery is sitting in a chair, which decreases chance for UTI because gravity is helping the ureters drain.

A patient has been admitted to the cardiac unit with the diagnosis of bradycardia. The patient states I am confused about what the doctor said is wrong with me, he said my pacemaker is not working. I dont have a pacemaker. What is the nurses best response? a.The sinoatrial node is the pacemaker of your heart. b.Myocardial contractility determines your heart rate. c.The atrioventricular node is the pacemaker of your heart. d.The ventricular Purkinje network determines your heart rate.

a.The sinoatrial node is the pacemaker of your heart. The conduction system originates with the sinoatrial node, the pacemaker of the heart. Electrical impulses are then transmitted along intraatrial pathways to the atrioventricular (AV) node. The AV node mediates impulse transmission between the atria and the ventricles. Delaying the impulse at the AV node before transmitting it through the bundle of His and ventricular Purkinje network. The heart rate is regulated by the sympathetic and parasympathetic systems, not by myocardial contractility and ventricular Purkinje network. Myocardial contractility is the ability of the heart to squeeze blood from the ventricles and prepare for the next contraction. This is difficult to measure because preload, afterload, and heart rate must remain constant.

Which patient is most likely to have developmental effects due to prolonged immobility? a.Toddler patient in traction for a congenital skeletal anomaly b.Young adult patient with burns on the hands c.Teenage patient with a bacterial infection in isolation d.Middle-age adult patient with a fractured ankle on crutches

a.Toddler patient in traction for a congenital skeletal anomaly The toddler would be most affected because of the prolonged immobility. Developmental effects of immobility more commonly affect the very young and the older adult. When the infant, toddler, or preschooler is immobilized, it is usually because of trauma or the need to correct a congenital skeletal abnormality. Prolonged immobilization delays the childs motor skill and intellectual development. The immobilized young or middle-age adult experiences few, if any, developmental changes. The young and middle adult patients are mobile, not immobile. The teenage patient may have developmental effects due to the isolation but the question asked for prolonged immobility, which this patient does not have.

A patient with a tracheostomy is experiencing thick and tenacious secretions. To maintain this patients airway, what is the most appropriate action for the nurse? a.Tracheal suctioning b.Oropharyngeal suctioning c.Nasotracheal suctioning d.Orotracheal suctioning

a.Tracheal suctioning Tracheal suctioning is performed through an artificial airway such as a tracheostomy. Oropharyngeal suctioning clears secretions from the mouth and upper airway. Nasotracheal suctioning introduces the catheter through the naris into the trachea. Orotracheal suctioning introduces the catheter through the mouth into the trachea.

Which term should the nurse use to best describe the movement of air in and out of the patients lungs? a.Ventilation b.Diffusion c.Respiration d.Perfusion

a.Ventilation The primary functions of the lungs include ventilation, the movement of air in and out of the lungs, and diffusion, the movement of gases between air spaces and the bloodstream. Respiration is the exchange of oxygen and carbon dioxide during cellular metabolism. The heart supports perfusion, the movement of blood into and out of the lungs to the organs and tissues of the body.

.A young adult patient was involved in a motorcycle accident. The patient was in the intensive care unit of the hospital for 2 months with immobility and was just discharged to a rehabilitation hospital. The patient asks the nurse, Why am I so weak? What is the best response from the nurse? a.When you are in bed for a long time, your body begins to break down its own protein. b.When you dont use it, you lose it. c.You havent eaten much for the past couple of months. d.Your body has spent energy trying to heal itself by increasing the metabolic rate.

a.When you are in bed for a long time, your body begins to break down its own protein. Immobility disrupts normal metabolic functioning, decreasing the metabolic rate and altering the metabolism of carbohydrates, proteins, and fats. A patients basal metabolic rate (BMR) decreases in response to reduced cellular energy because of the bodys decreased ability to produce insulin and metabolize glucose. The body then begins to breakdown its protein stores for energy resulting in a negative nitrogen balance and increased oxygen demands. However, in the presence of an infection, immobilized patients have an increased BMR. It is the immobility that has caused the weakness, not what was eaten. Not using it leads to losing it is a clich and should be avoided. The metabolic rate is decreased in immobility not increased.

The guardians of a premature infant who was delivered at 31-weeks gestation is expected to be discharged from the hospital within the next few days. The guardians have voiced concern regarding how to prevent respiratory syncytial virus (RSV) exposure to the baby. What is the best response from the nurse? a.You will need to limit the babys exposure to crowds of people. b.You need to make sure that the car seat is facing backward in the back seat of your car. c.You do not need to be concerned; the baby has a natural protection against this disease. d.You must sterilize all the bottles for the first 6 months.

a.You will need to limit the babys exposure to crowds of people. Premature infants are at risk for development of respiratory illnesses such as respiratory syncytial virus (RSV) as a result of the underdevelopment of the lung. Limiting exposure to crowds of people will limit the exposure to respiratory viruses. Premature infants do not have protection against this virus. The virus is respiratory, so the car seat and bottles will not affect this illness.

A patient reports chest pain. The nurse is attempting to assess the pain to differentiate the pain as cardiac, respiratory, or gastrointestinal. The nurse can properly identify the pain as cardiac in origin when the patient states that the cardiac pain: a.does not occur with respiratory variations. b.is peripheral and may radiate to the scapular areas. c.is aggravated by inspiratory movements. d.is nonradiating and occurs during inspiration.

a.does not occur with respiratory variations. Cardiac pain does not occur with respiratory variations. Cardiac chest pain is most often substernal and radiates to the left arm and jaw in men, but some women have epigastric pain, complaints of indigestion, or a choking feeling and dyspnea. Pericardial pain results from an inflammation of the pericardial sac and is usually nonradiating and often occurs with inspiration. Pleuritic chest pain is peripheral and usually radiates to the scapular region. Inspiratory maneuvers aggravate pleuritic chest pain.

The nurse is providing prenatal education to a patient whose pregnancy has been confirmed. This is the patients first pregnancy and she is in her first trimester. The nurse instructs the patient that she should stop smoking, avoid alcohol, and avoid eating king mackerel because of the high mercury content in the fish. Although this advice should be followed during the entire pregnancy, the fetus is most vulnerable to adverse effects in the trimester. a.first b.second c.third d.final

a.first Exposure to potential teratogens can affect fetal development during any of the trimesters; however, vulnerability is increased during the first trimester when fetal cells are differentiating and organs are forming.

Preconception counseling is a growing trend in health care with the goal being to secure the best outcome for mother, fetus, and significant others through good prenatal care and teaching that: a.teratogens can affect fetal development during any trimester. b.the placenta prevents teratogens from passing to the fetus. c.teratogens are all man-made, preventable, and do not include viruses or bacteria. d.smoking has been shown to have no effect on fetal development.

a.teratogens can affect fetal development during any trimester. Teratogens are chemical or physiological agents capable of having adverse effects on the fetus. Exposure to potential teratogens can affect fetal development during any of the trimesters; however, vulnerability is increased during the first trimester when fetal cells are differentiating and organs are forming. Because the placenta is extremely porous, teratogens pass easily from mother to fetus. Some examples of teratogens are viruses, drugs (prescribed, over-the-counter, and street drugs), alcohol, and environmental pollutants, such as lead. The fetal effect of these harmful agents depends on the developmental stage in which exposure takes place. In addition, there is evidence that mothers who smoke deliver infants with lower birth weights than nonsmoking mothers.

Which assessment question enables the nurse to determine provocative factors of the patient's pain? a. "What does your pain feel like?" b. "Does anything make your pain worse?" c. "Can you show me where the pain is?" d. "Is the pain constant or does it come and go?"

b. "Does anything make your pain worse?" The comprehensive assessment of pain aims to gather information about the cause of a person's pain and determine its effect on his or her ability to function. • Palliative or provocative factors—What makes your pain worse or better? • Quality—How do you describe your pain? • Relief measures—What do you take at home to gain pain relief? • Region (location)—Show me where you hurt. • Severity—On a scale of 0 to 10, with 10 being worst, how bad is your pain now? • Timing (onset, duration, and pattern)—Is your pain constant, intermittent, or both? • U (effect of pain on patient)—What are you unable to do because of your pain?

A nurse is educating a 21-year-old lifeguard about the risk of skin cancer and the need to wear sunscreen. Which statement by the patient indicates that the need for additional teaching? a. "I wear a hat and sit under the umbrella when not in the water." b. "I don't bother with sunscreen on overcast days." c. "I use a sunscreen with the highest SPF number." d. "I wear a UV shirt and limit exposure to the sun by covering up."

b. "I don't bother with sunscreen on overcast days." The sun's rays are as damaging to skin on cloudy, hazy days as on sunny days. The other options will all prevent skin cancer.

A Muslim patient has requested to have only female nursing staff provide personal care. Which is the appropriate response of the nurse? a. "I know that it may be difficult to understand but both male and female nurses are employed at this hospital." b. "I will make sure that the charge nurse is notified about your request so that it can be accommodated." c. "It will be difficult for people like you to adjust to our ways, but there are limitations for all of us." d. "You may be perceived as a 'difficult patient' if you insist on limiting which nurses can take care of you."

b. "I will make sure that the charge nurse is notified about your request so that it can be accommodated." Nurses who provide culturally competent care bridge cultural gaps to provide meaningful and supportive care for all patients. The Muslim patient's request to have female nursing staff members provide personal care should be accommodated. The patient should not be threatened with a perception of being labeled as difficult. Even though both male and female nurses are employed at the hospital, female nurses can be assigned to provide personal care for the Muslim patient.

The patient refuses to get out of bed to ambulate after surgery. Which is the appropriate response of the nurse? a. "No problem. You should rest quietly in bed today so that you can heal." b. "It is important to get out of bed and walk to prevent blood clots or pneumonia." c. "I will notify your doctor that you refused and make a notation in your chart." d. "You can have your next dose of pain medication after you get up and walk.

b. "It is important to get out of bed and walk to prevent blood clots or pneumonia." The nurse should inform the patient of the risks associated with prolonged bed rest. The nurse should not readily allow the patient to rest in bed when ambulation has been ordered. The patient should be informed of the risks before the nurse notifies the doctor of the patient's refusal to get out of bed. Pain medication should be administered prior to ambulation whenever possible and should never be withheld due to patient's refusal to complete a prescribed activity.

The patient had diarrhea for 5 days and developed an acid-base imbalance. Which statement would indicate that the nurse's teaching about the acid-base imbalance has been effective? a. "To prevent another problem, I should eat less sodium during diarrhea." b. "My blood became too acid because I lost some base in the diarrhea fluid." c. "Diarrhea removes fluid from the body, so I should drink more ice water." d. "I should try to slow my breathing so my acids and bases will be balanced."

b. "My blood became too acid because I lost some base in the diarrhea fluid." Diarrhea causes metabolic acidosis through loss of bicarbonate, which is a base. Eating less sodium during diarrhea increases the risk of ECV deficit. Although diarrhea does remove fluid from the body, it also removes sodium and bicarbonate which need to be replaced. Rapid deep respirations are the compensatory mechanism for metabolic acidosis and should be encouraged rather than stopped.

When a diabetic patient asks about maintaining adequate blood glucose levels, which of the following statements by the nurse relates most directly to the necessity of maintaining blood glucose levels no lower than about 74 mg/dL? a. "Glucose is the only type of fuel used by body cells to produce the energy needed for physiologic activity." b. "The central nervous system cannot store glucose and needs a continuous supply of glucose for fuel." c. "Without a minimum level of glucose circulating in the blood, erythrocytes cannot produce ATP." d. "The presence of glucose in the blood counteracts the formation of lactic acid and prevents acidosis."

b. "The central nervous system cannot store glucose and needs a continuous supply of glucose for fuel." The brain cannot synthesize or store significant amounts of glucose; thus a continuous supply from the body's circulation is needed to meet the fuel demands of the central nervous system.

A patient who was diagnosed with senile dementia has become incontinent of urine. The patient's daughter asks the nurse why this is happening. What is the nurse's best response? a. "The patient is angry about the dementia diagnosis." b. "The patient is losing sphincter control due to the dementia." c. "The patient forgets where the bathroom is located due to the dementia." d. "The patient wants to leave the hospital."

b. "The patient is losing sphincter control due to the dementia." Anger, wanting to leave the hospital, and forgetting where the bathroom is really have no bearing on the urinary incontinence. The patient is incontinent due to the mental ability to voluntarily control the sphincter. This is happening because of the dementia.

Which statement made by the patient indicates readiness for learning about colostomy care? a. "I don't want to look at it and I can't imagine caring for it." b. "The sooner I can take care of it, the sooner I can go home." c. "I never thought I would have to take care of something like this." d. "I hope I can still wear a bathing suit with this thing."

b. "The sooner I can take care of it, the sooner I can go home." "The sooner I can take care of it, the sooner I can go home" indicates that the patient is ready to learn about how to take care of the colostomy. The patient realizes that discharge from the hospital depends on the ability to care for the colostomy so the patient is amenable to teaching about how to care for it. "I don't want to look at it and I can't imagine caring for it" indicates that the patient does not wish to learn about colostomy care. "I never thought I would have to take care of something like this" is an emotional statement that indicates a need for support from the nurse. "I hope I can still wear a bathing suit with this thing" addresses a need for reassurance about appearance and activity after colostomy surgery

Which patient would benefit from a sitz bath? a. A patient who has not had a bowel movement for the last 4 days b. A patient with painful, swollen hemorrhoids after vaginal childbirth c. A patient with perineal skin breakdown due to continuous oozing of stool d. A patient who is having difficulty adhering the ostomy appliance to the skin

b. A patient with painful, swollen hemorrhoids after vaginal childbirth A sitz bath is a shallow bath of warm water used to clean and soothe irritation of the perineum or rectum. A patient with painful hemorrhoids would benefit from soaking in a sitz bath. A sitz bath does not relieve constipation or help adhere an ostomy appliance to the skin. A patient with continuous oozing of stool should not be given a sitz bath, as bacteria from the stool could be introduced to the vaginal and/or urethral openings.

The nurse assists a patient who collapsed in cardiac arrest. Which is the first action of the nurse? a. Determine the patient's cardiac rhythm. b. Administer fast, deep chest compressions. c. Ensure that a patent airway is maintained. d. Ventilate the patient using a barrier device.

b. Administer fast, deep chest compressions The first action of the nurse is to begin chest compressions. Determining the cardiac rhythm, ensuring a patent airway, and ventilating the patient are done after chest compressions are initiated.

The nursing staff is caring for a patient who collapsed in cardiac arrest. When will breaths be delivered via the bag-valve mask device? a. After the patient is intubated b. After every 30 chest compressions c. When the patient's lips start to become cyanotic d. When another nurse takes over chest compressions

b. After every 30 chest compressions Two breaths are delivered via the bag-valve mask device after each set of 30 chest compressions. The mask device is not used after the patient is intubated. Frequency of breaths is not determined by the presence or absence of cyanosis or change of staff providing chest compressions.

Which technique is best to calm a preschooler's fears before auscultating bowel sounds? a. Reassure the child that listening to bowel sounds will not hurt in the least. b. Allow the child to use the stethoscope to listen to a teddy bear's abdomen. c. Explain how a stethoscope is used to listen to the functioning of the bowel. d. Seat the child in the parent's lap and have the parent hold the child's hands.

b. Allow the child to use the stethoscope to listen to a teddy bear's abdomen The preschooler should be allowed to use the stethoscope to listen to the teddy bear's abdomen. This will enable the child to understand how the stethoscope works and reassure the child that the procedure will cause no discomfort. A preschooler should never be restrained when other options are available. Touching the stethoscope will be more effective for reducing fears than explanations and reassurance.

The nurse is caring for an incontinent male patient with a large sacral pressure injury. Which is the safest intervention that will maintain skin integrity and facilitate healing of the ulcer? a. Obtain a surgical consult for placement of a suprapubic urinary catheter. b. Apply a condom catheter attached to a bedside urinary drainage bag. c. Insert an indwelling urinary catheter attached to a small volume drainage bag. d. Perform intermittent straight catheterization of the patient every 4 to 6 hours.

b. Apply a condom catheter attached to a bedside urinary drainage bag. The safest option for this patient is to apply a condom catheter to facilitate containment of urine. Indwelling, suprapubic, and straight catheters will increase risk of urinary tract infection.

The nurse is conducting a health history for a patient who does not speak English. What is the best action of the nurse? a. Communicate with the patient using simple gestures and drawings. b. Arrange for a medical translator to be present during the health history interview. c. Ask an English-speaking family member to translate the nurse's questions. d. Obtain the patient's health history from an English-speaking family member

b. Arrange for a medical translator to be present during the health history interview The nurse should utilize the services of a medical translator to obtain the health history for a patient who does not speak English. Communication using simple gestures and drawings will not be sufficient to obtain an accurate health history. Family members should not be used to translate except in the case of emergency due to their lack of knowledge about medical terminology and patient confidentiality.

Which technique will the nurse use to assess the patient's olfactory nerve? a. Ask to shrug the shoulders and turn the head against the nurse's hand. b. Ask the patient to identify the smell of peppermint oil and orange peel. c. Ask the patient to read the smallest set of letters on a Snellen eye chart. d. Ask the patient to stick out the tongue and move it side to side from midline.

b. Ask the patient to identify the smell of peppermint oil and orange peel. The olfactory nerve communicates the patient's sense of smell. Damage to the nerve will diminish the patient's ability to smell and differentiate odors. Asking the patient to identify the smell of peppermint and orange will assess the function of the olfactory nerve. Asking the patient to read the smallest set of letters on a Snellen eye chart will assess the optic nerve. Having the patient stick out the tongue and move it from side to side will assess the hypoglossal nerve. Checking shoulder strength tests the spinal accessory nerve.

The nurse is caring for a patient who has shallow breaths following abdominal surgery. Which respiratory complication is most likely to occur as a result of the patient's breathing pattern? a. Aspiration b. Atelectasis c. Cor pulmonale d. Pulmonary fibrosis

b. Atelectasis Atelectasis is collapse of the alveoli due to shallow breathing after surgery and general anesthesia. The patient should be encouraged to take deep breaths to avoid development of atelectasis. Shallow breathing will not lead to aspiration, cor pulmonale, or pulmonary fibrosis

The nurse is applying soft wrist restraints to the patient. Where will the restraints be tied to the patient's bed? a. Side rails b. Bedframe c. Footboard d. Headboard

b. Bedframe Attach restraint straps to the portion of the bedframe that moves when raising or lowering the head of the bed. Do not attach to the side rails. Attaching the restraint straps to a portion of the bedframe that does not move (headboard or footboard) will injure the patient.

Which intervention will most effectively maintain breathing function for a patient with muscle weakness due to amyotrophic lateral sclerosis (ALS)? a. Teaching pursed-lip breathing exercises b. BiPAP (bi-level positive airway pressure) c. Administration of oxygen via nasal cannula d. CPAP (continuous positive airway pressure)

b. BiPAP (bi-level positive airway pressure) ALS results in progressive respiratory muscle weakness. The patient with ALS will benefit from BiPAP to provide assistance during expiration and prevent the alveoli from collapsing during exhalation. CPAP is used to treat sleep apnea. Oxygen administration and pursed-lip breathing will not assist respiratory muscle weakness.

The nurse has orders to titrate the patient's oxygen to maintain a pulse oximetry level greater than 94%. The patient's pulse oximetry will not rise above 90% despite use of a nonrebreather mask. Which is the appropriate action of the nurse? a. Insert an oral airway and apply a full face oxygen mask. b. Call respiratory therapy to consider BiPAP support for the patient. c. Remove the nonrebreather mask and replace it with a Venturi mask. d. Place an oxygen nasal cannula underneath the patient's nonrebreather mask

b. Call respiratory therapy to consider BiPAP support for the patient. BiPAP is a form of noninvasive ventilation that is used to assist patients in respiratory failure without intubation. The nurse should respiratory therapy to consider BiPAP support for the patient because oxygen via nonrebreather mask is not sufficient to maintain oxygenation. Inserting an oral airway will cause the patient to gag. The nonrebreather mask covers the mouth and nose already. Venturi mask will deliver less oxygen than a nonrebreather mask. An oxygen nasal cannula should not be placed underneath the nonrebreather mask.

The nurse is caring for a patient who has just undergone knee-replacement surgery. The patient has incontinent of continuous oozing stool for the last few days. Which is the appropriate action of the nurse? a. Administer loperamide 8 mg PO BID. b. Check the patient's rectum for presence of impacted stool. c. Liberally apply skin barrier cream to prevent perineal irritation. d. Encourage the patient to drink at least 2 L of fluid each day.

b. Check the patient's rectum for presence of impacted stool. Continuous oozing stool for several days indicates the possibility of fecal impaction, especially as the patient has been recuperating after joint surgery. Loperamide would cause fecal impaction to worsen and should not be administered. Potential skin breakdown does not address the root cause of the oozing stool. Fluid intake, in the presence of continuous oozing stool, is less likely to help the patient versus assessing for a fecal impaction.

The nurse is caring for a patient who had surgery to remove most of the large intestine. Which finding will the nurse expect to note when assessing the patient? a. Soft formed stools b. Chronic loose stools c. Frequent stool impaction d. Intermittent constipation

b. Chronic loose stools The large intestine is responsible for resorption of water from the stool. Removal of the large intestine results in loose stools as the water is not sufficiently removed from the stool before evacuation. Removal of the large intestine does not cause impaction or constipation.

The patient reports using history of ibuprofen for arthritis pain after telling the nurse about a severe allergy to NSAID medications. The nurse asks the patient to further explain the allergy and use of ibuprofen. Which action is demonstrated by the nurse? a. Focusing b. Clarifying c. Summarizing d. Sharing observations

b. Clarifying The nurse's request for more information is used to clarify the patient's conflicting statements. Clarifying validates whether the person interpreted the message correctly. Focusing directs conversation to a specific topic or issues when a discussion becomes unclear. Summarizing provides a concise review of main ideas. Sharing observations is commenting on a patient's appearance and how he or she sounds and acts such as, "I see you didn't eat any breakfast."

The nurse notes that the patient has bilateral entropion. What will the nurse plan to do as a result of this assessment finding? a. Instruct the patient to rinse the mouth gently with warm saline solution. b. Clean the patient's eyelids gently and apply antibiotic ointment. c. Place a wedge pillow between the patient's legs to prevent crossing at the hip. d. Elevate the head of the patient's bed and administer supplemental oxygen.

b. Clean the patient's eyelids gently and apply antibiotic ointment. Entropion is the turning inward of eyelashes so that they irritate the conjunctiva and cornea. The nurse should clean the patient's eyelids gently and apply antibiotic ointment. The other interventions are not appropriate to address entropion

The nurse is caring for a patient who has pneumonia and chronic bronchitis. The patient is very congested, coughing up copious amounts of thick green sputum. Which breath sounds will the nurse expect to hear? a. Fine crackles b. Coarse rhonchi c. Diminished bases d. Scattered wheezes

b. Coarse rhonchi Coarse rhonchi are heard when thick sputum is heard in the airways. Fine crackles are heard when there is watery fluid in the airways. Diminished bases are heard when the patient is taking shallow breaths. Scattered wheezes are heard when there is narrowing of the airways such as from asthma

A patient in the outpatient setting was diagnosed with atopic dermatitis. What interventions will the plan of care focus primarily on? a. Decreasing pain b. Decreasing pruritus c. Preventing infection d. Promoting drying of lesions

b. Decreasing pruritus Pruritus is the major manifestation of atopic dermatitis and causes the greatest morbidity. The urge to scratch may be mild and self-limiting, or it may be intense, leading to severely excoriated lesions, infection, and scarring.

Which assessment findings indicate that the patient has developed fluid overload as a result of transfusing two units of packed red blood cells? a. Hypotension and thirst b. Dyspnea and tachycardia c. Shivering and high fever d. Hypotension and urticaria

b. Dyspnea and tachycardia Dyspnea and tachycardia are signs of pulmonary edema and fluid overload after transfusion. Hypotension and urticaria indicate anaphylactic shock. Shivering and high fever are signs of a transfusion reaction rather than fluid overload. Hypotension and thirst indicate fluid deficit.

The nurse suspects that the postoperative patient has developed a deep vein thrombosis. Which is the priority action of the nurse? a. Obtain an order for STAT bleeding time, D-dimer, and platelet count. b. Elevate the patient's leg and assess for chest pain or shortness of breath. c. Apply a mobile compression device (MCD) to the patient's affected leg. d. Use a Doppler machine to confirm the presence of bilateral pedal pulses.

b. Elevate the patient's leg and assess for chest pain or shortness of breath The nurse should elevate the patient's leg if DVT is suspected. The nurse should assess for chest pain or shortness of breath to determine if a clot has broken off and become a pulmonary embolism. Ordering laboratory tests and assessing pedal pulses is less important than assessing for chest pain and elevating the leg. The MCD should be removed from the leg if DVT is suspected.

The patient verbalized frustration to the nurse about the lengthy recovery time after surgery. The nurse's response was "I understand how you want to be feeling better already." Which communication technique was used by the nurse? a. Sympathy b. Empathy c. Focusing d. Self-disclosure

b. Empathy Empathy is the ability to understand and accept another person's perspective. Although no one can ever totally know another's experiences, a nurse can try to understand what the person is experiencing. Focusing directs conversation to a specific topic or issue when a discussion becomes unclear. Self-disclosures are personal statements intentionally revealed to the other person. Sympathy is the concern, sorrow, or pity that you feel for a patient when you personally identify with his or her needs. Unlike empathy, which tries to understand a patient's experience, sympathy takes a subjective look at the patient's world.

Which is the appropriate intervention for a patient with risk for constipation related to prolonged immobility? a. Check the patient's rectum for the presence of a fecal impaction. b. Encourage the patient to consume plenty of fluids and dietary fiber. c. Instruct the patient about the body's need for daily bowel movements. d. Recommend the use of daily laxatives to prevent constipation or impaction

b. Encourage the patient to consume plenty of fluids and dietary fiber. Intake of fluids and fiber will help prevent development of constipation. The nurse should not check the rectum for fecal impaction unless one is suspected. It is not necessary to have daily bowel movement and the use of daily laxatives is not recommended.

Which action of the nurse will help to reduce the effects of orthostatic hypotension? a. Perform isometric and range-of-motion exercises. b. Encourage the patient to move from a sitting position to standing position slowly. c. Place trochanter rolls on either side of the patient's hips. d. Participate in chest physiotherapy and incentive spirometry.

b. Encourage the patient to move from a sitting position to standing position slowly Moving positions slowly will help with orthostatic hypotension by allowing the body to adapt. Isometric exercises improve activity tolerance but do not have any effect on preventing orthostatic hypotension. Participating in chest physiotherapy assists patients with decreasing effects of pulmonary complications. Trochanter rolls help prevent external rotation of the patient's hips when in the supine position.

Which is the appropriate intervention for the patient with the diagnosis powerlessness related to pain and weakness after right hip fracture? a. Place the patient's personal items within view on the left side of the bed. b. Encourage the patient to participate in self-care and recreational activities. c. Assess the patient's sleep patterns and potential for obstructive sleep apnea. d. Reposition the patient and pad bony prominences to prevent skin breakdown.

b. Encourage the patient to participate in self-care and recreational activities Powerlessness can be minimized by encouraging the patient to participate in self-care and recreational activities as much as possible. This will provide motivation and enable the patient to feel a sense of control over the situation. Placing the patient's personal items within reach, assessing sleep patterns, and repositioning the patient will not minimize powerlessness.

The patient reports feeling an urge to urinate even though an indwelling urinary catheter is in place. Which is the priority action of the nurse? a. Measure the patient's urinary output. b. Ensure that the catheter tubing is not kinked. c. Provide perineal care to the patient for comfort. d. Reassure the patient that the sensation is to be expected.

b. Ensure that the catheter tubing is not kinked. The patient will feel an urge to urinate if the catheter tubing becomes kinked and the urine is not able to drain. The nurse should first check to make sure that the tubing is patent and draining freely. Once this is done the nurse can measure urinary output and provide perineal care. If there is no evidence of urethral irritation or kinked tubing, the patient may be reassured that the sensation may be expected.

Which teaching approach is demonstrated when the nurse provides guidance while the patient performs the sterile dressing change? a. Telling b. Entrusting c. Reinforcing d. Participating

b. Entrusting The entrusting approach provides the patient the opportunity to manage self-care. The patient accepts responsibilities and correctly performs the task while a nurse observes the patient's progress and remains available for assistance. Telling involves explicit instructions with no feedback. Participating involves mutual goal setting with the patient helping decide the content. Reinforcing is using a stimulus that increases the probability of a response.

Approximately 30 minutes into the transfusion of this blood product, the patient becomes short of breath with wheezing, low BP, and hives. Which medication must be administered to the patient immediately? a. 500 mL 0.9 NS IV fluid bolus b. Epinephrine 0.4 mg IM c. Diphenhydramine 50 mg IV d. Methylprednisolone 40 mg IV

b. Epinephrine 0.4 mg IM The patient is experiencing an anaphylactic reaction to the transfusion and requires epinephrine immediately to prevent the development of shock. Methylprednisolone, normal saline IV, and diphenhydramine may be given after epinephrine.

The nurse is caring for a patient who has suffered a spinal cord injury and is concerned about the patient's elimination status. What is the nurse's best action? a. Speak with the patient's family about food choices. b. Establish a bowel and bladder program for the patient. c. Speak with the patient about past elimination habits. d. Establish a bedtime ritual for the patient.

b. Establish a bowel and bladder program for the patient. Establishing a bowel and bladder program for the patient is a priority to be sure that adequate elimination is happening for the patient with a spinal cord injury. Speaking with the family to determine food choices is not the primary concern. Speaking with the patient to know past elimination habits does not apply, because the spinal cord injury changes elimination habits. Establishing a bedtime ritual does not apply to elimination.

Lactated Ringer's solution is administered to the patient during surgery. What is the purpose of this infusion? a. Cause cells to shrink and reduce swelling. b. Expand the body's intravascular fluid volume. c. Move fluid from intravascular space into cells. d. Pull fluid from cells into the intravascular space.

b. Expand the body's intravascular fluid volume Fluids that have the same osmolality as normal blood are called isotonic. Isotonic solutions such as Lactated Ringer's expand the body's extracellular fluid volume without causing water to shift in or out of cells. Infusion of hypertonic intravenous solutions (more concentrated than normal blood), such as 3% sodium chloride, pulls fluid from cells by osmosis, causing them to shrink. Physiologically hypotonic solutions (less concentrated than normal blood after they are infused) move water from the extracellular compartment into the cells by osmosis, causing them to swell.

Which is an example of an adjuvant medication for pain management? a. Naloxone b. Gabapentin c. Morphine sulfate liquid d. Fentanyl transdermal patch

b. Gabapentin Adjuvant medications enhance the properties of analgesic medications. Gabapentin is an anticonvulsant and is a type of adjuvant medication. Fentanyl transdermal patch and morphine sulfate liquid are examples of narcotic pain medications. Naloxone is used to reverse symptoms of narcotic overdose.

The nurse hears a blowing sound when auscultating the patient's carotid artery. What is the appropriate action of the nurse? a. Notify the health care provider immediately as the patient may be having a stroke. b. Gently feel over the patient's carotid artery to check for a faint vibration. c. Massage the patient's carotid artery to determine if the blowing sound subsides. d. Palpate the patient's carotid pulse while the patient's neck is hyperextended.

b. Gently feel over the patient's carotid artery to check for a faint vibration. A bruit is a blowing or swooshing sound heard with the stethoscope over a blood vessel. A bruit is caused by turbulent blood flow through a vein or artery. If a bruit is heard over the carotid artery, the nurse should gently palpate the area for a thrill. A thrill is also known as a palpable bruit, a soft vibration caused by the turbulent blood flow through a vein or artery

The patient tells the nurse that it is much easier to read books on the tablet computer after applying a matte screen protector. Which is the best explanation for this? a. Glare causes headaches. b. Glare reduces visual acuity. c. Bright light overstimulates the retina. d. Too much light damages the iris.

b. Glare reduces visual acuity. When a patient ages, the pupil loses the ability to adjust to bright light. An antiglare screen protector therefore makes it easier for the patient to read text on the tablet computer. While glare can cause headaches for some people, it does not explain why the patient has an easier time reading text on the computer after the antiglare screen protector is applied. Overly bright light does not cause overstimulation of the retina or damage to the iris.

Which technique is most appropriate when performing a physical assessment for a male Muslim patient? a. Use short, simple sentences and complete the physical assessment as quickly as possible. b. Have a male nurse perform the assessment and expose only the body part being examined. c. Stand close to the patient, speak clearly, and be careful not to exaggerate lip movements or facial expressions. d. Use therapeutic touch to reassure the patient and convey the nurse's sense of concern.

b. Have a male nurse perform the assessment and expose only the body part being examined. The male Muslim patient must not expose himself to women other than his wife. It is important for a male nurse to perform the assessment. Modesty is also extremely important so the body should remain covered except for the part being examined. There is no need to complete the assessment quickly or limit facial expressions. Therapeutic touch is not the priority assessment technique and should be used sparingly with patients from other cultures.

Which is the highest priority concern for the nurse who is educating the homeless patient about medications, appointments, and therapies for management of diabetes? a. Motivation b. Health literacy c. Developmental stage d. Psychomotor learning

b. Health literacy Health literacy includes patients' reading and math skills, comprehension, the ability to make health-related decisions, and successful functioning as a consumer of health care. It is a strong predictor of health status and patient outcomes. The homeless patient is at high risk for having minimal health literacy. Psychomotor learning occurs when patients acquire skills that require the integration of knowledge and physical skills. Developmental stage is not as important as health literacy and developmental stage is more important when teaching children. Motivation is an internal impulse, such as an emotion or need, which prompts, guides, and sustains human behavior.

The nurse assesses a patient during suctioning. Which finding indicates that the procedure should be stopped immediately? a. Pulse oximetry decreases from 98% to 92%. b. Heart rate decreases from 78 to 40 beats/minute. c. Respiratory rate increases from 16 to 20 breaths/minute. d. Blood pressure increases from 110/70 to 120/80 mm Hg.

b. Heart rate decreases from 78 to 40 beats/minute. Decreased heart rate to 40 beats/minute indicates that the patient is not tolerating the suction procedure and it should be stopped immediately. Pulse of 92%, respiratory rate of 20, and BP 120/80 do not warrant immediately stopping suction.

The nurse is caring for a patient with a necrotic wound. Which dressing would be the best choice for the nurse to use on this type of wound to help with debridement? a. Transparent film b. Hydrogel dressing c. Dry nonstick gauze d. Hydrocolloid dressing

b. Hydrogel dressing Hydrogel dressings contain a high percentage of water and are indicated for wounds that have a high percentage of necrotic tissue (the hydrogel facilitates debridement by softening the dead tissue). Gauze dressings are best for wounds with moderate drainage, deep wounds, undermining, and tunnels. Transparent film dressings are used as a primary dressing in wounds with minimal tissue loss that have very little wound drainage. Hydrocolloid dressings are used for stage 1, 2, and 3 pressure injuries

Which medication order will provide the most immediate relief of the patient's acute pain? a. Morphine sulfate 5 mg PO b. Hydromorphone 0.5 mg IV c. Buprenorphine transdermal patch 10 mg d. Oxymorphone 30 mg extended release

b. Hydromorphone 0.5 mg IV The IV route of administration allows for the quickest onset of action for medications. IV hydromorphone will provide the most immediate relief of the patient's pain. Morphine sulfate will provide relief when given orally but not as fast as IV. Buprenorphine transdermal patch, and oxymorphone extended-release are formulations that will take longer to work.

A nurse is caring for a patient who is visually impaired. Which technique will the nurse use to facilitate communication? a. Touch the patient before speaking. b. Identify self when entering the room. c. Quietly leave the room when finished. d. Keep the room dimly lit for calmness.

b. Identify self when entering the room. For a visually impaired patient, identify yourself when entering the room. The nurse should communicate verbally before touching the patient who is visually impaired. Notify the patient when leaving the room; do not quietly leave the room when finished as the patient will think you are still in the room. Ensure that lighting is adequate for the patient to see the speaker; do not keep it dimly lit.

The patient's blood pressure is 180/100. Why does the patient's heart have to work harder due to the high blood pressure? a. Increased preload b. Increased afterload c. Decreased contractility d. Increased stroke volume

b. Increased afterload Afterload is the resistance to the blood being pumped from the left ventricle. The patient's hypertension increases afterload and increases the heart must work harder to push against it. Hypertension does not increase cardiac workload due to increased preload, decreased contractility, or increased stroke volume.

Which is the highest priority nursing diagnosis for the patient admitted with pneumonia? a. Activity intolerance related to increased oxygen demand with exertion b. Ineffective airway clearance related to inability to cough up thick secretions c. Risk for fluid volume deficit related to inadequate intake of fluids with fever d. Imbalanced nutrition related to loss of appetite and increased metabolic demand

b. Ineffective airway clearance related to inability to cough up thick secretions Ineffective airway clearance is the highest priority nursing diagnosis as it relates to oxygen, the body's most basic need. Risk for fluid volume deficit, imbalanced nutrition, and activity intolerance may be addressed once the airway is clear.

The nurse is caring for a patient who had a stroke because of lack of understanding about how to take the prescribed blood pressure medication. Which is the priority nursing diagnosis for this patient? a. Noncompliance related to patient's refusal to follow the prescribed treatment regimen b. Ineffective therapeutic regimen management related to lack of understanding about prescribed medications c. Ineffective health maintenance related to lack of expressed interest in taking prescribed medications correctly d. Readiness for enhanced decision making related to desire to choose the course of action that best meets health need

b. Ineffective therapeutic regimen management related to lack of understanding about prescribed medications Ineffective therapeutic regimen management related to lack of understanding about prescribed medications is the priority nursing diagnosis for the patient because the patient's knowledge deficit about the prescribed medications led to the stroke. The nurse will help teach the patient about the medications and ensure that they are taken exactly as prescribed. Nothing indicates that the patient refused to follow the prescribed treatment plan or that the patient was not interested in taking the prescribed medications. Readiness for enhanced decision making is not the priority diagnosis as it does not address the patient's need to take prescribed medications correctly.

Which complication is suspected when the patient's IV site is swollen, cool, and pale with no blood return? a. Phlebitis b. Infiltration c. Thrombophlebitis d. Local inflammation

b. Infiltration An infiltration occurs when IV fluids leak into the subcutaneous tissue around the venipuncture site because the catheter tip no longer is in the vein. Infiltration causes swelling (from increased interstitial fluid), paleness, and coolness (from decreased circulation) around the venipuncture site. The IV infusion may slow or stop. Pain may occur, increasing as the infiltration progresses. Phlebitis is inflammation of a vein. Signs and symptoms include redness, tenderness, and warmth along the course of the vein starting at the access site, with possibly a red streak and/or palpable cord along the vein. Phlebitis can be dangerous because blood clots (thrombophlebitis) can form, increasing the risk for an embolus, a clot that becomes dislodged and can travel to the lungs. Local infection at the VAD site is possible. The insertion site will be red and/or edematous; exudate may occur.

A nurse has prepared a discharge teaching plan for an adult patient who is not being compliant. Which strategy should the nurse include to help increase compliance with following discharge instructions? a. Individualized handout b. Instructional videos c. Internet resources d. Self-help books

b. Instructional videos An instructional video would provide a visual/auditory approach for discharge instructions. Repeatedly not following written instructions is a clue that the patient may not be able to read or understand the information. While reviewing the literacy level of an adult patient can be challenging, the information that they have not been able to follow previous written instructions would suggest that the nurse use an alternate strategy that does not require a high degree of literacy. An individualized handout would be written, very similar to previous instructions, and would not address a concern about literacy. Internet resources generally require an individual to be able to read, and although videos are available through the Internet, this is not the best response. Self-help books would be appropriate for an individual who reads. There is a question about whether this patient is literate, so these would not be the best choice

The patient expresses frustration about not being able to function as the family breadwinner any longer due to chronic severe pain. Which psychosocial nursing diagnosis is most appropriate for this patient's concern? a. Risk for loneliness related to need for prescription pain medications b. Interrupted family processes related to changes in assigned roles c. Disturbed sensory perception related to insufficient environmental stimuli d. Moral distress related to time constraints for ethical decision making

b. Interrupted family processes related to changes in assigned roles Interrupted family processes is the appropriate nursing diagnosis because the patient no longer fills the role of family breadwinner. This has led to a change in the functioning of the family. The scenario does not indicate risk for loneliness, disturbed sensory perception, or moral distress.

Why does the nurse utilize a head-to-toe approach when performing physical examinations of patients? a. The head-to-toe format excludes unnecessary body systems. b. It is a methodical way to include all body systems. c. It reduces time by allowing examination of only one side. d. It requires that painful procedures be done first.

b. It is a methodical way to include all body systems. A head-to-toe approach includes all body systems and helps to anticipate each step. In an adult, a nurse begins by assessing the head and neck, progressing methodically down the body to include all body systems. Both sides of the body must be compared for symmetry. Any painful procedures should be performed near the end of the examination.

The patient has severe hyperthyroidism and will have surgery tomorrow. What assessment is most important for the nurse to perform in order to detect development of the highest risk acid-base imbalance? a. Urine output and color b. Level of consciousness c. Heart rate and blood pressure d. Lung sounds in lung bases

b. Level of consciousness Thyroid hormone increases metabolic rate, causing a patient with severe hyperthyroidism to have high risk of metabolic acidosis from increased production of metabolic acids. Metabolic acidosis decreases level of consciousness. Changes in urine output, urine color, and lung sounds are not signs of metabolic acidosis. Although metabolic acidosis often causes tachycardia, many other factors influence heart rate and blood pressure, including thyroid hormone.

Which type of reinforcement is used when the nurse gives a sticker to a pediatric patient every time the incentive spirometer is used? a. Social b. Material c. Activity d. Negative

b. Material Examples of material reinforcers are food, toys, and music. These work best with young children. Use social reinforcers (e.g., smiles, compliments, words of encouragement, or physical contact) to acknowledge a learned behavior. Activity reinforcers (e.g., physical therapy) rely on the principle that a person is motivated to engage in an activity if there is an opportunity to participate in more desirable activity upon completion of this first activity. Negative reinforcement (frowning) may work but people usually respond better to positive reinforcement.

A 70-year-old retired patient is interested in nondrug, mind-body therapies, self-management, and alternative strategies to deal with joint discomfort from rheumatoid arthritis. What options should the nurse consider in the plan of care considering the patient's expressed wishes? a. Stationary exercise bicycle, free weights, and spinning class b. Mind-body therapies such as music therapy, distraction techniques, meditation, prayer, hypnosis, guided imagery, relaxation techniques, and pet therapy c. Chamomile tea and IcyHot gel d. Acupuncture and attending church services

b. Mind-body therapies such as music therapy, distraction techniques, meditation, prayer, hypnosis, guided imagery, relaxation techniques, and pet therapy Mind-body therapies are designed to enhance the mind's capacity to affect bodily function and symptoms and include music therapy, distraction techniques, meditation, prayer, hypnosis, guided imagery, relaxation techniques, and pet therapy, among many others. Stationary exercise bicycle, free weights, and spinning are not mind-body therapies. They are classified as exercise therapies. Chamomile tea and IcyHot gel are not mid-body therapies per se. They are classified as herbal and topical thermal treatments. Acupuncture is an ancient Chinese complementary therapy, while attending church services is a religious prayer mind-body therapy capable of enhancing the mind's capacity to affect bodily function and symptoms.

When conducting a spiritual assessment of a hospitalized patient, the nurse should remain aware of which potential barrier to effective communication? a. Clarifying the meaning of a patient's statement b. Multi-tasking while talking to the patient c. Listening to patients' complete statements d. Discussing patient's feelings while hospitalized

b. Multi-tasking while talking to the patient Several barriers may result in the nurses' inability to be totally present and communicate effectively with the patient. First, the nurse may be distracted by other things and may not pay attention to the patient. Multi-tasking while trying to listen to a patient may be a barrier to effective communication. Second, the nurses may miss the meaning of the patient's message because of failure to clarify the meaning of a word, a phrase, or a facial expression. Third, the nurse may interject personal feelings and reactions into the patient's situation rather than allow the patient to explore and discuss his own feelings and reactions. The last barrier occurs when the nurse is busy formulating a response while the patient is still talking. In this instance, the nurse never hears the patient's message.

A 62-year-old male has fallen while trimming tree branches sustaining tissue injury. He describes his condition as an aching, throbbing back. This is characteristic of what type of pain? a. Neuropathic pain b. Nociceptive pain c. Chronic pain d. Mixed pain syndrome

b. Nociceptive pain Nociceptive pain refers to the normal functioning of physiological systems that leads to the perception of noxious stimuli (tissue injury) as being painful. Patients describe this type of pain as aching, cramping, or throbbing. Neuropathic pain is pathologic and results from abnormal processing of sensory input by the nervous system as a result of damage to the brain, spinal cord, or peripheral nerves. Patients describe this type of pain as burning, sharp, and shooting. Chronic pain is constant and unrelenting such as pain associated with cancer. Mixed pain syndrome is not easily recognized, is unique with multiple underlying and poorly understood mechanisms like fibromyalgia and low back pain.

What causes the patient to feel thirsty and drink more water? a. Colloid osmotic pressure b. Osmoreceptor stimulation c. Increased oncotic pressure d. Decreased hydrostatic pressure

b. Osmoreceptor stimulation Thirst, a conscious desire for water, regulates fluid intake when plasma osmolality increases (osmoreceptor-mediated thirst) or the blood volume decreases (baroreceptor-mediated thirst and angiotensin II-mediated thirst). The thirst-control mechanism is in the hypothalamus of the brain. Osmoreceptors there continually monitor plasma osmolality; when osmolality increases, the hypothalamus stimulates thirst. Colloid osmotic pressure (oncotic pressure) is an inward-pulling force caused by the presence of protein molecules. Hydrostatic pressure is the force of a fluid pressing outward against the walls of its container. Thus capillary hydrostatic pressure is an outward-pushing force

The patient receives an infusion of albumin to pull water from the tissues into the bloodstream. Which process is demonstrated by this treatment? a. Diffusion b. Osmosis c. Filtration d. Clarification

b. Osmosis Osmosis is movement of water across a semipermeable membrane from a compartment of lower particle concentration to one that has a higher particle concentration. Diffusion is passive movement of electrolytes or other particles from an area of higher concentration to an area of lower concentration. Filtration is the net effect of several forces that tend to move fluid across a membrane. Clarification is explanation of a concept in greater detail or removal of impurities.

The nurse is caring for a patient who relies on laxatives to ensure daily bowel movements. Which is the appropriate nursing diagnosis for this patient? a. Risk for constipation related to irregular bowel elimination patterns b. Perceived constipation related to expectation of daily bowel movements c. Toileting self-care deficit related to inability to set regular defecation regimen d. Powerlessness related to inability to have daily bowel movements without laxatives

b. Perceived constipation related to expectation of daily bowel movements Perceived constipation is the patient's use of laxatives to ensure daily bowel movements. This leads to laxative abuse and dependence with possible damage to the colon. Risk for constipation and toileting self-care deficit are less appropriate for the patient than perceived constipation. Powerlessness is not a priority nursing diagnosis for this patient

Which is the appropriate method to obtain a urinalysis specimen for culture and sensitivity from an incontinent female patient? a. Obtain a midstream specimen. b. Perform straight catheterization. c. Obtain a double-voided specimen. d. Leave a fresh bedpan under the patient

b. Perform straight catheterization. The nurse should perform straight catheterization to obtain a clean catch urinalysis from an incontinent female patient. The patient will not be able to provide a midstream or double-voided specimen. Leaving the bedpan under the patient will lead to skin breakdown and contamination of the specimen.

The patient is brought to the emergency department after a motor vehicle accident. The patient is diagnosed with internal bleeding. What is the priority of care for this patient? a. Mental alertness b. Perfusion c. Pain d. Reaction to medications

b. Perfusion Perfusion is the correct answer, because with internal bleeding, the nurse should monitor vital signs to be sure perfusion is happening. Mental alertness, pain, and medication reactions are important but not the primary concern.

The nurse is caring for a patient who has a dangerously low platelet count. Which assessment finding will the nurse expect to note during physical examination of the patient? a. Bright yellow jaundice in the sclera of the patient's eyes b. Pinpoint red spots on the skin of the patient's torso c. Dry, flaky skin with evidence of frequent scratching d. Thick indurated skin across the patient's back

b. Pinpoint red spots on the skin of the patient's torso Dangerously low platelet count leads to easy bruising and bleeding. Tiny hemorrhages in the skin layers cause pinpoint red or purple dots on the skin called petechiae. Dry flaky skin, thick, indurated skin and jaundice are not indicative of a low platelet count.

The nurse reviewed four patients at the beginning of the shift. Which finding should the nurse report immediately to the physician? a. Swollen ankles in patient with compensated heart failure b. Positive Chvostek sign in patient with acute pancreatitis c. Dry mucous membranes in patient taking a new diuretic d. Constipation in patient who has advanced breast cancer

b. Positive Chvostek sign in patient with acute pancreatitis Positive Chvostek sign indicates increased neuromuscular excitability, which can progress to dangerous laryngospasm or seizures and thus needs to be reported first. The other assessment findings are less urgent and need further assessment. Bilateral ankle edema is a sign of ECV excess, and follow-up is needed, but the situation is not immediately life-threatening. Dry mucous membranes in a patient taking a diuretic may be associated with ECV deficit; however, additional assessments of ECV deficit are required before reporting to the physician. Constipation has many causes, including hypercalcemia and opioid analgesics, and it needs action, but not as urgently as a positive Chvostek sign.

Which assessment finding supports the nursing diagnosis overflow urinary incontinence related to urethral obstruction? a. Advanced dementia prevents the patient from indicating need to urinate b. Postvoid residual of 900 mL after incontinence of small amounts of urine c. Leakage of urine around the urostomy appliance leading to skin irritation d. Incontinence of large amounts of urine every time the patient coughs or sneezes

b. Postvoid residual of 900 mL after incontinence of small amounts of urine Overflow urinary incontinence occurs when small amounts of urine are lost due to retention of large amount of urine in the bladder. A postvoid residual of 900 mL after incontinence of small amounts of urine supports the overflow urinary incontinence diagnosis. Overflow urinary incontinence does not apply to patients with urostomy appliances. Incontinence of urine with coughing or sneezing demonstrates stress incontinence. Inability to indicate need to urine leads to functional incontinence.

Which assessment finding is expected for a patient with a chest tube for treatment of hemothorax? a. Constant bubbling in the water-seal chamber b. Presence of bloody drainage from the chest tube c. The patient denies having pain at the chest tube site d. Subcutaneous emphysema is present around the chest tube site

b. Presence of bloody drainage from the chest tube Hemothorax occurs when the pleural space is filled with blood. The nurse expects the patient's chest tube drainage to be bloody during treatment of hemothorax. Constant bubbling in the water-seal chamber and subcutaneous emphysema indicate leakage of air from the tube into the chest. It would be unusual for the patient to deny having pain at the chest tube site.

Which technique should the nurse use when providing information to a patient with a health literacy level of fifth grade? a. Use the passive voice of language. b. Present the most important information first. c. Use medical terminology to explain the concepts. d. Shift from subject to subject until the patient responds.

b. Present the most important information first To promote understanding in a patient with a health literacy level of fifth grade is to present the most important information first. Use the active voice instead of passive. Break complex information into understandable chunks; do not shift from subject to subject. Use simple language, avoid medical jargon.

The wrong type of medication was administered to the patient. Which type of error is this? a. Exposure-related accident b. Procedure-related accident c. Equipment-related accident d. Organization-related accident

b. Procedure-related accident A procedure-related accident is caused by health care providers and includes medication and fluid administration errors, not putting external devices on correctly, and improperly performing procedures such as dressing changes. An equipment-related accident results from misuse, disrepair, malfunction, or electrical hazard. There is no classification of exposure-related accident or organization-related accident.

Which bedtime action by the nurse may make it more difficult for the patient to fall asleep? a. Giving the patient a gentle backrub b. Providing a warm cup of hot chocolate c. Encouraging the patient to use the bathroom d. Giving the patient an extra blanket when cold

b. Providing a warm cup of hot chocolate Coffee, tea, cola, and chocolate cause a person to stay awake or awaken throughout the night. Promote comfort by encouraging the patient to wear loose-fitting nightwear, void before bedtime, give a relaxing backrub, and offer an extra blanket to prevent chilling when trying to fall asleep.

The patient's wound has thick creamy yellow drainage present on the dressing. How will the nurse document this finding? a. Serous drainage b. Purulent drainage c. Sanguineous drainage d. Serosanguineous drainage

b. Purulent drainage Purulent is thick, yellow, green, or brown, indicating the presence of dead or living organisms and white blood cells. Serosanguineous is pale, more watery, and a combination of plasma and red cells, which may be blood streaked. Serous is clear, watery plasma. Sanguineous is fresh bleeding

Into which seating position will the nurse teach a family to place their 18-month-old toddler in the family car? a. Front seat facing backward b. Rear seat facing backward c. Front seat facing forward d. Rear seat facing forward

b. Rear seat facing backward Infants and toddlers should be in a rear-facing car safety seat until 2 years of age. The rear seat is the safest location for children less than 13 years of age.

The nurse educates the patient about what to expect during suctioning of the tracheostomy tube. Which term best describes the patient's communication role? a. Channel b. Receiver c. Message d. Sender

b. Receiver The patient is the receiver in this scenario, as they have received the message. The nurse is the sender in this scenario. The sender is the person who delivers the message. The message is the content of the conversation; in this scenario explaining what will happen is the message. The channel is the means of conveying and receiving messages through visual, auditory, and tactile senses, the nurse's spoken words in this scenario.

The nurse is caring for a middle-aged adult who verbalizes a desire to start jogging and has a goal to run a half marathon. Which is the most appropriate action of the nurse? a. Tell the patient that exercising during middle age could lead to injury. b. Recommend that the patient have a complete physical examination first. c. Inform the patient that it is unwise to take up new sports during middle age. d. Explain that the disability from normal aging prevents taking up new sport

b. Recommend that the patient have a complete physical examination first Taking up new sports and activities during middle age is acceptable but the patient should have a physical examination first to ensure that there are no significant health impairments. Taking up a new sport during middle age could lead to injury but that should not prevent the patient from taking on new activities. The normal aging process does not necessarily prevent taking up new sports

The patient's incision is fading to a pale pink following surgery 2 months previously. Which stage of the healing describes the current status of the patient's wound? a. Hemostasis phase b. Remodeling phase c. Proliferative phase d. Inflammation phase

b. Remodeling phase The patient's wound is at the remodeling phase as the healing process continues to strengthen the scar tissue in the wound. The proliferative phase causes new capillary networks to form that provide oxygen and nutrients for new tissue and contribute to the synthesis of collagen. In the inflammation phase the goal is to establish a clean wound bed and obtain bacterial balance. Hemostasis phase controls bleeding.

A patient refuses to remove a religious headscarf when being bathed. Which is appropriate action of the nurse? a. Remove the headscarf because its presence hinders hygiene. b. Respect the patient's wishes and work around the headscarf. c. Explain to the patient that the headscarf has no real spiritual value. d. Identify the refusal to remove the headscarf as a sign of spiritual distress.

b. Respect the patient's wishes and work around the headscarf. To care for and meet the spiritual needs of your patients, it is essential to respect each patient's personal beliefs. People experience the world and find meaning in life in different ways and the spiritual garment has meaning for the patient. Caring for your patients' spiritual needs requires you to be compassionate and remove any personal biases or misconceptions. You need to recognize that not all patients have spiritual problems. Patients bring certain spiritual resources that help them assume healthier lives, recover from illness, or face impending death.

The nurse is caring for a dyspneic patient with a long history of smoking. The blood gas report shows a pH of 7.33, PaCO2 of 47, PaO2 of 78, and HCO3 - of 26. What is the patient's acid-base status? a. Respiratory alkalosis b. Respiratory acidosis c. Metabolic alkalosis d. Metabolic acidosis

b. Respiratory acidosis Normal arterial blood pH value is 7.35 to 7.45 (acidic is less than 7.35, and alkalotic is greater than 7.45). Respiratory acidosis is an increased PaCO2 and an increased hydrogen ion concentration (pH below 7.35) that reflect the excess carbonic acid (H2CO3) in the blood. Hypoventilation produces respiratory acidosis, which causes the cerebrospinal fluid and brain cells to become acidic, thus decreasing the level of consciousness. Respiratory alkalosis is a decreased PaCO2 and increased pH (above 7.45) that reflect the deficit of carbonic acid (H2CO3) in the blood. Metabolic acidosis results from conditions that increase metabolic acids in the body or decrease the amount of base (bicarbonate). The bicarbonate level is always low because the bicarbonate system buffers metabolic acids. Metabolic alkalosis results from a gain of bicarbonate or excessive excretion of metabolic acid

Which patient care need may be delegated to the nursing assistant? a. Providing discharge teaching about fall precautions in the home b. Responding to the patient's bed alarm as the patient attempts to get up c. Performing a mental status assessment to check for confusion or delirium d. Obtaining a consult for physical therapy for strengthening/balance exercises

b. Responding to the patient's bed alarm as the patient attempts to get up The nursing assistant can respond to the patient's bed alarm as the patient attempts to get up. The registered nurse should provide teaching, perform assessments, and obtain consults for the patient

The nurse must awaken a patient from Stage 4 non-rapid eye movement sleep in order to prepare the patient for a procedure. The patient is disoriented. What is the nurse's best action? a. Notify the healthcare provider. b. Review again the patient's orientation. c. Administer an anti-anxiety medication. d. Cancel the patient's procedure.

b. Review again the patient's orientation. Stage 3 (N3) and Stage 4 (N4) are the periods of deep sleep. N3 is characterized by very slow brain waves called delta waves interspersed with smaller, faster waves. In Stage 4 (N4), the EEG shows almost exclusively delta waves. In this type of sleep it is difficult to awaken the individual and muscle activity is very limited or may be completely absent. A person awakened from Stage 3 or 4 of sleep could be disoriented for a brief period of time before regaining awareness

While auscultating a patient with pneumonia, a nurse hears low-pitched, rumbling coarse sounds during inspiration and expiration. Which term will the nurse use to document this assessment finding? a. Crackles b. Rhonchi c. Wheezes d. Friction rub

b. Rhonchi Rhonchi are loud, low-pitched, rumbling coarse sounds heard either during inspiration or expiration. They may be cleared by coughing. Fine crackles are high-pitched fine, short, interrupted crackling sounds heard during end of inspiration; usually not cleared with coughing. Moist crackles are lower, more moist sounds heard during middle of inspiration; not cleared with coughing. Coarse crackles are loud, bubbly sounds heard during inspiration; not cleared with coughing. Wheezes are high-pitched, continuous musical sounds like a squeak heard continuously during inspiration or expiration; usually louder on expiration. A pleural friction rub has a dry, grating quality heard during inspiration; does not clear with coughing; heard loudest over lower lateral anterior surface.

Which nursing diagnosis is the highest priority for a patient who just received local anesthesia to the back of the throat for a diagnostic procedure? a. Feeding self-care deficit related to pain and discomfort b. Risk for aspiration related to depressed gag reflex c. Impaired social interaction related to slurred speech d. Impaired oral mucus membrane related to dry mouth

b. Risk for aspiration related to depressed gag reflex Local anesthesia works to block motor and sensory function of the affected tissue. Local anesthesia applied to the back of the throat will impair swallowing, depress the gag reflex, and put the patient at risk for aspiration. Feeding self-care deficit, impaired social interaction, and impaired oral mucus membrane diagnoses are not high priority.

Which is the priority nursing diagnosis for an adolescent who gets up at 5:00 a.m. every morning for school and studies until midnight every night? a. Fatigue related to insufficient rest and stress of academic demands b. Risk for injury related to inattention and excessive daytime sleepiness c. Deficient diversional activity related to lack of time for recreation and leisure d. Impaired social interaction related to time required to study and maintain grades

b. Risk for injury related to inattention and excessive daytime sleepiness Adolescents need between 8 to 10 hours of sleep each night. An adolescent who gets only 5 hours of sleep at night is at risk of injury due to intention and effects of excessive daytime sleepiness. This is particularly true if the adolescent is driving. Fatigue, deficient diversional activity, and impaired social interaction are all less important than the risk for injury

Which is the highest priority nursing diagnosis for a college student who is living away from home for the first time? a. Sleep deprivation related to noisy dormitory environment b. Risk-prone health behavior related to weekend binge drinking c. Relocation stress syndrome related to moving away from home d. Risk for loneliness related to being away from family and old friends

b. Risk-prone health behavior related to weekend binge drinking Weekend binge drinking indicates a risk-prone health behavior that could lead to liver damage, injury, or death. Sleep deprivation is not as important as binge drinking. Relocation stress syndrome and risk for loneliness relate to psychosocial needs, making these diagnoses lower priority than physical need for safety and rest

Prior to drug administration the nurse reviews the seven rights, which include right patient, right medication, right time, right dose, right education, right documentation, and what other right? a. Room b. Route c. Physician d. Manufacturer

b. Route The right route (e.g., oral or intramuscular) is an essential component to verify prior to the administration of any drug. The patient does not need to be in a specific location. There may be a number of physicians caring for a patient who prescribe medications for any given patient. A similar drug may be made by a number of different companies, and checking the manufacturer is not considered one of the seven rights. However, the nurse will want to be aware of a difference, because different companies prepare the same medication in different ways with different inactive ingredients, which can affect patient response.

A nurse is caring for a patient who signs and lip reads. When communicating, the most appropriate nursing action is to do which of the following? a. Rely on friends or family members to interpret for the patient. b. Sit facing the patient when speaking and ensure there is adequate light. c. Repeat the entire conversation if it is not clearly understood the first time. d. Speak louder and more distinctly than normal with exaggerated lip movements.

b. Sit facing the patient when speaking and ensure there is adequate light. Nurses can use a variety of communication techniques, including reading notes and writing notes, as well as reading lips and signing. When communicating, nurses should speak slowly and articulate clearly. The nurse should be seated facing the patient and ensure that there is enough light for the patient to see the nurse's lips clearly. When you are not understood, rephrase rather than repeating the entire conversation. Some patients with hearing impairments are able to speak normally. To more clearly hear what a person communicates, family and friends need to learn to move away from background noise, rephrase rather than repeat sentences, be positive, and have patience. On the other hand, some deaf patients have serious speech alterations.

Which action by the nurse will help to reduce the fears of a hospitalized young child? a. Stand over the bed when talking to the patient. b. Sit in a chair next to the bed when talking to the patient. c. Maintain constant eye contact with the patient at all times. d. Stay within 12 inches of the patient when talking to the patient.

b. Sit in a chair next to the bed when talking to the patient. The nurse should sit in a chair next to the bed. A nurse appears less dominant and less threatening when interacting at the patient's eye level. Looking down on a person (standing by the bed) establishes authority, but interacting at the same eye level indicates equality in the relationship. Constant eye contact can be intrusive or threatening to some people. Twelve inches is within the intimate zone and can be threatening.

Which behavior best demonstrates active listening by the nurse? a. Keeping arms crossed b. Sitting facing the patient c. Standing facing the patient d. Leaning away from the patient

b. Sitting facing the patient The best behavior is sitting facing the patient. Active listening enhances trust because the nurse communicates acceptance and respect for the patient. Several nonverbal skills facilitate attentive listening, which are identified by the acronym SOLER: Sit facing the patient. Observe an open posture. Lean toward the patient. Establish and maintain eye contact. Relax. Keeping arms crossed is a closed posture. Leaning toward, not away, from the patient is active listening. Sitting, not standing, is best.

The nurse is making rounds on the hospital unit and observes a patient sleeping. The patient's pulse and respiratory rates are slower than baseline. The nurse realizes the patient has most likely just entered which stage of non-rapid eye movement sleep? a. Stage 1 b. Stage 2 c. Stage 3 d. Stage 4

b. Stage In Stage 2 (N2), eye movement ceases; brain waves become even slower with the exception of an occasional burst or more rapid brain waves. Pulse rate and respirations slow and body temperature decreases as the individual moves toward deeper stages of sleep. Stage 3 (N3) and Stage 4 (N4) are the periods of deep sleep.

The patient has a large red, blistered area on the left hip. Which pressure injury stage will be recorded in the patient's chart? a. Stage 1 b. Stage 2 c. Stage 3 d. Stage 4

b. Stage 2 Stage 2: Partial-thickness loss of dermis presenting as a shallow open ulcer with a red-pink wound bed, without slough; may also present as an intact or open/ruptured serum-filled blister Stage 1: Intact skin with nonblanchable redness of a localized area, usually over a bony prominence Stage 3: Full-thickness tissue loss; subcutaneous fat may be visible, but bone, tendon, or muscle is not exposed; slough may be present but does not obscure the depth of tissue loss; may include undermining and tunneling Stage 4: Full-thickness tissue loss with exposed bone, tendon, or muscle; slough or eschar may be present; often includes undermining and tunneling

The patient is receiving an intravenous infusion of 40 mEq of potassium chloride in a 1000 mL solution of 0.9% saline. The patient states that the area around the IV site burns. What intervention does the nurse perform first? a. Notify the physician. b. Stop the IV infusion. c. Document the finding. d. Increase the IV drip rate.

b. Stop the IV infusion The nurse's first action should be to stop the IV infusion and assess the IV site carefully. The physician does not need to be notified. The IV drip rate should not be increased as it will increase tissue damage and phlebitis risk. The finding should be documented after the patient's IV line has been taken care of

Which nursing diagnosis indicates that the patient will have difficulty learning how to perform sterile dressing changes at home? a. Deficient knowledge related to diabetic wound management b. Stress overload related to ongoing emotional abuse and bullying c. Readiness for enhanced knowledge related to diabetes management d. Impaired physical mobility related to need to use a cane for ambulation

b. Stress overload related to ongoing emotional abuse and bullying The patient who is overly stressed will have difficulty learning procedures or concepts. The nurse should expect to spend extra time helping the patient to learn. Impaired physical mobility will not impair learning ability. The patient's deficient knowledge about wound management justifies the need for teaching. Readiness for enhanced knowledge indicates that the patient is ready to learn.

The nurse attempts to teach the patient about wound care in a loud semiprivate room with many distractions. Which is the appropriate action of the nurse? a. Explain to the patient that all of the information about wound care is in the handout provided. b. Take the patient to a quiet private treatment room to teach the patient about how to perform wound care. c. Ask the distraught roommate to please be considerate of the patient while the nurse is teaching about wound care. d. Arrange for the home-health nurse to provide teaching about wound care after discharge from the hospital.

b. Take the patient to a quiet private treatment room to teach the patient about how to perform wound care. A quiet area is needed for learning. Before learning anything, patients must be able to pay attention to or concentrate on the information they will learn. Physical discomfort, anxiety, and environmental distractions make it more difficult for a patient to concentrate. It is not appropriate to refer the patient to a handout. Asking the roommate to be considerate is inappropriate because the roommate is distraught. Deferring patient teaching to the home-health nurse is not appropriate.

Which action by the nurse demonstrates appropriate timing for effective communication? a. The nurse sits in a chair next to the patient's bed to maintain eye contact. b. The nurse waits to begin teaching until the patient's nausea has subsided. c. The nurse speaks slowly and loudly for a patient who is hard of hearing. d. The nurse maintains privacy during all conversations with the patient.

b. The nurse waits to begin teaching until the patient's nausea has subsided. The nurse demonstrates appropriate timing by waiting to begin teaching until the patient's nausea has subsided. Timing must be appropriate in order for the receiver to understand the message. The nurse demonstrates pacing by speaking slowly for a patient who is hard of hearing. Timing does not refer to ensuring privacy or maintaining eye contact.

The rehabilitation nurse is working with a patient to regain bowel continence after a stroke. Which intervention will the nurse include as part of the patient's bowel training program? a. The nurse will administer docusate sodium 100 mg PO BID. b. The nurse will assist the patient to the toilet every morning after breakfast. c. The nurse will check for the presence of a fecal impaction every other day. d. The nurse will apply skin barrier cream to the perineal area after each loose stool.

b. The nurse will assist the patient to the toilet every morning after breakfast. The patient's bowel training program will help the patient to set up a daily routine for defecation. Assisting the patient to the toilet every morning after breakfast is a good way to help set up this routine. Administering stool softeners, routinely checking for fecal impaction, and applying skin barrier creams will not help the patient to set up a daily routine for defecation.

Which chart entry documents patient achievement of cognitive learning? a. The patient verbalized decreased desire to commit self-harm. b. The patient described three symptoms of diabetic ketoacidosis. c. The patient demonstrated how to perform active range of motion. d. The patient expressed satisfaction with ability to share feelings with others

b. The patient described three symptoms of diabetic ketoacidosis. Cognitive learning includes what the patient knows and understands. All intellectual behaviors are in the cognitive domain. Describing symptoms of DKA is an example of cognitive learning. Psychomotor learning occurs when patients acquire skills that require the integration of knowledge and physical skills. Examples of psychomotor learning include how to perform active range of motion. Affective learning includes the patient's feelings, attitudes, opinions, and values such as decreased desires and satisfaction.

Which assessment finding explains the cause of the patient's stress urinary incontinence? a. The patient uses a wheelchair and cannot get to the toilet in time to void. b. The patient gave birth to six babies who weighed more than 9 pounds. c. The patient suffered a spinal cord injury and has no sensation below the waist. d. The patient self-catheterizes due to urinary retention from multiple sclerosis.

b. The patient gave birth to six babies who weighed more than 9 pounds Giving birth to six large babies causes weakness of pelvic organs and subsequent stress incontinence. Inability to reach the toilet in time to void indicates a functional incontinence. Self-catheterization and spinal cord injury do not lead to stress incontinence.

The nurse is caring for a patient who continues to have severe pain at the site of a fracture long after it healed. The patient's physicians can find no rationale for the pain. What is the most likely cause of the patient's discomfort? a. The patient is trying to obtain unneeded pain medications. b. The patient has developed a complex regional pain syndrome. c. The patient is in denial that the fracture has healed completely. d. The patient is experiencing referred pain from a fracture elsewhere.

b. The patient has developed a complex regional pain syndrome An example of idiopathic pain is complex regional pain syndrome. Idiopathic pain is chronic pain in the absence of an identifiable physical or psychological cause or pain perceived as excessive for the extent of an organic pathological condition. There is no evidence that the patient is trying to obtain unneeded pain medications, is in denial, or is experiencing referred pain from another fracture.

Which assessment finding leads the nurse to add ineffective protection to the patient's care plan? a. The patient follows a gluten-free, low-sodium, antiinflammatory diet. b. The patient has not received immunizations against influenza or pneumonia. c. The patient recently divorced after being in an unhappy marriage for 4 years. d. The patient takes levothyroxine daily to treat hypothyroid disease.

b. The patient has not received immunizations against influenza or pneumonia The patient is susceptible to influenza and pneumonia due to lack of immunization against these diseases. Ineffective protection is an appropriate nursing diagnosis for the patient as there is a decreased ability to protect itself from infection. Levothyroxine, divorce, and dietary preferences do not lower the patient's defenses.

Which assessment finding leads the nurse to question an order for an abdominal flat plate test? a. The patient is very claustrophobic. b. The patient is 8 weeks pregnant. c. The patient has a history of renal failure. d. The patient is allergic to iodine and shrimp

b. The patient is 8 weeks pregnant. The abdominal flat plate test is an x-ray diagnostic test so the nurse should question the order for a pregnant patient. Claustrophobia, renal failure, and shrimp allergy are not contraindications for the test.

The patient's urinalysis indicates increased specific gravity of the urine. Which finding does the nurse anticipate will be found upon assessment? a. The patient uses supplemental oxygen due to COPD. b. The patient is thirsty with dry oral mucus membranes. c. The patient has a history of benign prostatic enlargement. d. The patient just completed antibiotics for a bladder infection

b. The patient is thirsty with dry oral mucus membranes. Increased specific gravity occurs when the urine is concentrated. This can occur with dehydration, SIADH, or fluid restrictions. The patient's thirst and dry oral mucus membranes are consistent with increased specific gravity. History of BPH, resolved UTI, and COPD do not correlate with increased specific gravity of the urine.

When the nurse takes the patient's hand, the patient quickly pulls it back. How will the nurse interpret this patient's behavior? a. The patient is unable to express feelings. b. The patient is uncomfortable with being touched. c. The patient has impaired social skills with others. d. The patient has difficulty with nonverbal communication.

b. The patient is uncomfortable with being touched Nurses need to remain sensitive to their actions as well as the patient's feelings. If a patient refuses to hold a nurse's hand while in pain or pulls away from physical contact, this signals that the patient is uncomfortable with being touched by the nurse. It does not imply impaired social skills, inability to express feelings, or difficulty with nonverbal communication.

Which assessment finding indicates that the patient is at risk for developing polypharmacy? a. The patient stores medications in the kitchen cabinet. b. The patient takes four different medications for congestive heart failure. c. The patient requires financial assistance to pay for prescriptions. d. The patient obtains all prescription medications from a mail-order pharmacy

b. The patient takes four different medications for congestive heart failure. Polypharmacy is the use of multiple prescription medications, putting the patient at risk for developing harmful drug interactions. The four different medications for congestive heart failure put the patient at risk of developing polypharmacy. Financial assistance, storage location, and use of mail-order pharmacy do not increase the risk of polypharmacy

Which assessment finding indicates that the patient is experiencing hypercapnia during sleep? a. The patient sleeps in the lateral position with at least two pillows. b. The patient wakes up feeling hung over after consuming no alcohol. c. The patient has difficulty falling asleep and wakes up early each morning. d. The patient works the night shift and is unable to sleep well during the day

b. The patient wakes up feeling hung over after consuming no alcohol. Waking up feeling hung over after no alcohol consumption can indicate hypercapnia during sleep. The patient feels better as the day progresses as the elevated carbon dioxide level slowly returns to normal. Sleeping in the lateral position, working the night shift, and insomnia do not demonstrate hypercapnia

Which outcome is most appropriate for a patient with the diagnosis impaired wheelchair mobility related to neuromuscular impairment and fatigue? a. The patient will feel comfortable navigating the motorized wheelchair. b. The patient will demonstrate ability to safely operate the motorized wheelchair. c. The patient will understand the need to use a motorized wheelchair for mobility. d. The patient will demonstrate correct use of the trapeze bar for repositioning self.

b. The patient will demonstrate ability to safely operate the motorized wheelchair The most appropriate goal for impaired wheelchair mobility is for the patient to demonstrate how to safely operate the motorized wheelchair. The trapeze bar is used for repositioning in the bed. Feeling comfortable and understanding concepts are not examples of measurable outcomes.

Which is the most appropriate goal for an elderly adult with the nursing diagnosis of sedentary lifestyle related to deconditioning and lack of physical exercise? a. The patient will participate in social activities with others. b. The patient will increase walking to at least 5000 steps per day. c. The patient will identify ways to conserve energy and prevent fatigue. d. The patient will develop meaningful relationships with the nursing staff.

b. The patient will increase walking to at least 5000 steps per day. The most appropriate goal for the sedentary lifestyle diagnosis is to increase activity. The patient can achieve this by walking at least 5000 steps each day. Participation in social activities does not necessarily require physical movement or walking. Conserving energy and developing relationships are not goals for the sedentary lifestyle diagnosis

Which is the highest priority goal for the young adult patient with the nursing diagnosis risk for suicide related to depression and inability to make friends? a. The patient will express feelings and maintain self-control. b. The patient will not cause any sort of harm or injury to self. c. The patient will develop a therapeutic relationship with the nurse. d. The patient will verbalize less anxiety and fear around other people.

b. The patient will not cause any sort of harm or injury to self. The highest priority goal is that the patient will not cause any harm or injury to self. Expressing feelings, developing relationships, and verbalizing less anxiety are all less important than maintaining safety of the patient.

Which outcome is most appropriate for the patient with the nursing diagnosis insomnia related to night shift work? a. The patient will rotate day/night work shifts frequently. b. The patient will obtain at least 48 hours of sleep per week. c. The patient will use caffeine sparingly to wake up before shifts. d. The patient will use bright lights to stay awake through the night

b. The patient will obtain at least 48 hours of sleep per week The patient should obtain at least 48 hours of sleep per week, during the day and night as shifts permit. Frequent rotation of night/day shifts will worsen insomnia. Use of bright lights and caffeine are interventions rather than goals.

Which is the most appropriate goal for a patient with the nursing diagnosis risk for loneliness related to loss of spouse and admission to long-term nursing facility? a. The patient will use effective coping strategies to prevent self-harm. b. The patient will participate in at least one group activity every week. c. The patient will assist staff with activities of daily living every morning. d. The patient will express the desire to achieve increased levels of comfort.

b. The patient will participate in at least one group activity every week. The appropriate goal for a patient at risk of loneliness is for the patient to participate in at least one group activity every week. This will allow the patient to meet other residents and hopefully develop some friendships. The prevention of self-harm, assisting with activities of daily living, and desiring increased comfort are not appropriate goals for risk of loneliness.

Which is an appropriate goal for a patient's preoperative teaching? a. The nurse will provide written materials about nonpharmacological pain-management techniques. b. The patient will verbalize understanding of the pain-management techniques to be used after surgery. c. The nurse will demonstrate correct use of the patient-controlled anesthesia (PCA) pump. d. The patient will rate current pain of less than 3 out of 10 on the descriptive pain intensity scale

b. The patient will verbalize understanding of the pain-management techniques to be used after surgery. An appropriate goal of preoperative teaching is that the patient will verbalize understanding of the pain-management techniques to be used after surgery. The patient has not undergone surgery yet so current pain assessment is not a goal of preoperative teaching. Demonstration of the PCA pump and providing written materials are interventions rather than goals.

Which nursing intervention can cause an unsafe sleeping environment for the hospitalized patient? a. A small night-light left on in the bedroom. b. The patient's bed is in high position with side rails up. c. All clutter is removed between the bed and the bathroom. d. Call bell at the bedside for the patient to contact the nurse.

b. The patient's bed is in high position with side rails up. A bed in high position with side rails up is a safety hazard. Safety precautions are important for patients who awaken during the night to use the bathroom and for those with excessive daytime sleepiness. Set beds lower to the floor to lessen the chance of the patient falling when first standing. Remove clutter, and move equipment from the path a patient uses to walk from the bed to the bathroom. If patient needs assistance in ambulating from the bed to the bathroom, make sure the call light is within the patient's reach. The call light helps alert the nursing staff, not the family. A small night-light is beneficial to help with vision.

. Which assessment finding indicates that the sinoatrial node was damaged as a result of the patient's heart attack? a. The patient's jugular vein is distended. b. The patient's heart rate is 34 beats/minute. c. Faint wheezes are heard in the patient's lungs. d. The patient has developed a new heart murmur.

b. The patient's heart rate is 34 beats/minute The conduction system originates with the sinoatrial node, the "pacemaker" of the heart. Damage to the sinoatrial node results in bradycardia as the ventricles set the rhythm. Damage to the sinoatrial node would not lead to jugular venous distention, wheezes, or new heart murmur.

Which is the appropriate outcome for the patient with the nursing diagnosis risk for ineffective peripheral tissue perfusion related to prolonged immobility? a. The patient's urinary output will remain at least 30 mL/hour. b. The patient's legs will maintain strong peripheral pulses and no edema. c. The patient's abdomen will stay soft with bowel sounds present 4 quadrants. d. The patient will remain alert and appropriate with no changes in mental status

b. The patient's legs will maintain strong peripheral pulses and no edema The appropriate outcome for the patient with the nursing diagnosis risk for ineffective peripheral tissue perfusion is that the patient's legs will maintain strong peripheral pulses and no edema. Urinary output indicates renal perfusion. Bowel sounds indicate gastrointestinal perfusion. Mental status indicates cerebral perfusion

Which assessment finding by the nurse indicates that the patient's colonoscopy preparation is complete? a. The patient has stopped vomiting. b. The patient's stool is watery clear yellow. c. The patient had a large soft formed stool. d. The patient's abdomen is softly distended.

b. The patient's stool is watery clear yellow. The colonoscopy preparation is complete when the patient's colon is free of stool. This is indicated by passage of watery clear yellow liquid from the colon. Passage of soft formed stool indicates that additional cleansing of the colon is required. The abdomen should not be distended and the colonoscopy preparation should not cause vomiting.

At change-of-shift report, the nurse learns the medical diagnoses for four patients. Which patient should the nurse review most carefully for development of hyponatremia? a. Vomiting all day and not replacing any fluid b. Tumor that secretes excessive antidiuretic hormone (ADH) c. Tumor that secretes excessive aldosterone d. Tumor that destroyed the posterior pituitary gland

b. Tumor that secretes excessive antidiuretic hormone (ADH) ADH causes renal reabsorption of water, which dilutes the body fluids. Excessive ADH thus causes hyponatremia. Excessive aldosterone causes ECV excess rather than hyponatremia. The posterior pituitary gland releases ADH; lack of ADH causes hypernatremia. Vomiting without fluid replacement causes ECV deficit and hypernatremia.

The nurse is caring for a patient who has a severe right-sided stroke with left-sided hemiplegia. The patient uses the right extremities well but does not realize that the left arm and leg even exist. Which is the most appropriate nursing diagnosis for this patient? a. Deficient knowledge related to presence of paralyzed left arm and leg b. Unilateral neglect related to brain tissue damage after right-sided stroke c. Ineffective denial related to inability to accept paralysis of left arm and leg d. Noncompliance related to inability to follow directions to use left arm and leg

b. Unilateral neglect related to brain tissue damage after right-sided stroke Unilateral neglect occurs when the patient is unaware that a body area exists after a neurological injury or stroke. This patient demonstrates unilateral neglect by not realizing that the left arm and leg exist. Knowledge deficit, noncompliance, and ineffective denial do not explain the patient's lack of realization of the left-sided extremities.

Which nursing intervention is the highest priority when caring for an impulsive, forgetful stroke patient with right-sided paralysis? a. Complete a fall risk assessment such as the Hendrich II Fall Risk Model. b. Utilize a bed alarm and respond immediately when it is triggered. c. Place the call light within easy reach and remind the patient to use it. d. Apply a soft restraint to the patient's left wrist to prevent getting out of bed

b. Utilize a bed alarm and respond immediately when it is triggered The nurse should utilize a bed alarm for the patient and respond immediately when it is triggered indicating that the patient is starting to get up. This way the nurse will be able to intervene before the patient is able to get out of the bed. The patient is already known to be a fall risk so completing an additional fall risk assessment is unnecessary. The patient is forgetful and likely will not remember to use the call light. Restraints should be used only as a last result as they can cause significant injury and distress to the patient.

Which is the priority assessment to be documented prior to administering a blood product to a patient? a. Strength of peripheral pulses b. Vital signs and pulse oximetry c. Presence/absence of bowel sounds d. Height, weight, and body mass index (BMI)

b. Vital signs and pulse oximetry Vital signs and pulse oximetry must be documented before administering a blood product to the patient. This will allow the nurse to compare subsequent findings after the transfusion has started. Peripheral pulses, bowel sounds, and height/weight are not as important as vital signs and pulse oximetry prior to starting a transfusion.

Which technique can the nurse use to facilitate assessment of the thyroid gland? a. Have the patient tip the head back and cough twice. b. Watch the patient's neck when sipping water from a straw. c. Have the patient slowly turn the head from side to side. d. Gently palpate from underneath the jaw to behind the ear.

b. Watch the patient's neck when sipping water from a straw. Observing the patient's neck during swallowing can help visualize the shape of the thyroid gland. Having the patient tip the head back or turn the head from side to side will not help visualize the shape of the thyroid gland. The thyroid gland lies just in front of the trachea so palpation behind the ear will not be helpful.

When will the nurse clamp the patient's chest tube? a. When the patient ambulates in the hallway b. When changing the drainage collection unit c. Before assisting the patient to take a shower d. When disconnecting the chest tube from suction

b. When changing the drainage collection unit The drainage collection unit should be changed when it is full or damaged. The chest tube must be clamped when changing the unit in order to prevent air from entering the chest cavity. The tube should not be clamped during ambulation or when disconnecting the chest tube from suction. The patient with a chest tube cannot shower as the site must be kept clean and dry

A mother complains to the nurse at the pediatric clinic that her 4-year-old child always talks to her toys and makes up stories. The mother wants her child to have a psychological evaluation. The nurse's best initial response is to a. refer the child to a psychologist immediately. b. explain that playing make believe is normal at this age. c. complete a developmental screening using a validated tool. d. separate the child from the mother to get more information.

b. explain that playing make believe is normal at this age. By the end of the fourth year, it is expected that a child will engage in fantasy, so this is normal at this age. A referral to a psychologist would be premature based only on the complaint of the mother. Completing a developmental screening would be very appropriate but not the initial response. The nurse would certainly want to get more information, but separating the child from the mother is not necessary at this time.

When teaching an Asian patient with newly diagnosed diabetes, the nurse notes the patient nodding yes to everything that is being said. With a better understanding of cultural interdependence in self-concept, a nurse should immediately a. write everything down for the patient to refer to later. b. prompt further to elicit additional questions or concerns. c. call the recognized elder for this patient. d. call the oldest male relative for help with decision making.

b. prompt further to elicit additional questions or concerns. When a nurse provides nutritional education to a patient who is from a culture that values greater power distance, it might appear that the patient is willing to accept all that the nurse suggests, when further prompting would elicit additional questions or concerns. The patient from a collectivist culture will usually consult family members for a best course of action. It is not acceptable for nurses to take it upon themselves to call the recognized elder or oldest male relative for help with decision making. While writing everything down may be OK for some cultures, with Asian patients it may be best to prompt further to elicit additional questions or concerns

The transition from childhood to adulthood, in terms of the psychological maturation of the individual, is known as which of the following? a.Puberty b.Adolescence c.Menarche d.Preadolescence

b.Adolescence The term adolescence refers to the psychological maturation of the individual, whereas puberty refers to the point at which reproduction is possible. Menarche refers to the onset of menstruation. The transitional period between childhood and adolescence is preadolescence. Others refer to this period as late childhood, early adolescence, pubescence, and transescence.

A healthy 33-year-old mother of three children reports having no energy. She asks the nurse, how she could increase her energy level. What is the best response for the nurse to give? a.You should decrease iron intake. b.Daily exercise has been shown to increase a persons energy level. c.Energy caffeinated drinks are a good substitute for exercise. d.Try to exercise 90 minutes every other day.

b.Daily exercise has been shown to increase a persons energy level. A physical exercise program has many benefits. People who exercise daily for 30 to 60 minutes, not 90, have a lower heart rate, lower blood pressure, decreased cholesterol, increased blood flow, and greater oxygen extraction by working muscles. Fully conditioned people are able to increase oxygen consumption by 10% to 20% because of increased cardiac output and efficiency of the myocardium. If the diet does not supply iron needed for hemoglobin synthesis, RBC synthesis is reduced, and oxygen-carrying capacity decreases. A risk factor for cardiopulmonary disease is excessive use of caffeinated energy drinks.

The nurse is educating a patient who has recently been diagnosed with chronic obstructive pulmonary disease (COPD). The nurse explains how the gasses in the lungs move between the air spaces and the bloodstream. Which process is the nurse describing? a.Ventilation b.Diffusion c.Respiration d.Perfusion

b.Diffusion Diffusion is the movement of gases between air spaces and the bloodstream. One of the primary functions of the lungs includes ventilation, the movement of air in and out of the lungs. Respiration is the exchange of oxygen and carbon dioxide during cellular metabolism. The heart supports perfusion, the movement of blood into and out of the lungs to the organs and tissues of the body.

A student nurse caring for a patient with a chest tube has been asked what equipment should be at the bedside to assess for an air leak. Which information indicates the student nurse has a correct understanding of the equipment needed? a.Suction equipment wrapped in plastic to keep instrument clean b.Hemostat covered with gauze to prevent penetration of the chest tube c.Cup of water to place the end of the chest tube d.Petroleum gauze to use as a dressing

b.Hemostat covered with gauze to prevent penetration of the chest tube Covered hemostats are used to assess for air leaks, not a cup of water, suction equipment, or petroleum gauze. Hemostats have a covering to prevent hemostat from penetrating the chest tube. The use of these hemostats or other clamp prevents air from re-entering the pleural space. Suctioning does not determine an air leak; it is used to clear secretions. If there is a break in the chest drainage device, place the end of the chest tube in a bottle of sterile saline, not water. If a chest tube becomes dislodged apply pressure to chest tube site wound using petroleum gauze, a dry gauze dressing, and adhesive tape.

A 36-year-old patient diagnosed with meningitis has a fever of 102.3 F. A family member verbalizes a concern that the patient is breathing fast. Upon assessment, the nurse notes a respiratory rate of 20 breaths/min, pulse oximetry is 92% on 2L/O 2 , and lungs clear to auscultation. What is the best explanation for the rapid respiratory rate that the nurse can give the family member? a.He is most likely anxious because he is in the hospital. b.His fever has increased his metabolic rate and is causing him to breathe faster. c.He is hyperventilating because he needs more oxygen. d.He has an acid-base imbalance, which is causing him to hyperventilate.

b.His fever has increased his metabolic rate and is causing him to breathe faster. An increase of 1 F in body temperature causes a 7% increase in the metabolic rate, thereby increasing carbon dioxide production. The clinical response is increased rate and depth of respiration. Anxiety is not the reason for increased respirations in this scenario. The fever (102.3 F) is the cause of the increased respirations. The patient may have increased breathing but the patient is not hyperventilating. Hyperventilation occurs when the respiratory rate is greater than 20 breaths per minute in an adult, causing an increase in carbon dioxide elimination.

A student nurse is caring for a young adult patient who is immobile with a back injury. On auscultation, the student nurse hears rhonchi in the lower lobes. The student nurse reports this symptom because the patient is developing which complication? a.Increased lung expansion b.Hypostatic pneumonia c.Aspiration pneumonia d.Increased diuresis

b.Hypostatic pneumonia Decreased, not increased, lung expansion, generalized respiratory muscle weakness, and dependent stasis o secretions occur with immobility. These conditions often contribute to the development of atelectasis (collapse of alveoli) and hypostatic pneumonia (inflammation of the lung from stasis or pooling of secretions). Aspiration pneumonia results from aspiration, not from immobility. Diuresis is increased urine excretion.

A home care nurse visits a new patient. The family asks how the home can be made safer. The nurses best advice includes which of the following? a.Using throw rugs to prevent tripping b.Installing extra incandescent lighting c.Painting the floor black and white to add perception d.Installing handrails painted the same color as the walls

b.Installing extra incandescent lighting Good lighting at front and back entrances and light switches at the top and bottom of stairwells and long hallways add an additional safety element. Throw rugs, footstools, and electrical cords present tripping hazards. Handrails painted the same color as the walls may pose a problem for the visually impaired. Using color contrasts such as tape, paint, or nail enamel can highlight items.

A nurse is being oriented to work for an intensive care unit. The hemodynamic data indicate that the patient has a decreased preload. Which information indicates the nurse has a correct understanding of the concept of preload? a.It is the amount of blood ejected from the left ventricle each minute. b.It is the amount of blood in the heart at the end of ventricular diastole. c.It is the resistance to the ejection of blood from the left ventricle. d.It is the rhythmic relaxation and contraction of the heart chambers.

b.It is the amount of blood in the heart at the end of ventricular diastole. Preload is the amount of blood at the end of ventricular diastole. Cardiac output is the amount of blood ejected from the left ventricle each minute. Cardiac conduction is the rhythmic relaxation and contraction of the atria and ventricles dependent on transmission of electrical impulses. Afterload is the resistance of the ejection of blood from the left ventricle.

Which of the following people developed the theory of cognitive development that describes childrens intellectual organization and how they think, reason, and perceive the world? a.Sigmund Freud b.Jean Piaget c.Erik Erikson d.Lawrence Kohlberg

b.Jean Piaget Jean Piaget (1896-1980) developed the theory of cognitive development, which describes childrens intellectual organization and how they think, reason, and perceive the world. Sigmund Freud (1856-1939) provided the first formal structured theory of personality development. Freuds psychoanalytic model of personality development is grounded in the belief that two internal biological forces drive the psychological change in a child: sexual (libido) and aggressive energies. Erik Erikson divided life into eight stages, known as Eriksons eight stages of development. According to this theory, individuals need to accomplish a particular task before successfully completing each stage. Each task is framed with opposing conflicts, such as trust versus mistrust. Each stage builds upon the successful attainment of the previous developmental conflict. Lawrence Kohlberg (1927-1987) expanded on Piagets work. According to Kohlberg (1964), moral development is one component of psychosocial development. It involves the reasons an individual makes a decision about right and wrong behaviors within a culture.

A patient presents with an acute myocardial infarction that resulted in right ventricular damage. The nurse needs to assess the patient for right-sided heart failure, which includes which of the following? a.Crackles on auscultation b.Jugular neck vein distention c.Increased myocardial perfusion d.Orthopnea

b.Jugular neck vein distention Right-sided heart failure causes distended jugular veins and peripheral edema. Right-sided heart failure results from impaired functioning of the right ventricle, which is typically caused by pulmonary disease or pulmonary hypertension. An increase in pressure in the pulmonary system causes increased resistance in the right ventricle. The right ventricle fails as a result of this pressure. Left-sided heart failure results in crackles on auscultation and patient complaints of fatigue, dyspnea, and orthopnea. Increased myocardial perfusion does not occur.

An 11-year-old boy is being seen at the clinic for his annual check-up. As part of anticipatory guidance, the nurse instructs the boys father that accidents and injuries are major health problems affecting school-age children and that the number one cause of death is in this age group is which of the following? a.Drowning b.Motor vehicle accidents c.Fire d.Firearms

b.Motor vehicle accidents Accidents and injuries are major health problems affecting school-age children and are the causative factor in a large number of deaths in this age group. Motor vehicle accidents, followed by drowning, fires, burns, and firearms are the most frequent fatal accidents.

A patient in the intensive care unit requires mechanical ventilation, a wound VAC system, patient-controlled analgesia, and an intravenous infusion device. Which safety precaution should the nurse implement in the health care setting? a.Using two-pronged plugs b.Never operating equipment without previous instruction c.Using an extension cord to accommodate plugs for all the equipment d.Never using equipment without having another nurse assist

b.Never operating equipment without previous instruction To avoid accidents, do not operate medical equipment without adequate instruction. Decrease the incidence of electrical hazards by using a three-pronged grounded plug. Many types of equipment have both electric outlet and battery power sources. Extension cords are a common cause for falls. If an extension cord must be utilized, it should be placed next to the wall to decrease tripping. Using equipment without having another nurse assist is safe behavior.

The spouse of a homebound elderly patient voices a concern to the visiting nurse, Im having a hard time getting the patient to eat a balanced diet. All the patient wants to eat are sweets. What is the best explanation the nurse can give to the spouse? a.Maybe she has a sweet tooth. b.Older adults seem to be able to taste sweet foods best. c.I wouldnt worry about it as long as she is eating something. d.She is probably getting all the nutrients that she needs.

b.Older adults seem to be able to taste sweet foods best. A small decrease in the number of taste cells occurs with aging, beginning around age 60. Reduced sour, salty, and bitter taste discrimination is common. The ability to detect sweet tastes seems to remain intact. Promote sense of taste through good oral hygiene, serving well-seasoned and differently textured foods, chewing food thoroughly, and avoiding blending or mixing foods. Enhance the sense of smell by removing unpleasant odors from the environment and introducing pleasant smells such as mild room deodorizers or fragrant flowers.

A student nurse has been asked by the registered nurse with whom the student nurse is working to apply wrist restraints to a patient who is confused and is trying to remove the endotracheal tube. The student nurse knows that it is important to tie the restraints to which part of the bed? a.Side rails b.Part of bed frame that moves up and down with the patient c.Footboard d.Headboard

b.Part of bed frame that moves up and down with the patient Attach restraint straps to the portion of the bed frame that moves when raising or lowering the head of the bed. Do not attach to the side rails. Attaching the restraint straps to a portion of the bed frame that does not move (headboard or footboard) will injure the patient.

The nurse is evaluating a patient who has a chest tube. To properly maintain chest tube function, what is the nurses best action? a.Strip the tube every hour to maintain drainage. b.Place the device below the patients chest. c.Double clamp the tubes except during assessments. d.Remove the tubing from the drainage device to check for proper suctioning.

b.Place the device below the patients chest. Observe the chest drainage system to be sure it is upright and below the level of tube insertion. Most institution have stopped stripping the chest tube because this greatly increases intrapleural pressure unless the patient is fresh from postoperative thoracic surgery or has chest trauma. Chest tubes are only clamped under specific circumstances per health care providers order or nursing policy to assess for an air leak, to quickly empty or change disposable drainage systems, or to assess if the chest tube is ready to be removed. Clamping the chest tube is not recommended because it may result in a tension pneumothorax, a life-threatening event. Removing the tubing would cause a disruption in the suctioning of the chest tube and should not be done.

An 85-year-old retired man with arthritis has recently been prescribed a new medication by his health care provider for pain management. The health care provider identifies that the patient is currently taking 13 different medications on a daily basis. The health care provider is concerned about the patients safety in the home. Which of the following is most important to assess? a.Marital status b.Potential for falls c.Skin breakdown d.Cultural beliefs

b.Potential for falls The physiological changes associated with aging (85 years old), effects of multiple medications (13 different medications), psychological factors, and acute or chronic disease (arthritis) increase the older adults risk for falls and other types of accidents. Fear of falling is common among community-dwelling older adults, who both do and do not have a history of falling. As a result of their fear, many older adults avoid activities or change the way in which they walk and position themselves, making them more at risk for falling. It is important to learn what conditions increase a persons fear of falling so that steps can be taken to remove or change any hazards in the home. Marital status and cultural beliefs are not priority assessment issues for home safety. Skin breakdown could occur if the patient was immobile but this is not the priority home safety issue for this patient and there is no data in the question to indicate the patient is immobile.

The nurse is concerned because a 77-year-old patient is weak after abdominal surgery. Which of the following should be done to ensure that one of the preventable conditions identified by the Centers for Medicare and Medicaid does not occur? a.Use the rights of medication administration. b.Provide frequent opportunities to use the bathroom. c.Document thoroughly. d.Complete discharge teaching as quickly as possible.

b.Provide frequent opportunities to use the bathroom. Providing frequent opportunities to use the bathroom helps prevent pressure ulcers, falls, trauma, and even may help prevent an infection from the insertion of a catheterall are listed on the preventable conditions. The Centers for Medicare and Medicaid Services (CMS) names select serious reportable events (SREs) as Never Events (adverse events that should never occur in a health care setting) (US Department of Health and Human Services, 2008). The CMS now denies payment to hospitals for any hospital-acquired conditions resulting from or complicated by the occurrence of certain Never Events that were not present on admission. Many of the hospital-acquired conditions are nurse-sensitive indicators, meaning that nursing interventions directly affect their development. Whereas the rights of medication administration demonstrates safety it does not relate to the preventable conditions identified by CMS, but does relate to the National Patient Safety Goals. Thorough documentation is a legal issue, not a CMS preventable condition. Discharge teaching is not listed on the preventable list and it should not be done quickly.

The nurse is caring for an older adult who seems depressed and states that I hate going to Bingo. Id really like to get back in shape. Maybe if I could join a health club, I could get back to feeling like myself. The nurse should do which of the following? a.Inform the patient that at his age its probably not a good idea to join a health club. b.Recommend that the patient have a complete physical examination. c.Explain that physical impairments would prevent any worthwhile exercising. d.Tell the patient that at his age he would probably hurt himself.

b.Recommend that the patient have a complete physical examination. It is not too late for an older person to begin an exercise program; however, older adults need to have a complete physical examination, which usually includes a stress cardiogram or stress test. Assessment of activity tolerance will help you and the patient plan a program that meets physical needs while allowing for physical impairments. Most older adults are capable of taking charge of their lives and assume responsibility for preventing disability.

A pregnant teenager asks the clinic nurse why she cannot smoke during the first trimester. Remembering growth and development, what is the nurses best response? a.Smoking is a bad habit, but it probably wont affect the baby. b.Smoking may affect organ systems that are beginning to develop. c.Smoking will only affect the baby in the third trimester. d.Smoking mothers usually produce overweight babies

b.Smoking may affect organ systems that are beginning to develop. Teratogens are chemical or physiological agents capable of having adverse effects on the fetus. Exposure to potential teratogens can affect fetal development during any of the trimesters; however, vulnerability is increased during the first trimester when fetal cells are differentiating and organs are forming. In addition, there is evidence that mothers who smoke deliver infants with lower birth weights than nonsmoking mothers.

The patient is recovering from a cerebrovascular accident (stroke). The patient is having problems with balance and coordination. The patient asks the nurse what part of the brain has been damaged. How should the nurse respond? a.The hypothalamus has been damaged. b.The cerebellum has been damaged. c.The thalamus has been damaged. d.The medulla oblongata has been damaged.

b.The cerebellum has been damaged Damage to the cerebellum causes problems with balance, and motor impairment is directly related to the amount of destruction of the motor strip. The hypothalamus controls temperature. The thalamus controls the five senses: hearing, seeing, taste, smell, and touch. The medulla oblongata is part of the brainstem and controls breathing, heart rate, and digestion.

A college student who smokes asks a healthcare professional if there really is a connection between smoking and lung cancer. What is the healthcare workers best response? a.The risk for lung cancer for smokers than for nonsmokers is 5 times greater. b.The risk for lung cancer for smokers than for nonsmokers is 10 times greater. c.The risk for lung cancer for smokers than for nonsmokers is 50 times greater. d.Lung cancer affects smokers and non-smokers equally due to occupational hazards.

b.The risk for lung cancer for smokers than for nonsmokers is 10 times greater. The risk for lung cancer is 10 times greater for a person who smokes than for a nonsmoker. Exposure to secondhand smoke increases the risk for lung cancer in the nonsmoker and worsens other pulmonary problems such as asthma or COPD: 5 times is too small; 50 times is too great. Lung cancer does not affect smokers and nonsmokers equally

An older adult patient has been admitted to a busy medical unit. To control environmental stimuli a nurse should do which of the following? a.Leave the hospital room lights on at all times. b.Turn off bedside equipment not in use. c.Leave the window curtains closed at all times. d.Leave the door open so the patient can hear the staff and feel secure.

b.Turn off bedside equipment not in use. Try to control extraneous noise in and around a patients room, such as television volume and visitors. Turn off bedside equipment not in use. Close a patients room door if necessary. Hospital staff members need to control loud laughter or conversation at the nurses station. In addition to controlling excess stimuli, try to introduce meaningful stimulation that makes the environment pleasing and comfortable. Open drapes and close door if indicated. Control extraneous noise in and around room such as television volume and visitors. Turn off bedside equipment not in use.

A confused patient was found wandering in the hallways several times during the shift. What is the most appropriate nursing intervention to prevent a fall by this patient? a.Reassigning the patient to a room closer to the nursing station b.Using an electronic monitor that sounds an alarm when the patient reaches a near-vertical position c.Raising two or four side rails d.Placing wrist restraint on the patient during the nighttime hours of sleep

b.Using an electronic monitor that sounds an alarm when the patient reaches a near-vertical position Alarm devices warn nursing staff that a patient is attempting to leave a bed or chair unassisted. There are a variety of types, including a device with a knee band that sounds an alarm when the patient reaches a near-vertical position. An infrared type of alarm is affixed to a headboard or bed frame, allowing a patient to move freely within a bed. If a patient tries to leave the bed, the infrared beam detects motion and sends out an alarm tone. Moving the patient to a room closer to the nursing station does not solve the problem. Raising side rails has the potential to trap parts of the patients body, producing a hazard. The use of side rails alone for a disoriented patient often causes more confusion and further injury. Restraints are not a solution to a patient problem, but a temporary means to patient safety. Restraints are a last resort to prevent injury.

A specific phase or period when the presence of a function or reasoning has its greatest effect on a specific aspect of development is referred to as: a.Freuds psychoanalytic model of personality. b.a critical period of development. c.Eriksons stages of development. d.Piagets theory of cognitive development.

b.a critical period of development. A critical period of development refers to a specific phase or period when the presence of a function or reasoning has its greatest effect on a specific aspect of development. For example, if a child does not walk by 20 months, there is delayed gross motor ability, which slows exploration and manipulation of the environment. Freuds psychoanalytic model of personality development is grounded in the belief that two internal biological forces drive the psychological change in a child: sexual (libido) and instinctive forces. Erik Erikson (1902-1994) expanded Freuds psychoanalytic stages into a psychosocial model that covered the whole life span. In this theory, Erikson divided life into eight stages, known as Eriksons eight stages of development. Jean Piaget (1896-1980) developed the theory of cognitive development, which describes childrens intellectual organization and how they think, reason, and perceive the world.

Polypharmacy is: a.the use of unprescribed medication for recreational use. b.the prescription, use, or administration of more medications than are needed. c.very rare in older adults. d.rarely a problem as long as the medications are taken together.

b.the prescription, use, or administration of more medications than are needed. Polypharmacy, the prescription, use, or administration of more medications than are indicated clinically, is acommon problem of older adults. The combined use of multiple drugs causes serious problematic effects.

A patient diagnosed with hypertension asks the nurse how this disease could have happened to them. What is the nurse's best response? a. "Hypertension happens to everyone sooner or later. Don't be concerned about it." b. "Hypertension can happen from eating a poor diet, so change what you are eating." c. "Hypertension can happen from arterial changes that block the blood flow." d. "Hypertension happens when people do not exercise, so you should walk every day."

c. "Hypertension can happen from arterial changes that block the blood flow." Hardening of the arteries from atherosclerosis can cause hypertension in the patient. Hypertension does not happen to everyone. Changing the patient's diet and exercising may be a positive life change, but these answers do not explain to the patient how the disease could have happened.

A mother tells the nurse she is concerned because her 8-month-old infant sleeps all day and night and is only awake about 2-3 hours per day. What is the nurse's best response? a. "This sleep pattern is very normal for an infant at this age." b. "Adding an additional feeding will keep the child awake more" c. "I recommend that you notify the child's pediatrician." d. "Be sure you are laying the child on his back to sleep at night."

c. "I recommend that you notify the child's pediatrician." By approximately 6 months of age, the infant should sleep through the night with at least one nap during the day. 2-3 hours of wakefulness per day is not an expected finding at age 8 months and should be reported to the pediatrician to determine the underlying cause. An additional feeding may be warranted; however the pediatrician should be notified first. Laying the infant on the back to sleep is recommended to prevent sudden infant death syndrome; however the priority concern is the length of time the child is sleeping.

An older patient has developed age spots and is concerned about skin cancer. How would the nurse instruct the patient to perform skin checks to review for signs of skin cancer? a. "Limit the time you spend in the sun." b. "Monitor for signs of infection." c. "Monitor spots for color change." d. "Use skin creams to prevent drying

c. "Monitor spots for color change." The ABCDE method (check for asymmetry, border irregularity, color variation, diameter, and evolving) should be used to review lesions for signs associated with cancer. Color change could be a sign of cancer and needs to be looked at by a dermatologist. Limiting time spent in the sun is a preventative measure but will not assist the patient in checking the skin or detecting skin cancer. Infection is usually not found in skin cancer. Skin creams have not been shown to prevent cancer nor would they assist in detecting skin cancer.

A nurse is explaining the concept of perfusion to a student nurse. The nurse knows the student understands the concept of perfusion when the student makes which statement? a. "Perfusion is a normal function of the body, and I don't have to be concerned about it." b. "Perfusion is monitored by the physician." c. "Perfusion is monitored by vital signs and capillary refill." d. "Perfusion varies as a person ages, so I would expect changes in the body."

c. "Perfusion is monitored by vital signs and capillary refill." The best method to monitor perfusion is to monitor vital signs and capillary refill. This allows the nurse to know if perfusion is adequate to maintain vital organs. The nurse does have to be concerned about perfusion. Perfusion is not only monitored by the physician but the nurse too. Perfusion does not always change as the person ages.

The mother of a school-age child asks the nurse why the child's blood pressure is measured at a routine physical examination appointment. What is the best answer of the nurse? a. "Insurance companies require blood pressure checks of children as well as adults to calculate premium rates." b. "The state health department requires blood pressure checks before immunizations can be administered." c. "Sometimes children develop high blood pressure that can lead to health problems in adulthood." d. "Blood pressure checks are started in childhood so that adults will not be afraid of them later on."

c. "Sometimes children develop high blood pressure that can lead to health problems in adulthood." Blood pressure elevation in childhood is the single best predictor of adult hypertension. This recognition has reinforced the significance of making blood pressure measurement a part of every annual assessment of the child. Measure on at least three separate occasions with the appropriate-size cuff and in a relaxed situation before concluding that the child's blood pressure is elevated and needs further medical attention. Childhood obesity is a prominent health problem, which increases the child's risk for hypertension, diabetes, coronary artery disease, and other chronic health problems. Daily exercise and maintaining normal body weight are important as both interventions and prevention even while in the preteen years.

The mother of a healthy toddler reports that the child is eating less than a few months ago but the child has not lost any weight. Which is the most appropriate response of the nurse? a. "You need to make him eat more every day. At this stage, he is growing too fast to not eat." b. "I could show you a growth chart, but each child is different so it doesn't mean much." c. "Toddlers have periods when they aren't growing as fast and they don't need to eat as much." d. "Make him eat with a spoon and don't feed him snacks. He will be hungrier at meal time."

c. "Toddlers have periods when they aren't growing as fast and they don't need to eat as much." Slower growth rates often occur with a decrease in caloric needs and a smaller food intake. Confirming the child's pattern of growth with standard growth charts is reassuring to parents concerned about their toddler's decreased appetite (physiological anorexia). Encourage parents to offer a variety of nutritious foods, in reasonable servings, for mealtime and snacks. Finger foods allow the toddler to be independent.

Which instruction will the nurse give to the patient about proper use of patient-controlled analgesia (PCA)? a. "Wait until the pain becomes severe before pushing the PCA button." b. "The PCA will deliver medication through the IV until the pain is all gone." c. "You or a designated family member are the only one who gets to push the PCA button—nobody else may do so." d. "The PCA will give additional pain medication whenever the button is pushed."

c. "You or a designated family member are the only one who gets to push the PCA button—nobody else may do so." The patient should be instructed not to let anyone else push the PCA pain button to avoid overdosage. One family member or significant other can be the patient's primary pain manager and is allowed to push the PCA button. The patient should not wait until the pain becomes severe to push the PCA button. The PCA will have preset time limitations to determine how frequently the patient will receive a dose of pain medication no matter how often the button is pushed. The PCA will not continue to administer medication until the pain is completely relieved.

The home care nurse is trying to determine the necessary services for a 65-year-old patient who was admitted to the home care service after left knee replacement. Which tool is the best for the nurse to utilize? a. Minimum Data Set (MDS) b. Functional Status Scale (FSS) c. 24-Hour Functional Ability Questionnaire (24hFAQ) d. The Edmonton Functional Assessment Tool

c. 24-Hour Functional Ability Questionnaire (24hFAQ) The 24hFAQ reviews the postoperative patient in the home setting. The MDS is for nursing home patients. The FSS is for children. The Edmonton is for cancer patients.

The patient has been smoking 2 packs of cigarettes for the last 15 years. How will the nurse chart the patient's tobacco use history in pack-years? a. 7.5 pack-years b. 17 pack-years c. 30 pack-years d. 35 pack-years

c. 30 pack-years Pack-years equal the number of years smoking number of packs per day. This patient smoked 2 packs of cigarettes for the last 15 years so the nurse would document 30 pack-years in the chart.

When is the best time for a male patient to perform a testicular self-assessment? a. Before getting out of bed in the morning b. After having a bowel movement c. After getting out of a hot tub d. Immediately following ejaculation

c. After getting out of a hot tub The best time for the male patient to perform a testicular self-assessment is after getting out of a warm bath or hot tub as the scrotal sac will be relaxed. This will make it easier to carefully palpate the entire surface of each testicle. The other times are not optimal for testicular self-assessment

Which action by the nurse will best allay a young child's fear about auscultation of breath sounds? a. Do nothing because the more fuss that is made about a procedure, the more anxiety it causes the patient. b. Explain to the patient that the stethoscope is used to listen to air going in and out of the lungs. c. Allow the child to listen to sounds with the stethoscope before the nurse uses it for assessment. d. Ask the child's mother to step outside the room because children frequently do better when alone.

c. Allow the child to listen to sounds with the stethoscope before the nurse uses it for assessment. Describe physical sensations that will occur during the procedure by telling the child that the stethoscope will not hurt. Providing information about procedures helps patients feel less anxious because they understand what to expect during the procedure. When preparatory instructions accurately describe the actual experience, the patient is able to cope more effectively with the stress from procedures and therapies. Doing nothing does not prepare the patient properly or address the anxiety. Involve the parents with young children.

Which patient has the greatest risk for extracellular fluid volume (ECV) deficit? a. A female with Crohn's disease b. A male with peptic ulcer disease c. An infant with fever and vomiting d. An adolescent with acute appendicitis

c. An infant with fever and vomiting Fever increases insensible water loss through the skin and lungs. Infants and very young children have relatively more body surface area and higher percentage of body water than older children and adults. They have greater water needs and immature kidneys. Infants are at greater risk for ECV deficit and hypernatremia because their body water loss is proportionately greater per kilogram of weight. Although acute appendicitis may cause vomiting and an active episode of Crohn's disease may cause diarrhea, adolescents and young adults have less risk of ECV deficit than an infant. An older adult has increased risk of ECV deficit if there is a large fluid output; however, peptic ulcer disease ordinarily does not cause a large fluid output

A patient was given a patch test to determine what allergen was responsible for their atopic dermatitis. The provider prescribes a steroid cream. What important instructions should the nurse give to the patient? a. Apply the cream generously to affected areas. b. Apply a thin coat to affected areas, especially the face. c. Apply a thin coat to affected areas; avoid the face and groin. d. Apply an antihistamine along with applying a thin coat of steroid to affected areas.

c. Apply a thin coat to affected areas; avoid the face and groin The patient should avoid the face and groin area as these areas are sensitive and may become irritated or excoriated. An antihistamine cream would also excoriate the area if the pruritus is cause by an allergen. There may be a need to administer oral steroid if the rash is generalized.

The nurse is reviewing a patient's spirituality and observes the patient meditating before any treatment. What is the nurse's best action? a. Document that the patient is not religious. b. Offer the patient a copy of the Bible to read. c. Arrange for quiet time for the patient as needed. d. Limit the time patient can meditate before procedures.

c. Arrange for quiet time for the patient as needed The nurse can best promote the patient's spirituality practices by arranging for the patient to be left alone when possible to meditate. Meditation is an exemplar of spirituality, not necessarily of the Christian faith. The Bible is most often read by believers in the Christian faith. Meditation does not imply that the patient is not religious. Time for meditation should not be limited, whenever possible.

The nurse is caring for an older Chinese adult male who is grimacing and appears restless after abdominal surgery. What is the nurse's best action? a. Ask the patient if he is anxious about his hospital stay. b. Ask a translator to conduct a FACES pain scale assessment. c. Ask the patient about pain and review vital signs. d. Ask the patient about any history of depression or anxiety.

c. Ask the patient about pain and review vital signs In the Chinese culture, elderly Chinese people believe that they must be stoic about pain and there is a stigma about talking about any mental health problems. The nurse should ask the patient about pain and also review vital signs for physiological signs of pain, since the patient may not admit to any pain. Assuming the patient is depressed or anxious is not the best action when considering individual cultural differences and the risk of pain after major surgery. The registered nurse should never delegate assessment to any unlicensed member of the healthcare team such as a translator. The translator may assist with communication, but the nurse is responsible for the pain assessment

Which intervention is most appropriate for the patient with the nursing diagnosis Noncompliance related to the patient's preference for cultural therapies rather than prescription medications? a. Create a clearly written medication schedule and instruction list with dosages and side effects. b. Describe the negative health outcomes that will be experienced if the cultural therapies are continued. c. Attempt to coordinate the patient's cultural therapies with the prescribed treatments. d. Inform the patient that the hospital will not be able to continue providing care if the noncompliant behavior continues.

c. Attempt to coordinate the patient's cultural therapies with the prescribed treatments. The best option for the nurse is to attempt to coordinate the patient's cultural therapies with the prescribed treatments. The nurse needs to start with the patient's own beliefs and then try to coordinate prescribed treatments so that they will be congruent. Threatening discontinuation of care and negative health outcomes will alienate the patient and worsen noncompliance. Creating a medication schedule will not increase compliance as it does not address the patient's cultural beliefs.

A nurse is caring for a patient who survived cardiopulmonary resuscitation after almost drowning. How can the nurse best help the patient to deal with the aftereffects of the experience? a. Recommend that the patient avoid discussing the experience with family. b. Assume that the near-death experience was a positive experience for the patient. c. Be nonjudgmental and help the patient to work through the near-death experience. d. Explain that people who have not had a near-death experience will not understand.

c. Be nonjudgmental and help the patient to work through the near-death experience. After patients have survived a near-death experience (NDE), promote spiritual well-being by remaining open, giving patients a chance to explore what happened, and supporting patients as they share the experience with significant others. Patients who have an NDE are often reluctant to discuss it, thinking family or caregivers will not understand. Isolation and depression often occur. Furthermore, not all NDEs are positive experiences. However, individuals experiencing an NDE who discuss it openly with family or caregivers find acceptance and meaning from this powerful experience.

The nurse is teaching a patient with vomiting and diarrhea about appropriate dietary fluids. Which option chosen by the patient would indicate further teaching is needed? a. Ice chips b. Tap water c. Black coffee d. Chicken broth

c. Black coffee Coffee should be avoided by a patient with vomiting and diarrhea as caffeine is a diuretic that can worsen dehydration. The patient should consume fluids high in Na+ (e.g., electrolyte oral solution, chicken broth). Strategies to encourage fluid intake include offering small sips of fluid frequently, popsicles, and ice chips.

Which scale is used to assess the patient's risk for development of pressure injury? a. Baker b. Morse c. Braden d. Hendrich

c. Braden The Braden Scale is used to assess the patient's risk for development of pressure injury. The Morse and Hendrich scales are used to assess the patient's risk for falls. The Wong-Baker scale is used to assess the patient's pain.

Which assessment finding is expected for a patient who may have a lower extremity deep vein thrombosis (DVT)? a. Deep burning pain in the extremity that worsens with walking and exercise b. Weak pedal pulses and an absence of hair on the affected extremity c. Calf pain when the knee is flexed slightly and the foot is pointed downward d. Numbness and tingling of the extremity with hyperreactive reflexes

c. Calf pain when the knee is flexed slightly and the foot is pointed downward The nurse may perform a Homans' test to check for the presence of a lower extremity deep vein thrombosis (DVT). The knee is flexed slightly and the foot is pointed downward. If the patient reports pain in the calf when the leg is in this position, the test is positive and the patient may have a DVT. Deep pain in the leg that increases with walking, absence of hair, and weak pedal pulses indicate arterial insufficiency. Numbness and tingling of the leg indicates a neurological problem.

The middle-aged patient is overwhelmed by the stresses of caring for aging parents as well as young children living at home. Which nursing diagnosis is most appropriate for this patient? a. Disturbed body image related to developmental changes associated with middle age b. Deficient diversional activity related to desire to enhance relaxation and contentment c. Caregiver role strain related to amount of attention and care needed by both parents and children d. Readiness for enhanced power related to desire for freedom from family responsibilities

c. Caregiver role strain related to amount of attention and care needed by both parents and children The patient feels overwhelmed by the care required by young children and aging parents so caregiver role strain is the appropriate diagnosis. The patient is not ready for enhanced power and does not need additional diversional activities. The patient does not demonstrate any sign of disturbed body image.

The nurse is caring for a dehydrated, confused patient whose breath smells like fruity bubble gum. What is the priority action of the nurse based on these assessment findings? a. Check the patient's pulse oximetry. b. Check the patient's oral mucosa for thrush. c. Check the patient's blood sugar. d. Check the patient's sclerae for jaundice.

c. Check the patient's blood sugar Confusion and dehydration with fruity breath is indicative of diabetic ketoacidosis. The nurse should check the patient's blood sugar immediately. Pulse oximetry, oral assessment for thrush, and checking for jaundice will not facilitate assessment and treatment of diabetic ketoacidosis.

Which is the best strategy for the nurse to use when communicating with a patient from different culture? a. Using a cultural joke to break the ice b. Stereotyping the patient within his or her culture c. Considering the context of the patient's background d. Assuming the patient or the family member speaks English

c. Considering the context of the patient's background When a patient is from another culture, the nurse should consider the context of the patient's background. Accept patients' rights to adhere to cultural customs and norms. People of different cultures use different types of verbal and nonverbal cues to convey meaning. A nurse should make a conscious effort not to interpret messages through his or her own cultural perspective; instead, a nurse considers the context of the other individual's background. Avoid stereotyping people from other cultures or making jokes about them. With patients from another culture, the nurse cannot assume the patient or family members can speak English

The nurse is caring for a patient with end-stage chronic obstructive pulmonary disease (COPD). The patient's pulse oximetry reading is 90% on room air. What is the priority action of the nurse? a. Administer 4L/NC oxygen immediately. b. Assist the patient into a recumbent position. c. Determine the patient's normal pulse oximetry values. d. Obtain an order for STAT arterial blood gases (ABGs).

c. Determine the patient's normal pulse oximetry values. The nurse must determine the patient's normal pulse oximetry values. The patient with end-stage COPD uses low oxygen levels to stimulate breathing rather than elevated carbon dioxide levels. When caring for patients with COPD and chronically elevated PaCO2 levels, remember that inappropriate administration of excessive oxygen will result in hypoventilation. Patients with COPD and hypercapnia (high carbon dioxide levels) have adapted to the higher carbon dioxide level. Patients with respiratory difficulty should be assisted to sit up to facilitate lung expansion. STAT arterial blood gases are not needed.

A patient tells the nurse that he experiences daytime fatigue even after 7-8 hours of sleep each night. What is the best assessment question for the nurse to ask? a. Have you tried getting 10 hours of sleep instead of 8 hours? b. How long are you in the rapid eye movement (REM) stage? c. Do you also have any recent lifestyle or behavior changes? d. Do any of your close relatives have any sleep disorders?

c. Do you also have any recent lifestyle or behavior changes? The best question to elicit the most pertinent information is "Do you also have any recent lifestyle or behavior changes?" The patient is getting 7-9 hours/sleep each night, which is expected for the average adult. The patient will not be able to recall an unconscious state such as REM sleep. The patient may have close relatives with sleep disorders but this does not necessarily affect the patient's own sleep habits.

The patient's tuberculosis test appears red and flat after the injection 48 hours ago. Which is the appropriate action of the nurse? a. Repeat the tuberculosis test because the results are inconclusive. b. Measure the reddened area in millimeters and document the result. c. Document the results as a negative reaction to the tuberculosis test. d. Contact the state health department about the patient's positive test.

c. Document the results as a negative reaction to the tuberculosis test A reddened flat area is not a positive reaction that does not need to be measured. A positive tuberculosis skin result is a palpable, elevated, hardened area around the injection site.

Which nursing action indicates that a nurse is more likely to incur a medication error during medication administration? a. Checks the original medication order on the patient's chart b. Asks the patient to state his/her name and date of birth c. Does not scan the barcode of the patient prior to administering the medication d. Does not provide the patient with a glass of water

c. Does not scan the barcode of the patient prior to administering the medication Use of barcode scanning of both the medication and the patient's hospital band is critical to maintaining safe practice during medication administration. The nurse by not scanning the barcode is not maintaining the required elements and as a result is more likely to incur a medication error. Checking the original order and asking the patient to provide identification are required elements. Not providing a glass of water to the patient is not related to a medication error but does not represent best practice unless the patient is NPO except meds which would require sips of water.

The nurse is inserting an indwelling urinary catheter before the patient has abdominal surgery. Which type of catheter will the nurse utilize for the procedure? a. Straight catheter b. Single-lumen catheter c. Double-lumen catheter d. Triple-lumen catheter

c. Double-lumen catheter The nurse will use a double-lumen catheter for the patient. One lumen will inflate the balloon and the other will drain the urine from the bladder. Straight or single-lumen catheters are used for intermittent catheterization. Triple-lumen catheters are used for continuous bladder irrigation.

The patient is brought in after collapsing outside on a very hot day. Which assessment finding will the nurse expect to note during physical examination of the patient? a. Pallor of the patient's extremities b. Cyanosis of the patient's nail beds and lips c. Dry mucus membranes and poor skin turgor d. Lower extremity edema and a generalized itchy rash

c. Dry mucus membranes and poor skin turgor The patient would expect to find signs of dehydration after the patient collapsed on a very hot day. These signs include dry mucus membranes and poor skin turgor. Pallor and cyanosis indicates poor blood flow and impaired oxygenation. Lower extremity edema and generalized itchy rash are not consistent with dehydration due to heat exposure.

Which intervention is most appropriate to treat ongoing insomnia for a middle-aged adult with a busy career? a. Obtain a prescription for zolpidem to be taken at bedtime. b. Suggest having warm milk with a shot of whisky before going to bed. c. Encourage the patient to practice peaceful meditation before bedtime. d. Recommend the use of sleep aids such as triazolam

c. Encourage the patient to practice peaceful meditation before bedtime The patient should be encouraged to try nonpharmaceutical methods to reduce stress and reduce insomnia. Meditation is a calming activity that can help prepare the patient to fall asleep. Zolpidem and triazolam should not be taken for long periods of time and carry the risk of side effects. Alcohol disrupts sleep patterns and should be avoided.

What is the best intervention to help a school-age child fall asleep at night? a. Encourage exercise in the evening to promote fatigue. b. Provide the child with a high-protein snack before bed. c. Have the parent read a quiet bedtime story to the child. d. Have the child complete homework before going to bed.

c. Have the parent read a quiet bedtime story to the child. A 6-year-old child averages 11 to 12 hours of sleep nightly. Encouraging quiet activities usually persuades the 6- or 7-year-old child to go to bed. Playing an active game, doing homework right before bed, and eating are not quiet activities.

Which test can the nurse use to assess the patient's balance? a. Have the patient reach out to touch the nurse's finger then the patient's nose. b. Have the patient track the nurse's finger as it moves through the field of vision. c. Have the patient stand with feet together, arms out, and eyes closed. d. Have the patient run the heel of the foot along the shin of the other leg.

c. Have the patient stand with feet together, arms out, and eyes closed. Having the patient stand with feet together, arms out, and eyes closed is a good test of balance. Having the patient touch or track the nurse's finger and run the heel up and down the shin are tests for coordination and skilled movements

Which complication may develop as a result of frequent middle ear infections as a child? a. Meniere's disease b. Serous otitis media c. Hearing impairment d. Impaction of cerumen

c. Hearing impairment Hearing impairment is common in the United States. At-risk children include those with a family history of childhood hearing impairment, perinatal infection (rubella, herpes, or cytomegalovirus), low birth weight, chronic ear infections, and Down syndrome. Frequent middle ear infections do not lead to impaction of cerumen or Meniere's disease. Serous otitis media is another name for a middle ear infection.

The nurse is assigned a group of patients. Which patient finding would the nurse identify as a factor leading to increased risk for impaired gas exchange? a. Blood glucose of 350 mg/dL b. Anticoagulant therapy for 10 days c. Hemoglobin of 8.5 g/dL d. Heart rate of 100 beats/min and blood pressure of 100/60

c. Hemoglobin of 8.5 g/dL The hemoglobin is low (anemia); therefore the ability of the blood to carry oxygen is decreased. High blood glucose and/or anticoagulants do not alter the oxygen carrying capacity of the blood. A heart rate of 100 beats/min and blood pressure of 100/60 are not indicative of oxygen carrying capacity of the blood.

An elderly Chinese woman is interested in biologically based therapies to relieve osteoarthritis (OA) pain. You are preparing a plan of care for her OA. Options most conducive to her expressed wishes may include which actions or activities? a. Pilates, breathing exercises, and aloe vera b. Guided imagery, relaxation breathing, and meditation c. Herbs, vitamins, and tai chi d. Alternating ice and heat to relieve pain and inflammation

c. Herbs, vitamins, and tai chi Nonpharmacologic strategies encompass a wide variety of nondrug treatments that may contribute to comfort and pain relief. These include the body-based (physical) modalities, such as massage, acupuncture, and application of heat and cold, and the mind-body methods, such as guided imagery, relaxation breathing, and meditation. There are also biologically based therapies which involve the use of herbs and vitamins, and energy therapies such as reiki and tai chi. Pilates, breathing exercises, aloe vera, guided imagery, relaxation breathing, meditation, and alternating ice and heat are multimodal therapies for pain management. They are not exclusively biologically based, which involves the use of herbs and vitamins.

Which is the best position for a patient who is acutely short of breath? a. Lateral b. Prone c. High-Fowler's d. Sims'

c. High-Fowler's In high-Fowler's position the head of the bed is at a 60- to 90-degree angle. Use this position for patients who cannot tolerate a supine position, such as those with cardiac and respiratory problems. Prone position is incorrect because it can compromise lung expansion. Lateral position is incorrect because patients who are obese or older do not tolerate this position for any length of time. Sims' position is incorrect because it is a semi-prone position that can compromise lung expansion.

Which patient assessment finding leads the nurse to suspect that the patient may have central sleep apnea? a. Use of nicotine chewing gum to stop smoking b. History of iron deficiency anemia and hemoglobin 13 g/dL c. History of cervical spine degeneration d. Use of nitrofurantoin to treat urinary tract infection

c. History of cervical spine degeneration Degeneration of the cervical spine can lead to central sleep apnea as the brain fails to send stimuli to breathe. Iron deficiency anemia, nicotine chewing gum, and nitrofurantoin do not lead to central sleep apnea.

A nurse is preparing to implement a teaching plan. Which factor might be considered to be a barrier to patient education? a. Family resources b. High school education c. Hunger and pain d. Need perceived by patient

c. Hunger and pain A patient who is hungry or in pain has limited ability to concentrate or learn. Family resources would be considered in developing a plan of care and could be an asset or a barrier to patient education. The patient's educational level would be considered in planning teaching strategies but would not be a barrier to education. A need perceived by a patient would provide motivation for learning and would not be a barrier.

The patient keeps more than 30 cats in the home and is unable to adequately care for them. Which is the priority nursing diagnosis for this patient? a. Disturbed sensory perception related to inability to smell cat feces b. Caregiver role strain related to inability to adequately care for 30 cats c. Impaired home maintenance related to unhygienic, unclean surroundings d. Risk for situational low self-esteem related to neglected home environment

c. Impaired home maintenance related to unhygienic, unclean surroundings Impaired home maintenance related to unhygienic, unclean surroundings is the priority nursing diagnosis because the unhygienic conditions can lead to illness, injury, or infection. The inability to smell cat feces is not as important as the overall impaired home maintenance. Caregiver role strain is not appropriate for this patient. Risk for situational low self-esteem is not a high priority

Which nursing diagnosis is most appropriate for a patient with xerostomia? a. Total urinary incontinence related to inability to feel urge to urinate b. Bathing self-care deficit related to inability to perceive left-sided body parts c. Impaired oral mucus membranes related to decreased salivation and dry mouth d. Disturbed sensory perception related to feeling of electric pain in feet and hands

c. Impaired oral mucus membranes related to decreased salivation and dry mouth Xerostomia is a decrease in salivary production leading to dry mouth. This can cause damage to oral mucus membranes. Xerostomia does not include urinary disturbances or neuropathic pain.

When a parent asks how to seat a 6-year-old child in the family car, how will the nurse respond? a. In the rear seat with lap and shoulder seat belts b. In the front seat with lap and shoulder seat belts c. In the rear seat with a belt-positioning booster seat d. In the front seat with a belt-positioning booster seat

c. In the rear seat with a belt-positioning booster seat All children under age 13 should ride in the rear seat. Children less than 8 years of age should be in a belt-positioning booster seat as they are not tall enough to fit into the lap and shoulder seat belts.

A diabetic patient is brought into the emergency department unresponsive. The arterial pH is 7.28. Besides the blood pH, which clinical manifestation is seen in uncontrolled diabetes mellitus and ketoacidosis? a. Decreased hunger sensation b. Report of no urine output c. Increased respiratory rate d. Decreased thirst

c. Increased respiratory rate Ketoacidosis decreases the pH of the blood, stimulating the respiratory control area of the brain to buffer the effects of the increasing acidosis. The rate and depth of respirations are increased (Kussmaul respirations) to excrete more acids by exhalation. Usually polydipsia (increased thirst), polyphagia (increased hunger), and polyuria (increased urine output) are seen with hyperglycemia and ketoacidosis.

The nurse is caring for a patient who must lay flat in bed for several days after spinal surgery. Which is the priority nursing diagnosis for this patient? a. Impaired social interaction related to prolonged bed rest b. Toileting self-care deficit related to inability to use the restroom c. Ineffective breathing pattern related to prolonged supine position d. Ineffective thermoregulation related to lengthy period of immobility

c. Ineffective breathing pattern related to prolonged supine position Prolonged bed rest leads to decreased lung expansion and ineffective breathing pattern. Toileting self-care deficit and impaired social interaction are not as important as oxygenation. Prolonged immobility does not affect thermoregulation

The nurse manager rehearses what to say to a nurse who made a serious medication error. Which form of communication is being used by the nurse manager? a. Intonation b. Nonverbal c. Intrapersonal d. Orientation

c. Intrapersonal Intrapersonal communication, also called self-talk, is a powerful form of communication that occurs within an individual. People "talk to themselves" by forming thoughts internally that strongly influence perceptions, feelings, behavior, self-concept, and performance. Self-talk is a mental rehearsal for difficult tasks or situations so that individuals deal with them more effectively. Nonverbal communication uses body language, gestures, and eye contact to convey messages rather than spoken phrases. Intonation is inflection and pitch of the voice used to help convey a message. Orientation is the interview phase when the patient and nurse meet and get to know each other.

The nurse is performing urinary catheterization for a female patient. The catheter will not advance any further but there is no urine output. What is the appropriate action of the nurse? a. Withdraw the catheter and notify the health care provider immediately. b. Palpate the patient's bladder to assess for fullness, tenderness, or distention. c. Leave the catheter in place and reattempt insertion with a new sterile catheter. d. Utilize the bladder scanner to determine how much urine is in the patient's bladder.

c. Leave the catheter in place and reattempt insertion with a new sterile catheter The catheter has been inadvertently inserted into the patient's vagina. The nurse should leave the catheter in place and reattempt insertion with a new sterile catheter. There is no need to notify the health care provider immediately. Palpation of the bladder and bladder scanning should be completed prior to insertion of the catheter

How can the parents best protect their premature infant from developing respiratory syncytial virus (RSV)? a. Immunize the infant against RSV. b. Ensure that the infant's bottles are sterilized. c. Limit the baby's exposure to crowds of people. d. Daily administration of prophylactic antibiotics.

c. Limit the baby's exposure to crowds of people. Limiting exposure to crowds of people will limit the exposure to respiratory viruses including RSV. Sterilizing bottles will not protect the infant from RSV. There is no vaccine available for RSV. Antibiotics are not effective for viral infections such as RSV

Which is the priority nursing assessment for a patient wearing an abdominal binder after abdominal surgery? a. Mental status and orientation b. Hourly fluid intake and output c. Lung sounds and pulse oximetry d. Presence of peripheral pedal pulses

c. Lung sounds and pulse oximetry Evaluate the patient's ability to ventilate properly, including deep breathing and coughing. An abdominal binder supports a large incision that is vulnerable to stress when a patient moves or coughs. Mental status, fluid balance, and peripheral pulses are not affected by the abdominal binder

How can the nurse best assist a hospitalized young child to fall asleep? a. Eliminate a daytime nap. b. Offer the child warm chocolate milk. c. Maintain the child's home bedtime routine. d. Allow the child to sleep longer in the morning.

c. Maintain the child's home bedtime routine A bedtime routine (e.g., same hour for bedtime or quiet activity) used consistently helps toddlers and preschool children avoid delaying sleep. Parents need to reinforce patterns of preparing for bedtime. Reading stories, allowing children to sit in a parent's lap while listening to music or praying, and coloring are routines associated with preparing for bed. Toddlers still need naps. Sleeping longer will continue to disrupt the normal routine. Chocolate can cause a person to stay awake or wake up throughout the night.

Which intervention will facilitate the physical examination of a patient with mobility issues? a. Be sure that the head of the examination table may be elevated for the patient's comfort. b. Tune the radio to the nurse's favorite station as a relaxation intervention for the patient. c. Make sure that the patient has sufficient space and assistance to transfer onto the examination table. d. Instruct the patient on the safest way to transfer onto the examination table.

c. Make sure that the patient has sufficient space and assistance to transfer onto the examination table Patients with mobility issues often have wheelchairs, walkers, or other pieces of equipment to help them move. The nurse should ensure that there is enough space for the patient to use mobility devices as needed. The nurse should eliminate other sources of noise, take precautions to prevent interruptions, and make sure the room is warm enough to maintain comfort. Playing music during the examination is distracting for the patient and the nurse. Patients with mobility impairments require safe transfer to an examination table. The patient is the expert and should be asked how to safely move from the bed to the table, either with a standing assisted transfer or by being lifted, as with a child or small adult.

What is the most appropriate resource to include when planning to provide patient education related to a goal in the psychomotor domain? a. Diagnosis-related support groups b. Internet resources c. Manikin practice sessions d. Self-directed learning modules

c. Manikin practice sessions A teaching goal in the psychomotor domain should be matched with teaching strategies in the psychomotor domain, such as demonstration, practice sessions with a manikin, and return demonstrations. Diagnosis-related support groups would be most effective with goals in the affective domain. Internet resources would be most effective for goals in the cognitive domain. Self-directed learning modules would be most effective for goals in the cognitive domain.

Which is the best method for measuring the patient's daily weight? a. Weigh the patient daily using different scales for comparison. b. Teach that daily weights are done in hospitals, but not at home. c. Monitor daily weight, comparing with the previous day's weight. d. Weigh the patient at different times of the day to determine trends

c. Monitor daily weight, comparing with the previous day's weight. Daily weights are an important indicator of fluid status. Each kilogram (2.2 pounds) of weight gained or lost overnight is equal to 1 L of fluid gained or lost. Weigh heart failure patients daily, as well as other patients who are at high risk for or actually have ECV excess. Obtain the weight at the same time each day with the same calibrated scale after a patient voids. Teach heart failure patients to take and record daily weights at home and to contact their health care provider if weight increases suddenly according to parameters their providers set. Classic research shows that heart failure patients who are hospitalized for decompensated heart failure often experience steady increases in daily weights during the week before hospitalization.

The patient with which diagnosis should have the highest priority for teaching regarding foods that are high in magnesium? a. Severe hemorrhage b. Diabetes insipidus c. Oliguric renal disease d. Adrenal insufficiency

c. Oliguric renal disease When renal excretion is decreased, magnesium intake must be decreased also, to prevent hypermagnesemia. The other conditions are not likely to require adjustment of magnesium intake

Which is an example of nociceptive pain? a. Neuropathy due to uncontrolled diabetes b. Phantom pain after amputation of a limb c. Pain from rheumatoid arthritis joint damage d. Chronic nerve pain after shingles infection

c. Pain from rheumatoid arthritis joint damage Nociceptive pain is due to damaged tissues such as bones, skin, and organs. An example of nociceptive pain would be pain resulting from rheumatoid arthritis joint damage. Neuropathy, phantom pain, and chronic nerve pain are all examples of neuropathic pain

Which patient would benefit from BiPAP therapy? a. Surgical patient under general anesthesia b. Confused, agitated patient with no gag reflex c. Patient with pulmonary edema due to CHF exacerbation d. Stroke patient who frequently aspirates fluids and saliva

c. Patient with pulmonary edema due to CHF exacerbation A patient with pulmonary edema would benefit from BiPAP therapy as it will assist with inspiration, prevent alveolar collapse during exhalation, and improve oxygenation. BiPAP is not appropriate for patients who are confused or unable to swallow. Patients under general anesthesia must be intubated in order to maintain the airway as well as ventilation

Which type of sterile dressing will be applied to the chest wall after removal of the patient's chest tube? a. Dry gauze dressing b. Absorbent foam dressing c. Petroleum gauze dressing d. Nonadherent gauze dressing

c. Petroleum gauze dressing An occlusive dressing must be applied to the chest tube site following removal to prevent entry of air into the chest cavity. Gauze impregnated with petroleum jelly is an example of an occlusive dressing. Dry gauze is not occlusive and will allow air into the chest cavity. Absorbent foam dressings are used to absorb large amounts of drainage from wounds and are not appropriate for chest tube sites

The nurse is caring for a patient who is ordered to remain on bed rest. Which is the appropriate action of the nurse to facilitate emptying of the patient's bladder? a. Assist the patient to the bedside commode for voiding. b. Insert an indwelling urinary catheter to bedside drainage. c. Place the patient in high Fowler's position when using the bedpan. d. Place the patient in Trendelenburg's position when using the bedpan.

c. Place the patient in high Fowler's position when using the bedpan The nurse should place the patient's bed in high Fowler's position when using the bedpan in order to facilitate emptying of the patient's bladder. The patient's head is lower than the feet in Trendelenburg's position and that would make it impossible for the patient to use the bedpan. The patient should not be assisted to the commode unless the orders specifically allow the nurse to do so. An indwelling urinary catheter should be avoided due to the risk of urinary tract infection.

The patient is aggressively attempting to pull out IV lines and hurt staff members. Which is the first action of the nurse? a. Conduct a thorough mental status assessment. b. Contact the health care provider to obtain an order for restraints. c. Place the patient in soft restraints to prevent injury. d. Document the patient's actions in the medical record.

c. Place the patient in soft restraints to prevent injury. The first priority of the nurse is to restrain the patient to prevent injury to the patient or others. The health care provider can then be contacted to obtain an order for restraints. The patient's actions can later be documented in the medical record. A thorough mental status assessment can be performed once the patient has been restrained and there is no risk of injury.

Which is the appropriate disposal method for used insulin syringes at home? a. Engage the safety cap over the needle and place it in the recycle bin. b. Remove the needle from the syringe and then flush it down the toilet. c. Place the used syringes in a sharps container that is mailed back for destruction. d. Place the used needles in a plastic can that is placed in the center of the trash bin.

c. Place the used syringes in a sharps container that is mailed back for destruction The safest method is to place the used syringes in a sharps container that is mailed back for destruction. The next best option is to place the used needles in a coffee can (not a plastic can) that is placed in the center of the trash bin. This will help protect sanitation workers from needlestick injury. Syringes and needles should never be flushed down the toilet. Individual syringes should never be placed in the trash even if the safety cap is engaged

A patient is to receive phototherapy for the treatment of psoriasis. What is the nursing priority for this patient? a. Obtaining a complete blood count (CBC) b. Protection from excessive heat c. Protection from excessive ultraviolet (UV) exposure d. Instructing the patient to take their multivitamin prior to treatment

c. Protection from excessive ultraviolet (UV) exposure Protection from excessive UV exposure is important to prevent tissue damage. Protection from heat is not the most important priority for this patient. There is no need for vitamins or a CBC for patients with psoriasis.

The nurse associates which assessment finding in the diabetic patient with decreasing renal function? a. Ketone bodies in the urine during acidosis b. Glucose in the urine during hyperglycemia c. Protein in the urine during a random urinalysis d. White blood cells in the urine during a random urinalysis

c. Protein in the urine during a random urinalysis v Urine should not contain protein. Proteinuria in a diabetic heralds the beginning of renal insufficiency or diabetic nephropathy with subsequent progression to end-stage renal disease. Chronic elevated blood glucose levels can cause renal hypertension and excess kidney perfusion with leakage from the renal vasculature. This leaking allows protein to be filtered into the urine.

Which action communicates to the patient that the nurse wants to leave the patient's room to care for other patients? a. Sitting in a chair next to the patient's bed b. Making sure the door is completely shut for privacy c. Repeatedly checking the clock to see what time it is d. Holding the patient's hand when the patient starts to cry

c. Repeatedly checking the clock to see what time it is Repeatedly checking the clock to see what time it is communicates to the patient that the nurse wishes to leave the room. Sitting next to the patient, ensuring privacy, and holding the patients hand are supportive nonverbal communications.

The patient's home is filled with papers and trash that has accumulated over the last 20 years. Which is the priority nursing diagnosis for the patient? a. Unilateral neglect related to inadequate support systems b. Ineffective coping related to hoarding behaviors c. Risk for falls related to cluttered walkways and untidy environment d. Readiness for enhanced comfort related to desire for nicer surroundings

c. Risk for falls related to cluttered walkways and untidy environment The highest priority nursing diagnosis is risk for falls related to cluttered walkways and untidy environment. Fall risk is more important than ineffective coping or readiness for enhanced comfort. Unilateral neglect is the lack of awareness of a body part following a stroke.

A patient has had no visitors during a lengthy hospitalization. The patient is bored, restless, and irritable. Which term best describes the patient's feelings? a. Sensory deficits b. Sensory overload c. Sensory deprivation d. Changes in attitudes

c. Sensory deprivation Sensory deprivation occurs when inadequate quality or quantity of stimuli impairs a patient's perception. It can cause affective changes (e.g., boredom, restlessness, increased anxiety, emotional lability) and/or perceptual changes (e.g., reduced attention span, disorganized visual and motor coordination, confusion of sleeping and waking states). Sensory deficits such as low vision and blindness are very common forms of disability. When a person receives multiple sensory stimuli, the brain has difficulty distinguishing the stimuli, leading to sensory overload. A person with sensory overload no longer perceives the environment in a way that makes sense. Sensory deprivation can be caused from living in a nonstimulating environment. Ask the patient how to improve the quality of stimulation in the environment.

The nurse educator uses manikins to teach patients how to correctly perform CPR on a victim of cardiac arrest. Which teaching technique is used by the nurse? a. Analogy b. Role play c. Simulation d. Enunciation

c. Simulation Simulation is a useful technique for teaching problem solving, application, and independent thinking. During individual or group discussion, the nurse presents a problem or situation pertaining to the patients' learning for patients to solve. In this case, the manikins are used to simulate a victim of cardiac arrest. During role play, your patients play themselves or someone else in the situation. Analogies add to verbal instruction by providing familiar images that make complex information more real and understandable. Enunciation is pronouncing words clearly.

The process of digestion is important for every living organism for the purpose of nourishment. Where does most digestion take place in the body? a. Large intestine b. Stomach c. Small intestine d. Pancreas

c. Small intestine Most digestion takes place in the small intestine. The main function of the large intestine is water absorption. The pancreas contains digestive enzymes; the stomach secrets hydrochloric acid to assist with food breakdown.

The patient has a large left hip decubitus ulcer with tunneling but no involvement of bone, tendon, or muscle. Which pressure injury stage will be recorded in the patient's chart? a. Stage 1 b. Stage 2 c. Stage 3 d. Stage 4

c. Stage Stage 3: Full-thickness tissue loss; subcutaneous fat may be visible, but bone, tendon, or muscle is not exposed; slough may be present but does not obscure the depth of tissue loss; may include undermining and tunneling Stage 1: Intact skin with nonblanchable redness of a localized area, usually over a bony prominence Stage 2: Partial-thickness loss of dermis presenting as a shallow open ulcer with a red-pink wound bed, without slough; may also present as an intact or open/ruptured serum-filled blister Stage 4: Full-thickness tissue loss with exposed bone, tendon, or muscle; slough or eschar may be present; often includes undermining and tunneling

A patient has cellulitis on the right forearm. The nurse would anticipate orders to administer medications to eradicate which organism? a. Candida albicans b. Group A -hemolytic streptococci c. Staphylococcus aureus d. E. Coli

c. Staphylococcus aureus Staphylococcus aureus is the usual cause of cellulitis, although other pathogens may be responsible. A small abrasion or lesion can provide a portal for opportunistic or pathogenic infectious organisms to infect deeper tissues.

Which is the priority action of the nurse for a patient with ventricular tachycardia? a. Assess the patient for signs of digoxin toxicity. b. Draw serum electrolytes to check for hyperkalemia. c. Start chest compressions if there is no palpable pulse. d. Check the patient's BP and administer sublingual nitroglycerin.

c. Start chest compressions if there is no palpable pulse. The patient with ventricular tachycardia may or may not have a pulse. The most appropriate option is for the nurse to start chest compressions if there is no palpable pulse. Checking the patient's BP, drawing blood for laboratory testing, and assessing for digoxin toxicity are not priority interventions for a patient without a palpable pulse.

Which position should be avoided for a patient who is unconscious? a. Semi-Fowler's b. Sims' c. Supine d. Lateral

c. Supine The risk for aspiration is greater in the supine position; thus avoid this position when the patient is confused, agitated, experiencing a decreased level of consciousness, or is at risk for aspiration. Semi-Fowler's is an incorrect answer because that is the position of choice to prevent aspiration. Sims' is incorrect because it is a semi-prone position that would allow the stomach contents to exit the body if the patient experienced emesis. Lateral is incorrect because the patient would be placed on his or her side, which would promote the exit of stomach contents if the patient experiences emesis

Which is the role of the nurse regarding a malfunctioning IV pump? a. Contact the IV pump manufacturer. b. Initiate a work order on the IV pump. c. Tag the IV pump and remove it from the area. d. Clean the fixed IV pump and return it to the floor

c. Tag the IV pump and remove it from the area Initiating the work order on the pump is important, but the first priority is to tag and remove the pump from service. Leaving the pump in the equipment closet could allow the pump to mistakenly be put back into service without be fixed. It is not within the nurse's role to call the pump manufacturer to report the issues. Accidents that are equipment related result from the malfunction, disrepair, or misuse of equipment or from an electrical hazard. To avoid accidents, do not operate medical equipment without adequate instruction. If you discover a faulty piece of equipment, replace it with the proper working equipment, place a tag on the faulty one, take it out of service, and promptly report any malfunctions

Which action demonstrates Piaget's formal operations period of development? a. Touching everything and putting everything into the mouth to learn about the surroundings b. Learning not to touch a hot radiator cover again after suffering a mild burn the first time c. Taking insulin as prescribed because otherwise dangerous complications of diabetes will develop d. Understanding that a family vacation is in the future and asking daily if vacation will be "today."

c. Taking insulin as prescribed because otherwise dangerous complications of diabetes will develop The formal operations stage of development is typically achieved during adolescence through adulthood. The individual is able to understand abstract concepts and realize the consequences of actions. Touching everything demonstrates the sensorimotor stage experienced by infants. Memory about events is achieved during the preoperational stage when the child learns not to touch a hot item after previously suffering a burn. Preoperational stage is also demonstrated by knowing that something will happen in the future but not being able to fully understand when it will occur.

The nurse is caring for a patient who suffered a fractured arm. Which assessment finding is expected after the patient's cast is removed after 6 weeks? a. The skin is thin with no hair growth. b. The radial pulse is weak and thready. c. The arm muscles are atrophied and weak. d. The fingernail beds are thick and clubbed

c. The arm muscles are atrophied and weak The patient's arm muscles will have atrophied and become weak due to 6 weeks of immobilization in the cast. Thin skin, weak radial pulse, and clubbed nail beds are not expected after immobilization.

Which assessment finding leads the nurse to include the risk for delayed development nursing diagnosis in the care plan for an infant? a. The baby's father works from home b. The baby has three older sisters at home c. The baby was born at only 30 weeks' gestation d. The baby cannot tolerate formula that contains lactose

c. The baby was born at only 30 weeks' gestation Preterm birth is an important risk factor for delayed development in infants. The father's employment, older siblings, and need for lactose-free formula are not risk factors for delayed development.

Which is an example of a procedure-inherent accident? a. The patient suffered a burn due to a malfunctioning heating pad. b. The patient suffered a tongue laceration during a grand mal seizure. c. The nurse suffered a back injury when repositioning a heavy patient in bed. d. The physician suffered a broken wrist after it was caught in the elevator door.

c. The nurse suffered a back injury when repositioning a heavy patient in bed. A procedure-inherent accident occurs when a patient or staff member is injured in the process of providing patient care. An example of a procedure-inherent accident is a back injury caused by moving a heavy patient in bed. A malfunctioning heating pad caused an equipment-related accident. A tongue laceration from a seizure is a patient-inherent accident. A broken wrist in an elevator door could be considered an equipment-related accident

Which assessment finding best indicates to the nurse that the teaching about a dressing change was successful? a. The patient understands how to change the dressing using sterile technique. b. The patient verbalizes understanding about how to change the sterile dressing. c. The patient correctly demonstrates the dressing change using sterile technique. d. The patient acknowledges the principles of sterile technique for dressing changes

c. The patient correctly demonstrates the dressing change using sterile technique. Demonstration is the best method to evaluate a psychomotor skill. Examples of evaluating the effectiveness of teaching include having patients show how to perform a newly learned skill (e.g., self-catheterization) or asking patients to explain how they will incorporate newly ordered medications into their daily routines. Evaluating the effectiveness of teaching for a psychomotor skill includes a demonstration, not understanding or acknowledging. Just stating, "Yes" does not indicate learning like a demonstration does.

A nurse enters a patient's room and sees the patient grimacing with each movement. When the nurse asks how the patient is feeling, the patient states "I feel fine." Which finding will the nurse classify as nonverbal communication? a. The patient states "I feel fine." b. The nurse asks how the patient is feeling. c. The patient grimaces with each movement. d. The nurse is present at the patient's bedside.

c. The patient grimaces with each movement. The patient grimacing with each movement is nonverbal communication. Nonverbal communication includes messages sent through the language of the body, without the use of words. Nonverbal forms of communication include use of facial expressions, eyes, gestures, posture, and physical appearance. Nonverbal communication often reveals physical feelings. Tone of voice, asking questions, and saying that he or she feels fine are examples of verbal communication. Verbal communication involves the use of words or phrases and includes intonation, pacing, denotative and connotative meanings, volume, clarity, brevity, timing, and relevance. The nurse's presence at the patient's bedside is not nonverbal communication.

The patient's pupils are the size of tiny pinpoints. Which factor could lead to this assessment finding? a. The patient has been taking high doses of steroids. b. The patient suffered massive head trauma and is brain dead. c. The patient injected heroin intravenously 1 hour ago. d. The patient has developed acute narrow angle glaucoma

c. The patient injected heroin intravenously 1 hour ago. Pinpoint pupils are a common sign of opioid intoxication. Brain death causes dilation of the pupils that are unresponsive to light. High doses of steroids can lead to cataracts, clouding of the lens within the eye. Acute narrow angle glaucoma will not cause tiny pinpoint pupils.

Why will the patient with sleep apnea be encouraged to use the CPAP machine while in the hospital after surgery? a. It will keep the patient in deep levels of REM, which will decrease the need for pain medication. b. It will help decrease noise from the roommate and hospital environment that may keep the patient awake. c. The patient needs ventilator support owing to the increased chance of postoperative respiratory complications. d. The patient needs to follow the same bedtime routine to promote a safe environment for sleep.

c. The patient needs ventilator support owing to the increased chance of postoperative respiratory complications These patients need ventilator support in the postoperative period because obstructive sleep apnea is linked to increased postoperative respiratory complications. After surgery, the patient achieves very deep levels of REM sleep that lead to muscle relaxation and airway obstruction. In these patients, the anesthesia in combination with pain medications used after surgery reduces the patient's defenses against airway obstruction. Make sure that patients use their home CPAP equipment. Use pain medication carefully in these patients. Promoting the home bedtime routine is beneficial, but that is not the primary reason for using the CPAP; it is to prevent complications from surgery.

Which assessment finding is expected for a patient with impaired lung compliance? a. The patient's respirations are very deep and rapid. b. The patient reports sharp left-sided rib and chest pain. c. The patient struggles to take a deep breath and exhale. d. The patient's breathing pattern is irregular with periods of apnea.

c. The patient struggles to take a deep breath and exhale Compliance refers to the elasticity of the lungs to expand and contract with each breath. A patient with impaired lung compliance struggles to inhale and exhale due to stiffness of the lungs. Lung compliance does not affect breathing patterns or cause left-sided rib pain.

Which assessment finding leads the nurse to add risk for poisoning to the patient's care plan? a. The patient takes alprazolam 0.25 mg every 8 hours. b. The patient rinses with a fluoride mouthwash after brushing the teeth. c. The patient takes acetaminophen 1000 mg every 4 hours around the clock. d. The patient frequently uses an alcohol-based sanitizer for hand hygiene.

c. The patient takes acetaminophen 1000 mg every 4 hours around the clock The safe maximum daily dosage of acetaminophen is 4000 mg daily. The patient is taking 6000 mg daily, leading to a risk of poisoning. The patient is taking an appropriate dosage of alprazolam daily. Use of fluoride mouthwash daily and an alcohol-based sanitizer for hand hygiene do not put the patient at risk of poisoning.

Which is the appropriate outcome for a patient with the nursing diagnosis insomnia related to overwhelming parental and job responsibilities? a. The patient will sleep longer throughout the night. b. The patient will wake up more refreshed in the morning. c. The patient will fall asleep within 30 minutes of going to bed. d. The patient will take a warm bath nightly before going to bed.

c. The patient will fall asleep within 30 minutes of going to bed. Falling asleep within 30 minutes of going to bed is a measurable, appropriate goal. Waking up more refreshed or sleeping longer is not measurable. Taking a warm bath before bed is an intervention rather than a goal.

Which is the appropriate goal for a nonverbal, confused patient with the nursing diagnosis chronic pain related to widespread tissue damage? a. The patient's pain will be reduced to a minimal level. b. The nurse will assess the patient's pain every 2 hours. c. The patient will not demonstrate moaning or grimacing. d. The patient will use a 0-to-10 pain scale to identify pain levels.

c. The patient will not demonstrate moaning or grimacing. Objective assessments of discomfort may be used when the patient is unable to verbalize sensations or rate pain on a scale. The absence of moaning or grimacing is objective and the nurse is able to determine whether or not the goal has been met. Pain assessment is an intervention rather than a goal. Minimal pain is not objective. The nonverbal confused patient will not be able to rate pain using a 0-to-10 pain scale.

Which outcome is appropriate for the patient with the nursing diagnosis ineffective protection related to use of anticoagulant medications? a. The patient will remain free of signs or symptoms of infection. b. The patient will have a soft formed stool at least every other day. c. The patient will verbalize precautions to take in order to prevent bleeding. d. The patient will have clear lung sounds bilaterally and no sign of cyanosis

c. The patient will verbalize precautions to take in order to prevent bleeding The patient taking anticoagulant medications is at risk for bleeding. An appropriate goal is that the patient will verbalize precautions to take in order to prevent bleeding. Infection, constipation, and oxygenation are not appropriate goals for a patient at risk for bleeding.

Which assessment finding leads the nurse to conclude that digital disimpaction of stool is unsafe for the patient? a. The patient has a large mass of hard, dry stool in the rectum. b. The patient has not had a bowel movement for the last 6 days. c. The patient's pulse is 50 beats/minute due to a history of heart block. d. The patient has taken senna every morning for the last 3 days.

c. The patient's pulse is 50 beats/minute due to a history of heart block Digital removal of stool from the rectum can cause lowering of the heart rate due to stimulation of the vagus nerve. Patients with bradycardia can experience dangerously low heart rates due to vagal stimulation. Presence of hard, dry stool in the rectum, use of senna, and no bowel movement for the last 6 days are not contraindications for digital removal of stool

Which assessment finding will the nurse expect to note during physical examination of the patient with a cast on the lower left leg? a. The patient's left foot has a musty, fetid smell b. The patient's left foot has 3+ pitting edema and pallor. c. The temperature of the left foot is the same as the right foot. d. The patient's left foot is cool with thin, shiny skin.

c. The temperature of the left foot is the same as the right foot. Compare symmetrical body parts, which should be the same in assessment. Always assess skin temperature for patients at risk for impaired circulation, such as after a cast application or vascular surgery. Normally the skin temperature is warm. Skin temperature is the same throughout the body. Accurately assess temperature by palpating the skin with the dorsum, or back, of the hand. A fetid, musty smell indicates an infection under the cast. The left foot should not have edema, pallor, shiny skin, or be cool to the touch.

The nurse is to administer 40 mEq of IV KCl to the patient with severe hypokalemia. Why did the pharmacy send up four 100 mL IV bags with 10 mEq of KCl in each bag? a. To prevent the patient from receiving too much IV fluid. b. The four smaller bags are less expensive than one large one. c. To prevent accidentally infusing the 40 mEq of KCl too quickly. d. The doctor may change his mind and lower the prescribed KCl dose.

c. To prevent accidentally infusing the 40 mEq of KCl too quickly Rapid infusion of KCl can cause life-threatening cardiac dysrhythmias. The pharmacy sent up four separate IV bags containing 10 mEq KCl in each to prevent accidentally infusing the 40 mEq of KCL too quickly. Four smaller bags are not necessarily less expensive than one large bag but the cost is less important than patient safety. The small amount of extra fluid is negligible. The infusion may be stopped if the physician lowers the ordered dose of KCl.

Which pain relieving option should be avoided by a patient with chronic back pain who must continue to work as a truck driver? a. Transcutaneous electrical nerve stimulation (TENS) unit b. Naproxen sodium 200 mg PO every 12 hours c. Tramadol extended release 200 mg PO daily d. Application of hot and cold packs to the lower back area

c. Tramadol extended release 200 mg PO daily Tramadol is a controlled substance that can cause seizures, hallucinations, and sedation. Tramadol should not be taken before driving a vehicle or operating machinery. TENS units, NSAIDs such as naproxen sodium, and hot/cold packs are all acceptable options for the truck driver.

Loud wails erupt whenever the nurse tries to pick up the infant from the mother. Which developmental stage is the infant experiencing? a. Affiliation and love b. Production and care c. Trust versus mistrust d. Autonomy versus doubt

c. Trust versus mistrust The infant is at the first stage of Erikson's psychosocial development, trust versus mistrust. The infant trusts the mother and must learn to trust others. The infant is not able to address more advanced developmental needs such as autonomy, affiliation, and production.

The nurse is reviewing a patient before hanging an IV solution of 0.9% NaCl with KCl in it. Which assessment finding should cause the nurse to hold the IV solution and contact the physician? a. Weight gain of 2 pounds since last week b. Dry mucous membranes and skin tenting c. Urine output 8 mL/hr d. Blood pressure 98/58

c. Urine output 8 mL/hr Administering IV potassium to a patient who has oliguria is not safe, because potassium intake faster than potassium output can cause hyperkalemia with dangerous cardiac dysrhythmias. Dry mucous membranes, skin tenting, and blood pressure 98/58 are consistent with the need for IV 0.9% NaCl. Weight gain of 2 pounds in a week does not necessarily indicate fluid overload, because it can be from increased nutritional intake. An overnight weight gain indicates a fluid gain.

Which assessment finding needs to be communicated promptly to the patient's health care provider? a. Postvoid residual of 15 mL of urine. b. Leakage of small amounts of urine when coughing. c. Urine output of 160 mL over the last 8 hours. d. Patient's report of an urge to void during palpation of the bladder.

c. Urine output of 160 mL over the last 8 hours. Urinary output should be at least 30 mL/hour so urine output of 160 mL for 8 hours should be reported promptly to the health care provider. A postvoid residual of 15 mL is not a problem. Stress incontinence does not need to be promptly communicated to the health care provider. It is normal for patients to feel an urge to void during palpation of the bladder.

How can the nurse most accurately assess the skin tone for an older adult African American patient with deeply pigmented skin? a. Utilize fluorescent lighting. b. Turn up the heat in the room. c. Utilize natural sunlight. d. Turn down the temperature in the room.

c. Utilize natural sunlight The recommended light is natural sunlight, with halogen lighting being another option. Sunlight is the best choice for detecting skin changes in patients with darker skin. A room that is too warm causes superficial vasodilation, resulting in an increased redness of the skin. Patients who become too cold by air conditioning can develop cyanosis (bluish color) around the lips and nail beds.

Which intervention will the nurse use for an abscessed leg wound? a. Warm water sitz baths b. Cold moist compresses c. Warm moist compresses d. Epsom salt solution soaks

c. Warm moist compresses A warm moist compress improves circulation, relieves edema, and promotes concentration of pus and drainage. Warm moist compresses are appropriate for an abscessed leg wound. Epsom salt soaks increases muscle relaxation and loosens stiff joints. The patient who has had rectal surgery or an episiotomy during childbirth or who has painful hemorrhoids or vaginal inflammation will benefit from a sitz bath, a bath in which only the pelvic area is immersed in warm fluid. Cold moist compresses are used to relieve inflammation and swelling

Women who are given the job of caretaker for aging relatives are subject to caregiver strain due to a. feminine attributes. b. unequal gender. c. fixed gender roles. d. female inequality

c. fixed gender roles In cultures with more fixed gender roles, women are usually given the role of caretaker for aging relatives and may suffer the stresses of caregiver strain. Feminine attributes refers to harmonious relationships, modesty, and taking care of others. Unequal gender refers to roles of males and females being unevenly distributed. Female inequality refers to female gender and roles being less than or unequal to male roles.

To plan early intervention and care for a child with a developmental delay, the nurse would consider knowledge of the concepts most significantly impacted by development, including a. culture. b. environment. c. functional status. d. nutrition.

c. functional status Function is one of the concepts most significantly impacted by development. Others include sensory-perceptual, cognition, mobility, reproduction, and sexuality. Knowledge of these concepts can help the nurse anticipate areas that need to be addressed. Culture is a concept that is considered to significantly affect development; the difference is the concepts that affect development are those that represent major influencing factors (causes); hence determination of development would be the focus of preventive interventions. Environment is considered to significantly affect development. Nutrition is considered to significantly affect development

The emphasis on understanding cultural influence on health care is important because of a. disability entitlements. b. HIPAA requirements. c. increasing global diversity. d. litigious society.

c. increasing global diversity Culture is an essential aspect of health care because of increasing diversity. Disability entitlements refer to defined benefits for eligible mental or physically disabled beneficiaries in relation to housing, employment, and health care. HIPAA requirements refers to the HIPAA Privacy Rule, which protects the privacy of individually identifiable health information; the HIPAA Security Rule, which sets national standards for the security of electronic protected health information; and the confidentiality provisions of the Patient Safety Rule, which protect identifiable information being used to analyze patient safety events and improve patient safety. Litigious society refers to excessively ready to go to law or initiate a lawsuit.

The most appropriate response of the nurse when a mother asks what the Denver II does is that it a. can diagnose developmental disabilities. b. identifies a need for physical therapy. c. is a developmental screening tool. d. provides a framework for health teaching.

c. is a developmental screening tool. The Denver II is the most commonly used measure of developmental status used by healthcare professionals; it is a screening tool. Screening tools do not provide a diagnosis. Diagnosis requires a thorough neurodevelopment history and physical examination. Developmental delay, which is suggested by screening, is a symptom, not a diagnosis. The need for any therapy would be identified with a comprehensive evaluation, not a screening tool. Some providers use the Denver II as a framework for teaching about expected development, but this is not the primary purpose of the tool.

The patient has type B chronic obstructive pulmonary disease (COPD) exacerbated by an acute upper respiratory infection. Which blood gas values should the nurse expect to see? a. pH high, PaCO2 high, HCO3 - high b. pH low, PaCO2 low, HCO3 - low c. pH low, PaCO2 high, HCO3 - high d. pH low, PaCO2 high, HCO3 - normal

c. pH low, PaCO2 high, HCO3 - high Type B COPD is a chronic disease that causes impaired excretion of carbonic acid, thus causing respiratory acidosis, with PaCO2 high and pH low. This chronic disease exists long enough for some renal compensation to occur, manifested by high HCO3 - . Answers that include low or normal bicarbonate are not correct, because the renal compensation for respiratory acidosis involves excretion of more hydrogen ions than usual, with retention of bicarbonate in the blood. High pH occurs with alkalosis, not acidosis.

The nurse preparing a teaching plan for a preschooler knows that, according to Piaget, the expected stage of development for a preschooler is a. concrete operational. b. formal operational. c. preoperational. d. sensorimotor.

c. preoperational The expected stage of development for a preschooler (3-4 years old) is pre-operational. Concrete operational describes the thinking of a school-age child (7-11 years old). Formal operational describes the thinking of an individual after about 11 years of age. Sensorimotor describes the earliest pattern of thinking from birth to 2 years old.

A 17-year-old girl is hospitalized for appendicitis, and her mother asks the nurse why she is so needy and acting like a child. The best response of the nurse is that in the hospital, adolescents a. have separation anxiety. b. rebel against rules. c. regress because of stress. d. want to know everything.

c. regress because of stress Regression to an earlier stage of development is a common response to stress. Separation anxiety is most common in infants and toddlers. Rebellion against hospital rules is usually not an issue if the adolescent understands the rules and would not create childlike behaviors. An adolescent may want to "know everything" with their logical thinking and deductive reasoning, but that would not explain why they would act like a child.

A student nurse who works in a pediatric clinic is assisting with an assessment on a young child who is not yet walking. She knows that it is considered a delayed gross motor ability if the child does not walk by months. a.16 b.18 c.20 d.22

c.20 A critical period of development refers to a specific phase or period when the presence of a function or reasoning has its greatest effect on a specific aspect of development. For example, if a child does not walk by the age of 20 months, there is delayed gross motor ability, which slows exploration and manipulation of the environment. The success or failure experienced within a phase affects the childs ability to complete the next phases.

A nursing student is volunteering with a local agency to help prepare the community for a potential bioterrorist attack. On which of the following threats would be the nursing students primary focus? a.Hurricane b.Earthquake c.Anthrax d.Tornado

c.Anthrax A new potential environmental health threat is the possibility of a bioterrorist attack. Threats of this type come in the form of biological, chemical, and radiological attacks. Bioterrorism, or the use of biological agents to create fear and threat, is the most likely form of a terrorist attack to occur. The other responses (hurricane, earthquake, and tornado) are classified as natural disasters.

A 58-year-old Caucasian woman is at the clinic for her annual check-up. She tells the nurse that she noticed her blood pressure is higher than it was when she was younger. She asks the nurse why this would happen. What is the nurses best response? a.Your race and gender are predisposing factors for heart disease. b.Well, if you stop smoking your blood pressure would go down. c.As we age, our blood vessels become less elastic, which causes higher blood pressure. d.I dont think its anything to worry about.

c.As we age, our blood vessels become less elastic, which causes higher blood pressure Arterial vessels in the older adult become calcified and lose elastin. This may lead to hypertension and a rise in systolic blood pressure. Both of these normal changes of aging place the older adult at risk for heart failure. Do not be judgmental about smoking and the question does not indicate the female is a smoker. Males, not females, are more prone to heart disease. African-Americans, American Indians, and Mexican-Americans are at greater risk than Caucasians for developing heart disease. Telling the patient to not worry is not therapeutic

A fire erupts in a hospital waste receptacle in the hallway. What is the nurses first response? a.Report the fire. b.Attempt to extinguish the fire. c.Assist any patients to a safe area. d.Close the door to contain the fire.

c.Assist any patients to a safe area. Use the mnemonic RACE to set priorities in case of fire: RRescue and remove all patients in immediate danger. AActivate the alarm. Always do this before trying to extinguish even a minor fire. CConfine a fire by closing doors and windows and turning off oxygen and electrical equipment. EExtinguish a fire using an appropriate extinguisher. Reporting, attempting to extinguish, and closing the door all occur after assisting patients to a safe area.

A patient with pulmonary congestion needs to cough to clear secretions. The nurse instructs the patient to inhale and perform a series of coughs during exhalation. What type of cough did the nurse teach the patient? a.Quad b.Huff c.Cascade d.Splinting

c.Cascade With the cascade cough a patient takes a slow, deep breath and holds it for 2 seconds while contracting expiratory muscles. He or she then opens the mouth and performs a series of coughs throughout exhalation, thereby coughing at progressively lowered lung volumes. This technique promotes airway clearance and a patent airway in patients with large volumes of sputum. A quad cough is used for patients who have a spinal cord injury and no use of their abdominal muscles. While the patient breathes out with a maximal expiratory effort, the patient or you push inward and upward on the abdominal muscles toward the diaphragm, causing the cough. A huff cough stimulates a natural cough reflex. While exhaling, a patient opens the glottis by saying the word huff. Splinting occurs when the patientsupports the abdomen when coughing; it is not a type of cough.

A health care worker received an annual tuberculosis test administered 56 hours ago. The injection site is very red and flat. The certified nurse who is reading the test should take which action? a.Advise the health care worker another test must be done because the test was not read within the proper time. b.Tell the health care worker the results are positive and cannot return to work. c.Document the results as a negative reaction. d.Measure the area in millimeters.

c.Document the results as a negative reaction. A reddened flat area is not a positive reaction, and you do not need to measure it. Tuberculosis skin testing positive results is a palpable, elevated, hardened area around the injection site, caused by edema and inflammation from the antigen-antibody reaction, measured in millimeters. Tuberculin skin tests are read between 48 and 72 hours so the health care worker came at the right time (56 hours).

A nurse is caring for an immobile patient. What is the most appropriate nursing intervention to implement? a.Turn the patient every 4 hours. b.Apply an abdominal binder while the patient is lying in bed. c.Encourage the regular use of incentive spirometry while awake. d.Maintain the patients maximum fluid intake of 1000 mL daily.

c.Encourage the regular use of incentive spirometry while awake You can also promote lung expansion through regular deep breathing exercises, use of an incentive spirometer, and forceful coughing. Changing the position of the patient at least every 2 hours, not 4, allows the dependent lung regions to re-expand, maintains the elastic recoil property of the lungs, and clears the dependent lung regions of pulmonary secretions. Your assessment will determine if patients need more frequent position changes. An application of an abdominal binder will restrict chest expansion. Make sure that the immobile patient has a fluid intake of at least 2000 mL per day, if not contraindicated, to help keep mucociliary clearance intact.

The RN and nursing assistive personnel (NAP) are caring for six patients on the pulmonary unit. Which of the following tasks would be most appropriate for the nurse to delegate to the NAP? a.Taking vital signs on a 56-year-old man with severe dyspnea b.Suctioning a patient with hemoptysis c.Encouraging a postoperative patient to use the incentive spirometer d.Performing chest percussion on a patient with atelectasis

c.Encouraging a postoperative patient to use the incentive spirometer Base your decision to delegate responsibility to nursing assistive personnel (NAP) on your assessment of the patient and the type of care the patient will receive. Consider which tasks are safe to delegate within the skill set of the NAP and how the patient will feel about the care that you have delegated. Incentive spirometry is a method of encouraging voluntary deep breathing by providing visual feedback to patients about inspiratory volume. It is an effective method for promoting deep breathing to prevent or treat atelectasis in the postoperative patient. Incentive spirometry encourages patients to breathe to their normal inspiratory capacities and can be delegated to a NAP. The priority is to maintain or improve the patients oxygenation and meet the patients needs. You are ultimately responsible for all the total patient care. In all the other responses the patient is experiencing difficulties that require the nurses attention: severe dyspnea (difficulty breathing), hemoptysis (bloody sputum), and chest percussion (done by nurses or respiratory therapists)

According to one growth and development theorist, individuals need to accomplish a particular task before successfully completing the stage of growth and development. Each task is framed with opposing conflicts, such as trust versus mistrust. Who developed this theory? a.Sigmund Freud b.Jean Piaget c.Erik Erikson d.Lawrence Kohlberg

c.Erik Erikson Erik Erikson divided life into eight stages, known as Eriksons eight stages of development. According to this theory, individuals need to accomplish a particular task before successfully completing each stage. Each task is framed with opposing conflicts, such as trust versus mistrust. Each stage builds upon the successful attainment of the previous developmental conflict. Freuds psychoanalytic model of personality development is grounded in the belief that two internal biological forces drive the psychological change in a child: sexual (libido) and instinctive forces. Each of the five stages is associated with a pleasurable zone, serving as the focus of gratification. Jean Piaget (1896-1980) developed the theory of cognitive development, which describes childrens intellectual organization and how they think, reason, and perceive the world. The theory includes four periods: sensorimotor, preoperational, concrete operations and formal operations. Lawrence Kohlberg (1927-1987) expanded on Piagets work. According to Kohlberg (1964), moral development is one component of psychosocial development. It involves the reasons an individual makes a decision about right and wrong behaviors within a culture.

A young adult was involved in a motor vehicle accident and suffers from brain trauma. The patient has decrease mobility in all joints. The nurse should assess for which common, debilitating contracture? a.Lordosis b.Bowlegs c.Footdrop d.Kyphosis

c.Footdrop One common and debilitating contracture is footdrop. When footdrop occurs, the foot is permanently in plantar flexion. Patients are no longer able to walk when this occurs. Lordosis is the exaggeration of the anterior convex curve of the lumbar spine caused by a congenital condition or a temporary condition such as pregnancy. Bowlegs (genu varum) is one or both legs bent outward at the knee, which is normal until 2 to 3 years of age and is caused by a congenital condition or rickets. Kyphosis is the increased convexity in curvature of thoracic spine for a congenital condition, rickets, osteoporosis, or tuberculosis of the spine.

A patient on the surgical unit is 1 day postoperative for surgery to remove stomach cancer. In addition to the physiological stress this patient has undergone, the nurse recognizes that this patient will have to deal with the psychological stress of finding out that the cancer has metastasized to the liver. Which physiological change would the nurse expect to see as a response to stress? a.Decreased heart rate b.Increased hemoptysis output c.Increased respiratory rate d.Decreased cardiac output

c.Increased respiratory rate A continuous state of stress increases the bodys metabolic rate and the oxygen demand. The body responds to stress by an increased rate and depth of respiration and increased cardiac output. Hemoptysis, bloody sputum, does not occur in stress.

A 3-month-old infant is being seen for a well-child check at the pediatric clinic. The nurse is assessing the guardians knowledge level about the infants growth and development. One of the topics that the nurse has chosen to address in this session is the risk for airway obstruction. What is the main reason that the nurse has chosen this topic? a.Infants can have severe allergic reactions to food based on exposure to secondhand smoke. b.Infants are prone to lower airway infections that can become obstructive. c.Infants have a tendency to place foreign objects in their mouths. d.Infants can have airway obstruction from excessive drooling associated with teething.

c.Infants have a tendency to place foreign objects in their mouths. Infants and toddlers are at risk for airway obstruction because of their tendency to place a foreign object in their mouth. Infants and toddlers are at risk for upper respiratory tract infections, not allergic food reactions, as a result of frequent exposure to other children and exposure to secondhand smoke. During the teething process some infants develop nasal congestion, which encourages bacterial growth and increases the risk for respiratory tract infection, but not for airway obstruction. Infants are prone to upper respiratory tract infections, not lower airway infections, which usually are not dangerous, and infants and toddlers recover with little difficulty.

A nurse is giving anticipatory guidance to the mother of a 10-month-old child. The nurse is focusing on providing a safe environment for the child. Which of the following is the best statement regarding childhood safety? a.The car seat should be placed in a forward-facing position. b.The majority of deaths in children between the ages of 1 and 3 years old are caused by contagious diseases. c.Injuries are a major cause of death during infancy, especially for children 6 to 12 months old. d.Measles causes more deaths in children younger than 5 years old than all other diseases combined.

c.Injuries are a major cause of death during infancy, especially for children 6 to 12 months old. Injuries, not contagious diseases, are a major cause of death during infancy, especially for children 6 to 12 months old. The leading causes of injury to infants are falls, ingestion injuries (poison, foreign body ingestion, and medication), and burns. Aspiration often occurs from the ingestion of foreign material such as small toys and food items. The question is asking for a safe environment assessment and childhood safety; measles is an illness/disease caused by a pathogen and does not relate to providing a safe environment or information about childhood safety. All infants and toddlers should ride in a rear-facing car safety seat until they are 2 years of age or until they reach the highest weight or height allowed by the manufacturer of the car seat.

All hospital employees are concerned about the safety of patients in the hospital, especially regarding the transmission of pathogens. What is the most common means of transmission of pathogens in this environment? a.Contaminated blood products b.Enteric transmission c.Insufficient hand hygiene d.Aerosols

c.Insufficient hand hygiene A pathogen is any microorganism capable of producing an illness. The most common means of transmission of pathogens is by the hands. Pathogens are also transmitted through a humans blood and body fluids and by insects (e.g., mosquitoes carrying malaria) and rodents. Although the other responses could spread pathogens, the most correct answer and most common method is insufficient hand hygiene.

A patient has decrease mobility in all joints. Because of the lack of mobility, the nurse expects the health care provider to order what medication to prevent venous thromboembolisms that will reduce the side effect of hemorrhage? a.Oral anticoagulant b.Aspirin c.Low-molecular-weight heparin d.Unfractionated heparin

c.Low-molecular-weight heparin Newer low-molecular-weight (LMW) heparins such as ardeparin and enoxaparin are being prescribed in place of older forms of unfractionated heparin. The LMW heparins have a more predictable anticoagulant effect. Low- molecular-weight heparin compared with unfractionated heparin reduces the occurrence of major hemorrhage as a side effect. Heparin is an anticoagulant that suppresses clot formation. This therapy requires a health care providers order. Aspirin and oral anticoagulants increase the risk for hemorrhage.

While planning care for an immobilized patient, which physiological process will the nurse consider about the patients musculoskeletal system? a.Increased muscle mass b.Decreased rate of bone resorption c.Muscle atrophy d.Bone tissue density elevated

c.Muscle atrophy During immobility, the muscle atrophies, and the size of the muscle decreases. As immobility progresses and muscles are not exercised, muscle mass continues to decrease, not increase. Immobilization increases (not decreases) the rate of bone resorption, which results in reduced (not elevated) bone tissue density.

A student nurse is in her community health clinical rotation. She is visiting a family with a new baby. Which of the following statements made by the mother of a 1-month-old infant indicates the need for client education? a.My baby should double his birth weight by the time he is 6 months old. b.I shouldnt give my baby any cows milk until he is at least a year old. c.My baby has been fussy lately; I believe he is probably cutting his teeth. d.I shouldnt put my baby on a fluffy pillow to sleep.

c.My baby has been fussy lately; I believe he is probably cutting his teeth. The first tooth to erupt is usually one of the lower central incisors at the average age of 7 months. Most babies have six teeth by their first birthday. Typically infants double their birth weight by 5 to 6 months and triple it by 12 months. Infants should not have any type of cows milk during the first year because the high protein content may increase the chance of food allergies. The American Academy of Pediatrics recommends infants not sleep with a blanket until they are a year old.

A nurse notes a typical cardiovascular change in an immobilized postoperative patient. Which of the following did the nurse find upon assessment? a.Atelectasis b.Negative nitrogen balance c.Orthostatic hypotension d.Bleeding

c.Orthostatic hypotension Orthostatic hypotension, a common cardiovascular response, occurs in patients on bed rest and after prolonged sitting. Orthostatic hypotension is an increase in heart rate of more than 15% and a drop of 15 mm Hg or more in systolic blood pressure or a decrease of 10 mm Hg in diastolic blood pressure when the patient rises from a lying or sitting position to a standing position. Respiratory changes, not cardiovascular, include decreased lung expansion, generalized muscle weakness, and stasis of secretions. These conditions are consistent with the development of atelectasis. Three factors contribute to thrombus formation (not bleeding), which include loss of integrity of vessel wall, abnormalities of blood flow, and changes in clotting factors/increased platelet activity

A 5-year-old child was admitted to the pediatric unit of the hospital with the diagnosis of fever of unknown origin. Currently the patients temperature is 105 F. Which of the following is the best way to prevent a patient-inherent accident from occurring? a.Keep all electric receptacles covered in the patients room. b.Clean up patient spills as they occur. c.Pad all bed side rails. d.Do not allow the child in the playroom

c.Pad all bed side rails. One of the more common precipitating factors for a patient-inherent accident is a seizure. Place patients with a seizure disorder on seizure precautions, which are designed to protect patients when seizures occur. Keeping electric receptacles and cleaning up patient spills, and not allowing the child to play in the playroom are all extrinsic factors that are environmentally related and include room clutter, loose electrical cords, and spills

A nurse working on the medical unit mistakenly administers the wrong medication to a patient. This type of error would be classified as which of the following? a.Poisoning accident b.Equipment-related accident c.Procedure-related accident d.Accident related to time management

c.Procedure-related accident A procedure-related accident is caused by health care providers and includes medication and fluid administration errors, not putting external devices on correctly, and improperly performing procedures such as dressing changes. A poisoning accident is related to inhaled or ingested substances. An equipment-related accident results from misuse, disrepair, malfunction, or electrical hazard. An accident related to time management deals with the nurses inability to follow an organizations policy and procedures

A patient is confused and has been restrained to prevent injury. Which of the following is a priority as the nurse plans care for the shift? a.Calling the physician for an order for a chemical restraint b.Applying the most restrictive restraint to prevent injury c.Removing the restraints on the patient at least every 2 hours d.Checking on the restrained patient last

c.Removing the restraints on the patient at least every 2 hours Assess proper placement of restraint, skin integrity, pulses, temperature, color, and sensation of the restrained body part. Remove restraints at least every 2 hours or more frequently as determined by agency policy. If patient is violent or noncompliant, remove one restraint at a time and/or have other staff present while removing restraints. Chemical restraints are medications, such as anxiolytics and sedatives, used to manage a patients behavior and are not a standard treatment for a patients condition. Always attempt restraint alternatives before using a restraint. If a restraint is needed, always use the least restrictive device. Checking restrained patients last is not appropriate. Legislation emphasizes reducing the use of restraints. The Joint Commission and Centers for Medicare and Medicaid Services enforce standards for the safe use of restraint devices.

The exchange of oxygen and carbon dioxide during cellular metabolism is best described as which of the following? a.Ventilation b.Diffusion c.Respiration d.Perfusion

c.Respiration Respiration is the exchange of oxygen and carbon dioxide during cellular metabolism. The primary functions of the lungs include ventilation, the movement of air in and out of the lungs, and diffusion, the movement of gases between air spaces and the bloodstream. The heart supports perfusion, the movement of blood into and out of the lungs to the organs and tissues of the body.

A health care provider orders that a confused and disoriented patient be placed in a full hand restraint because of excessive scratching of skin. The nurse acknowledges which of the following? a.Restraints are used on an as-needed basis. b.No orders or patient consents are needed. c.Restraints must be removed every 2 hours to allow for skin assessment, toileting, and nutrition. d.An order for restraints may be used indefinitely until the patient no longer needs to be restrained.

c.Restraints must be removed every 2 hours to allow for skin assessment, toileting, and nutrition. Restraints must be removed every 2 hours to allow for skin assessment, toileting, and nutrition. Restraints are only used when other less restrictive measures fail to prevent interruption of therapies. The physicians or health care providers orders are necessary. The need for restraints must be reevaluated every 24 hours.

A patient has been hospitalized for 5 days and has had no visitors. The nurse observes the patient to be bored, restless, and anxious. The nurse identifies this behavior as which of the following? a.Sensory deficits b.Sensory overload c.Sensory deprivation d.Changes in attitude

c.Sensory deprivation Sensory deprivation occurs when inadequate quality or quantity of stimuli impairs a patients perception. It can cause affective changes (e.g., boredom, restlessness, increased anxiety, emotional lability) and/or perceptual changes (e.g., reduced attention span, disorganized visual and motor coordination, confusion of sleeping and waking states). Sensory deficits such as low vision and blindness are very common forms of disability. When a person receives multiple sensory stimuli, the brain has difficulty distinguishing the stimuli, leading to sensory overload. A person with sensory overload no longer perceives the environment in a way that makes sense. Sensory deprivation can be caused from living in a nonstimulating environment. Ask the patient how to improve the quality of stimulation in the envirment

The assistive personnel reports that an older patient is complaining of shortness of breath and palpitations. The nurse connects the patient to an electrocardiogram monitor and analyzes the rhythm with normal P wave and normal QRS and T waves. The rate is 116 beats per minute and regular. The nurse identifies this rhythm as which of the following? a.Sinus bradycardia b.Ventricular tachycardia c.Sinus tachycardia d.Normal sinus rhythm

c.Sinus tachycardia Sinus tachycardia is a rate between 100 and 180 beats per minute with normal P, QRS, and T waves. Ventricular Tachycardia has a rhythm slightly irregular, rate 100 to 200 beats/min, P wave absent, QRS complex wide and bizarre, greater than 0.12 sec. Sinus bradycardia is a rate less than 60 beats per minute with normal P, QRS, and T waves. Normal sinus rhythm is a rate between 60 and 100 beats per minute with normal P, QRS, and T waves.

A nurse is caring for a patient who signs and lip reads. When communicating, the most appropriate nursing action is to do which of the following? a.Rely on family members to interpret. b.Speak louder and more distinctly than normal. c.Sit facing the patient when speaking. d.Repeat the entire conversation if it is not understood the first time.

c.Sit facing the patient when speaking. Nurses can use a variety of communication techniques, including reading notes and writing notes, as well as reading lips and signing. When communicating, nurses should speak slowly and articulate clearly. When you are not understood, rephrase rather than repeating the entire conversation. Some patients with hearing impairments are able to speak normally. To more clearly hear what a person communicates, family and friends need to learn to move away from background noise, rephrase rather than repeat sentences, be positive, and have patience. On the other hand, some deaf patients have serious speech alterations. Patients who are deaf use sign language, read lips, write with pad and pencil, or learn to use a computer for communication

A toddler is ready to be discharged home after outpatient surgery. When conducting a home safety assessment the childs guardian states, I keep the cleaning supplies under the sink for easy access, and how soon can the child resume swimming in the local pond? Based on this statement, what is the most important safety issue for the nurse to identify? a.Standing water in the neighborhood b.Reasons for outbursts in behavior c.Storage of cleaning supplies in the house d.Childs use of safety equipment when riding or skating

c.Storage of cleaning supplies in the house Growing, curious children need adults to protect them from injury. Educate young parents or guardians about reducing risks of injuries to children, and teach ways to promote safety in the home. An example is preventing access to poisonous substances like cleaning supplies. Standing water, reasons for outbursts, and use of safety equipment are not issues based upon the guardians response, which focuses on cleaning supplies and swimming

A patient who is 7 months pregnant with her first child is visiting the health care provider for her scheduled prenatal checkup. She tells the nurse that she is short of breath and fatigued. What is the best response from the nurse? a.You should have let us know immediately instead of waiting until your appointment. b.Ill make a note of it on your chart. c.That is normal; your uterus is causing pressure on your diaphragm, making it more difficult to breathe. d.Oxygen is needed for you and the baby. I will give you some oxygen to help you.

c.That is normal; your uterus is causing pressure on your diaphragm, making it more difficult to breathe. Pregnancy causes changes in ventilation. As the fetus grows during pregnancy, the greater size of the uterus pushes abdominal contents up against the diaphragm. During the last trimester of pregnancy the inspiratory capacity declines, resulting in dyspnea on exertion and increased fatigue. Letting the nurse know immediately and applying oxygen are not needed because this is a normal finding. Making a note of it on the chart does not address the patients need.

The nursing student is formulating a nursing care plan for a patient with pneumonia. The care plan is directed toward meeting the potential oxygenation needs of the patient. Which of the following examples would be the best way for the nursing student to write an expected outcome for the care plan? a.The patient will have less pain. b.The patient will be able to breathe better. c.The patients pulse oximetry reading will remain greater than 92%. d.The patients interactions will be normal.

c.The patients pulse oximetry reading will remain greater than 92%. All goals need to have measurable outcomes for you to be able to determine whether they have been met. These include objective data, such as oxygen saturation levels, arterial blood gas levels, laboratory findings, chest radiographs, electrocardiogram patterns, blood pressure, and pulse. Quantify subjective findings, such as the reported degree of breathlessness or pain on visual analog scales. Less pain, breathe better, and normal interactions do not include objective data, making them hard to measure.

A 16-year-old patient is being seen in the emergency department (ED) after being involved in a minor motor vehicle accident. The guardian has voiced that the patient has been spending more time in his or her room, has difficulties getting along with friends, and has declining grades over the past 3 months. The patient seems distant and angry all the time. Which of the following topics is most important for the nurse to discuss with the guardian? a.Accident prevention measures b.Enrolling the patient in a defensive driving course c.The possibility of substance abuse d.The importance of automobile insurance

c.The possibility of substance abuse To assess for possible substance abuse, have parents look for environmental and psychosocial clues. Environmental clues include the presence of drug-oriented magazines, beer and liquor bottles, drug paraphernalia, blood spots on clothing, and the continual wearing of long-sleeved shirts in hot weather and dark glasses indoors. Psychosocial clues include failing grades, change in dress, increased absenteeism from school, isolation, increased aggressiveness, and changes in interpersonal relationships. Fatal crash rates for teens are high largely because of their immaturity combined with driving inexperience. Accident prevention measures, a defensive driving course, and automobile insurance are not the most important topics. Accident prevention, defensive driving, and insurance will not be effective if substance abuse is not addressed.

The family member of a 73-year-old patient with chronic obstructive pulmonary disease (COPD) is concerned about the patients recent weight loss. When questioned by the nurse, the patient denies any change in diet or activity, but admits to losing 10 lb in the past 6 weeks. What is the nurses best response to the family members concern? a.Maybe the patient has a higher metabolic rate than you. b.It doesnt seem fair that some people can lose weight so easily. c.This disease affects the respiratory system and causes the body to burn more calories to supply the energy to breathe. d.You need to discuss this with the health care provider so testing can be ordered for tuberculosis.

c.This disease affects the respiratory system and causes the body to burn more calories to supply the energy to breathe. A patient with chronic lung disease usually requires a diet higher in calories because of the increased work of breathing. Inadequate nutrition occurs when nutritional intake does not meet nutritional needs. Without essential nutrients, a patient may experience respiratory muscle wasting, resulting in decreased muscle strength and respiratory excursion. One symptom of tuberculosis is rapid weight loss, which is not appropriate based on the question. The effort of breathing, not metabolic rate, is the reason for the weight loss. Saying that it doesnt seem fair does not address the family members concern.

The registered nurse from the home health agency is performing an initial assessment on a 72-year-old patient who was released from a nursing home. The patient had been admitted to the nursing home for therapy after surgery for repair of a fractured left hip. During a survey of the home environment, which finding would cause the nurse to intervene? a.Bedside lamp plugged into the wall outlet behind the bed b.Handrail on one side of the stairs only c.Throw rugs in the bedroom d.No handrail near the toilet

c.Throw rugs in the bedroom Common physical hazards that lead to falls in the home include inadequate lighting, barriers along normal walking paths and stairways, and a lack of safety devices. All other answers do not indicate a safety risk to the home.

The mother of a toddler is concerned that her son is not eating enough, although he has not lost any weight. She tells the nurse that her son used to have a very good appetite, but now does not eat as much as he did a couple of months ago. What is the best response for the nurse to provide? a.You need to make him eat. At this stage, he is growing too fast to not eat. b.I could show you a growth chart, but each child is different so it doesnt mean much. c.Toddlers have periods when they arent growing as fast and they dont need to eat as much. d.Make him eat with a spoon and dont feed him snacks. He will be hungrier at meal time.

c.Toddlers have periods when they arent growing as fast and they dont need to eat as much. Slower growth rates often occur with a decrease in caloric needs and a smaller food intake. Confirming the childs pattern of growth with standard growth charts is reassuring to parents concerned about their toddlers decreased appetite (physiological anorexia). Encourage parents to offer a variety of nutritious foods, in reasonable servings, for mealtime and snacks. Finger foods allow the toddler to be independent.

Of the following, who is most at risk for accidental poisoning? a.Supervised 16-month-old toddler eating dry cereal in the highchair b.Unsupervised 2-month-old infant left near a closed bottle of prescription medication c.Unsupervised 4-year-old child playing dress-up with mothers makeup d.Supervised 6-year-old child playing with watercolor paints

c.Unsupervised 4-year-old child playing dress-up with mothers makeup In the home, accidental poisoning is a greater risk for the toddler, preschooler, and young school-age child, who often ingest household cleaning solutions, medications, or personal hygiene products. Two of the responses have the word supervised in the response, which makes them incorrect for an accidental poisoning. The response which has the childs age of 2 months decreases the chance of accidental poisoning due to lack of coordination and dexterity (a 2-month-old cannot open a closed medication bottle).

A 45-year-old patient was diagnosed with an anterior myocardial infarction. The patient asks the nurse why his chest hurt when he had his heart attack. What is the best response from the nurse? a.One of your heart valves wasnt working properly and caused an obstructed blood flow. b.One of your coronary arteries had a spasm, and your heart muscle wasnt able to get enough blood. c.Your heart muscle was deprived of oxygen, which caused chest pain. d.The heart muscle is sensitive to changes in electrical conduction.

c.Your heart muscle was deprived of oxygen, which caused chest pain. When decreased myocardial blood perfusion is extensive or perfusion is completely blocked, the tissue becomes necrotic and a myocardial infarction occurs. Angina or angina pectoris is the result of decreased blood flow to the myocardium as a result of coronary artery spasms or temporary constriction. When stenosis occurs in the aortic and pulmonic valves, the adjacent ventricles work harder to move the ventricular volume beyond the stenotic valve. When regurgitation occurs, there is a backflow of blood into an adjacent chamber, which causes either pulmonary or systemic congestion. A dysrhythmia is a disturbance in the electrical impulse of the heart rhythm. Any rhythm not generated at the sinoatrial node is classified as such. Dysrhythmias are primary conduction disturbances that occur as a response to ischemia, valvular abnormalities, anxiety, and drug toxicity (e.g., digoxin toxicity).

A 16-year-old mother and her newborn come into the clinic for a routine checkup. The mother is concerned that her baby could be deaf because her uncle lost his hearing at a young age. The nurse hits a buzzer and the baby turns toward the sound. The nurse assures the mother that the baby can hear because the baby: a.was discharged from the hospital without any known problems. b.is producing ear wax. c.responds to loud noises. d.is too long young to determine any type of hearing loss.

c.responds to loud noises. Neonates without hearing impairments respond to loud noises. Atrophy of the cerumen glands, seen mainly in older adults, cause thicker and dryer wax, which is more difficult to remove and may completely obstruct the auditory canal. Hearing loss can be determined at any age with additional testing by an EENT specialist.

A patient states, "I had a bad nightmare. When I woke up, I felt emotionally drained, as though I hadn't rested well." Which response by the nurse would be an example of interpersonal therapeutic communication? a. "It sounds as though you were uncomfortable with the content of your dream." b. "I understand what you're saying. Bad dreams leave me feeling tired, too." c. "So, all in all, you feel as though you had a rather poor night's sleep?" d. "Can you give me an example of what you mean by a 'bad nightmare'?"

d. "Can you give me an example of what you mean by a 'bad nightmare'?" The technique of clarification is therapeutic and helps the nurse examine meaning. The distracters focus on patient feelings but fail to clarify the meaning of the patient's comment.

The nurse has telephone messages from four patients who requested information and assistance. Which one should the nurse refer to a social worker or community agency first? a. "Is there a place that I can dispose of my unused morphine pills?" b. "I want to lose at least 20 pounds without getting sick this time." c. "I think I have asthma because I cough when dogs are near." d. "I ran out of money and am cutting my insulin dose in half."

d. "I ran out of money and am cutting my insulin dose in half." Decreasing an insulin dose by half creates high risk of diabetic ketoacidosis, and this patient has the highest priority. The other patients have less priority due to lower risk situations with longer time course before development of an acid-base imbalance. The coughing when dogs are near is not a sign of a severe asthma episode that causes respiratory acidosis, although this patient does need attention after the insulin situation is handled. Disposing of morphine properly helps prevent respiratory acidosis from opioid overdose. Guidance regarding weight loss helps prevent starvation ketoacidosis.

Which statement by the patient indicates that additional teaching is needed about the application of an elastic bandage to the ankle? a. "I will take the bandage off if my toes start to tingle." b. "I need to make sure the bandage is applied smoothly." c. "I need to watch my toes for swelling and feeling cold." d. "I will to wrap the bandage from my shin toward my toes."

d. "I will to wrap the bandage from my shin toward my toes." The bandage should be wrapped from the toes up to the shin. The toes must be observed for swelling and the bandage must be applied smoothly. The bandage should be removed if the toes start to tingle.

The nurse is admitting a patient to the medical/surgical unit. Which communication technique would be considered appropriate for this interaction? a. "I've also had traumatic life experiences. Maybe it would help if I told you about them." b. "Why do you think you had so much difficulty adjusting to this change in your life?" c. "You will feel better after getting accustomed to how this unit operates." d. "I'd like to sit with you for a while to help you get comfortable talking to me."

d. "I'd like to sit with you for a while to help you get comfortable talking to me." Because the patient is newly admitted to the unit, allowing the patient to become comfortable with the setting is a technique that can assist in establishing the nurse-patient relationship. It helps build trust and conveys that the nurse cares about the patient. The nurse should not reveal their life experiences as this is not therapeutic. Asking why the patient is having difficulty may provide insight; however, this would be best saved for an established relationship with the patient. Assuring the patient that they will feel better may not be true depending on the reason for the admission.

A patient asks the nurse what the purpose of the Wood light is. Which response by the nurse is accurate? a. "We will put an anesthetic on your skin to prevent pain." b. "The lamp can help detect skin cancers." c. "Some patients feel a pressure-like sensation." d. "It is used to identify the presence of infectious organisms and proteins associated with specific skin conditions."

d. "It is used to identify the presence of infectious organisms and proteins associated with specific skin conditions." The Wood light examination is the use of a black light and darkened room to assist with physical examination of the skin. The examination does not cause discomfort.

The parent is concerned because the child has been referred to an optometrist after a routine eye screening at school. What is the nurse's most appropriate response to the parent? a. "Most children have a mild form of color blindness as their eyes mature." b. "You should wash the child's eyelids every morning with a damp washcloth." c. "It is normal for children to squint to see but it should be checked out anyway." d. "Most likely your child will need glasses to see the teacher and board at school."

d. "Most likely your child will need glasses to see the teacher and board at school." The most common visual problem during childhood is a refractive error such as nearsightedness. The parent can be told that the child may likely require glasses in order to see clearly over far distances. Color blindness is not common in childhood and only affects males. Squinting is not normal at any age. Washing the child's eyelids every morning will not affect the child's need for corrective lenses.

Which statement indicates the patient's world view? a. "My religion prohibits me from eating meat or drinking alcohol." b. "My family has a strong history of hypertension and type 2 diabetes." c. "A doctoral degree will allow me to advance my professional career." d. "The government is not interested in caring for immigrants to the United States"

d. "The government is not interested in caring for immigrants to the United States" The patient's world view includes how others are perceived and their beliefs about life around them including opinions about the federal government. Personal religious beliefs, family medical history, and educational plans for career advancement are not examples of world views.

Which statement by the nurse is an example of an SBAR recommendation? a. "The patient has become increasingly short of breath over the last few hours." b. "The patient has a history of chronic respiratory failure due to emphysema." c. "The patient's pulse oximetry is 84% and crackles are heard over all lung fields." d. "The patient needs oxygen titrated to maintain oximetry between 90% and 92%."

d. "The patient needs oxygen titrated to maintain oximetry between 90% and 92%." The SBAR recommendation statement is what the nurse believes should happen next for the patient to address the crucial situation at hand. The nurse recommends that the patient receive oxygen to maintain pulse oximetry between 90% and 92%. The SBAR situation statement is the increasing shortness of breath. The SBAR background statement is the patient's history of respiratory failure. The SBAR assessment statement is the current pulse oximetry and lung sounds.

The patient presents with fever for the last few days, sore throat, and enlarged lymph nodes under the jaw. What is the appropriate response of the nurse when the patient asks about the cause of the enlarged lymph nodes? a. "Enlarged lymph nodes are associated with hypertensive heart disease." b. "You probably have a blockage in the thoracic lymph duct." c. "You should probably see an oncologist to rule out lymphedema." d. "They are most likely enlarged as a result of the throat infection."

d. "They are most likely enlarged as a result of the throat infection." Lymph nodes that are large, fixed, inflamed, or tender indicate a problem such as local infection, systemic disease, or neoplasm. Enlarged lymph nodes are not normal. Normally lymph nodes are not easily palpable. An extensive system of lymph nodes collects lymph from the head, ears, nose, cheeks, and lips. The thoracic lymph duct drains lymph from the torso and extremities so a blockage there would not cause enlargement of lymph nodes under the jaw. Lymphedema would be readily apparent with swelling of the affected area. There is no mention of edema and an oncologist is not needed to rule it out. Enlarged lymph nodes are not associated with hypertensive heart disease.

The mother of a small infant is surprised when the clinic scale indicates a 1-pound weight gain from the scale used that morning at home. What is the appropriate response of the nurse? a. "Babies have significantly different weights throughout the day." b. "Variations occur because of the weight of the baby's clothing." c. "Weight variation of 1 to 2 pounds is common for most scales." d. "Weight measurements can vary with different scales."

d. "Weight measurements can vary with different scales." Different scales can give different weights for patients. To ensure accurate clinical decisions, weigh infants at the same time of day, on the same scale, and without a diaper. The scale can measure in weight increments to the nearest 0.1 pound or 0.1 kg. Weight variation of 1 to 2 pounds for a scale is unacceptable

Which question will the nurse ask the patient to assess abstract thinking ability? a. "Can you tell me the color of the blanket on your bed?" b. "Can you tell me what you ate for breakfast this morning?" c. "Can you tell me about what it was like to grow up in your neighborhood?" d. "What do I mean when I say that something costs an arm and a leg?"

d. "What do I mean when I say that something costs an arm and a leg?" Abstract thinking ability is tested by asking the patient to interpret common sayings or idioms. Memory is tested by asking the patient what was eaten for breakfast that morning and what it was like growing up. Asking the patient to identify the color of the blanket assesses orientation and speech.

A patient who survived a near-death experience tells the nurse about feeling a deep sense of peace, light, and unconditional love while watching the health care providers perform resuscitation procedures. Which is the best response of the nurse? a. "Your experiences were probably due to side effects of the medications." b. "I will ask the doctor for a psychiatric evaluation since you are hallucinating." c. "It sounds like you are fearful of suffering associated with the dying process." d. "Your experience is similar to others who have survived near-death experiences."

d. "Your experience is similar to others who have survived near-death experiences." Patients who survive near-death experiences relate comparable experiences during resuscitation efforts. The nurse should reassure the patient that similar to those reported by others who have survived near-death experiences. The patient's experiences were not hallucinations or due to side effects of medications. The patient does not show any evidence of fearing suffering associated with the dying process.

The nurse is caring for a patient with a puncture wound. How much time must have passed since the patient's last tetanus toxoid vaccination for the patient to require an additional injection before being discharged from the emergency department? a. 1 year b. 3 years c. 5 years d. 10 years

d. 10 years When an injury results from trauma from a dirty penetrating object, determine if the patient has received a tetanus toxoid injection within the last 10 years. Within the past year, 3 years, or 5 years is too early.

A 3-month-old infant is at increased risk for developing anemia. The nurse would identify which principle contributing to this risk? a. The infant is becoming more active. b. There is an increase in intake of breast milk or formula. c. The infant is unable to maintain an adequate iron intake. d. A depletion of fetal hemoglobin occurs.

d. A depletion of fetal hemoglobin occurs. Fetal hemoglobin is present for about 5 months. The fetal hemoglobin begins deteriorating, and around 2-3 months the infant is at increased risk of developing an anemia due to decreasing levels of hemoglobin. Breast milk or formula is the primary food intake up to around 6 months. Often iron supplemented formula is offered, and/or an iron supplement is given if the infant is breastfed

Which patient would benefit from soaking in a sitz bath? a. A patient with an abscessed tooth b. A patient with a fractured right arm c. A patient with painful back muscle spasms d. A patient who just had hemorrhoid surgery

d. A patient who just had hemorrhoid surgery A patient who just had hemorrhoid surgery would benefit from a sitz bath as the perineal area is immersed in warm fluid. A sitz bath would not be appropriate for a fractured arm, abscessed tooth, or back spasms.

Which patient should receive an IV solution of D5W? a. A trauma patient with massive hemorrhage b. A patient being resuscitated after cardiac arrest c. A patient with CHF and acute pulmonary edema d. A patient with a serum sodium level of 162 mEq/L

d. A patient with a serum sodium level of 162 mEq/L D5W solution is used to lower elevated serum sodium levels. Normal saline solution should be administered to hypovolemic patients and during resuscitation after cardiac arrest. D5W solution will worsen fluid overload with CHF and acute pulmonary edema.

Which action of the nurse demonstrates culturally competent care? a. Helping a Hindu patient to eat meatloaf and gravy for dinner b. Cutting up ham and vegetables for an orthodox Jewish patient c. Bringing a grieving Mormon family a tray of coffee and snacks d. Adding fruit to a Buddhist patient's bowl of oatmeal for breakfast

d. Adding fruit to a Buddhist patient's bowl of oatmeal for breakfast Buddhist patients are often vegetarian so oatmeal and fruit would be an appropriate meal. Mormon families do not believe in consumption of alcohol or caffeine. Orthodox Jewish patients do not eat pork. Hindu patients are often vegetarian

A sentinel event refers to which situation? a. An event that could have harmed a patient, but serious harm didn't occur because of chance. b. An event that harms a patient as a result of underlying disease or condition. c. An event that harms a patient by omission or commission, not an underlying disease or condition. d. An event that signals the need for immediate investigation and response.

d. An event that signals the need for immediate investigation and response. A sentinel event is an unexpected occurrence involving death or serious physical or psychological injury or the risk thereof called sentinel, because it signals the need for immediate investigation and response. A near-miss refers to an error or commission or omission that could have harmed the patient, but serious harm did not occur as a result of chance. Harm that relates to an underlying disease or condition provides the rationale for the close monitoring and supervision provided in a healthcare setting. An adverse event is one that results in unintended harm because of the commission or omission of an act.

The nurse is caring for a patient who just underwent knee-replacement surgery. The patient complains of pain at the operative site but it is too soon for the nurse to administer the next dose of pain medication. What is the appropriate action of the nurse? a. Give the next dose of prescribed pain medication early. b. Contact the surgeon immediately to assess the patient's knee. c. Caution the patient about the risk of addiction to pain medications. d. Apply an ice pack to the knee and elevate the patient's knee on pillows.

d. Apply an ice pack to the knee and elevate the patient's knee on pillows. Nonpharmaceutical methods are often effective for relieving musculoskeletal pain. Elevation of the joint and ice should be applied before contacting the surgeon. The next dose of pain medication should not be given early without the physician's approval.

To help decrease the threat of a melanoma in a blonde-haired, fair-skinned patient at risk, the nurse would advise the patient to do which of the following? a. Apply sunscreen 1 hour prior to exposure. b. Drink plenty of water to prevent hot skin. c. Use vitamins to help prevent sunburn by replacing lost nutrients. d. Apply sunscreen 30 minutes prior to exposure.

d. Apply sunscreen 30 minutes prior to exposure. Wearing sunglasses and sunscreen are recommended by the National Cancer Institute. Drinking water will help with heat exhaustion but will not prevent melanoma. Green tea, fish oil, soy products, and vitamin E are thought to be helpful in minimizing the risk of developing melanoma; however, vitamins do not prevent burn

Which action by the nurse is appropriate for a family with a terminally ill newborn? a. Call the rabbi to come to the bedside. b. Arrange for the infant to be baptized immediately. c. Call the hospital chaplain to pray with the family. d. Ask the family how their spiritual needs can be supported.

d. Ask the family how their spiritual needs can be supported. Nurses need to differentiate their personal spiritual beliefs from those of the patients and their families. The nurse's role is not to solve the spiritual problems of patients, but to provide an environment for your patients to express their spirituality. Having the baby baptized or calling the rabbi indicates that the nurse is applying personal spiritual values on the patient and family. Calling the chaplain assumes that the patient and family value a religious denomination.

Which is the first action of the nurse when teaching the patient how to perform colostomy care? a. Determine the patient's educational background and learning abilities. b. Identify a responsible family member to reinforce colostomy care teaching. c. Have the patient watch a video that demonstrates how to perform colostomy care. d. Assess the patient's level of comfort with looking at and caring for the colostomy.

d. Assess the patient's level of comfort with looking at and caring for the colostomy. The first action of the nurse is to determine the patient's readiness to learn about colostomy care. The nurse should assess the patient's level of comfort with looking at and caring for the colostomy before initiating any teaching. Identifying a family member to assist, determining the patient's educational background, and having the patient watch a video should all be done after assessing the patient's readiness to learn.

A patient with a Foley catheter needs a urine sample for culture and sensitivity. What is the appropriate action for the nurse to take? a. Disconnect the drainage tube from the catheter. b. Withdraw urine from the closed system drainage bag. c. Empty contents of the drainage bag into the specimen cup. d. Attach a sterile syringe to the catheter port to withdraw urine.

d. Attach a sterile syringe to the catheter port to withdraw urine. The nurse should attach a sterile syringe to the catheter port to withdraw urine. Separating the drainage tube from the catheter breaches sterile technique and can cause introduction of bacteria into the system. Because urine in the drainage bag can rapidly grow bacteria, never take a urine sample from the drainage bag

A patient's serum electrolytes are being monitored. The nurse notices that the potassium level is low. What should the nurse monitor for in this patient? a. Tissue ischemia b. Brain malformations c. Intestinal blockage d. Cardiac dysrhythmia

d. Cardiac dysrhythmia Cardiac dysthymia is a possibility when serum potassium is high or low. Tissue ischemia, brain malformations, or intestinal blockage do not have a direct correlation to potassium irregularities.

The nurse assesses distended neck veins in a patient sitting in a chair to eat. Which is the priority intervention of the nurse? a. Document the observation in the chart. b. Assess the patient's deep tendon reflexes. c. Measure urine specific gravity and volume. d. Check the patient's pulse and blood pressure.

d. Check the patient's pulse and blood pressure. Distended neck veins indicate venous congestion in the systemic circulation and possibly right-sided heart failure. The nurse's priority action is to check the patient's pulse and blood pressure. Distended neck veins may be documented in the chart but that is not the priority action of the nurse. There is no need to check specific gravity or deep tendon reflexes

The nurse is caring for a dying patient in hospice. The patient's respirations are slow and uneven with deep breaths and long periods of apnea. Which term best describes this breathing pattern? a. Rhonchal bradypnea b. Forrest-Shiley breaths c. Kussmaul's respirations d. Cheyne-Stokes breathing

d. Cheyne-Stokes breathing Cheyne-Stokes respirations have uneven with deep breaths and long periods of apnea. Kussmaul respirations are deep and rapid to blow off excess carbon dioxide with metabolic acidosis. Rhonchal bradypnea and Forrest-Shiley breaths are not accepted terms.

The nurse is caring for a patient with an ileostomy. Which assessment finding is expected when emptying the patient's ostomy appliance? a. Infrequent hard pellets of stool b. Daily soft formed stool c. Frequent unformed stool d. Constant watery liquid stool

d. Constant watery liquid stool An ileostomy bypasses the entire large intestine creating frequent, liquid stools. Frequent unformed stool would be expected after removal of some of the large intestine. The patient would have a soft, formed stool after sigmoid colostomy. Infrequent hard pellets of stool is not be expected in a patient with an ostomy.

A small amount of mercury was spilled on the floor after an old sphygmomanometer was broken. What is the priority action of the nurse? a. Disinfect the area with a solution of chlorine bleach. b. Contact the housekeeping staff to mop up the liquid. c. Wipe up the liquid using paper towels and nitrile gloves. d. Consult the agency's materials safety data sheets (MSDS).

d. Consult the agency's materials safety data sheets (MSDS). Mercury is a toxic chemical so the nurse should consult the MSDS to determine how the spill should be cleaned. Mercury cannot be mopped up by a housekeeper or wiped with paper towels. The area should not be disinfected with chlorine bleach.

The home care nurse is caring for a patient with an indwelling urinary catheter after spinal cord injury. The catheter is patent with clear yellow urine after being in place for 8 weeks. Which is the appropriate action of the nurse? a. Request an order for a urinalysis with culture and sensitivity. b. Irrigate the patient's catheter using 60 mL of sterile normal saline. c. Remove the catheter immediately and notify the health care provider. d. Contact the health care provider for an order to change the catheter.

d. Contact the health care provider for an order to change the catheter. Indwelling urinary catheters should be changed every 4 to 6 weeks. The nurse should contact the health care provider for an order to change the catheter after it has been in place for 8 weeks. Removal of the catheter is not an option for the patient with a spinal cord injury as the damage to the nerves is permanent. Irrigation and urinalysis are not needed.

The patient takes morphine sulfate controlled release every 12 hours for chronic severe cancer pain. The last dose was 8 hours ago. The patient is presently moaning and states that the pain is very bad. Which is the best action of the nurse? a. Give the next scheduled dose of morphine sulfate controlled release now. b. Wait for the last dose of morphine sulfate controlled release to start taking effect. c. Carefully reposition the patient and reassess the pain in 1 hour. d. Contact the physician for a breakthrough pain medication order.

d. Contact the physician for a breakthrough pain medication order. Morphine sulfate controlled release is an extended-release medication that provides pain relief for 8 to 12 hours. The patient is experiencing breakthrough pain and needs a dose of short-acting pain medication right away to relieve the immediate discomfort. The next dose of morphine sulfate controlled release should not be administered early as it is not ordered and will take many hours to take effect. The patient's pain is so bad that repositioning and waiting an hour is inappropriate

What interrelated constructs facilitate a nurse to become culturally competent? a. Cultural diversity, self-awareness, cultural skill, and cultural knowledge b. Cultural desire, self-awareness, cultural knowledge, and cultural identity c. Cultural desire, self-awareness, cultural knowledge, and cultural diversity d. Cultural desire, self-awareness, cultural knowledge, and cultural skill

d. Cultural desire, self-awareness, cultural knowledge, and cultural skill The process of cultural competence consists of four interrelated constructs: cultural desire, self-awareness, cultural knowledge, and cultural skill. Cultural diversity in the context of health care refers to achieving the highest level of health care for all people by addressing societal inequalities and historical and contemporary injustices. Cultural identity is the norms, values, beliefs, and behaviors of a culture learned through families and group members.

The patient is receiving tube feedings due to a jaw surgery. What change in assessment findings should prompt the nurse to request an order for serum sodium concentration? a. Development of ankle or sacral edema b. Increased skin tenting and dry mouth c. Postural hypotension and tachycardia d. Decreased level of consciousness

d. Decreased level of consciousness Tube feedings pose a risk for hypernatremia unless adequate water is administered between tube feedings. Hypernatremia causes the level of consciousness to decrease. The serum sodium concentration is a laboratory measure for osmolality imbalances, not ECV imbalances. Edema is a sign of ECV excess, not hypernatremia. Skin tenting, dry mouth, postural hypotension, and tachycardia all can be signs of ECV deficit.

Which is the best method to begin teaching the adult patient how to self-administer tube feeding through a new gastrostomy tube? a. Analogies b. Detachment c. Role play d. Demonstration

d. Demonstration The nurse should begin to teach the patient by demonstrating how to administer tube feedings. The patient is then encouraged to assist until a return demonstration of the skill can be performed. Detachment is not a teaching approach. Role playing and analogies are not appropriate for teaching tube feeding administration

A young child learns how to use scissors and catch a ball. Which term best describes these developments? a. Puberty b. Growth c. Adolescence d. Development

d. Development Development is the gradual increase in cognitive and motor abilities over time, such as the ability to use scissors or catch a ball. Growth is an increase in the individual's physical size. Puberty is the developmental point when reproduction is possible. Adolescence is the time period that occurs between puberty and adulthood.

Which is an example of a primary sexual characteristic? a. Enlarged larynx and deeper voice b. Enlargement of female breast tissue c. Growth of hair on the face and chest d. Development of the penis and testicles

d. Development of the penis and testicles Primary sexual characteristics include the presence of a penis and testicles rather than a clitoris and vagina. Enlarged larynx, facial hair, and enlargement of female breast tissue are all secondary sexual characteristics.

The adolescent with a BMI of 18.5 feels overweight after seeing excessively slender actresses on TV and in the movies. Which nursing diagnosis is appropriate for this adolescent? a. Decisional conflict related to how much weight should be lost to look pretty b. Defensive coping related to denial of imbalanced weight to height proportion c. Powerlessness related to perception that weight loss is needed to gain friends d. Disturbed body image related to cultural perception of ideal body proportions

d. Disturbed body image related to cultural perception of ideal body proportions The adolescent with a BMI of 18.5 is at a healthy weight for height. The adolescent's feelings of overweight are due to a disturbed body image brought on by seeing the excessively slender actresses. The adolescent's weight is appropriate for height so there is no denial or defensive coping. The adolescent does not need to lose weight to gain friends and there is no decisional conflict about how much weight should be lost.

Which teaching will the nurse provide to the patient before having an intravenous pyelogram (IVP)? a. Drink water and do not void so the bladder will be full during the test. b. An urge to void may be felt as the endoscope passes through the urethra. c. The urine may have an orange or pink for a day or two following the test. d. Drink plenty of water afterward to prevent kidney damage from the contrast dye.

d. Drink plenty of water afterward to prevent kidney damage from the contrast dye. Contrast dye used for the IVP can be harmful to the kidneys so the patient should drink lots of water afterward to increase urine output. The bladder should not be full for the IVP test and an endoscope is not used. No urine discoloration is expected after IVP testing.

The nurse is caring for a patient with a colostomy. The nursing diagnosis risk for impaired skin integrity related to leakage of effluent from appliance is included in the patient's care plan. Which is the appropriate intervention of the nurse? a. Apply antifungal cream to the skin before attaching the ostomy appliance. b. Liberally apply a rich skin barrier cream to the skin surrounding the stoma. c. Measure the width and the length of the stoma each week for the first 6 weeks. d. Empty the effluent into the toilet before the ostomy appliance becomes half full.

d. Empty the effluent into the toilet before the ostomy appliance becomes half full. The ostomy appliance is liable to leak if it becomes more than half full. Leakage of stool from the ostomy appliance puts the patient's skin at risk of breakdown. The nurse can minimize the risk of skin breakdown by emptying the effluent into the toilet before the ostomy appliance becomes half full. Application of creams to the skin under the appliance will prevent adhesion and lead to leakage around the stoma. Measurement of the stoma will not help to prevent skin breakdown

. To promote safety, the nurse manager sensitive to point of care (sharp end) and systems level (blunt end) exemplars works closely with staff to address which point of care exemplar? a. Care coordination b. Documentation c. Electronic records d. Fall prevention

d. Fall prevention The most common safety issues at the sharp end include prevention of decubitus ulcers, medication administration, fall prevention, invasive procedures, diagnostic workup, recognition of/action on adverse events, and communication. These are the most common issues the staff nurse providing direct patient care encounters. Each of the other options are classified as systems level exemplars.

How will the nurse obtain a culture of the patient's wound? a. Obtain a sample from the patient's wound drainage bag. b. Obtain a sample of the drainage around the edge of the wound. c. Obtain a sample of the drainage from the dressing on the wound. d. Gently swab the center of the wound after irrigating with sterile saline.

d. Gently swab the center of the wound after irrigating with sterile saline The nurse should gently swab the center of the wound after irrigating with sterile saline in order to obtain an accurate culture. Samples should never be taken from the edge of the wound, from the dressing, or from the wound drainage bag.

The nurse is reviewing a patient's functional ability. Which patient best demonstrates the definition of functional ability? a. Considers self as a healthy individual; uses cane for stability b. College educated; travels frequently; can balance a checkbook c. Works out daily, reads well, cooks, and cleans house on the weekends d. Healthy individual, volunteers at church, works part time, takes care of family and house

d. Healthy individual, volunteers at church, works part time, takes care of family and house Functional ability refers to the individual's ability to perform the normal daily activities required to meet basic needs; fulfill usual roles in the family, workplace, and community; and maintain health and well-being. The other options are good; however, healthy individual, church volunteer, part time worker, and the patient who takes care of the family and house fully meets the criteria for functional ability.

The nurse is caring for a patient with painful hemorrhoids. Which is the appropriate recommendation of the nurse to prevent their recurrence? a. Stool softener daily at bedtime b. Low-carbohydrate ketogenic diet c. Periodic bowel cleansing programs d. High-fiber diet with plenty of liquids

d. High-fiber diet with plenty of liquids Hemorrhoids are caused and aggravated by straining for defecation with constipation, pregnancy, or chronic illness. A high-fiber diet with plenty of liquids will keep stools soft and prevent straining for defecation. Low-carbohydrate ketogenic diets are often low in fiber which can lead to constipation. Bowel cleansing programs and daily stool softeners are not recommended, as they can lead to chronic constipation, and fluid-electrolyte imbalances.

The nurse is caring for a patient who develops slurred speech with right-sided facial drooping. The patient is also unable to make a fist with the right hand. What is the priority action of the nurse? a. Place a cool washcloth on the patient's forehead and turn down the lights. b. Obtain an order from the provider for a consultation with a speech therapist. c. Feed the patient by placing the food on the unaffected side of the mouth. d. Immediately obtain vital signs and notify the hospital's acute stroke team.

d. Immediately obtain vital signs and notify the hospital's acute stroke team. Slurred speech with right-sided facial drooping and weakness on one side of the body are indicative of an acute stroke. The nurse should immediately obtain the patient's vital signs and notify the hospital's acute stroke team. Any delay in diagnosis or treatment can cause stroke symptoms to worsen or become permanent. Consultation with speech therapy, feeding techniques, and comfort measures can be initiated once the patient has been evaluated and cleared by the stroke team.

The nurse is caring for a patient with nonblanchable redness of the coccyx after a lengthy surgical procedure in the supine position. Which nursing diagnosis is appropriate for this patient? a. Risk for impaired skin integrity related to lengthy surgical procedure b. Noncompliance related to failure to frequently reposition self c. Ineffective therapeutic regimen management related to improper positioning d. Impaired skin integrity related to tissue pressure from prolonged supine position

d. Impaired skin integrity related to tissue pressure from prolonged supine position The patient has impaired skin integrity related to the reddened area on the coccyx. The patient was not noncompliant as immobility was required for the surgical procedure. Ineffective therapeutic regimen management refers to daily compliance with the prescribed treatment regimen.

The family requests insertion of a Foley catheter to address the elderly patient's frequent episodes of incontinence. Which is the best action of the nurse? a. Obtain an order for an indwelling urinary catheter. b. Teach family to perform intermittent straight catheterization. c. Utilize disposable absorbent undergarments for the patient. d. Implement a bladder training program to promote continence.

d. Implement a bladder training program to promote continence. The nurse can act as a patient advocate by suggesting noninvasive alternatives to catheterization use. Bladder training programs can be used to promote continence for children and adults. Catheterization should be avoided whenever possible due to the risk of urinary infection. Disposable undergarments should be used as a last resort if bladder training is unsuccessful.

The patient's laboratory report today indicates severe hypokalemia, and the nurse has notified the physician. Nursing assessment indicates that heart rhythm is regular. What is the priority important nursing intervention? a. Raise bed side rails due to potential decreased level of consciousness and confusion. b. Examine sacral area and patient's heels for skin breakdown due to potential edema. c. Establish seizure precautions due to potential muscle twitching, cramps, and seizures. d. Institute fall precautions due to potential postural hypotension and weak leg muscles.

d. Institute fall precautions due to potential postural hypotension and weak leg muscles. Hypokalemia can cause postural hypotension and bilateral muscle weakness, especially in the lower extremities. Both of these increase the risk of falls. Hypokalemia does not cause edema, decreased level of consciousness, or seizures.

A nurse gives a hand-off report to the oncoming staff nurse. Which type of communication does this illustrate? a. Gossip b. Courtesy c. Validation d. Intrapersonal

d. Intrapersonal Interpersonal communication is interaction that occurs between two people or within a small group. Gossiping violates confidentiality. The act of validation requires comparing data with another source. Professional courtesy conveys respect between colleagues. The nurse should always utilize professional courtesy when providing report to the oncoming shift.

A nurse administers an incorrect medication to a patient. In reviewing this medication error, the nurse finds out that incorrect medication was placed in the Pyxis system. What type of error has the nurse committed? a. Latent error b. Blunt end c. Did not follow nursing process d. Latent error resulting in active error

d. Latent error resulting in active error The situation described is a latent error which resulted in an active error as incorrect medication was placed in the Pyxis system. Latent errors are also referred to as blunt end whereas active errors are applied as occurring at the sharp end. There is no provided information to suggest that the nurse did not follow nursing process

Although the patient can see movement in the periphery, the patient can no longer see to read books or do crossword puzzles. Which is the most likely cause of the patient's vision loss? a. Cataracts b. Glaucoma c. Diabetic retinopathy d. Macular degeneration

d. Macular degeneration Age-related macular degeneration occurs when the macula (specialized portion of the retina responsible for central vision) degenerates as a result of aging and loses its ability to function efficiently. An early sign includes distortion that causes edges or lines to appear wavy. In later stages, patients may see dark or empty spaces that block the center of vision. Cataract is clouding of the lens in the eye that affects vision. Interferes with passage of light through the lens and reduces the light that reaches the retina. Cataracts usually develop gradually and often result in cloudy or blurry vision, glare, double vision, and poor night vision. Glaucoma is a slowly progressive increase in intraocular pressure that causes progressive pressure against the optic nerve. At first, vision stays normal, and there is no pain. If left untreated, there may be a loss of peripheral (side vision). Diabetic retinopathy are pathological changes of the blood vessels of the retina secondary to increased pressure resulting in hemorrhage, macular edema, and reduced vision or vision loss.

The nurse is developing an interdisciplinary plan of care using the Roper-Logan-Tierney Model of Nursing for a patient who is currently unconscious. Which interventions would be most critical to developing a plan of care for this patient? a. Eating and drinking, personal cleansing and dressing, working and playing b. Toileting, transferring, dressing, and bathing activities c. Sleeping, expressing sexuality, socializing with peers d. Maintaining a safe environment, breathing, maintaining temperature

d. Maintaining a safe environment, breathing, maintaining temperature The most critical aspects of care for an unconscious patient are safe environment, breathing, and temperature. Eating and drinking are contraindicated in unconscious patients. Toileting, transferring, dressing, and bathing activities are BADLs. Sleeping, expressing sexuality, and socializing with peers are a part of the Roper-Logan-Tierney Model of Nursing; however, these are not the most critical for developing the plan of care in an unconscious patient

A patient had surgery two days ago. Which intervention will be included in the plan of care for this patient wearing compression stockings? a. The patient's stockings should be removed at night for washing. b. Slippers should not be applied over the stockings for ambulation. c. Regular size stockings are for females and large size are for males. d. Make sure that the patient's toes are not sticking out of the stockings.

d. Make sure that the patient's toes are not sticking out of the stockings. The patient's toes must not stick out from the compression stockings as circulation will be impaired. The stockings should not be taken off at night in the immediate postoperative period. Slippers should be worn over the stockings for ambulation. Failure to do so may cause a patient to fall. The patient's legs should be measured to ensure that the appropriate size compression stockings are applied. DIF: Cognitive Level: Understand (C

The nurse is caring for a patient with bulimia. The patient vomits after eating and is experiencing tingling of the fingers and toes and muscle cramps. What is the patient's expected acid-base status? a. Respiratory acidosis b. Metabolic acidosis c. Respiratory alkalosis d. Metabolic alkalosis

d. Metabolic alkalosis ANS: D Metabolic alkalosis results from acid loss from the body or an increase in levels of bicarbonate. The most common causes are vomiting and gastric suction. Respiratory acidosis results from respiratory diseases or other conditions that reduce alveolar ventilation (hypoventilation), preventing excretion of the carbonic acid continuously produced by cells. Metabolic acidosis results from conditions that increase metabolic acids in the body or decrease the amount of base (bicarbonate). Diabetic ketoacidosis is a common cause of metabolic acidosis. Hyperventilation produces respiratory alkalosis, which causes cerebrospinal fluid and brain cells to become alkalotic, decreasing the level of consciousness

The nurse recognizes which patient as having the greatest risk for undiagnosed diabetes mellitus? a. Young white man b. Middle-aged African-American man c. Young African-American woman d. Middle-aged Native American woman

d. Middle-aged Native American woman The highest incidence of diabetes in the United States occurs in Native Americans. With age, the incidence of diabetes increases in all races and ethnic groups.

The nurse notes a reddened area on the right heel that does not turn lighter in color when pressed with a finger. Which term will the nurse use to describe this area? a. Reactive hyperemia b. Secondary erythema c. Blanchable hyperemia d. Nonblanchable erythema

d. Nonblanchable erythema Nonblanchable erythema is redness that persists after palpation and indicates tissue damage. When you press a finger against the red or purple area, it does not turn lighter in color. Deep tissue damage is present and is commonly the first stage of pressure injury development. There is no such condition as secondary erythema. Reactive hyperemia is a redness of the skin resulting from dilation of the superficial capillaries. Reactive hyperemia blanches. In blanchable hyperemia, the area that appears red and warm will blanch (turn lighter in color) following fingertip palpation.

What is a primary prevention tool used for colon cancer screening? a. Abdominal x-rays b. Blood, urea, and nitrogen (BUN) testing c. Serum electrolytes d. Occult blood testing

d. Occult blood testing Occult blood testing will reveal unseen blood in the stool, and this may signal a potentially serious bowel problem like colon cancer. BUN is used to evaluate kidney function. Serum electrolytes and abdominal x-rays are not related to colon cancer screening.

When describing patient education approaches, the nurse educator would explain that informal teaching is an approach that involves which quality? a. Addresses group needs b. Follows formalized plans c. Has standardized content d. Often occurs one-to-one

d. Often occurs one-to-one Informal teaching is individualized one-on-one teaching which represents the majority of patient education done by nurses that occurs when an intervention is explained or a question is answered. Group needs are often the focus of formal patient education courses or classes. Informal teaching does not necessarily follow a specific formalized plan. It may be planned with specific content, but it is individualized responses to patient needs. Formal teaching involves the use of a curriculum/course plan with standardized content

Which is the appropriate intervention for a patient with the nursing diagnosis wandering related to disorientation, memory loss, and urge incontinence? a. Raise three of the four side rails on the patient's bed. b. Assign the patient to a room close to the nursing station. c. Remind the patient to always ask for help before getting up. d. Place a bed alarm to notify staff when the patient is getting up.

d. Place a bed alarm to notify staff when the patient is getting up. Alarm devices warn nursing staff that a patient is attempting to leave a bed or chair unassisted. Staff can then provide assistance before the patient is fully out of the bed. Although moving the patient to a room closer to the nursing station allows the nurse to keep a closer eye on the patient, this action does not discourage wandering behavior. Raising side rails has the potential to trap parts of the patient's body, producing a hazard. The use of side rails alone for a disoriented patient often causes more confusion and further injury. Reminders are not effective for this patient due to memory loss.

The patient reports muscle weakness after taking prescribed furosemide daily. Which laboratory finding will the nurse expect to see in the patient's chart? a. Chloride 84 mEq/L b. Sodium 124 mEq/L c. Calcium 12.6 mg/dL d. Potassium 2.8 mEq/L

d. Potassium 2.8 mEq/L Loop diuretics cause hypokalemia. The nurse will expect to find the patient's potassium level to be 2.8 mEq/L as a result. The patient would not have hyponatremia (sodium 124 mEq/L), hypercalcemia (calcium 12.6 mg/dL), or hypochloremia (chloride 84 mEq/L) as a result of taking loop diuretics.

The emergency room nurse obtains report from the paramedics as the patient is on the way to the hospital. The nurse is in which phase of the therapeutic relationship? a. Working b. Orientation c. Termination d. Preinteraction

d. Preinteraction In the preinteraction stage the nurse gathers information from various sources about the patient such as from the paramedics. The orientation phase when you and the patient meet and get to know one another is the time when the contract is formed. The nurse and patient work together during the working phase to solve problems and accomplish goals. During the termination phase the helping relationship is ended.

Which clinical management prevention concept would the nurse identify as representative of secondary prevention? a. Decreasing venous stasis and risk for pulmonary emboli b. Implementation of strict hand washing routines c. Maintaining current vaccination schedules d. Prevention of pneumonia in patients with chronic lung disease

d. Prevention of pneumonia in patients with chronic lung disease Prevention of and treatment of existing health problems to avoid further complications is an example of secondary prevention. Primary prevention includes infection control (hand washing), smoking cessation, immunizations, and prevention of postoperative complications.

The nurse includes "The patient will demonstrate correct technique for self-injection of insulin" as a goal in the patient's care plan. Which type of learning is addressed by this goal? a. Cognitive b. Affective c. Perceptive d. Psychomotor

d. Psychomotor Psychomotor learning is the acquisition of motor skill such as injection of insulin. Cognitive learning is thinking in new ways. Affective learning is expression of emotions or beliefs. Perceptive means the ability to sense of show insight.

Which term will the nurse use to document the patient's drooping right eyelid? a. Ectropion b. Esotropia c. Photophobia d. Ptosis

d. Ptosis An abnormal drooping of the lid over the pupil is called ptosis, caused by edema or impairment of the third cranial nerve. In the older adult, ptosis results from a loss of elasticity that accompanies aging. An older adult frequently has lid margins that turn out (ectropion) or in (entropion). Esotropia refers to inward turn of the eye. Photophobia refers to light sensitivity.

When developing a plan of care, the nurse should consider which attribute of the concept of spirituality? a. Spirituality is not a well-known universal concept. b. Chronic versus acute illnesses affect spirituality. c. Convincing patients to pray is a priority intervention. d. Referrals may be needed to spiritual counselors.

d. Referrals may be needed to spiritual counselors. Spirituality is universal. All individuals, even those who profess no religious belief, are driven to derive meaning and purpose from life. • Illness impacts spirituality in a variety of ways. Some patients and families will draw closer to God or however they conceive that Higher Power to be in an effort to seek support, healing, and comfort. Others may blame and feel anger toward that Higher Power for any illness and misfortune that may have befallen a loved one or their entire family. Still others will be neutral in their spiritual reactions. • There has to be willingness on the part of patient and/or family to share and/or act on spiritual beliefs and practices. • The nurse needs to be aware that specific spiritual beliefs and practices are impacted by family and culture. • The nurse needs to be willing to review the concept of spirituality in patients and families and based on this ongoing assessment to integrate the spiritual beliefs of patients and families into care. • The nurse needs to be willing to refer the patient or family to a Spiritual Expert i.e. a Minister, Priest, Rabbi, an Imam. • Community-based religious organizations can provide supportive care to families and patients and nurses need to be aware of these resources.

The nurse is caring for a patient a serum potassium level of 7.4 mEq/L. Which is the highest priority nursing diagnosis for this patient? a. Nausea related to side effects from medications b. Ineffective tissue perfusion related to altered mental status c. Fluid volume excess related to increased isotonic fluid retention d. Risk for decreased cardiac output related to altered heart rhythm

d. Risk for decreased cardiac output related to altered heart rhythm The patient's dangerously high potassium level causes risk of ventricular fibrillation. The highest priority nursing diagnosis for the patient is risk for decreased cardiac output related to altered heart rhythm. Nausea is not the highest priority diagnosis. Fluid volume excess is not expected with a potassium level of 7.4 mEq/L. There is no indication of the patient having an altered mental status.

The nurse is caring for a patient with a serum potassium level of 7.1 mEq/L. Which is the highest priority nursing diagnosis for this patient? a. Risk for spiritual distress related to chronic illness despair b. Powerlessness related to illness-related physical limitations c. Impaired social interaction related to limited physical activity d. Risk for decreased cardiac output related to altered heart rhythm

d. Risk for decreased cardiac output related to altered heart rhythm The patient's potassium level is dangerously high, which can cause ventricular fibrillation. For this reason the highest priority diagnosis for the patient is risk for decreased cardiac output related to altered heart rhythm. Risk for spiritual distress, powerlessness, and impaired social interaction are not the priority

Which nursing diagnosis is appropriate for a patient using CPAP therapy to treat sleep apnea? a. Readiness for enhanced sleep related to desire for restful sleep b. Disturbed body image related to use of CPAP mask for sleeping c. Risk for disuse syndrome related to discomfort from CPAP mask d. Risk for impaired skin integrity related to tight-fitting mask on face

d. Risk for impaired skin integrity related to tight-fitting mask on face The patient's facial skin may become irritated from the tight-fitting CPAP mask. This is the highest priority diagnosis as it addresses a patient safety need. Risk for disuse syndrome is inappropriate for sleep apnea as the diagnosis relates to deterioration of a body part because of inactivity. Disturbed body image and readiness for enhanced sleep are lower priority nursing diagnoses for this patient.

The nurse is caring for a patient with a sodium level of 156 mEq/L due to diabetes insipidus. Which is the highest priority nursing diagnosis for this patient? a. Activity intolerance related to fatigue from chronic illness b. Imbalanced nutrition related to excessive carbohydrate intake c. Excess fluid volume related to increased isotonic fluid retention d. Risk for ineffective airway clearance related to secretions during seizure

d. Risk for ineffective airway clearance related to secretions during seizure Diabetes insipidus is caused by a lack of antidiuretic hormone (ADH). Excessive urination causes dehydration and hypernatremia. The patient's sodium level of 156 mEq/L puts the patient at risk for seizures and risk for ineffective airway clearance related to secretions during seizure. Diabetes insipidus is not related to blood sugar levels unlike diabetes mellitus so imbalanced nutrition related to excessive carbohydrate intake is not appropriate for the patient. Excess fluid volume related to increased isotonic fluid retention is appropriate for a patient with SIADH rather than diabetes insipidus. Activity intolerance is not as important as risk for ineffective airway clearance

Which is the highest priority nursing diagnosis for a toddler who is at the autonomy versus doubt stage of development? a. Toileting self-care deficit related to readiness for transition from diapers b. Impaired verbal communication related to stage of cognitive development c. Readiness for enhanced sleep related to desire to sleep in a big kids' bed d. Risk for poisoning related to unawareness of environmental risks within reach

d. Risk for poisoning related to unawareness of environmental risks within reach Toddlers cannot understand the risks of their environment and want to put everything in their mouths. This presents a very significant risk of poisoning as toddlers are not aware when substances may be toxic or poisonous. Impaired verbal communication is expected for toddlers as they struggle to learn to speak clearly. Toddlers are ready to transition from a crib to a big kids' bed and to stop using diapers. Communication, toileting, and sleep are less of a priority than protection from poisoning.

The nurse feels a deep sense of altruism and wonderment after successfully resuscitating a young athlete who suddenly collapsed in cardiac arrest. Which term best describes the sensation experienced by the nurse? a. Holy conviction b. Spiritual distress c. Divine expectation d. Self-transcendence

d. Self-transcendence Self-transcendence refers to connecting to your inner self, which allows you to go beyond yourself to understand the meanings of experiences, whereas transcendence is the belief that there is a positive force outside of and greater than oneself that allows you to develop new perspectives that are beyond physical boundaries. Spiritual distress is suffering due to lack of connectedness with a faith system. The nurse did not experience holy conviction or divine expectation.

The nurse educates the patient about what to expect during insertion of a nasogastric tube. Which term best describes the nurse's communication role? a. Channel b. Receiver c. Message d. Sender

d. Sender The nurse is the sender in this scenario. The sender is the person who delivers the message. The message is sent to a receiver, in this case the patient. The message is the content of the conversation; in this scenario explaining what will happen is the message. The channel is the means of conveying and receiving messages through visual, auditory, and tactile senses, the nurse's spoken words in this scenario

Which factor contributes to pressure injury formation when patient's body slides downward to the foot of the bed? a. Momentum b. Acceleration c. Applied force d. Shearing force

d. Shearing force Shear is the force exerted against the skin while the skin remains stationary and the bony structures move. For example, when the head of the bed is elevated, gravity causes the bony skeleton to pull toward the foot of the bed, while the skin remains against the sheets. An applied force is the movement of one object as the result of another object acting upon it. Momentum is the force of an object as it moves forward. Acceleration is increased speed of an object as it moves.

The nurse is caring for a patient with the nursing diagnosis of disturbed sensory perception related to loud, bright hospital environment. Which is the priority intervention for the patient's care plan? a. Maintain eye contact with the patient and avoid chewing gum. b. Ask the patient to repeat information back to ensure understanding. c. Repeatedly orient the patient to time, place, and the hospital room surroundings. d. Shut the patient's door and avoid turning on the bright overhead lights in the room.

d. Shut the patient's door and avoid turning on the bright overhead lights in the room The nurse should try to reduce the patient's sensory overload by closing the patient's door and avoiding the use of bright overhead lights. Dimmer reading lights should be used whenever possible. Maintaining eye contact, having the patient repeat information, and orienting the patient to the surroundings will not reduce the sensory overload of the bright, loud hospital environment.

The patient's sacral pressure injury is open with exposed bone. Which pressure injury stage will be recorded in the patient's chart? a. Stage 1 b. Stage 2 c. Stage 3 d. Stage 4

d. Stage 4 Stage 4: Full-thickness tissue loss with exposed bone, tendon, or muscle; slough or eschar may be present; often includes undermining and tunneling Stage 1: Intact skin with nonblanchable redness of a localized area, usually over a bony prominence Stage 2: Partial-thickness loss of dermis presenting as a shallow open ulcer with a red-pink wound bed, without slough; may also present as an intact or open/ruptured serum-filled blister Stage 3: Full-thickness tissue loss; subcutaneous fat may be visible, but bone, tendon, or muscle is not exposed; slough may be present but does not obscure the depth of tissue loss; may include undermining and tunneling

Why is acute pain particularly dangerous for a patient having a heart attack? a. Release of endorphins causes dangerous elevation of blood pressure. b. Release of substance P narrows the airways and leads to hypoxemia. c. Release of prostaglandins lowers the patient's heart rate and blood pressure. d. Stimulation of the sympathetic nervous system will increase cardiac workload.

d. Stimulation of the sympathetic nervous system will increase cardiac workload. Acute pain of low-to-moderate intensity and superficial pain cause the fight-or-flight response from the sympathetic branch of the autonomic nervous system and elevates pulse. Endorphins are natural morphine-like substances that decrease pain. Substance P helps transmit pain impulses from periphery to higher brain centers and causes vasodilation and edema. Prostaglandins increase sensitivity to pain.

Which is the highest priority intervention for a patient with diabetic neuropathy who has lost sensation in both feet? a. Encourage the patient to participate in tai chi exercises to promote balance. b. Instruct the patient to wear a medical alert bracelet that identifies risk for falls. c. Evaluate the patient's blood pressure for orthostatic hypotension. d. Teach the patient to wear low-heeled, comfortable, supportive footwear at all times

d. Teach the patient to wear low-heeled, comfortable, supportive footwear at all times It is essential for patients with diabetic neuropathy to wear supportive footwear at all times to prevent injury to the feet. Patients with neuropathy will not realize if the foot has been injured or punctured, leading to the risk of ulceration, serious wound, or even amputation. Tai chi exercises are not a priority. Fall alert bracelets may be worn in the hospital but are not appropriate for use at home. There is no need to check the patient for orthostatic hypotension.

A nurse manager is reviewing interrelated concepts to the professional nursing role. Which factor should the nurse manager consider when addressing concerns about the quality of patient education? a. Adherence b. Developmental level c. Motivation d. Technology

d. Technology The interrelated concepts to the professional role of a nurse include health promotion, leadership, technology/informatics, quality, collaboration, and communication. Adherence, culture, developmental level, family dynamics, and motivation are considered interrelated concepts to patient attributes and preference.

The patient comes to the hospital with a variety of symptoms. Which symptom will the nurse assess first? a. The patient experiences joint stiffness after sitting still for long periods of time. b. The patient developed an itchy rash after taking the second dose of antibiotics. c. The patient is nauseated and vomited a small amount earlier this morning. d. The patient feels short of breath and has audible expiratory wheezes.

d. The patient feels short of breath and has audible expiratory wheezes. The nurse needs to assess the patient's shortness of breath before addressing the other symptoms. Oxygenation is the body's most basic need and takes priority over other patient needs

Which assessment finding indicates why the patient does not have signs of respiratory alkalosis despite a respiratory rate of 30 breaths/minute? a. The patient's hematocrit is 28%. b. The patient's oral temperature is 99.2° F. c. The patient is experiencing a panic attack. d. The patient has a large pulmonary embolism.

d. The patient has a large pulmonary embolism. A large pulmonary embolism reduces blood flow through the lungs leading the patient to feel acutely short of breath. The patient will not develop signs of respiratory alkalosis because the rapid breathing rate is compensating for impaired perfusion. Elevated hematocrit level, slightly elevated temperature, or panic attack would all cause the patient to have signs of hyperventilation with a respiratory rate of 30 breaths/minute

Which assessment finding leads the nurse to include hopelessness as a nursing diagnosis in the patient's plan of care? a. The patient does not wish to attend any type of religious services. b. The patient does not believe in a higher power, spirit guide, or God. c. The patient has a source of structure and guidance for difficult times. d. The patient has no motivation or resources to achieve any life goals.

d. The patient has no motivation or resources to achieve any life goals. Hope is energizing, giving individuals a motivation to achieve and the resources to use toward that achievement. The patient experiences hopelessness when there is no motivation or resources to achieve any life goals. Religion refers to the system of organized beliefs and worship that a person practices to outwardly express spirituality. Faith involves a belief in a higher power, spirit guide, God, or Allah. Spirituality offers a sense of connectedness. Spirituality is unique for each person. It is a unifying theme in life and a state of being.

The nurse is caring for a patient who is recovering from a cerebrovascular accident. Which assessment finding indicates that the patient's cerebellum was damaged by the stroke? a. The patient has continuous double vision. b. The patient has slurred speech and dysphagia. c. The patient is incontinent of bowel and bladder. d. The patient has poor balance and has an unsteady gait.

d. The patient has poor balance and has an unsteady gait. Damage to the cerebellum causes problems with balance, and motor impairment is directly related to the amount of destruction of the motor strip. The cerebellum does not control speech, continence, or vision.

Which assessment finding indicates to the nurse why the patient is having difficulty sleeping at night? a. The patient follows an organic, low-carbohydrate diet. b. The patient enjoys doing crossword puzzles and reading. c. The patient's job includes many hours of hard labor each day. d. The patient now works in Alaska with extended daylight hours.

d. The patient now works in Alaska with extended daylight hours. Northern Alaska has extended daylight hours. Light and temperature affect all circadian rhythms, including the sleep-wake cycle. The most familiar rhythm is the 24-hour, day-night cycle known as the diurnal or circadian rhythm. When the sleep-wake cycle becomes disrupted (e.g., by working rotating shifts), other physiological functions change as well. Low-carbohydrate diet, reading, and manual labor will not cause the patient's sleep problems.

Which assessment finding indicates that the patient's CPAP is effectively treating the patient's sleep apnea? a. The patient is able to fall asleep within 30 minutes of going to bed. b. The patient is getting an average of 7 to 8 hours of sleep each night. c. The patient does not require trazodone to fall asleep. d. The patient sleeps through the night without having to get up and urinate.

d. The patient sleeps through the night without having to get up and urinate. Nocturia is a sign of sleep apnea so the patient's ability to sleep through the night without having to get up and urinate indicates that the CPAP therapy is effective. Getting 7 to 8 hours of sleep at night, not requiring trazodone, and falling asleep readily do not indicate effective treatment of sleep apnea

Which assessment finding explains why the patient developed right-sided heart failure? a. The patient's resting heart rate is usually 55 to 60 beats/minute. b. The patient's resting heart rate is usually 58 to 60 beats/minute. c. The patient has 2+ pitting edema of the legs, feet, and abdomen. d. The patient was diagnosed with cystic fibrosis at 2 years of age

d. The patient was diagnosed with cystic fibrosis at 2 years of age Chronic respiratory conditions such as cystic fibrosis predispose the patient to right-sided heart failure as the heart must work harder to compensate for the respiratory disease. Resting heart rate of 55 to 60 beats/minute is normal for some patients and will not lead to right-sided heart failure. 2+ pitting edema of the legs, feet, and abdomen is a symptom of right-sided heart failure rather than a cause of it. Athletic training will not lead to right-sided heart failure.

The nurse is caring for an elderly patient who has become withdrawn and refuses to eat after being admitted to a long-term nursing facility. Which is the highest priority goal for the patient? a. The patient will participate in social activities with other residents. b. The patient will ask questions about the prescribed care and treatment. c. The patient will develop meaningful relationships with the nursing staff. d. The patient will maintain usual weight and show no signs of dehydration.

d. The patient will maintain usual weight and show no signs of dehydration. The highest priority goal for this patient is maintenance of weight and prevention of dehydration. When this goal is met, the nurse can help the patient to participate in activities with other residents, ask questions about care, and develop relationships with the staff

Which patient learning goal is measurable? a. The patient will understand the importance of daily iron supplements. b. The patient will be able to learn sufficient information to be discharged. c. The patient will feel comforted by the nurses' presence during anxious periods. d. The patient will verbalize responsibility for obtaining daily weights each morning.

d. The patient will verbalize responsibility for obtaining daily weights each morning. Patient care plan goals must be measurable so that the nurse can determine whether or not the goal has been met. Measurable goals use objective terms such as verbalize, demonstrate, list, articulate, and perform. The patient's verbalization of responsibility for obtaining daily weights each morning is a measurable goal. The nurse cannot objectively determine if the patient understands, feel comforted, or learn sufficient information in order to determine whether or not the goal has been achieved.

The nurse is caring for a patient with congestive heart failure. The patient reports frequently waking up at night gasping for air. What is the cause of the patient's awakenings? a. The patient is going to bed too early every night. b. The patient consumed caffeine just before going to bed. c. The patient consumed too much fiber before going to bed. d. The pulse, respirations, and blood pressure drop during sleep.

d. The pulse, respirations, and blood pressure drop during sleep. Heart rate, respirations, and blood pressure drop during sleep. This can cause the patient with congestive heart failure to wake up gasping during the night as the blood oxygenation drops to dangerously low levels. Going to bed early and consuming fiber or caffeine before bed would not cause the patient to wake up gasping for air.

The nurse is caring for a patient experiencing an allergic reaction to a bee sting who has an order for diphenhydramine (BenaDRYL). The only medication in the patient's medication bin is labeled BenaZEPRIL. The nurse contacts the pharmacy for the correct medication to avoid what type of error? a. Communication b. Diagnostic c. Preventive d. Treatment

d. Treatment The nurse avoided a treatment error, giving the wrong medication. Benazepril is an ace inhibitor used to treat blood pressure. The Institute of Medicine (IOM) report referred to Leape identification of four types of errors. Treatment errors occur in the performance of an operation, procedure, or test; in administering a treatment; in the dose or method of administering a drug; or in avoidable delay in treatment or in responding to an abnormal test. Communication errors refer to those that occur from a failure to communicate. Diagnostic errors are the result of a delay in diagnosis, failure to employ indicated tests, use of outmoded tests, or failure to act on results of monitoring or testing. Preventive errors occur when there is inadequate monitoring or failure to provide prophylactic treatment or follow-up of treatment.

The nurse is instructing the nursing assistant to prevent pressure ulcers in a frail older patient. Which action indicates the nursing assistant has understood the nurse's teaching? a. Bathing and drying the skin vigorously to stimulate circulation b. Keeping the head of the bed elevated 30 degrees c. Limiting intake of fluid and offer frequent snacks d. Turning the patient at least every 2 hours

d. Turning the patient at least every 2 hours The patient should be turned at least every 2 hours as permanent damage can occur in 2 hours or less. If skin assessment reveals a stage I ulcer while on a 2-hour turning schedule, the patient must be turned more frequently. Limiting fluids will prevent healing; however, offering snacks is indicated to increase healing particularly if they are protein based, because protein plays a role in healing. Use of doughnuts, elevated heads of beds, and overstimulation of skin may all stimulate, if not actually encourage, dermal decline.

The patient has a deep decubitus ulcer on the heel that is covered in thick necrotic tissue. Which term will the nurse use to describe the ulcer in the patient's medical record? a. Fluctuant b. Indurated c. Macerated d. Unstageable

d. Unstageable The nurse will document the ulcer as unstageable as the depth of the wound cannot be determined. Indurated skin feels hard to the touch but is not necrotic. Macerated skin has broken down due to long exposure to moisture. Fluctuant means to change or vary

The patient has cloudy yellow urine with a faint odor of ammonia. Which laboratory test will the nurse expect to note in the patient's admission orders? a. Serum albumin and pre-albumin b. Serum calcium and magnesium levels c. Fasting blood sugar every morning d. Urine sample for culture and sensitivity

d. Urine sample for culture and sensitivity Cloudy yellow urine with a faint odor of ammonia is a sign that the patient has a urinary tract infection. The patient's urine should be sent for culture and sensitivity. Daily fasting blood sugars are checked for patients with diabetes or hyperglycemia. Serum calcium, magnesium, albumin, and pre-albumin do not test for the presence or a urinary tract infection.

Which technique is the best way for the nurse to obtain information from the patient? a. Ask personal questions so as to show interest. b. Use medical vocabulary to appear competent. c. Ask why the patient waited so long to get treatment. d. Use silence while the patient collects his or her thoughts

d. Use silence while the patient collects his or her thoughts Most people have a natural tendency to fill empty spaces with words, but sometimes silence is useful when they face decisions that require much thought. Nontherapeutic techniques discourage further expression of feelings and ideas and engender negative responses or behaviors in others. Asking irrelevant personal questions simply to satisfy your curiosity is inappropriate and invasive and nontherapeutic. Limit questions to health-related information. Health care professionals have their own culture and language. Using technical words in discussions with patients can cause confusion and anxiety. Avoid excessive use of such terms or translate them into lay terms. Sometimes asking "why" implies an accusation and results in resentment, insecurity, and mistrust.

Which approach will be most successful for the nurse to teach a preschooler about tube feeding through a gastrostomy tube? a. Offer opportunities to discuss tube feeding options and answer questions. b. Hold the child while smiling and speaking softly to convey a sense of trust. c. Collaborate with the child to develop an individualized tube feeding schedule. d. Use simple terms and show the child a gastrostomy tube inserted into a teddy bear

d. Use simple terms and show the child a gastrostomy tube inserted into a teddy bear The nurse should allow the child to see and touch a gastrostomy tube inserted into a teddy bear to facilitate teaching about tube feeding. Holding the child while smiling is an appropriate teaching technique for an infant. The preschooler is not mature enough to develop an individualized tube feeding schedule or discuss tube feeding options.

The nurse has just completed discharge teaching about self-injection of insulin to a patient who speaks little English. How can the nurse best determine that the teaching has been successful? a. Ask the patient if there are any questions about the procedure. b. Provide written instructions in the patient's preferred language. c. Have the patient watch a video about how to self-inject insulin. d. Watch the patient self-administer the next scheduled dose of insulin.

d. Watch the patient self-administer the next scheduled dose of insulin. The best way for the nurse to determine that the teaching has been successful is to watch the patient self-administer the next scheduled dose of insulin. This way the nurse can observe for any difficulties and provide feedback immediately. Asking the patient if there are any questions is not recommended due to the patient's language difficulty. Written instructions and videos will not determine if the patient is able to self-administer insulin correctly.

How can the nurse assess if an infant is experiencing hearing loss? a. Use an otoscope to ensure that the infant's tympanic membrane is intact. b. Review the infant's medication list for medications that cause ototoxicity. c. Examine the infant's outer ears to check for excessive amounts of cerumen. d. Watch to see if the infant reacts when the nurse's hands are clapped together.

d. Watch to see if the infant reacts when the nurse's hands are clapped together. Neonates without hearing impairments respond to loud noises. Atrophy of the cerumen glands, seen mainly in older adults, cause thicker and dryer wax, which is more difficult to remove and may completely obstruct the auditory canal. Hearing loss can be determined at any age with additional testing by an ENT specialist. Using an otoscope, reviewing the medication list, and examining the infant's outer ears will not assess for hearing loss

Which assessment is most important to determine if a patient is receiving sufficient sleep? a. Sleep-wake pattern b. Frequency of nocturia c. Hours of sleep each night d. Whether the patient feels rested

d. Whether the patient feels rested Because sleep is a subjective experience, only the patient is able to report whether it is sufficient and restful. Patients are your best resource for describing a sleep problem and any change from their usual sleep and waking patterns. Number of hours of sleep, sleep-wake pattern, and number of times awake for nocturia are not the most important to assess to determine effectiveness of the patient's sleep, the subjective experience of the patient is the most important.

To promote a culture of safety, the nurse manager preparing the staff schedule considers the anticipated census in planning the number and experience of staff on any given shift. Which is the human factor primarily addressed with this consideration? a. Available supplies b. Interdisciplinary communication c. Interruptions in work d. Workload fluctuations

d. Workload fluctuations Including an adequate number of staff members with experience caring for anticipated patients is a strategy to manage the workload and potential fluctuations. A safety culture requires organizational leadership (e.g., the nurse manager) that gives attention to human factors such as managing workload fluctuations. This strategy also applies principles of crew resource management in that it addresses workload distribution. Lack of supplies can create a challenge for safe care but could not be addressed with the schedule. Concerns with communication and coordination across disciplines, including power gradients, and excessive professional courtesy can create hazards but would not be the best answer. Strategies to minimize interruptions in work are essential but would not be the best answer in this situation

To plan early intervention and care for an infant with Down syndrome, the nurse considers knowledge of other physical development exemplars such as a. cerebral palsy. b. failure to thrive. c. fetal alcohol syndrome. d. hydrocephaly

d. hydrocephaly Hydrocephaly is also a physical development exemplar. Cerebral palsy is an exemplar of adaptive developmental delay. Failure to thrive is an exemplar of social/emotional developmental delay. Fetal alcohol syndrome is an exemplar of cognitive developmental delay.

The school nurse talking with a high school class about the difference between growth and development would best describe growth as a. processes by which early cells specialize. b. psychosocial and cognitive changes. c. qualitative changes associated with aging. d. quantitative changes in size or weight

d. quantitative changes in size or weight Growth is a quantitative change in which an increase in cell number and size results in an increase in overall size or weight of the body or any of its parts. The processes by which early cells specialize are referred to as differentiation. Psychosocial and cognitive changes are referred to as development. Qualitative changes associated with aging are referred to as maturation.

A recent graduate nurse has been assigned to be a primary nurse on a geriatric unit. After completing a review of development and aging, the nurse recalls that changes during the climacteric signify which of the following? a.A characteristic of young adulthood b.The increased reproductive ability of the older adult c.A time of significant change in cognitive performance in middle age d.A decline of reproductive capacity caused by a decrease in sexual hormones

d.A decline of reproductive capacity caused by a decrease in sexual hormones Climacteric is a term used to describe the decline of reproductive capacity and accompanying changes brought about by the decrease in sexual hormones. This affects men and women differently. Men begin to experience decreased fertility, but they are able to continue to father children. Menopause, when a woman stops ovulating and menstruating, occurs only when 12 months have passed since the last menstrual flow. Climacteric is a characteristic of middle adulthood usually referring to those years between 40 and 65. Changes in the cognitive function of middle-age adults are few except during illness or trauma.

A patient who underwent surgery for a bowel obstruction yesterday has become confused and has made several attempts to climb out of bed. The nurse is considering options to prevent the patient from harm. Which of the following actions could be delegated to assistive nursing personnel working with the nurse? a.Assessing the patient for appropriateness of restraints b.Calling the physician for an order for a restraint alternative c.Discussing the need for restraints with the patients family d.Applying restraints after orders received by the nurse

d.Applying restraints after orders received by the nurse The skill of applying a restraint can be delegated to trained nursing assistive personnel. However, the nurse is responsible for assessing a patients behavior, determining the need for restraint, the type of restraint to use, and performing patient assessments while restraints are in place. Patients, who are confused, disoriented, or who repeatedly fall or try to remove medical devices (e.g., IV lines or dressings) may require the temporary use of restraints to keep them safe. Restraints are not a solution to a patient problem but rather a temporary means to maintain patient safety. All alternatives must be used before placing patients in restraints. Performing an assessment, obtaining orders from the physician and including the family in the discussion of why restraints are necessary are all jobs that cannot be delegated to a nursing assistive personnel (NAP); and must be performed by the nurse.

What is the best indicator the nurse can use to determine the adequacy of a patients cardiac output? a.Stroke volume b.Myocardial contractility c.Afterload d.Cardiac index

d.Cardiac index Cardiac index is a measure of adequacy of the cardiac output. It equals the cardiac output divided by the patients body surface area. This calculation provides the caregiver with a more accurate calculation of blood flow by considering the patients body surface area. Stroke volume (SV) is the amount of blood ejected from the ventricle with each contraction. The normal range for a healthy adult is 50 to 75 mL per contraction. Myocardial contractility is the ability of the heart to squeeze blood from the ventricles and prepare for the next contraction. This is difficult to measure because preload, afterload, and heart rate must remain constant. Afterload is the resistance to the ejection of blood from the left ventricle. The left ventricular pressure must be greater than the aortic pressure to eject blood from the heart.

A 47-year-old woman with a history of diabetes and hypertension calls the clinic complaining of epigastric pain and shortness of breath with activity. She has taken antacids with no relief. What type of pain is the patient experiencing? a.Pericardial pain b.Pleuritic chest pain c.Musculoskeletal pain d.Cardiac pain

d.Cardiac pain Some women have epigastric pain, complaints of indigestion, or a choking feeling and dyspnea when experiencing cardiac pain. Cardiac pain does not occur with respiratory variations. Cardiac pain is most often substernal and typically radiates to the left arm and jaw in men. Pericardial pain resulting from an inflammation of the pericardial sac is usually nonradiating and often occurs with inspiration or when leaning forward. Pleuritic chest pain is peripheral and usually radiates to the scapular regions. Inspiratory maneuvers such as coughing, yawning, and sighing aggravate pleuritic chest pain. An inflammation or infection in the pleural space usually causes pleuritic chest pain. Musculoskeletal pain is often present following exercise, rib trauma, and prolonged coughing episodes. Inspiratory movements aggravate the pain and are easily confused with pleuritic chest pain.

The nurse is admitting a patient with chronic obstructive pulmonary disease (COPD). During the initial head-to-toe assessment the patients pulse oximetry reading is 89% on room air. What is the nurses first priority? a.Administer oxygen immediately @ 4L/NC. b.Call the primary health care provider for an order for oxygen. c.Assist the patient into a recumbent position. d.Determine the patients normal pulse oximetry reading.

d.Determine the patients normal pulse oximetry reading. The nurse must determine what is normal for this patient. The patient has COPD and the breathing stimulus is low oxygen, not increased carbon dioxide. When caring for patients with COPD and chronically elevated PaCO 2 levels, remember that inappropriate administration of excessive oxygen will result in hypoventilation. Patients with COPD and hypercapnia (high carbon dioxide levels) have adapted to the higher carbon dioxide level. The carbon dioxidesensitive chemoreceptors are no longer sensitive to increased carbon dioxide as a stimulus to breathe. Their stimulus to breathe is a decreased PaO 2 . The most effective position for patients with cardiopulmonary diseases is the 45-degree semi-Fowlers position, using gravity to assist in lung expansion and reduce pressure from the abdomen on the diaphragm. Administering excessive oxygen to patients with COPD satisfies the oxygen requirement of the body and negates the stimulus to breathe. High concentrations of oxygen (e.g., greater than 24% to 28% [1 to 3 L/min]) prevent the PaO 2 from falling. As a result, this suppresses the stimulus to breathe, resulting in hypoventilation.

A female 36-year-old bank executive was recently promoted to vice president. She and her husband have two school-age children. The patient is being seen at the clinic and reports severe abdominal pain with diarrhea. During the assessment, the patient explains to the health care worker that she and her family will be moving to another state because of her promotion. Her children are upset about leaving their friends. The health care worker recognizes that which of the following information is a priority for patient teaching? a.Providing growth and development information about the school-age child b.Recommending a gastroenterologist c.Offering to call a moving company d.Discussing how a high level of stress can cause illness

d.Discussing how a high level of stress can cause illness An adult experiencing a high level of stress is at a greater risk for accidents and certain stress-related illnesses such as headaches, depression, gastrointestinal disorders, and infections. Although it is important to give the patient information regarding normal stages of growth and development in school-age children; this is not the cause of the symptoms. Referral to a gastroenterologist would occur after stress management techniques have been tried; or symptoms become worse. Calling a moving company does not address the real problem of stress.

A nurse is working in a health facility that creates a culture of safety. Which behavior will the nurse use in this type of facility? a.Find blame when problems occur. b.Reprimand co-workers when a mistake is made. c.Maximize adverse events. d.Focus on performance improvement efforts.

d.Focus on performance improvement efforts. These types of organizations foster a patient-centered safety culture by continually focusing on performance improvement efforts, risk-management findings, and safety reports to design a safe work environment. Health care organizations strive to create a culture of safety, one that consistently minimizes, not maximizes, adverse events despite carrying out complex and hazardous work. A culture of safety requires the determination to achieve consistently safe operations and a blame-free environment in which individuals can report errors without fear or reprimand.

A school nurse performs a routine screening on a newly transferred school-age child. This nurse is especially interested in discovering the childs medical history regarding middle ear infections. The nurse knows that chronic ear infections are a major contributing factor to which of the following? a.Respiratory diseases b.Strep throat c.High fevers d.Hearing impairment

d.Hearing impairment Hearing impairment is common in the United States. At-risk children include those with a family history of childhood hearing impairment, perinatal infection (rubella, herpes, or cytomegalovirus), low birth weight, chronic ear infections, and Down syndrome. Children need periodic auditory testing. Advise pregnant women of the importance of early prenatal care, avoidance of ototoxic drugs, and testing for syphilis and rubella. Strep throat, high fevers, and respiratory diseases are potential contributing factors for chronic ear infections.

A patient has been admitted to the pulmonary unit of the hospital with right lower lobe pneumonia and history of chronic obstructive pulmonary disease (COPD). During the initial assessment the nurse notes that the patient has a respiration rate of 18 with retractions, tachycardia, and complains of dyspnea and dizziness. The nurse identifies that these are clinical signs of which condition? a.Paroxysmal nocturnal dyspnea b.Orthopnea c.Hemoptysis d.Hypoxia

d.Hypoxia Dyspnea is a clinical sign of hypoxia. Dyspnea is the subjective sensation of breathlessness as perceived by the patient. Hypoxia is inadequate tissue oxygenation with a deficiency in oxygen delivery or oxygen utilization at the cellular level. Signs and symptoms of hypoxia include tachycardia, peripheral vasoconstriction, dizziness, and mental confusion. Dyspnea that occurs when a patient is sleeping is called paroxysmal nocturnal dyspnea. The patient awakens in a panic, feels as if he or she is suffocating, and has a strong need to sit up to relieve the breathlessness. Orthopnea is an abnormal condition in which a patient has difficulty breathing when lying down and has to use multiple pillows or sit to breathe. Hemoptysis is bloody sputum. It is associated with coughing and bleeding from the upper respiratory tract, from sinus drainage, or from the gastrointestinal tract.

A nurse has finished preoperative teaching for a surgical patient. Which statement by the patient indicates teaching was successful about the use of elastic stockings? a.I do not have to worry about wrinkles. b.I can roll them no lower than my calf. c.I will massage my legs regularly. d.I should remove and reapply them every 8 hours.

d.I should remove and reapply them every 8 hours. Remove and reapply elastic stockings at least every 8 hours. Elastic stocking aid in maintaining pressure on the muscles of the lower extremities and promote venous return. Rolled-down stockings constrict the vessels and impede venous return. The elastic stocking should be smooth, not wrinkled. The legs should not be massaged.

According to one theorist, moral development depends on the childs ability to accept social responsibility and to integrate personal principles of justice and fairness. Which of the following individuals is responsible for the theory of moral development? a.Sigmund Freud b.Jean Piaget c.Erik Erikson d.Lawrence Kohlberg

d.Lawrence Kohlberg According to Kohlberg (1964), moral development is one component of psychosocial development. It involves the reasons an individual makes a decision about right and wrong behaviors within a culture. Moral development depends on the childs ability to accept social responsibility and to integrate personal principles of justice and fairness. Sigmund Freud (1856-1939) provided the first formal structured theory of personality development. Freuds psychoanalytic model of personality development is grounded in the belief that two internal biological forces drive the psychological change in a child: sexual (libido) and aggressive energies. Jean Piaget (1896-1980) developed the theory of cognitive development, which describes childrens intellectual organization and how they think, reason, and perceive the world. Erik Erikson divided life into eight stages, known as Eriksons eight stages of development. According to this theory, individuals need to accomplish a particular task before successfully completing each stage. Each task is framed with opposing conflicts, such as trust versus mistrust. Each stage builds upon the successful attainment of the previous developmental conflict. Lawrence Kohlberg (1927-1987) expanded on Piagets work.

A 75-year-old patient in an acute care hospital who underwent surgery for an abdominal aneurysm developed a urinary tract infection 3 days after placement of a Foley catheter. The nurse believes that this is a reportable incident, and which of the following will happen as a result? a.Medicare will be denied to the patient. b.Medicare will take the hospital to court. c.Nothing; it is not a reportable incident. d.Medicare will not reimburse the hospital for this infection.

d.Medicare will not reimburse the hospital for this infection. The Centers for Medicare and Medicaid Services (CMS) names select serious reportable events (SREs) as Never Events (adverse events that should never occur in a health care setting) (US Department of Health and Human Services, 2008). The CMS now denies payment to hospitals for any hospital-acquired conditions resulting from or complicated by the occurrence of certain Never Events that were not present on admission. Many of the hospital- acquired conditions are nurse-sensitive indicators, meaning that nursing interventions directly affect their development. Medicare is not denied to the patient and this is a reportable incident. Medicare does not take hospitals to court; it just denies payment for the Never Event.

Patients on prolonged bed rest are at risk for a deep vein thrombosis. Which information indicates the nurse needs more teaching about the factors in Virchows triad? a.One of the factors is loss of integrity of the vessel wall. b.One of the factors is abnormalities of blood flow. c.One of the factors is alterations in blood constituents. d.One of the factors is atrophy of the muscles.

d.One of the factors is atrophy of the muscles. Three factors contribute to venous thrombus formation: (1) loss of integrity of the vessel wall (e.g., injury), (2) abnormalities of blood flow (e.g., slow blood flow in calf veins associated with bed rest), and (3) alterations in blood constituents (e.g., a change in clotting factors or increased platelet activity). These three factors are referred to as Virchows triad . Disuse, atrophy, and shortening of muscle fibers and surrounding joint tissues cause joint contracture, not deep vein thrombosis.

When giving CPR, compressions are causing the heart to pump blood into and out of the lungs to the bodys organs. This movement of oxygenated blood is best described as which of the following? a.Ventilation b.Diffusion c.Respiration d.Perfusion

d.Perfusion The heart supports perfusion, the movement of blood into and out of the lungs to the organs and tissues of the body. The primary functions of the lungs include ventilation, the movement of air in and out of the lungs, and diffusion, the movement of gases between air spaces and the bloodstream. Respiration is the exchange of oxygen and carbon dioxide during cellular metabolism.

Postmenopausal women are at risk for skeletal changes. When developing a plan of care for postmenopausal women, a nurse should remember which of the following? a.Exercise provides little benefit for the middle-age adult. b.Middle-age women should avoid dietary calcium to prevent osteoporosis. c.Exercise and fitness clubs have limited activities. d.Physical activities help improve balance and coordination

d.Physical activities help improve balance and coordination When middle-age adults seek health care, nurses need to develop goals for positive health behaviors. Simple things like increasing dietary calcium and calcium supplements are effective. In addition, exercise and fitness clubs, for example, give men and women the opportunity to participate in many physical activities. These activities help improve balance, coordination, and activity tolerance.

The nurse is observing a patients posture while sitting, standing, and assessing gait. What is the rationale for the nurses assessment? a.To determine type of assistance with anthropometric measurements b.To determine type of assistance with joint mobility c.To determine type of assistance with range of motion (ROM) d.To determine type of assistance with ambulation

d.To determine type of assistance with ambulation Observing the patients posture while sitting and standing and assessing gait helps to determine the type of assistance the patient requires for ambulation or transfer. Assessment of ROM is important as a baseline measurement to compare and evaluate whether loss in joint mobility has occurred. Anthropometric measurements are for nutrition, not for mobility. Anthropometric measurements include height, weight, mid upper-arm circumference, and triceps skinfold measurements.

The student nurse is assisting an elderly patient to get ready for bed. The patient states, Please make sure you clean my hearing aids. The student nurse knows it is important to keep in mind which of the following when cleaning a hearing aid? a.Keep the battery in the machine when turned off. b.Store the hearing aid on the overnight table for easy access at night. c.Clean the hearing aid with hot water. d.Use a soft dry cloth to wipe the hearing aid.

d.Use a soft dry cloth to wipe the hearing aid. Care for hearing aids include: Make sure your fingers are dry and clean before handling hearing aids. Insert and remove the hearing aid over a soft surface. Place the battery in the hearing aid when it is turned off. Remove the hearing aid battery when not in use and store it in a marked container in a safe place. Protect hearing aids from water and excessive heat or cold. Use a soft dry cloth to wipe hearing aids and a soft brush to clean difficult to reach areas.

The parent of a 13-year-old boy is concerned because the teenager wants to hang out with friends all the time and has stated that he wants to get his ear pierced because all his friends have piercings. What is the best response from the nurse? a.I think you need to seek counseling for your son. b.I think this is just a phase that will quickly pass. c.Your son needs to find new friends. d.Your sons behavior is normal; he is trying to assert his independence.

d.Your sons behavior is normal; he is trying to assert his independence. As children enter adolescence, they develop greater independence and a sense of identity. The adolescent begins to separate emotionally from the family, and the peer group begins to have a stronger influence. To relieve the tensions associated with the physical and psychosocial changes, as well as peer pressure, adolescents often engage in risk-taking behaviors such as smoking, drinking alcohol, and using drugs. This increases the risk for accidents such as drowning and motor vehicle accidents. Counseling and finding new friends are not needed because the boy is demonstrating normal signs of development. Adolescence does not quickly pass.

Order: IV 1000 mL 0.9% NS to infuse over 5 hours IV tubing: 10 drops/mL No infusion pump is used. What is the flow rate in drops/minute? ____

33 drops/minute 1000 mL/5 hours = 200 mL/hour 200 mL/60 minutes 10 drops/mL = 33 drops/minute

Order: 500 mL D5W to infuse at 50 mL/hour IV tubing: 10 drops/mL How many mL will infuse over the nurse's 12 hour shift?

12 hours 50 mL/hour = 600 mL

A nurse prepares to teach the patient about strategies to minimize feelings of powerlessness. Which techniques will the nurse implement that are the best for this type of learning? (Select all that apply.) a. Lecture b. Practice c. Discussion d. Role play e. Return demonstration

C, D Teaching methods for affective learning include role play and discussion. Lecture is effective for cognitive learning. Practice and return demonstration are best for psychomotor learning.

What term refers to a fever whose cause cannot be determined? a.Febrile b.Afebrile c.Fever of unknown origin (FUO) d.Pyrexic

C- Fever of unknown origin (FUO) The term fever of unknown origin (FUO) refers to a fever whose cause cannot be determined. When the fever breaks, the temperature returns to an acceptable range and the patient becomes afebrile (a- means not or without , so afebrile means without fever ).

Order: Merrem 500 mg in 100 mL D5W IVPB infuse over 30 minutes IV tubing: 10 drops/mL What will the infusion pump rate be set at—how many mL/hour?

200 mL/hour 100 mL/30 minutes 60 minutes/1 hour = 200 mL/hour

The nurse and is very concerned about infection control in the Surgery Department. Recently she provided education to the surgery staff on ways to eliminate transient hand flora. The most precise description for this is hand: a.hygiene. b.washing. c.antisepsis. d.rub.

C- antisepsis. Hand hygiene is a general term that applies to hand washing, antiseptic handwash, antiseptic hand rub, or surgical hand antisepsis. Hand washing refers to washing hands with plain soap and water.

Order: IV 500 mL 0.9% NS to infuse over 10 hours IV tubing: 10 drops/mL What will the infusion pump rate be set at—how many mL/hour? ____

50 mL/hour 500 mL/10 hours = 50 mL/hour

Order: 500 mL D5W to infuse at 100 mL/hour IV tubing: 10 drops/mL The IV bag was hung at 12:00 noon. What time will the IV bag be completed? __

5:00 p.m. or 1700 500 mL 1 hour/100 mL = 5 hours infusion time 12:00 start time + 5:00 run time 1700 finish time

Which patient populations are at risk for being marginalized? (Select all that apply.) a. Recent immigrants to the United States b. Individuals with type AB positive blood c. Professionals with a master's or doctoral degree d. Individuals who speak at least three different languages e. Individuals who have undergone sexual reassignment surgery

A, E Recent immigrants and transgender individuals are considered to be marginalized and at risk for having poor health outcomes. Blood type, advanced education, and fluency in multiple languages are not criteria for marginalization.

During an assessment, the patient states that his bowel movements cause discomfort because the stool is hard and difficult to pass. As the nurse, you make which of the following suggestions to assist the patient with improving the quality of his bowel movement? (Select all that apply.) a. Increase fiber intake. b. Increase water consumption. c. Decrease physical exercise. d. Refrain from alcohol. e. Refrain from smoking.

A, B Increasing fiber assists in adding bulk to the stool. Increasing water assists in softening the stool and moving it through the large intestine. Decreasing exercise will have the opposite effect of slowing bowel movements. Refraining from alcohol and smoking have no direct effect on the quality of bowel movements.

The nurse is reviewing the patients arterial blood gas results. The PaO2 is 96 mm Hg, pH is 7.20, PaCO2 is 55 mm Hg, and HCO3 is 25 mEq/L. What would the nurse expect to observe on assessment of this patient? A Disorientation and tremors B Tachycardia and decreased blood pressure C Increased anxiety and irritability D Hyperventilation and lethargy

A Disorientation and tremors The patient is experiencing respiratory acidosis ( pH, and PaCO2 ) which may be manifested by disorientation, tremors, possible seizures, and decreased level of consciousness. Tachycardia and decreased blood pressure are not characteristic of a problem of respiratory acidosis. Increased anxiety and hyperventilation will cause respiratory alkalosis, which is manifested by an increase in pH and a decrease in PaCO2.

A nurse is teaching the staff about Quality and Safety Education in Nursing, which identified six competencies for nursing. Which information should the nurse include in the teaching session? (Select all that apply.) a.Informatics b.Safety c.Health policies d.Informatics e.Quality improvement

A, B, D, E The Robert Wood Johnson Foundation sponsored the Quality and Safety Education for Nurses (QSEN) initiative to respond to reports about safety and quality patient care by the IOM. The QSEN initiative encompasses the competencies of patient-centered care, teamwork and collaboration, evidence-based practice, quality improvement, safety, and informatics. Health policy is health related issues at the government level, not a competency.

A nurse is meeting a patient for the first time. The importance of establishing baseline data is that it will enable the nurse to assess which of the following? (Select all that apply.) a.Know the baseline information about the patients health status. b.Supplement, confirm, or refute information learned during the history taking. c.Identify or confirm nursing diagnoses. d.Assess the patients understanding of the disease process. e.Focus on a specific body system.

A,B,C,D

A nurse is performing an admission assessment on a middle-age patient. A normal change seen in this age group includes which of the following? (Select all that apply.) a.A progressive decrease in skin turgor b.Decreased visual acuity c.Decreased ability to solve practical problems d.Decreased strength of abdominal muscles e.Loss of accommodation

A, B, D, E Middle adulthood usually refers to those years between 40 and 65. Expected physical changes include a slow, progressive decrease in skin turgor, decreased abdominal strength, decreased visual acuity, and loss of accommodation of lens to focus light on near objects. The ability to solve practical problems based on experience peaks at midlife because of the ability for integrative thinking.

A nurse is teaching the staff about the characteristics of a profession. Which information should the nurse include? (Select all that apply.) a.Extended education b.Theoretical body of knowledge c.Code of ethics for practice d.Practice developments e.Provision of a specific service

A,B,C,E

The nurse is assessing a patients functional abilities and asks the patient, How would you rate your ability to prepare a balanced meal? How would you rate your ability to balance a checkbook? How would you rate your ability to keep track of your appointments? Which tool would be indicated for the best results of this patients perception of their abilities? A Functional Activities Questionnaire (FAQ) B Mini Mental Status Exam (MMSE) C 24hFAQ D Performance-based functional measurement

A Functional Activities Questionnaire (FAQ) The FAQ is an example of a self-report tool which provides information about the patients perception of functional ability. The MMSE assesses cognitive impairment. The 24hFAQ is used to assess functional ability in postoperative patients. Performance-based tools involve actual observation of a standardized task, completion of which is judged by objective criteria.

Medications that reduce fever by interfering with the hypothalamic response include: a.salicylates. b.acetaminophen. c.ibuprofen. d.corticosteroids

D- corticosteroids Antipyretics are medications that reduce fever. Nonsteroidal drugs such as acetaminophen, salicylates, indomethacin, ibuprofen, and ketorolac reduce fever by increasing heat loss. Although not used to treat fever, corticosteroids reduce heat production by interfering with the hypothalamic response

The nurse is assessing a patients functional performance. What assessment parameters will be most important in this assessment? A Continence assessment, gait assessment, feeding assessment, dressing assessment, transfer assessment B Height, weight, body mass index (BMI), vital signs assessment C Sleep assessment, energy assessment, memory assessment, concentration assessment D Healthy individual, volunteers at church, works part time, takes care of family and house

A Continence assessment, gait assessment, feeding assessment, dressing assessment, transfer assessment Functional impairment, disability, or handicap refers to varying degrees of an individuals inability to perform the tasks required to complete normal life activities without assistance. Height, weight, BMI, and vital signs are physical assessment. Sleep, energy, memory, and concentration are part of a depression screening. Healthy, volunteering, working, and caring for family and house are functional abilities, not performance.

A nurse is an advanced practice registered nurse (APRN) who cares for geriatrics. This nurse is which type of advanced practice nurse? a.Clinical nurse specialist b.Nurse practitioner c.Certified nurse-midwife d.Certified registered nurse anesthetist

A The clinical nurse specialist The clinical nurse specialist (CNS) is an APRN who is an expert clinician in a specialized area of practice, such as geriatrics or pediatrics. The nurse practitioner (NP) is an APRN who provides health care to a group of patients, usually in an outpatient, ambulatory care, or community-based setting. A certified nurse-midwife (CNM) is an APRN who is educated in midwifery and is certified by the American College of Nurse-Midwives. A certified registered nurse anesthetist (CRNA) is an APRN with advanced education in a nurse anesthesia accredited program.

The nurse manager of a pediatric clinic could confirm that the new nurse recognized the purpose of the HEADSS Adolescent Risk Profile when the new nurse responds that it is used to assess for needs related to A anticipatory guidance. B low-risk adolescents. C physical development. D sexual development.

A anticipatory guidance. The HEADSS Adolescent Risk Profile is a psychosocial assessment screening tool which assesses home, education, activities, drugs, sex, and suicide for the purpose of identifying high- risk adolescents and the need for anticipatory guidance. It is used to identify high-risk, not low-risk, adolescents. Physical development is assessed with anthropometric data. Sexual development is assessed using physical examination.

Aspects of safety culture that contribute to a culture of safety in a health care organization include A communication. B fear of punishment. C malpractice implications. D team nursing.

A communication. Aspects that contribute to a culture of safety include leadership, teamwork, an evidence base, communication, learning, a just culture, and patient-centered care. Fear of professional or personal punishment and concern about malpractice implications are considered barriers to a culture of safety. No model of nursing care has been related to a culture of safety.

.A nurse is concerned with the chain of infection when taking care of contaminated care items. Semicritical items that require disinfection include: a.linens. b.bedpans. c.blood pressure cuffs. d.gastrointestinal endoscopes

D- gastrointestinal endoscopes

What percentage of hip fractures are the result of falls? A 50% B 80% C 90% D 100%

C 90%

Prior to drug administration the nurse reviews the seven rights, which include right patient, right medication, right time, right dose, right education, right documentation, and right A room. B route. C physician. D manufacturer.

B route. The right route (e.g., oral or intramuscular) is an essential component to verify prior to the administration of any drug. The patient does not need to be in a specific location. There may be a number of physicians caring for a patient who prescribe medications for any given patient. A similar drug may be made by a number of different companies, and checking the manufacturer is not considered one of the seven rights. However, the nurse will want to be aware of a difference, because different companies prepare the same medication in different ways with different inactive ingredients, which can affect patient response.

Match the activities listed with the appropriate functional level of ability: Use A for instrumental activities of daily living (IADLs) and use B for basic activities of daily living (BADLs). A. Uses a cane B. Bathes daily C. Takes medications as prescribed D. Dresses self E. Balances the checkbook F. Cleans the house

B, B, A, B, A, A Functional impairment, disability, or handicap refers to varying degrees of an individuals inability to perform the tasks required to complete normal life activities without assistance. IADLs

A 6-year-old child was taken to the after-hours pediatric clinic with a fever and a rash. She had been seen by her pediatrician earlier in the day and had been given a prescription for an antibiotic. Later that evening she developed a fever and a rash on her abdomen. The nurse who assesses the child in the clinic suspected the symptoms are associated with which of the following? a.Dehydration b.An allergic response to the prescribed medication c.Febrile seizures d.Fever of unknown origin (FUO)

B- An allergic response to the prescribed medication Sometimes a fever results from a hypersensitivity response to a medication, especially when the medication is taken for the first time.

A nurse notices that a patient has an irregular pulse. The nurse should do which of the following? a.Count the number of lub-dubs occurring in 30 seconds. b.Assess how often the dysrhythmia is occurring. c.Assess the radial pulse for a pulse deficit. d.Chart the abnormally low heart rate as tachycardia.

B- Assess how often the dysrhythmia is occurring A regular interval interrupted by an early beat, late beat, or a missed beat indicates an abnormal rhythm or dysrhythmia. A dysrhythmia alters cardiac function, particularly if it occurs repetitively. If your patient has a dysrhythmia you need to assess how often it is occurring.

A woman has been hospitalized with pneumonia. She has had oxygen on via nasal cannula at a rate of 2 L per minute. A nursing student is taking her vital signs. She notes that her respirations are labored and the rate is 32 respirations per minute. The nursing student recognizes this as which of the following? a.Normal. b.Tachypnea. c.Bradypnea. d.Apnea.

B- Tachypnea.

Standard precautions involve using personal protective equipment with all patients regardless of the presence of infections. Therefore when obtaining a blood sample, the nurse must wear: a.a mask. b.gloves. c.gloves and a mask. d.gloves, a mask, and a gown.

B- gloves. Apply disposable gloves when there is a risk for exposing the hands to blood as in blood drawing, body fluids, mucous membranes, non-intact skin, or potentially infectious material on objects or surfaces

A systolic blood pressure (BP) less than 90 mm Hg or a diastolic BP less than 60 mm Hg is known as: a.hypertension. b.hypotension. c.orthostatic hypotension. d.postural hypotension.

B- hypotension Hypotension is a systolic blood pressure (SBP) less than 90 mm Hg or a diastolic blood pressure (DBP) less than 60 mm Hg. The most common alteration in blood pressure is hypertension, an often asymptomatic disorder characterized by persistently elevated blood pressure

The nurse has had a nasal culture performed and has been found to be MRSA positive. Because the nurse has not been ill from the bacteria, the nurses nasal cavity can best be described as a: a.susceptible host. b.reservoir. c.portal of entry. d.mode of transmission.

B- reservoir A place in which microorganisms survive, multiply, and wait to transfer to a susceptible host is called a reservoir

A nurse is assigned to multiple patients on a busy surgical unit. To minimize the onset and spread of infection, the nurse should: a.insert indwelling catheters to prevent incontinence. b.use aseptic technique when performing procedures. c.use barriers sparingly to reduce the patients sense of isolation. d.keep mucus membranes dry to prevent maceration.

B- use aseptic technique when performing procedures Invasive treatment devices such as intravenous (IV) catheters or indwelling urinary catheters impair or bypass the bodys natural defenses against microorganisms

The mechanical movement of gases into and out of the lungs is known as: a.respiration. b.ventilation. c.perfusion. d.eupnea.

B- ventilation

The nurse is assessing a patient with a mobility dysfunction and wants to gain insight into the patients functional ability. What question would be the most appropriate? A Are you able to shop for yourself? B Do you use a cane, walker, or wheelchair to ambulate? C Do you know what todays date is? D Were you sad or depressed more than once in the last 3 days?

B Do you use a cane, walker, or wheelchair to ambulate? Do you use a cane, walker, or wheelchair to ambulate? will assist the nurse in determining the patients ability to perform self-care activities. A nutritional health risk assessment is not the functional assessment. Knowing the date is part of a mental status exam. Assessing sadness is a question to ask in the depression screening.

A mother complains to the nurse at the pediatric clinic that her 4-year-old child always talks to her toys and makes up stories. The mother wants her child to have a psychologic evaluation. The nurses best initial response is to A refer the child to a psychologist. B explain that playing make believe with dolls and people is normal at this age. C complete a developmental screening. D separate the child from the mother to get more information.

B explain that playing make believe with dolls and people is normal at this age By the end of the fourth year, it is expected that a child will engage in fantasy, so this is normal at this age. A referral to a psychologist would be premature based only on the complaint of the mother. Completing a developmental screening would be very appropriate but not the initial response. The nurse would certainly want to get more information, but separating the child from the mother is not necessary at this time.

The nursing assistive personnel (NAP) is working on a busy pediatric unit in a hospital. She has a cut on her hand that has not been kept covered. It hurts her to wash her hands or sanitize them, so she has been providing patient care without performing hand hygiene. Several of the patients on the pediatric unit have suffered hospitalassociated infections of rotavirus. This was thought to be a result of the NAPs lack of hand hygiene. This type of disease transmission can best be described as: a.indirect. b.natural active immunity. c.direct. d.natural passive immunity

C- direct. Hands of health care workers often transmit microorganisms. This mode of transmission is called direct transmission

When patients with normal blood pressure experience a drop in blood pressure upon rising to an upright position and have symptoms of light-headedness or dizziness, the condition is known as: a.hypertension. b.hypotension. c.orthostatic hypotension. d.the pulse pressure.

C- orthostatic hypotension. Orthostatic hypotension , also referred to as postural hypotension, is a reduction of systolic blood pressure (SBP) of at least 20 mm Hg or reduction of diastolic blood pressure (DBP) of at least 10 mm Hg within 3 minutes of quiet standing

One of the ways the body increases heat production is through: a.convection. b.radiation. c.shivering. d.evaporation

C- shivering. Shivering is an involuntary body response to temperature differences in the body. Shivering can increase heat production 4 to 5 times greater than normal.

Mobility for the patient changes throughout the life span; this is known as the process of A aging and illness. B illness and disease. C health and wellness. D growth and development.

D growth and development. Growth and development happens from infancy to death. Muscular changes are always happening, and these changes affect the individual and his or her performance in life. Aging, illness, health, and wellness do have an effect on a person, but they dont always affect mobility.

To plan early intervention and care for an infant with Down syndrome, the nurse considers knowledge of other physical development exemplars such as A cerebral palsy. B failure to thrive. C fetal alcohol syndrome. D hydrocephaly.

D hydrocephaly. Hydrocephaly is also a physical development exemplar. Cerebral palsy is an exemplar of adaptive developmental delay. Failure to thrive is an exemplar of social/emotional developmental delay. Fetal alcohol syndrome is an exemplar of cognitive developmental delay.

A 3-month-old infant is at increased risk for developing anemia. The nurse would identify which principle contributing to this risk? A The infant is becoming more active. B There is an increase in intake of breast milk or formula. C The infant is unable to maintain an adequate iron intake. D A depletion of fetal hemoglobin occurs.

D A depletion of fetal hemoglobin occurs Fetal hemoglobin is present for about 5 months. The fetal hemoglobin begins deteriorating, and around 2 to 3 months the infant is at increased risk of developing an anemia due to decreasing levels of hemoglobin. Breast milk or formula is the primary food intake up to around 6 months. Often iron supplemented formula is offered, and/or an iron supplement is given if the infant is breastfed.

The nurse has noticed slight redness when washing her hands. She is concerned about developing a latex allergy. To prevent this, the nurse should: a.wear only powdered gloves to help protect her skin. b.wear gloves constantly to decrease the number of handwashings. c.apply only oil-based hand care products to her hands. d.report to employee health services and/or seek immediate medical care

D- report to employee health services and/or seek immediate medical care The Association of Perioperative Registered Nurses (2009) suggests that nurses report to employee health services to seek medical treatment immediately if a reaction or dermatitis occurs

Under high pressure, the left ventricle ejects blood into the aorta; the peak pressure is known as: a.diastolic pressure. b.pulse pressure. c.hypertension. d.systolic pressure.

D- systolic pressure. Under high pressure, the left ventricle ejects blood into the aorta; the peak pressure is known as systolic pressure. When the ventricles relax, the blood remaining in the arteries exerts a minimum or diastolic pressure

The nurse is listening for bowel sounds in a postoperative patient. The bowel sounds are slow, as they are heard only every 3-4 minutes. The patient asks the nurse why this is happening. What is the nurse's best response? a. "Anesthesia during surgery and pain medication after surgery may slow peristalsis in the bowel." b. "Some people have a slower bowel than others, and this is nothing to be concerned about." c. "The foods you eat contribute to peristalsis, so you should eat more fiber in your diet." d. "Bowel peristalsis is slow because you are not walking. Get more exercise during the day."

a. "Anesthesia during surgery and pain medication after surgery may slow peristalsis in the bowel." Anesthesia and pain medication used in conjunction with the surgery are affecting the peristalsis of the bowel. Having a slower bowel, eating certain food, or lack of exercise will not have a direct effect on the bowel.

Which patient is at greatest risk for developing a pressure ulcer? a.Young adult paraplegic with pneumonia b.Middle age adult that can turn by self in bed c.Teenager with a sprained ankle on crutches d.Middle-age adult with breast cancer

a.Young adult paraplegic with pneumonia The paraplegic (paralyzed) is most at risk. The direct effect of pressure on the skin by immobility is compounded by metabolic changes. Older adult patients and patients with paralysis have a greater risk for developing pressure ulcers. A breast cancer patient is mobile as is the teenager on crutches, which decreases their risk. The middle-age adult is turning, decreasing the risk of pressure ulcers.

A 90-year-old patient constantly tells the nurse stories about life many years ago. The nurse encourages this behavior because reminiscence or life review is: a.a technique that prepares the individual for the end of life. b.a review of dispelling past experiences as meaningless. c.helpful although it is unnatural in the older adult. d.a way for the elderly to realize that conflicts cannot be reconciled.

a.a technique that prepares the individual for the end of life. Reminiscence, or life review, is a technique that facilitates the individuals preparation for the end of life. It is an adaptive function of older adults that allows them to recall the past for the purpose of assigning new meaning to past experiences. Reminiscence is the natural way older adults revive their past in an attempt to establish order and meaning and to reconcile conflicts and disappointments as they prepare for death.

The nurse is explaining to a student nurse about impaired central perfusion. The nurse knows the student understands this problem when the student makes which statement? a. "Central perfusion is monitored only by the physician." b. "Central perfusion involves the entire body." c. "Central perfusion is decreased with hypertension." d. "Central perfusion is toxic to the cardiac system."

b. "Central perfusion involves the entire body." Central perfusion does involve the entire body as all organs are supplied with oxygen and vital nutrients. The physician does not control the body's ability for perfusion. Central perfusion is not decreased with hypertension. Central perfusion is not toxic to the cardiac system.

The nurse is reviewing a patient with a mobility dysfunction and wants to gain insight into the patient's functional ability. What question would be the most appropriate? a. "Are you able to shop for yourself?" b. "Do you use a cane, walker, or wheelchair to ambulate?" c. "Do you know what today's date is?" d. "Were you sad or depressed more than once in the last 3 days?"

b. "Do you use a cane, walker, or wheelchair to ambulate?" "Do you use a cane, walker, or wheelchair to ambulate?" will assist the nurse in determining the patient's ability to perform self-care activities. A nutritional health risk assessment is not the functional assessment. Knowing the date is part of a mental status exam. Reviewing sadness is a question to ask in the depression screening

Which term is used to describe a person who does not believe in the existence of a higher power but has had a meaningful life by contributing to the lives of others? a. Deist b. Atheist c. Fatalist d. Humanist

b. Atheist An atheist is a person who does not believe in the existence of a higher power or Supreme Being. Atheists search for meaning in life through their work and relationships with others. A humanist is devoted to the study of human nature and human welfare. A fatalist believes that all events were predetermined. A deist believes that God created the world but does not control how it functions.

An older adult patient residing at an adult assisted living facility complains of hearing and visual disturbances. A nurse must be alert to the effects of sensory deprivation that are associated with which of the following? a.Stable affect b.Altered perception c.Improved task completion d.Decreased need for social interaction

b.Altered perception Sensory deprivation sometimes produces cognitive changes such as the inability to solve problems, poor task performance, and disorientation. It also can cause affective changes (e.g., boredom, restlessness, increased anxiety, emotional lability) and/or perceptual changes (e.g., reduced attention span, disorganized visual and motor coordination, confusion of sleeping and waking states). Patients may withdraw from social situations because of their inability to handle stimuli.

A nursing student is undergoing a community health clinical rotation. One of the patients is a 53-year-old grandmother who has recently assumed custody of her daughters two young children, ages 3 and 5 years old. Regarding the childrens welfare, which of the following is most important for the nursing student to assess on this visit? a.The patients financial ability to care for two young children b.The patients knowledge of safety precautions for young children c.The patients emotional stability d.The patients feelings regarding taking on this responsibility

b.The patients knowledge of safety precautions for young children Some patients are unaware of safety precautions, such as keeping medicine, poisonous plants, or other poisons away from children or reading the expiration date on food products. A nursing assessment will identify the patients level of knowledge regarding home safety so that safety problems can be corrected with an individualized care plan. Although the other responses are important, they are not the focus of home safety issues, but financial and emotional aspects of care.

An RN is assessing an 87-year-old patient who has gone to the clinic to see the health care provider for a follow-up appointment. The nurse notes that the patient looks tired and has dark circles under her eyes. She assesses her skin turgor, which is poor. She is concerned because she knows that poor skin turgor can predispose the patient to which of the following? a.Dehydration b.Edema c.Skin breakdown d.Direct trauma

c- Skin breakdown A decrease in turgor predisposes a patient to skin breakdown. Turgor is the skins elasticity. Normally the skin loses its elasticity with age. Edema or dehydration diminishes turgor. Direct trauma and impairment of venous return are two common causes of edem

The nurse is seeking clarification of a statement that was made by a patient. What is the best way for the nurse to seek clarification? a. "What are the common elements here?" b. "Tell me again about your experiences." c. "Am I correct in understanding that..." d. "Tell me everything from the beginning."

c. "Am I correct in understanding that Clarification ensures that both the nurse and the patient share mutual understanding of the communication. The distracters encourage comparison rather than clarification and present implied questions that suggest the nurse was not listening.

Which nursing intervention is appropriate for a postoperative patient with a history of sleep apnea? a. Padded side rails b. Bedside commode c. Continuous pulse oximetry d. Suction equipment at bedside

c. Continuous pulse oximetry Continuous pulse oximetry is important for postoperative patients with sleep apnea to monitor for hypoxemia. Pain medications and anesthesia are likely to worsen sleep apnea. Padded side rails, bedside commode, and suction equipment are not needed.

Which nonpharmacological pain-relief technique is appropriate for a confused, nonverbal patient? a. Tai chi b. Biofeedback c. Massage therapy d. Guided imagery

c. Massage therapy Massage therapy is appropriate for use with nonverbal, confused patients as it promotes relaxation and healing without the need for the patient to follow directions. A confused, nonverbal patient will not be able to follow the directions required for tai chi, biofeedback, or guided imagery

The patient has recently moved into a newly renovated home in the inner city. The patient is being seen in the clinic for complaints of ongoing headaches, nausea, dizziness and fatigue. The symptoms started shortly after moving into the new home. As the nurse gathers information, which of the following questions would be most appropriate to ask the patient? a.Have you changed the battery in your smoke alarm recently? b.Have you changed your diet since moving? c.What type of furnace do you have? d.When was the last time your house was painted?

c.What type of furnace do you have? A furnace, stove, or fireplace that is not properly vented introduces carbon monoxide into the environment. This gas binds strongly with hemoglobin; preventing the formation of oxyhemoglobin and thus reducing the supply of oxygen delivered to the tissues. Low concentrations cause nausea, dizziness, headache, and fatigue. The importance of having a proper working smoke detector will decrease the chance of smoke inhalation and potential death owing to a fire but does not produce the symptoms listed. A balanced diet and proper storage of food is essential to decrease the chance of infection to the gastrointestinal system but does not produce the symptoms listed. Painting would not produce the symptoms listed even if the paint is old and contains lead as it must be ingested.

The nurse educator would identify a need for additional teaching when the student lists which example as a type of learning? a. Affective b. Cognitive c. Psychomotor d. Self-directed

d. Self-directed Self-directed is one approach to learning but is not considered a type or domain of learning. Self-directed would be a cognitive way of learning. Affective (feelings/attitude), cognitive (knowledge), and psychomotor (skills/performance) are the main domains of learning.

Which action should the nurse implement to help prevent thrombus formation in postsurgical patients? a.Maintain complete bed rest. b.Place tight clothing on the legs and waist. c.Put pillows under the knees. d.Position properly with use of antiembolic stockings.

d.Position properly with use of antiembolic stockings. Proper positioning used with other therapies (e.g., anticoagulants and antiembolic stockings) helps reduce thrombus formation. When positioning patients, use caution to prevent pressure on the posterior knee and deep veins in the lower extremities. Teach patients to avoid crossing the legs, sitting for prolonged periods of time, wearing tight clothing that constricts the legs or waist, putting pillows under the knees, and massaging the legs. Complete bed rest increases the chance for thrombus formation.

A patient is experiencing incisional pain after an operation. When using Maslows hierarchy of needs, the nurse realizes that for the patient to return to a prehospitalized status, the patient needs to progress beyond which of the following? a.Belonging b.Self-esteem c.Self-actualization d.Safety and security

d.Safety and security omposed of five levels. When the most basic needs, such as hunger and oxygen, are met, the person strives to satisfy those needs for safety and security on the next highest level. Disturbances at lower levels interfere with the highest level, self-actualization or the realization of ones potential. Individuals need to satisfy each level before moving on to the next. Belonging occupies the third stage, where threats to relationships create anticipatory loneliness and alienation. Self-esteem occupies the fourth stage, and threats create alienation. Self-actualization is the highest level that one can achievethe realization that one has reached his or her highest potential. Safety and security occupies the second stage, and threats to security (such as pain) produce feelings of insecurity.

The lack of weight bearing leads to bone _________ and __________ from the skeletal system.

demineralization, calcium loss Weight bearing helps to strengthen the bone. Lack of weight bearing means that the bone is losing minerals and calcium that strengthen it.


Related study sets

Specific Neurodegenerative Disorders

View Set

Policy provisions, options and riders

View Set

Adult Health Exam 2 - respiratory, HTN, HF, PVD

View Set

ACG 302 Exam 2 Concept Questions

View Set

Chapter 20: Taxation and the Public Budget

View Set

Chapter 4: Savings Plans and Payment Accounts

View Set

the parts of the brain; unit 3, lesson 3

View Set